ΠŸΡ€ΠΈΠΌΠ΅Ρ€Ρ‹ с дробями Π½Π° слоТСниС Π²Ρ‹Ρ‡ΠΈΡ‚Π°Π½ΠΈΠ΅ Π΄Π΅Π»Π΅Π½ΠΈΠ΅ ΡƒΠΌΠ½ΠΎΠΆΠ΅Π½ΠΈΠ΅ 5 класс: ДСйствия с дробями

Π‘ΠΎΠ΄Π΅Ρ€ΠΆΠ°Π½ΠΈΠ΅

ДСйствия с дробями

Π”Ρ€ΠΎΠ±ΠΈ ΠΌΠΎΠΆΠ½ΠΎ ΡΠΊΠ»Π°Π΄Ρ‹Π²Π°Ρ‚ΡŒ, Π²Ρ‹Ρ‡ΠΈΡ‚Π°Ρ‚ΡŒ, ΡƒΠΌΠ½ΠΎΠΆΠ°Ρ‚ΡŒ ΠΈ Π΄Π΅Π»ΠΈΡ‚ΡŒ. Π’Π°ΠΊΠΆΠ΅, Π΄Ρ€ΠΎΠ±ΠΈ ΠΌΠΎΠΆΠ½ΠΎ ΡΡ€Π°Π²Π½ΠΈΠ²Π°Ρ‚ΡŒ ΠΌΠ΅ΠΆΠ΄Ρƒ собой. Π’ ΠΏΡ€ΠΈΠ½Ρ†ΠΈΠΏΠ΅, всё Ρ‡Ρ‚ΠΎ ΠΌΠΎΠΆΠ½ΠΎ Π΄Π΅Π»Π°Ρ‚ΡŒ с ΠΎΠ±Ρ‹Ρ‡Π½Ρ‹ΠΌΠΈ числами, ΠΌΠΎΠΆΠ½ΠΎ Π΄Π΅Π»Π°Ρ‚ΡŒ ΠΈ с дробями.

Π‘Π»ΠΎΠΆΠ΅Π½ΠΈΠ΅ Π΄Ρ€ΠΎΠ±Π΅ΠΉ с ΠΎΠ΄ΠΈΠ½Π°ΠΊΠΎΠ²Ρ‹ΠΌΠΈ знамСнатСлями

Π‘Π»ΠΎΠΆΠ΅Π½ΠΈΠ΅ Π΄Ρ€ΠΎΠ±Π΅ΠΉ Π±Ρ‹Π²Π°Π΅Ρ‚ Π΄Π²ΡƒΡ… Π²ΠΈΠ΄ΠΎΠ²:

  1. Π‘Π»ΠΎΠΆΠ΅Π½ΠΈΠ΅ Π΄Ρ€ΠΎΠ±Π΅ΠΉ с ΠΎΠ΄ΠΈΠ½Π°ΠΊΠΎΠ²Ρ‹ΠΌΠΈ знамСнатСлями;
  2. Π‘Π»ΠΎΠΆΠ΅Π½ΠΈΠ΅ Π΄Ρ€ΠΎΠ±Π΅ΠΉ с Ρ€Π°Π·Π½Ρ‹ΠΌΠΈ знамСнатСлями.

Π‘Π½Π°Ρ‡Π°Π»Π° ΠΈΠ·Γ½Ρ‡ΠΈΠΌ слоТСниС Π΄Ρ€ΠΎΠ±Π΅ΠΉ с ΠΎΠ΄ΠΈΠ½Π°ΠΊΠΎΠ²Ρ‹ΠΌΠΈ знамСнатСлями. Π’ΡƒΡ‚ всё просто. Π§Ρ‚ΠΎΠ±Ρ‹ ΡΠ»ΠΎΠΆΠΈΡ‚ΡŒ Π΄Ρ€ΠΎΠ±ΠΈ с ΠΎΠ΄ΠΈΠ½Π°ΠΊΠΎΠ²Ρ‹ΠΌΠΈ знамСнатСлями, Π½ΡƒΠΆΠ½ΠΎ ΡΠ»ΠΎΠΆΠΈΡ‚ΡŒ ΠΈΡ… числитСли, Π° Π·Π½Π°ΠΌΠ΅Π½Π°Ρ‚Π΅Π»ΡŒ ΠΎΡΡ‚Π°Π²ΠΈΡ‚ΡŒ Π±Π΅Π· измСнСния.

НапримСр, слóТим Π΄Ρ€ΠΎΠ±ΠΈΒ  Β  ΠΈΒ  . Π‘ΠΊΠ»Π°Π΄Ρ‹Π²Π°Π΅ΠΌ числитСли, Π° Π·Π½Π°ΠΌΠ΅Π½Π°Ρ‚Π΅Π»ΡŒ оставляСм Π±Π΅Π· измСнСния:

Π­Ρ‚ΠΎΡ‚ ΠΏΡ€ΠΈΠΌΠ΅Ρ€ ΠΌΠΎΠΆΠ½ΠΎ Π»Π΅Π³ΠΊΠΎ ΠΏΠΎΠ½ΡΡ‚ΡŒ, Ссли Π²ΡΠΏΠΎΠΌΠ½ΠΈΡ‚ΡŒ ΠΏΡ€ΠΎ ΠΏΠΈΡ†Ρ†Ρƒ, которая Ρ€Π°Π·Π΄Π΅Π»Π΅Π½Π° Π½Π° Ρ‡Π΅Ρ‚Ρ‹Ρ€Π΅ части. Если ΠΊΒ  Β ΠΏΠΈΡ†Ρ†Ρ‹ ΠΏΡ€ΠΈΠ±Π°Π²ΠΈΡ‚ΡŒ ΠΏΠΈΡ†Ρ†Ρ‹, Ρ‚ΠΎ получится ΠΏΠΈΡ†Ρ†Ρ‹:


ΠŸΡ€ΠΈΠΌΠ΅Ρ€ 2. Π‘Π»ΠΎΠΆΠΈΡ‚ΡŒ Π΄Ρ€ΠΎΠ±ΠΈ ΠΈ .

ΠžΠΏΡΡ‚ΡŒ ΠΆΠ΅ складываСм числитСли, Π° Π·Π½Π°ΠΌΠ΅Π½Π°Ρ‚Π΅Π»ΡŒ оставляСм Π±Π΅Π· измСнСния:

Π’ ΠΎΡ‚Π²Π΅Ρ‚Π΅ ΠΏΠΎΠ»ΡƒΡ‡ΠΈΠ»Π°ΡΡŒ Π½Π΅ΠΏΡ€Π°Π²ΠΈΠ»ΡŒΠ½Π°Ρ Π΄Ρ€ΠΎΠ±ΡŒ .

Β  Если наступаСт ΠΊΠΎΠ½Π΅Ρ† Π·Π°Π΄Π°Ρ‡ΠΈ, Ρ‚ΠΎ ΠΎΡ‚ Π½Π΅ΠΏΡ€Π°Π²ΠΈΠ»ΡŒΠ½Ρ‹Ρ… Π΄Ρ€ΠΎΠ±Π΅ΠΉ принято ΠΈΠ·Π±Π°Π²Π»ΡΡ‚ΡŒΡΡ. Π§Ρ‚ΠΎΠ±Ρ‹ избавится ΠΎΡ‚ Π½Π΅ΠΏΡ€Π°Π²ΠΈΠ»ΡŒΠ½ΠΎΠΉ Π΄Ρ€ΠΎΠ±ΠΈ, Π½ΡƒΠΆΠ½ΠΎ Π²Ρ‹Π΄Π΅Π»ΠΈΡ‚ΡŒ Π² Π½Π΅ΠΉ Ρ†Π΅Π»ΡƒΡŽ Ρ‡Π°ΡΡ‚ΡŒ. Π’ нашСм случаС цСлая Ρ‡Π°ΡΡ‚ΡŒ выдСляСтся Π»Π΅Π³ΠΊΠΎ β€” Π΄Π²Π° Ρ€Π°Π·Π΄Π΅Π»ΠΈΡ‚ΡŒ Π½Π° Π΄Π²Π° Π±ΡƒΠ΄Π΅Ρ‚ ΠΎΠ΄ΠΈΠ½:

Π­Ρ‚ΠΎΡ‚ ΠΏΡ€ΠΈΠΌΠ΅Ρ€ ΠΌΠΎΠΆΠ½ΠΎ Π»Π΅Π³ΠΊΠΎ ΠΏΠΎΠ½ΡΡ‚ΡŒ, Ссли Π²ΡΠΏΠΎΠΌΠ½ΠΈΡ‚ΡŒ ΠΏΡ€ΠΎ ΠΏΠΈΡ†Ρ†Ρƒ, которая Ρ€Π°Π·Π΄Π΅Π»Π΅Π½Π° Π½Π° Π΄Π²Π΅ части. Если ΠΊ ΠΏΠΈΡ†Ρ†Ρ‹ ΠΏΡ€ΠΈΠ±Π°Π²ΠΈΡ‚ΡŒ Π΅Ρ‰Π΅ ΠΏΠΈΡ†Ρ†Ρ‹, Ρ‚ΠΎ получится ΠΎΠ΄Π½Π° цСлая ΠΏΠΈΡ†Ρ†Π°:


ΠŸΡ€ΠΈΠΌΠ΅Ρ€ 3. Π‘Π»ΠΎΠΆΠΈΡ‚ΡŒ Π΄Ρ€ΠΎΠ±ΠΈΒ  Β  ΠΈΒ  .

ΠžΠΏΡΡ‚ΡŒ ΠΆΠ΅ складываСм числитСли, Π° Π·Π½Π°ΠΌΠ΅Π½Π°Ρ‚Π΅Π»ΡŒ оставляСм Π±Π΅Π· измСнСния:

Π­Ρ‚ΠΎΡ‚ ΠΏΡ€ΠΈΠΌΠ΅Ρ€ ΠΌΠΎΠΆΠ½ΠΎ Π»Π΅Π³ΠΊΠΎ ΠΏΠΎΠ½ΡΡ‚ΡŒ, Ссли Π²ΡΠΏΠΎΠΌΠ½ΠΈΡ‚ΡŒ ΠΏΡ€ΠΎ ΠΏΠΈΡ†Ρ†Ρƒ, которая Ρ€Π°Π·Π΄Π΅Π»Π΅Π½Π° Π½Π° Ρ‚Ρ€ΠΈ части. Если ΠΊ ΠΏΠΈΡ†Ρ†Ρ‹ ΠΏΡ€ΠΈΠ±Π°Π²ΠΈΡ‚ΡŒ Π΅Ρ‰Ρ‘ ΠΏΠΈΡ†Ρ†Ρ‹, Ρ‚ΠΎ получится ΠΏΠΈΡ†Ρ†Ρ‹:


ΠŸΡ€ΠΈΠΌΠ΅Ρ€ 4. Найти Π·Π½Π°Ρ‡Π΅Π½ΠΈΠ΅ выраТСния 

Π­Ρ‚ΠΎΡ‚ ΠΏΡ€ΠΈΠΌΠ΅Ρ€ Ρ€Π΅ΡˆΠ°Π΅Ρ‚ΡΡ Ρ‚ΠΎΡ‡Π½ΠΎ Ρ‚Π°ΠΊΠΆΠ΅, ΠΊΠ°ΠΊ ΠΈ ΠΏΡ€Π΅Π΄Ρ‹Π΄ΡƒΡ‰ΠΈΠ΅. ЧислитСли Π½Π΅ΠΎΠ±Ρ…ΠΎΠ΄ΠΈΠΌΠΎ ΡΠ»ΠΎΠΆΠΈΡ‚ΡŒ, Π° Π·Π½Π°ΠΌΠ΅Π½Π°Ρ‚Π΅Π»ΡŒ ΠΎΡΡ‚Π°Π²ΠΈΡ‚ΡŒ Π±Π΅Π· измСнСния:

ΠŸΠΎΠΏΡ€ΠΎΠ±ΡƒΠ΅ΠΌ ΠΈΠ·ΠΎΠ±Ρ€Π°Π·ΠΈΡ‚ΡŒ нашС Ρ€Π΅ΡˆΠ΅Π½ΠΈΠ΅ с ΠΏΠΎΠΌΠΎΡ‰ΡŒΡŽ рисунка. Если ΠΊΒ Β ΠΏΠΈΡ†Ρ†Ρ‹ ΠΏΡ€ΠΈΠ±Π°Π²ΠΈΡ‚ΡŒΒ Β ΠΏΠΈΡ†Ρ†Ρ‹ ΠΈ Π΅Ρ‰Ρ‘ ΠΏΡ€ΠΈΠ±Π°Π²ΠΈΡ‚ΡŒΒ Β ΠΏΠΈΡ†Ρ†Ρ‹, Ρ‚ΠΎ получится 1 цСлая ΠΈ Π΅Ρ‰Ρ‘Β Β ΠΏΠΈΡ†Ρ†Ρ‹.

Как Π²ΠΈΠ΄ΠΈΡ‚Π΅ Π² слоТСнии Π΄Ρ€ΠΎΠ±Π΅ΠΉ с ΠΎΠ΄ΠΈΠ½Π°ΠΊΠΎΠ²Ρ‹ΠΌΠΈ знамСнатСлями Π½Π΅Ρ‚ Π½ΠΈΡ‡Π΅Π³ΠΎ слоТного. Достаточно ΠΏΠΎΠ½ΠΈΠΌΠ°Ρ‚ΡŒ ΡΠ»Π΅Π΄ΡƒΡŽΡ‰ΠΈΠ΅ ΠΏΡ€Π°Π²ΠΈΠ»Π°:

  1. Π§Ρ‚ΠΎΠ±Ρ‹ ΡΠ»ΠΎΠΆΠΈΡ‚ΡŒ Π΄Ρ€ΠΎΠ±ΠΈ с ΠΎΠ΄ΠΈΠ½Π°ΠΊΠΎΠ²Ρ‹ΠΌΠΈ знамСнатСлями, Π½ΡƒΠΆΠ½ΠΎ ΡΠ»ΠΎΠΆΠΈΡ‚ΡŒ ΠΈΡ… числитСли, Π° Π·Π½Π°ΠΌΠ΅Π½Π°Ρ‚Π΅Π»ΡŒ ΠΎΡΡ‚Π°Π²ΠΈΡ‚ΡŒ Π±Π΅Π· измСнСния;
  2. Если Π² ΠΎΡ‚Π²Π΅Ρ‚Π΅ ΠΏΠΎΠ»ΡƒΡ‡ΠΈΠ»Π°ΡΡŒ Π½Π΅ΠΏΡ€Π°Π²ΠΈΠ»ΡŒΠ½Π°Ρ Π΄Ρ€ΠΎΠ±ΡŒ, Ρ‚ΠΎ Π½ΡƒΠΆΠ½ΠΎ Π²Ρ‹Π΄Π΅Π»ΠΈΡ‚ΡŒ Π² Π½Π΅ΠΉ Ρ†Π΅Π»ΡƒΡŽ Ρ‡Π°ΡΡ‚ΡŒ.

Π‘Π»ΠΎΠΆΠ΅Π½ΠΈΠ΅ Π΄Ρ€ΠΎΠ±Π΅ΠΉ с Ρ€Π°Π·Π½Ρ‹ΠΌΠΈ знамСнатСлями

Π’Π΅ΠΏΠ΅Ρ€ΡŒ научимся ΡΠΊΠ»Π°Π΄Ρ‹Π²Π°Ρ‚ΡŒ Π΄Ρ€ΠΎΠ±ΠΈ с Ρ€Π°Π·Π½Ρ‹ΠΌΠΈ знамСнатСлями. Когда ΡΠΊΠ»Π°Π΄Ρ‹Π²Π°ΡŽΡ‚ Π΄Ρ€ΠΎΠ±ΠΈ, Π·Π½Π°ΠΌΠ΅Π½Π°Ρ‚Π΅Π»ΠΈ этих Π΄Ρ€ΠΎΠ±Π΅ΠΉ Π΄ΠΎΠ»ΠΆΠ½Ρ‹ Π±Ρ‹Ρ‚ΡŒ ΠΎΠ΄ΠΈΠ½Π°ΠΊΠΎΠ²Ρ‹ΠΌΠΈ. Но ΠΎΠ΄ΠΈΠ½Π°ΠΊΠΎΠ²Ρ‹ΠΌΠΈ ΠΎΠ½ΠΈ Π±Ρ‹Π²Π°ΡŽΡ‚ Π½Π΅ всСгда.

НапримСр, Π΄Ρ€ΠΎΠ±ΠΈΒ  Β ΠΈΒ  ΡΠ»ΠΎΠΆΠΈΡ‚ΡŒ ΠΌΠΎΠΆΠ½ΠΎ, ΠΏΠΎΡΠΊΠΎΠ»ΡŒΠΊΡƒ Ρƒ Π½ΠΈΡ… ΠΎΠ΄ΠΈΠ½Π°ΠΊΠΎΠ²Ρ‹Π΅ Π·Π½Π°ΠΌΠ΅Π½Π°Ρ‚Π΅Π»ΠΈ.

А Π²ΠΎΡ‚ Π΄Ρ€ΠΎΠ±ΠΈΒ  Β  ΠΈΒ  Β  сразу ΡΠ»ΠΎΠΆΠΈΡ‚ΡŒ нСльзя, ΠΏΠΎΡΠΊΠΎΠ»ΡŒΠΊΡƒ Ρƒ этих Π΄Ρ€ΠΎΠ±Π΅ΠΉ Ρ€Π°Π·Π½Ρ‹Π΅ Π·Π½Π°ΠΌΠ΅Π½Π°Ρ‚Π΅Π»ΠΈ. Π’ Ρ‚Π°ΠΊΠΈΡ… случаях Π΄Ρ€ΠΎΠ±ΠΈ Π½ΡƒΠΆΠ½ΠΎ ΠΏΡ€ΠΈΠ²ΠΎΠ΄ΠΈΡ‚ΡŒ ΠΊ ΠΎΠ΄ΠΈΠ½Π°ΠΊΠΎΠ²ΠΎΠΌΡƒ (ΠΎΠ±Ρ‰Π΅ΠΌΡƒ) Π·Π½Π°ΠΌΠ΅Π½Π°Ρ‚Π΅Π»ΡŽ.

БущСствуСт нСсколько способов привСдСния Π΄Ρ€ΠΎΠ±Π΅ΠΉ ΠΊ ΠΎΠ΄ΠΈΠ½Π°ΠΊΠΎΠ²ΠΎΠΌΡƒ Π·Π½Π°ΠΌΠ΅Π½Π°Ρ‚Π΅Π»ΡŽ. БСгодня ΠΌΡ‹ рассмотрим Ρ‚ΠΎΠ»ΡŒΠΊΠΎ ΠΎΠ΄ΠΈΠ½ ΠΈΠ· Π½ΠΈΡ…, ΠΏΠΎΡΠΊΠΎΠ»ΡŒΠΊΡƒ ΠΎΡΡ‚Π°Π»ΡŒΠ½Ρ‹Π΅ способы ΠΌΠΎΠ³ΡƒΡ‚ ΠΏΠΎΠΊΠ°Π·Π°Ρ‚ΡŒΡΡ слоТными для Π½Π°Ρ‡ΠΈΠ½Π°ΡŽΡ‰Π΅Π³ΠΎ.

Π‘ΡƒΡ‚ΡŒ этого способа Π·Π°ΠΊΠ»ΡŽΡ‡Π°Π΅Ρ‚ΡΡ Π² Ρ‚ΠΎΠΌ, Ρ‡Ρ‚ΠΎ сначала ищСтся наимСньшСС ΠΎΠ±Ρ‰Π΅Π΅ ΠΊΡ€Π°Ρ‚Π½ΠΎΠ΅ (НОК) Π·Π½Π°ΠΌΠ΅Π½Π°Ρ‚Π΅Π»Π΅ΠΉ ΠΎΠ±Π΅ΠΈΡ… Π΄Ρ€ΠΎΠ±Π΅ΠΉ. Π—Π°Ρ‚Π΅ΠΌ НОК дСлят Π½Π° Π·Π½Π°ΠΌΠ΅Π½Π°Ρ‚Π΅Π»ΡŒ ΠΏΠ΅Ρ€Π²ΠΎΠΉ Π΄Ρ€ΠΎΠ±ΠΈ ΠΈ ΠΏΠΎΠ»ΡƒΡ‡Π°ΡŽΡ‚ ΠΏΠ΅Ρ€Π²Ρ‹ΠΉ Π΄ΠΎΠΏΠΎΠ»Π½ΠΈΡ‚Π΅Π»ΡŒΠ½Ρ‹ΠΉ ΠΌΠ½ΠΎΠΆΠΈΡ‚Π΅Π»ΡŒ. Аналогично ΠΏΠΎΡΡ‚ΡƒΠΏΠ°ΡŽΡ‚ ΠΈ со Π²Ρ‚ΠΎΡ€ΠΎΠΉ Π΄Ρ€ΠΎΠ±ΡŒΡŽ β€” НОК дСлят Π½Π° Π·Π½Π°ΠΌΠ΅Π½Π°Ρ‚Π΅Π»ΡŒ Π²Ρ‚ΠΎΡ€ΠΎΠΉ Π΄Ρ€ΠΎΠ±ΠΈ ΠΈ ΠΏΠΎΠ»ΡƒΡ‡Π°ΡŽΡ‚ Π²Ρ‚ΠΎΡ€ΠΎΠΉ Π΄ΠΎΠΏΠΎΠ»Π½ΠΈΡ‚Π΅Π»ΡŒΠ½Ρ‹ΠΉ ΠΌΠ½ΠΎΠΆΠΈΡ‚Π΅Π»ΡŒ.

Π—Π°Ρ‚Π΅ΠΌ числитСли ΠΈ Π·Π½Π°ΠΌΠ΅Π½Π°Ρ‚Π΅Π»ΠΈ Π΄Ρ€ΠΎΠ±Π΅ΠΉ ΡƒΠΌΠ½ΠΎΠΆΠ°ΡŽΡ‚ΡΡ Π½Π° свои Π΄ΠΎΠΏΠΎΠ»Π½ΠΈΡ‚Π΅Π»ΡŒΠ½Ρ‹Π΅ ΠΌΠ½ΠΎΠΆΠΈΡ‚Π΅Π»ΠΈ. Π’ Ρ€Π΅Π·ΡƒΠ»ΡŒΡ‚Π°Ρ‚Π΅ этих дСйствий, Π΄Ρ€ΠΎΠ±ΠΈ Ρƒ ΠΊΠΎΡ‚ΠΎΡ€Ρ‹Ρ… Π±Ρ‹Π»ΠΈ Ρ€Π°Π·Π½Ρ‹Π΅ Π·Π½Π°ΠΌΠ΅Π½Π°Ρ‚Π΅Π»ΠΈ, ΠΎΠ±Ρ€Π°Ρ‰Π°ΡŽΡ‚ΡΡ Π² Π΄Ρ€ΠΎΠ±ΠΈ, Ρƒ ΠΊΠΎΡ‚ΠΎΡ€Ρ‹Ρ…Β ΠΎΠ΄ΠΈΠ½Π°ΠΊΠΎΠ²Ρ‹Π΅ Π·Π½Π°ΠΌΠ΅Π½Π°Ρ‚Π΅Π»ΠΈ. А ΠΊΠ°ΠΊ ΡΠΊΠ»Π°Π΄Ρ‹Π²Π°Ρ‚ΡŒ Ρ‚Π°ΠΊΠΈΠ΅ Π΄Ρ€ΠΎΠ±ΠΈ ΠΌΡ‹ ΡƒΠΆΠ΅ Π·Π½Π°Π΅ΠΌ.

ΠŸΡ€ΠΈΠΌΠ΅Ρ€ 1. Π‘Π»ΠΎΠΆΠΈΠΌ Π΄Ρ€ΠΎΠ±ΠΈΒ Β ΠΈΒ 

Π£ этих Π΄Ρ€ΠΎΠ±Π΅ΠΉ Ρ€Π°Π·Π½Ρ‹Π΅ Π·Π½Π°ΠΌΠ΅Π½Π°Ρ‚Π΅Π»ΠΈ, поэтому Π½ΡƒΠΆΠ½ΠΎ привСсти ΠΈΡ… ΠΊ ΠΎΠ΄ΠΈΠ½Π°ΠΊΠΎΠ²ΠΎΠΌΡƒ (ΠΎΠ±Ρ‰Π΅ΠΌΡƒ) Π·Π½Π°ΠΌΠ΅Π½Π°Ρ‚Π΅Π»ΡŽ.

Π’ ΠΏΠ΅Ρ€Π²ΡƒΡŽ ΠΎΡ‡Π΅Ρ€Π΅Π΄ΡŒ Π½Π°Ρ…ΠΎΠ΄ΠΈΠΌ наимСньшСС ΠΎΠ±Ρ‰Π΅Π΅ ΠΊΡ€Π°Ρ‚Π½ΠΎΠ΅ Π·Π½Π°ΠΌΠ΅Π½Π°Ρ‚Π΅Π»Π΅ΠΉ ΠΎΠ±Π΅ΠΈΡ… Π΄Ρ€ΠΎΠ±Π΅ΠΉ. Π—Π½Π°ΠΌΠ΅Π½Π°Ρ‚Π΅Π»ΡŒ ΠΏΠ΅Ρ€Π²ΠΎΠΉ Π΄Ρ€ΠΎΠ±ΠΈ это число 3, Π° Π·Π½Π°ΠΌΠ΅Π½Π°Ρ‚Π΅Π»ΡŒ Π²Ρ‚ΠΎΡ€ΠΎΠΉ Π΄Ρ€ΠΎΠ±ΠΈ β€” число 2. НаимСньшСС ΠΎΠ±Ρ‰Π΅Π΅ ΠΊΡ€Π°Ρ‚Π½ΠΎΠ΅ этих чисСл Ρ€Π°Π²Π½ΠΎ 6

НОК (2 и 3) = 6

Π’Π΅ΠΏΠ΅Ρ€ΡŒ возвращаСмся ΠΊ дробям  и . Π‘Π½Π°Ρ‡Π°Π»Π° Ρ€Π°Π·Π΄Π΅Π»ΠΈΠΌ НОК Π½Π° Π·Π½Π°ΠΌΠ΅Π½Π°Ρ‚Π΅Π»ΡŒ ΠΏΠ΅Ρ€Π²ΠΎΠΉ Π΄Ρ€ΠΎΠ±ΠΈ ΠΈ ΠΏΠΎΠ»ΡƒΡ‡ΠΈΠΌ ΠΏΠ΅Ρ€Π²Ρ‹ΠΉ Π΄ΠΎΠΏΠΎΠ»Π½ΠΈΡ‚Π΅Π»ΡŒΠ½Ρ‹ΠΉ ΠΌΠ½ΠΎΠΆΠΈΡ‚Π΅Π»ΡŒ. НОК это число 6, Π° Π·Π½Π°ΠΌΠ΅Π½Π°Ρ‚Π΅Π»ΡŒ ΠΏΠ΅Ρ€Π²ΠΎΠΉ Π΄Ρ€ΠΎΠ±ΠΈ это число 3. Π”Π΅Π»ΠΈΠΌ 6 Π½Π° 3, ΠΏΠΎΠ»ΡƒΡ‡Π°Π΅ΠΌ 2.

ΠŸΠΎΠ»ΡƒΡ‡Π΅Π½Π½ΠΎΠ΅ число 2 это ΠΏΠ΅Ρ€Π²Ρ‹ΠΉ Π΄ΠΎΠΏΠΎΠ»Π½ΠΈΡ‚Π΅Π»ΡŒΠ½Ρ‹ΠΉ ΠΌΠ½ΠΎΠΆΠΈΡ‚Π΅Π»ΡŒ. ЗаписываСм Π΅Π³ΠΎ ΠΊ ΠΏΠ΅Ρ€Π²ΠΎΠΉ Π΄Ρ€ΠΎΠ±ΠΈ. Для этого Π΄Π΅Π»Π°Π΅ΠΌ Π½Π΅Π±ΠΎΠ»ΡŒΡˆΡƒΡŽ ΠΊΠΎΡΡƒΡŽ линию Π½Π°Π΄ Π΄Ρ€ΠΎΠ±ΡŒΡŽ ΠΈ записываСм над Π½Π΅ΠΉ Π½Π°ΠΉΠ΄Π΅Π½Π½Ρ‹ΠΉ Π΄ΠΎΠΏΠΎΠ»Π½ΠΈΡ‚Π΅Π»ΡŒΠ½Ρ‹ΠΉ ΠΌΠ½ΠΎΠΆΠΈΡ‚Π΅Π»ΡŒ:

Аналогично поступаСм ΠΈ со Π²Ρ‚ΠΎΡ€ΠΎΠΉ Π΄Ρ€ΠΎΠ±ΡŒΡŽ. Π”Π΅Π»ΠΈΠΌ НОК Π½Π° Π·Π½Π°ΠΌΠ΅Π½Π°Ρ‚Π΅Π»ΡŒ Π²Ρ‚ΠΎΡ€ΠΎΠΉ Π΄Ρ€ΠΎΠ±ΠΈ ΠΈ ΠΏΠΎΠ»ΡƒΡ‡Π°Π΅ΠΌ Π²Ρ‚ΠΎΡ€ΠΎΠΉ Π΄ΠΎΠΏΠΎΠ»Π½ΠΈΡ‚Π΅Π»ΡŒΠ½Ρ‹ΠΉ ΠΌΠ½ΠΎΠΆΠΈΡ‚Π΅Π»ΡŒ. НОК это число 6, Π° Π·Π½Π°ΠΌΠ΅Π½Π°Ρ‚Π΅Π»ΡŒ Π²Ρ‚ΠΎΡ€ΠΎΠΉ Π΄Ρ€ΠΎΠ±ΠΈ β€” число 2. Π”Π΅Π»ΠΈΠΌ 6 Π½Π° 2, ΠΏΠΎΠ»ΡƒΡ‡Π°Π΅ΠΌ 3.

ΠŸΠΎΠ»ΡƒΡ‡Π΅Π½Π½ΠΎΠ΅ число 3 это Π²Ρ‚ΠΎΡ€ΠΎΠΉ Π΄ΠΎΠΏΠΎΠ»Π½ΠΈΡ‚Π΅Π»ΡŒΠ½Ρ‹ΠΉ ΠΌΠ½ΠΎΠΆΠΈΡ‚Π΅Π»ΡŒ. ЗаписываСм Π΅Π³ΠΎ ΠΊΠΎ Π²Ρ‚ΠΎΡ€ΠΎΠΉ Π΄Ρ€ΠΎΠ±ΠΈ. ΠžΠΏΡΡ‚ΡŒ ΠΆΠ΅ Π΄Π΅Π»Π°Π΅ΠΌ Π½Π΅Π±ΠΎΠ»ΡŒΡˆΡƒΡŽ ΠΊΠΎΡΡƒΡŽ линию Π½Π°Π΄ Π²Ρ‚ΠΎΡ€ΠΎΠΉ Π΄Ρ€ΠΎΠ±ΡŒΡŽ ΠΈ записываСм Π½Π°Π΄ Π½Π΅ΠΉ Π½Π°ΠΉΠ΄Π΅Π½Π½Ρ‹ΠΉ Π΄ΠΎΠΏΠΎΠ»Π½ΠΈΡ‚Π΅Π»ΡŒΠ½Ρ‹ΠΉ ΠΌΠ½ΠΎΠΆΠΈΡ‚Π΅Π»ΡŒ:

Π’Π΅ΠΏΠ΅Ρ€ΡŒ Ρƒ нас всё Π³ΠΎΡ‚ΠΎΠ²ΠΎ для слоТСния. ΠžΡΡ‚Π°Π»ΠΎΡΡŒ ΡƒΠΌΠ½ΠΎΠΆΠΈΡ‚ΡŒ числитСли ΠΈ Π·Π½Π°ΠΌΠ΅Π½Π°Ρ‚Π΅Π»ΠΈ Π΄Ρ€ΠΎΠ±Π΅ΠΉ Π½Π° свои Π΄ΠΎΠΏΠΎΠ»Π½ΠΈΡ‚Π΅Π»ΡŒΠ½Ρ‹Π΅ ΠΌΠ½ΠΎΠΆΠΈΡ‚Π΅Π»ΠΈ:

ΠŸΠΎΡΠΌΠΎΡ‚Ρ€ΠΈΡ‚Π΅ Π²Π½ΠΈΠΌΠ°Ρ‚Π΅Π»ΡŒΠ½ΠΎ ΠΊ Ρ‡Π΅ΠΌΡƒ ΠΌΡ‹ ΠΏΡ€ΠΈΡˆΠ»ΠΈ. ΠœΡ‹ ΠΏΡ€ΠΈΡˆΠ»ΠΈ ΠΊ Ρ‚ΠΎΠΌΡƒ, Ρ‡Ρ‚ΠΎ Π΄Ρ€ΠΎΠ±ΠΈ Ρƒ ΠΊΠΎΡ‚ΠΎΡ€Ρ‹Ρ… Π±Ρ‹Π»ΠΈ Ρ€Π°Π·Π½Ρ‹Π΅ Π·Π½Π°ΠΌΠ΅Π½Π°Ρ‚Π΅Π»ΠΈ, ΠΏΡ€Π΅Π²Ρ€Π°Ρ‚ΠΈΠ»ΠΈΡΡŒ Π² Π΄Ρ€ΠΎΠ±ΠΈ Ρƒ ΠΊΠΎΡ‚ΠΎΡ€Ρ‹Ρ… ΠΎΠ΄ΠΈΠ½Π°ΠΊΠΎΠ²Ρ‹Π΅ Π·Π½Π°ΠΌΠ΅Π½Π°Ρ‚Π΅Π»ΠΈ. А ΠΊΠ°ΠΊ ΡΠΊΠ»Π°Π΄Ρ‹Π²Π°Ρ‚ΡŒ Ρ‚Π°ΠΊΠΈΠ΅ Π΄Ρ€ΠΎΠ±ΠΈ ΠΌΡ‹ ΡƒΠΆΠ΅ Π·Π½Π°Π΅ΠΌ. Π”Π°Π²Π°ΠΉΡ‚Π΅ Π΄ΠΎΡ€Π΅ΡˆΠ°Π΅ΠΌ этот ΠΏΡ€ΠΈΠΌΠ΅Ρ€ Π΄ΠΎ ΠΊΠΎΠ½Ρ†Π°:

Π’Π°ΠΊΠΈΠΌ ΠΎΠ±Ρ€Π°Π·ΠΎΠΌ, ΠΏΡ€ΠΈΠΌΠ΅Ρ€ Π·Π°Π²Π΅Ρ€ΡˆΠ°Π΅Ρ‚ΡΡ.Β  ΠšΒ Β ΠΏΡ€ΠΈΠ±Π°Π²ΠΈΡ‚ΡŒΒ Β ΠΏΠΎΠ»ΡƒΡ‡Π°Π΅Ρ‚ΡΡΒ .

ΠŸΠΎΠΏΡ€ΠΎΠ±ΡƒΠ΅ΠΌ ΠΈΠ·ΠΎΠ±Ρ€Π°Π·ΠΈΡ‚ΡŒ нашС Ρ€Π΅ΡˆΠ΅Π½ΠΈΠ΅ с ΠΏΠΎΠΌΠΎΡ‰ΡŒΡŽ рисунка. Если ΠΊΒ Β ΠΏΠΈΡ†Ρ†Ρ‹ ΠΏΡ€ΠΈΠ±Π°Π²ΠΈΡ‚ΡŒΒ Β ΠΏΠΈΡ†Ρ†Ρ‹, Ρ‚ΠΎ получится ΠΎΠ΄Π½Π° цСлая ΠΏΠΈΡ†Ρ†Π° ΠΈ Π΅Ρ‰Π΅ ΠΎΠ΄Π½Π° ΡˆΠ΅ΡΡ‚Π°Ρ ΠΏΠΈΡ†Ρ†Ρ‹:

ΠŸΡ€ΠΈΠ²Π΅Π΄Π΅Π½ΠΈΠ΅ Π΄Ρ€ΠΎΠ±Π΅ΠΉ ΠΊ ΠΎΠ΄ΠΈΠ½Π°ΠΊΠΎΠ²ΠΎΠΌΡƒ (ΠΎΠ±Ρ‰Π΅ΠΌΡƒ) Π·Π½Π°ΠΌΠ΅Π½Π°Ρ‚Π΅Π»ΡŽ Ρ‚Π°ΠΊΠΆΠ΅ ΠΌΠΎΠΆΠ½ΠΎ ΠΈΠ·ΠΎΠ±Ρ€Π°Π·ΠΈΡ‚ΡŒ с ΠΏΠΎΠΌΠΎΡ‰ΡŒΡŽ рисунка. ΠŸΡ€ΠΈΠ²Π΅Π΄Ρ Π΄Ρ€ΠΎΠ±ΠΈΒ Β ΠΈΒ Β ΠΊ ΠΎΠ±Ρ‰Π΅ΠΌΡƒ Π·Π½Π°ΠΌΠ΅Π½Π°Ρ‚Π΅Π»ΡŽ, ΠΌΡ‹ ΠΏΠΎΠ»ΡƒΡ‡ΠΈΠ»ΠΈ Π΄Ρ€ΠΎΠ±ΠΈΒ Β ΠΈΒ . Π­Ρ‚ΠΈ Π΄Π²Π΅ Π΄Ρ€ΠΎΠ±ΠΈ Π±ΡƒΠ΄ΡƒΡ‚ ΠΈΠ·ΠΎΠ±Ρ€Π°ΠΆΠ°Ρ‚ΡŒΡΡ Ρ‚Π΅ΠΌΠΈ ΠΆΠ΅ кусками ΠΏΠΈΡ†Ρ†. Π Π°Π·Π»ΠΈΡ‡ΠΈΠ΅ Π±ΡƒΠ΄Π΅Ρ‚ лишь Π² Ρ‚ΠΎΠΌ, Ρ‡Ρ‚ΠΎ Π² этот Ρ€Π°Π· ΠΎΠ½ΠΈ Π±ΡƒΠ΄ΡƒΡ‚ Ρ€Π°Π·Π΄Π΅Π»Π΅Π½Ρ‹ Π½Π° ΠΎΠ΄ΠΈΠ½Π°ΠΊΠΎΠ²Ρ‹Π΅ Π΄ΠΎΠ»ΠΈ (ΠΏΡ€ΠΈΠ²Π΅Π΄Π΅Π½Ρ‹ ΠΊ ΠΎΠ΄ΠΈΠ½Π°ΠΊΠΎΠ²ΠΎΠΌΡƒ Π·Π½Π°ΠΌΠ΅Π½Π°Ρ‚Π΅Π»ΡŽ).

ΠŸΠ΅Ρ€Π²Ρ‹ΠΉ рисунок ΠΈΠ·ΠΎΠ±Ρ€Π°ΠΆΠ°Π΅Ρ‚ Π΄Ρ€ΠΎΠ±ΡŒΒ Β (Ρ‡Π΅Ρ‚Ρ‹Ρ€Π΅ кусочка ΠΈΠ· ΡˆΠ΅ΡΡ‚ΠΈ), Π° Π²Ρ‚ΠΎΡ€ΠΎΠΉ рисунок ΠΈΠ·ΠΎΠ±Ρ€Π°ΠΆΠ°Π΅Ρ‚ Π΄Ρ€ΠΎΠ±ΡŒΒ Β (Ρ‚Ρ€ΠΈ кусочка ΠΈΠ· ΡˆΠ΅ΡΡ‚ΠΈ). Π‘Π»ΠΎΠΆΠΈΠ² эти кусочки ΠΌΡ‹ ΠΏΠΎΠ»ΡƒΡ‡Π°Π΅ΠΌΒ Β (сСмь кусочков ΠΈΠ· ΡˆΠ΅ΡΡ‚ΠΈ). Π­Ρ‚Π° Π΄Ρ€ΠΎΠ±ΡŒ Π½Π΅ΠΏΡ€Π°Π²ΠΈΠ»ΡŒΠ½Π°Ρ, поэтому ΠΌΡ‹ Π²Ρ‹Π΄Π΅Π»ΠΈΠ»ΠΈ Π² Π½Π΅ΠΉ Ρ†Π΅Π»ΡƒΡŽ Ρ‡Π°ΡΡ‚ΡŒ. Π’ Ρ€Π΅Π·ΡƒΠ»ΡŒΡ‚Π°Ρ‚Π΅ ΠΏΠΎΠ»ΡƒΡ‡ΠΈΠ»ΠΈΒ Β (ΠΎΠ΄Π½Ρƒ Ρ†Π΅Π»ΡƒΡŽ ΠΏΠΈΡ†Ρ†Ρƒ ΠΈ Π΅Ρ‰Π΅ ΠΎΠ΄Π½Ρƒ ΡˆΠ΅ΡΡ‚ΡƒΡŽ ΠΏΠΈΡ†Ρ†Ρ‹).

ΠžΡ‚ΠΌΠ΅Ρ‚ΠΈΠΌ, Ρ‡Ρ‚ΠΎ ΠΌΡ‹ с Π²Π°ΠΌΠΈ расписали Π΄Π°Π½Π½Ρ‹ΠΉ ΠΏΡ€ΠΈΠΌΠ΅Ρ€ слишком ΠΏΠΎΠ΄Ρ€ΠΎΠ±Π½ΠΎ. Π’ ΡƒΡ‡Π΅Π±Π½Ρ‹Ρ… завСдСниях Π½Π΅ принято ΠΏΠΈΡΠ°Ρ‚ΡŒ Ρ‚Π°ΠΊ Ρ€Π°Π·Π²Ρ‘Ρ€Π½ΡƒΡ‚ΠΎ. НуТно ΡƒΠΌΠ΅Ρ‚ΡŒ быстро Π½Π°Ρ…ΠΎΠ΄ΠΈΡ‚ΡŒ НОК ΠΎΠ±ΠΎΠΈΡ… Π·Π½Π°ΠΌΠ΅Π½Π°Ρ‚Π΅Π»Π΅ΠΉ ΠΈ Π΄ΠΎΠΏΠΎΠ»Π½ΠΈΡ‚Π΅Π»ΡŒΠ½Ρ‹Π΅ ΠΌΠ½ΠΎΠΆΠΈΡ‚Π΅Π»ΠΈ ΠΊ Π½ΠΈΠΌ, Π° Ρ‚Π°ΠΊΠΆΠ΅ быстро ΡƒΠΌΠ½ΠΎΠΆΠ°Ρ‚ΡŒ Π½Π°ΠΉΠ΄Π΅Π½Π½Ρ‹Π΅ Π΄ΠΎΠΏΠΎΠ»Π½ΠΈΡ‚Π΅Π»ΡŒΠ½Ρ‹Π΅ ΠΌΠ½ΠΎΠΆΠΈΡ‚Π΅Π»ΠΈ Π½Π° свои числитСли ΠΈ Π·Π½Π°ΠΌΠ΅Π½Π°Ρ‚Π΅Π»ΠΈ. ΠΠ°Ρ…ΠΎΠ΄ΡΡΡŒ Π² школС, Π΄Π°Π½Π½Ρ‹ΠΉ ΠΏΡ€ΠΈΠΌΠ΅Ρ€ Π½Π°ΠΌ ΠΏΡ€ΠΈΡˆΠ»ΠΎΡΡŒ Π±Ρ‹ Π·Π°ΠΏΠΈΡΠ°Ρ‚ΡŒ ΡΠ»Π΅Π΄ΡƒΡŽΡ‰ΠΈΠΌ ΠΎΠ±Ρ€Π°Π·ΠΎΠΌ:

Но Π΅ΡΡ‚ΡŒ ΠΈ обратная сторона ΠΌΠ΅Π΄Π°Π»ΠΈ. Если Π½Π° ΠΏΠ΅Ρ€Π²Ρ‹Ρ… этапах изучСния ΠΌΠ°Ρ‚Π΅ΠΌΠ°Ρ‚ΠΈΠΊΠΈ Π½Π΅ Π΄Π΅Π»Π°Ρ‚ΡŒ ΠΏΠΎΠ΄Ρ€ΠΎΠ±Π½Ρ‹Ρ… записСй, Ρ‚ΠΎ Π½Π°Ρ‡ΠΈΠ½Π°ΡŽΡ‚ ΠΏΠΎΡΠ²Π»ΡΡ‚ΡŒΡΡ вопросы Ρ€ΠΎΠ΄Π° Β«Π° ΠΎΡ‚ΠΊΡƒΠ΄Π° Π²ΠΎΠ½ Ρ‚Π° Ρ†ΠΈΡ„Ρ€Π°?Β», Β«ΠΏΠΎΡ‡Π΅ΠΌΡƒ Π΄Ρ€ΠΎΠ±ΠΈ Π²Π΄Ρ€ΡƒΠ³ ΠΏΡ€Π΅Π²Ρ€Π°Ρ‰Π°ΡŽΡ‚ΡΡ совсСм Π² Π΄Ρ€ΡƒΠ³ΠΈΠ΅ Π΄Ρ€ΠΎΠ±ΠΈ?Β«.

ΠŸΠΎΡΡ‚ΠΎΠΌΡƒ Π½Π° ΠΏΠ΅Ρ€Π²Ρ‹Ρ… этапах совСтуСм Π·Π°ΠΏΠΈΡΡ‹Π²Π°Ρ‚ΡŒ ΠΊΠ°ΠΆΠ΄ΡƒΡŽ ΠΌΠ΅Π»ΠΎΡ‡ΡŒ. Π₯Π²Π°ΡΡ‚Π°Ρ‚ΡŒΡΡ ΠΌΠΎΠΆΠ½ΠΎ лишь Π² Π±ΡƒΠ΄ΡƒΡ‰Π΅ΠΌ, ΠΊΠΎΠ³Π΄Π° Π±ΡƒΠ΄ΡƒΡ‚ усвоСны Π°Π·Ρ‹.

Π§Ρ‚ΠΎΠ±Ρ‹ Π»Π΅Π³Ρ‡Π΅ Π±Ρ‹Π»ΠΎ ΡΠΊΠ»Π°Π΄Ρ‹Π²Π°Ρ‚ΡŒ Π΄Ρ€ΠΎΠ±ΠΈ с Ρ€Π°Π·Π½Ρ‹ΠΌΠΈ знамСнатСлями, ΠΌΠΎΠΆΠ½ΠΎ Π²ΠΎΡΠΏΠΎΠ»ΡŒΠ·ΠΎΠ²Π°Ρ‚ΡŒΡΡ ΡΠ»Π΅Π΄ΡƒΡŽΡ‰Π΅ΠΉ пошаговой инструкциСй:

  1. Найти НОК Π·Π½Π°ΠΌΠ΅Π½Π°Ρ‚Π΅Π»Π΅ΠΉ Π΄Ρ€ΠΎΠ±Π΅ΠΉ;
  2. Π Π°Π·Π΄Π΅Π»ΠΈΡ‚ΡŒ НОК Π½Π° Π·Π½Π°ΠΌΠ΅Π½Π°Ρ‚Π΅Π»ΡŒ ΠΊΠ°ΠΆΠ΄ΠΎΠΉ Π΄Ρ€ΠΎΠ±ΠΈ ΠΈ ΠΏΠΎΠ»ΡƒΡ‡ΠΈΡ‚ΡŒ Π΄ΠΎΠΏΠΎΠ»Π½ΠΈΡ‚Π΅Π»ΡŒΠ½Ρ‹ΠΉ ΠΌΠ½ΠΎΠΆΠΈΡ‚Π΅Π»ΡŒ для ΠΊΠ°ΠΆΠ΄ΠΎΠΉ Π΄Ρ€ΠΎΠ±ΠΈ;
  3. Π£ΠΌΠ½ΠΎΠΆΠΈΡ‚ΡŒ числитСли ΠΈ Π·Π½Π°ΠΌΠ΅Π½Π°Ρ‚Π΅Π»ΠΈ Π΄Ρ€ΠΎΠ±Π΅ΠΉ Π½Π° свои Π΄ΠΎΠΏΠΎΠ»Π½ΠΈΡ‚Π΅Π»ΡŒΠ½Ρ‹Π΅ ΠΌΠ½ΠΎΠΆΠΈΡ‚Π΅Π»ΠΈ;
  4. Π‘Π»ΠΎΠΆΠΈΡ‚ΡŒ Π΄Ρ€ΠΎΠ±ΠΈ, Ρƒ ΠΊΠΎΡ‚ΠΎΡ€Ρ‹Ρ… ΠΎΠ΄ΠΈΠ½Π°ΠΊΠΎΠ²Ρ‹Π΅ Π·Π½Π°ΠΌΠ΅Π½Π°Ρ‚Π΅Π»ΠΈ;
  5. Если Π² ΠΎΡ‚Π²Π΅Ρ‚Π΅ ΠΏΠΎΠ»ΡƒΡ‡ΠΈΠ»Π°ΡΡŒ Π½Π΅ΠΏΡ€Π°Π²ΠΈΠ»ΡŒΠ½Π°Ρ Π΄Ρ€ΠΎΠ±ΡŒ, Ρ‚ΠΎ Π²Ρ‹Π΄Π΅Π»ΠΈΡ‚ΡŒ Π΅Ρ‘ Ρ†Π΅Π»ΡƒΡŽ Ρ‡Π°ΡΡ‚ΡŒ;

ΠŸΡ€ΠΈΠΌΠ΅Ρ€ 2. Найти Π·Π½Π°Ρ‡Π΅Π½ΠΈΠ΅ выраТСния .

Π’ΠΎΡΠΏΠΎΠ»ΡŒΠ·ΡƒΠ΅ΠΌΡΡ инструкциСй, которая ΠΏΡ€ΠΈΠ²Π΅Π΄Π΅Π½Π° Π²Ρ‹ΡˆΠ΅.

Π¨Π°Π³ 1. Найти НОК Π·Π½Π°ΠΌΠ΅Π½Π°Ρ‚Π΅Π»Π΅ΠΉ Π΄Ρ€ΠΎΠ±Π΅ΠΉ

Находим НОК Π·Π½Π°ΠΌΠ΅Π½Π°Ρ‚Π΅Π»Π΅ΠΉ ΠΎΠ±Π΅ΠΈΡ… Π΄Ρ€ΠΎΠ±Π΅ΠΉ. Π—Π½Π°ΠΌΠ΅Π½Π°Ρ‚Π΅Π»ΠΈ Π΄Ρ€ΠΎΠ±Π΅ΠΉ это числа 2, 3 ΠΈ 4

Π¨Π°Π³ 2. Π Π°Π·Π΄Π΅Π»ΠΈΡ‚ΡŒ НОК Π½Π° Π·Π½Π°ΠΌΠ΅Π½Π°Ρ‚Π΅Π»ΡŒ ΠΊΠ°ΠΆΠ΄ΠΎΠΉ Π΄Ρ€ΠΎΠ±ΠΈ ΠΈ ΠΏΠΎΠ»ΡƒΡ‡ΠΈΡ‚ΡŒ Π΄ΠΎΠΏΠΎΠ»Π½ΠΈΡ‚Π΅Π»ΡŒΠ½Ρ‹ΠΉ ΠΌΠ½ΠΎΠΆΠΈΡ‚Π΅Π»ΡŒ для ΠΊΠ°ΠΆΠ΄ΠΎΠΉ Π΄Ρ€ΠΎΠ±ΠΈ

Π”Π΅Π»ΠΈΠΌ НОК Π½Π° Π·Π½Π°ΠΌΠ΅Π½Π°Ρ‚Π΅Π»ΡŒ ΠΏΠ΅Ρ€Π²ΠΎΠΉ Π΄Ρ€ΠΎΠ±ΠΈ. НОК это число 12, Π° Π·Π½Π°ΠΌΠ΅Π½Π°Ρ‚Π΅Π»ΡŒ ΠΏΠ΅Ρ€Π²ΠΎΠΉ Π΄Ρ€ΠΎΠ±ΠΈ это число 2. Π”Π΅Π»ΠΈΠΌ 12 Π½Π° 2, ΠΏΠΎΠ»ΡƒΡ‡Π°Π΅ΠΌ 6. ΠŸΠΎΠ»ΡƒΡ‡ΠΈΠ»ΠΈ ΠΏΠ΅Ρ€Π²Ρ‹ΠΉ Π΄ΠΎΠΏΠΎΠ»Π½ΠΈΡ‚Π΅Π»ΡŒΠ½Ρ‹ΠΉ ΠΌΠ½ΠΎΠΆΠΈΡ‚Π΅Π»ΡŒ 6. ЗаписываСм Π΅Π³ΠΎ Π½Π°Π΄ ΠΏΠ΅Ρ€Π²ΠΎΠΉ Π΄Ρ€ΠΎΠ±ΡŒΡŽ:

Π’Π΅ΠΏΠ΅Ρ€ΡŒ Π΄Π΅Π»ΠΈΠΌ НОК Π½Π° Π·Π½Π°ΠΌΠ΅Π½Π°Ρ‚Π΅Π»ΡŒ Π²Ρ‚ΠΎΡ€ΠΎΠΉ Π΄Ρ€ΠΎΠ±ΠΈ. НОК это число 12, Π° Π·Π½Π°ΠΌΠ΅Π½Π°Ρ‚Π΅Π»ΡŒ Π²Ρ‚ΠΎΡ€ΠΎΠΉ Π΄Ρ€ΠΎΠ±ΠΈ это число 3. Π”Π΅Π»ΠΈΠΌ 12 Π½Π° 3, ΠΏΠΎΠ»ΡƒΡ‡Π°Π΅ΠΌ 4. ΠŸΠΎΠ»ΡƒΡ‡ΠΈΠ»ΠΈ Π²Ρ‚ΠΎΡ€ΠΎΠΉ Π΄ΠΎΠΏΠΎΠ»Π½ΠΈΡ‚Π΅Π»ΡŒΠ½Ρ‹ΠΉ ΠΌΠ½ΠΎΠΆΠΈΡ‚Π΅Π»ΡŒ 4. ЗаписываСм Π΅Π³ΠΎ Π½Π°Π΄ Π²Ρ‚ΠΎΡ€ΠΎΠΉ Π΄Ρ€ΠΎΠ±ΡŒΡŽ:

Π’Π΅ΠΏΠ΅Ρ€ΡŒ Π΄Π΅Π»ΠΈΠΌ НОК Π½Π° Π·Π½Π°ΠΌΠ΅Π½Π°Ρ‚Π΅Π»ΡŒ Ρ‚Ρ€Π΅Ρ‚ΡŒΠ΅ΠΉ Π΄Ρ€ΠΎΠ±ΠΈ. НОК это число 12, Π° Π·Π½Π°ΠΌΠ΅Π½Π°Ρ‚Π΅Π»ΡŒ Ρ‚Ρ€Π΅Ρ‚ΡŒΠ΅ΠΉ Π΄Ρ€ΠΎΠ±ΠΈ это число 4. Π”Π΅Π»ΠΈΠΌ 12 Π½Π° 4, ΠΏΠΎΠ»ΡƒΡ‡Π°Π΅ΠΌ 3. ΠŸΠΎΠ»ΡƒΡ‡ΠΈΠ»ΠΈ Ρ‚Ρ€Π΅Ρ‚ΠΈΠΉ Π΄ΠΎΠΏΠΎΠ»Π½ΠΈΡ‚Π΅Π»ΡŒΠ½Ρ‹ΠΉ ΠΌΠ½ΠΎΠΆΠΈΡ‚Π΅Π»ΡŒ 3.

ЗаписываСм Π΅Π³ΠΎ Π½Π°Π΄ Ρ‚Ρ€Π΅Ρ‚ΡŒΠ΅ΠΉ Π΄Ρ€ΠΎΠ±ΡŒΡŽ:

Π¨Π°Π³ 3. Π£ΠΌΠ½ΠΎΠΆΠΈΡ‚ΡŒ числитСли ΠΈ Π·Π½Π°ΠΌΠ΅Π½Π°Ρ‚Π΅Π»ΠΈ Π΄Ρ€ΠΎΠ±Π΅ΠΉ Π½Π° свои Π΄ΠΎΠΏΠΎΠ»Π½ΠΈΡ‚Π΅Π»ΡŒΠ½Ρ‹Π΅ ΠΌΠ½ΠΎΠΆΠΈΡ‚Π΅Π»ΠΈ

Π£ΠΌΠ½ΠΎΠΆΠ°Π΅ΠΌ числитСли ΠΈ Π·Π½Π°ΠΌΠ΅Π½Π°Ρ‚Π΅Π»ΠΈ Π½Π° свои Π΄ΠΎΠΏΠΎΠ»Π½ΠΈΡ‚Π΅Π»ΡŒΠ½Ρ‹Π΅ ΠΌΠ½ΠΎΠΆΠΈΡ‚Π΅Π»ΠΈ:

Π¨Π°Π³ 4. Π‘Π»ΠΎΠΆΠΈΡ‚ΡŒ Π΄Ρ€ΠΎΠ±ΠΈ Ρƒ ΠΊΠΎΡ‚ΠΎΡ€Ρ‹Ρ… ΠΎΠ΄ΠΈΠ½Π°ΠΊΠΎΠ²Ρ‹Π΅ Π·Π½Π°ΠΌΠ΅Π½Π°Ρ‚Π΅Π»ΠΈ

ΠœΡ‹ ΠΏΡ€ΠΈΡˆΠ»ΠΈ ΠΊ Ρ‚ΠΎΠΌΡƒ, Ρ‡Ρ‚ΠΎ Π΄Ρ€ΠΎΠ±ΠΈ Ρƒ ΠΊΠΎΡ‚ΠΎΡ€Ρ‹Ρ… Π±Ρ‹Π»ΠΈ Ρ€Π°Π·Π½Ρ‹Π΅ Π·Π½Π°ΠΌΠ΅Π½Π°Ρ‚Π΅Π»ΠΈ, ΠΏΡ€Π΅Π²Ρ€Π°Ρ‚ΠΈΠ»ΠΈΡΡŒ Π² Π΄Ρ€ΠΎΠ±ΠΈ, Ρƒ ΠΊΠΎΡ‚ΠΎΡ€Ρ‹Ρ… ΠΎΠ΄ΠΈΠ½Π°ΠΊΠΎΠ²Ρ‹Π΅ (ΠΎΠ±Ρ‰ΠΈΠ΅) Π·Π½Π°ΠΌΠ΅Π½Π°Ρ‚Π΅Π»ΠΈ. ΠžΡΡ‚Π°Π»ΠΎΡΡŒ ΡΠ»ΠΎΠΆΠΈΡ‚ΡŒ эти Π΄Ρ€ΠΎΠ±ΠΈ. Π‘ΠΊΠ»Π°Π΄Ρ‹Π²Π°Π΅ΠΌ:

Π‘Π»ΠΎΠΆΠ΅Π½ΠΈΠ΅ Π½Π΅ ΠΏΠΎΠΌΠ΅ΡΡ‚ΠΈΠ»ΠΎΡΡŒ Π½Π° ΠΎΠ΄Π½ΠΎΠΉ строкС, поэтому ΠΌΡ‹ пСрСнСсли ΠΎΡΡ‚Π°Π²ΡˆΠ΅Π΅ΡΡ Π²Ρ‹Ρ€Π°ΠΆΠ΅Π½ΠΈΠ΅ Π½Π° ΡΠ»Π΅Π΄ΡƒΡŽΡ‰ΡƒΡŽ строку. Π­Ρ‚ΠΎ допускаСтся Π² ΠΌΠ°Ρ‚Π΅ΠΌΠ°Ρ‚ΠΈΠΊΠ΅. Когда Π²Ρ‹Ρ€Π°ΠΆΠ΅Π½ΠΈΠ΅ Π½Π΅ помСщаСтся Π½Π° ΠΎΠ΄Π½Ρƒ строку, Π΅Π³ΠΎ пСрСносят Π½Π° ΡΠ»Π΅Π΄ΡƒΡŽΡ‰ΡƒΡŽ строку, ΠΏΡ€ΠΈ этом Π½Π°Π΄ΠΎ ΠΎΠ±ΡΠ·Π°Ρ‚Π΅Π»ΡŒΠ½ΠΎ ΠΏΠΎΡΡ‚Π°Π²ΠΈΡ‚ΡŒ Π·Π½Π°ΠΊ равСнства (=) Π½Π° ΠΊΠΎΠ½Ρ†Π΅ ΠΏΠ΅Ρ€Π²ΠΎΠΉ строки ΠΈ Π² Π½Π°Ρ‡Π°Π»Π΅ Π½ΠΎΠ²ΠΎΠΉ строки. Π—Π½Π°ΠΊ равСнства Π½Π° Π²Ρ‚ΠΎΡ€ΠΎΠΉ строкС Π³ΠΎΠ²ΠΎΡ€ΠΈΡ‚ ΠΎ Ρ‚ΠΎΠΌ, Ρ‡Ρ‚ΠΎ это ΠΏΡ€ΠΎΠ΄ΠΎΠ»ΠΆΠ΅Π½ΠΈΠ΅ выраТСния, ΠΊΠΎΡ‚ΠΎΡ€ΠΎΠ΅ Π±Ρ‹Π»ΠΎ Π½Π° ΠΏΠ΅Ρ€Π²ΠΎΠΉ строкС.

Π¨Π°Π³ 5. Если Π² ΠΎΡ‚Π²Π΅Ρ‚Π΅ ΠΏΠΎΠ»ΡƒΡ‡ΠΈΠ»Π°ΡΡŒ Π½Π΅ΠΏΡ€Π°Π²ΠΈΠ»ΡŒΠ½Π°Ρ Π΄Ρ€ΠΎΠ±ΡŒ, Ρ‚ΠΎ Π²Ρ‹Π΄Π΅Π»ΠΈΡ‚ΡŒ Π² Π½Π΅ΠΉ Ρ†Π΅Π»ΡƒΡŽ Ρ‡Π°ΡΡ‚ΡŒ

Π£ нас Π² ΠΎΡ‚Π²Π΅Ρ‚Π΅ ΠΏΠΎΠ»ΡƒΡ‡ΠΈΠ»Π°ΡΡŒ Π½Π΅ΠΏΡ€Π°Π²ΠΈΠ»ΡŒΠ½Π°Ρ Π΄Ρ€ΠΎΠ±ΡŒ. ΠœΡ‹ Π΄ΠΎΠ»ΠΆΠ½Ρ‹ Π²Ρ‹Π΄Π΅Π»ΠΈΡ‚ΡŒ Ρƒ Π½Π΅Ρ‘ Ρ†Π΅Π»ΡƒΡŽ Ρ‡Π°ΡΡ‚ΡŒ. ВыдСляСм:

ΠŸΠΎΠ»ΡƒΡ‡ΠΈΠ»ΠΈ ΠΎΡ‚Π²Π΅Ρ‚


Π’Ρ‹Ρ‡ΠΈΡ‚Π°Π½ΠΈΠ΅ Π΄Ρ€ΠΎΠ±Π΅ΠΉ с ΠΎΠ΄ΠΈΠ½Π°ΠΊΠΎΠ²Ρ‹ΠΌΠΈ знамСнатСлями

Π’Ρ‹Ρ‡ΠΈΡ‚Π°Π½ΠΈΠ΅ Π΄Ρ€ΠΎΠ±Π΅ΠΉ Π±Ρ‹Π²Π°Π΅Ρ‚ Π΄Π²ΡƒΡ… Π²ΠΈΠ΄ΠΎΠ²:

  1. Π’Ρ‹Ρ‡ΠΈΡ‚Π°Π½ΠΈΠ΅ Π΄Ρ€ΠΎΠ±Π΅ΠΉ с ΠΎΠ΄ΠΈΠ½Π°ΠΊΠΎΠ²Ρ‹ΠΌΠΈ знамСнатСлями
  2. Π’Ρ‹Ρ‡ΠΈΡ‚Π°Π½ΠΈΠ΅ Π΄Ρ€ΠΎΠ±Π΅ΠΉ с Ρ€Π°Π·Π½Ρ‹ΠΌΠΈ знамСнатСлями

Π‘Π½Π°Ρ‡Π°Π»Π° ΠΈΠ·ΡƒΡ‡ΠΈΠΌ Π²Ρ‹Ρ‡ΠΈΡ‚Π°Π½ΠΈΠ΅ Π΄Ρ€ΠΎΠ±Π΅ΠΉ с ΠΎΠ΄ΠΈΠ½Π°ΠΊΠΎΠ²Ρ‹ΠΌΠΈ знамСнатСлями.

Π§Ρ‚ΠΎΠ±Ρ‹ Π²Ρ‹Ρ‡Π΅ΡΡ‚ΡŒ ΠΈΠ· ΠΎΠ΄Π½ΠΎΠΉ Π΄Ρ€ΠΎΠ±ΠΈ Π΄Ρ€ΡƒΠ³ΡƒΡŽ, Π½ΡƒΠΆΠ½ΠΎ ΠΈΠ· числитСля ΠΏΠ΅Ρ€Π²ΠΎΠΉ Π΄Ρ€ΠΎΠ±ΠΈ Π²Ρ‹Ρ‡Π΅ΡΡ‚ΡŒ Ρ‡ΠΈΡΠ»ΠΈΡ‚Π΅Π»ΡŒ Π²Ρ‚ΠΎΡ€ΠΎΠΉ Π΄Ρ€ΠΎΠ±ΠΈ, Π° Π·Π½Π°ΠΌΠ΅Π½Π°Ρ‚Π΅Π»ΡŒ ΠΎΡΡ‚Π°Π²ΠΈΡ‚ΡŒ Π±Π΅Π· измСнСния.

НапримСр, Π½Π°ΠΉΠ΄Ρ‘ΠΌ Π·Π½Π°Ρ‡Π΅Π½ΠΈΠ΅ выраТСния  . Π§Ρ‚ΠΎΠ±Ρ‹ Ρ€Π΅ΡˆΠΈΡ‚ΡŒ этот ΠΏΡ€ΠΈΠΌΠ΅Ρ€, Π½Π°Π΄ΠΎ ΠΈΠ· числитСля ΠΏΠ΅Ρ€Π²ΠΎΠΉ Π΄Ρ€ΠΎΠ±ΠΈ Π²Ρ‹Ρ‡Π΅ΡΡ‚ΡŒ Ρ‡ΠΈΡΠ»ΠΈΡ‚Π΅Π»ΡŒ Π²Ρ‚ΠΎΡ€ΠΎΠΉ Π΄Ρ€ΠΎΠ±ΠΈ, Π° Π·Π½Π°ΠΌΠ΅Π½Π°Ρ‚Π΅Π»ΡŒ ΠΎΡΡ‚Π°Π²ΠΈΡ‚ΡŒ Π±Π΅Π· измСнСния. Π’Π°ΠΊ ΠΈ сдСлаСм:

Π­Ρ‚ΠΎΡ‚ ΠΏΡ€ΠΈΠΌΠ΅Ρ€ ΠΌΠΎΠΆΠ½ΠΎ Π»Π΅Π³ΠΊΠΎ ΠΏΠΎΠ½ΡΡ‚ΡŒ, Ссли Π²ΡΠΏΠΎΠΌΠ½ΠΈΡ‚ΡŒ ΠΏΡ€ΠΎ ΠΏΠΈΡ†Ρ†Ρƒ, которая Ρ€Π°Π·Π΄Π΅Π»Π΅Π½Π° Π½Π° Ρ‡Π΅Ρ‚Ρ‹Ρ€Π΅ части. Если ΠΎΡ‚Β  ΠΏΠΈΡ†Ρ†Ρ‹ ΠΎΡ‚Ρ€Π΅Π·Π°Ρ‚ΡŒΒ Β  ΠΏΠΈΡ†Ρ†Ρ‹, Ρ‚ΠΎ получится  пиццы:


ΠŸΡ€ΠΈΠΌΠ΅Ρ€ 2. Найти Π·Π½Π°Ρ‡Π΅Π½ΠΈΠ΅ выраТСния .

ΠžΠΏΡΡ‚ΡŒ ΠΆΠ΅ ΠΈΠ· числитСля ΠΏΠ΅Ρ€Π²ΠΎΠΉ Π΄Ρ€ΠΎΠ±ΠΈ Π²Ρ‹Ρ‡ΠΈΡ‚Π°Π΅ΠΌ Ρ‡ΠΈΡΠ»ΠΈΡ‚Π΅Π»ΡŒ Π²Ρ‚ΠΎΡ€ΠΎΠΉ Π΄Ρ€ΠΎΠ±ΠΈ, Π° Π·Π½Π°ΠΌΠ΅Π½Π°Ρ‚Π΅Π»ΡŒ оставляСм Π±Π΅Π· измСнСния:

Π­Ρ‚ΠΎΡ‚ ΠΏΡ€ΠΈΠΌΠ΅Ρ€ ΠΌΠΎΠΆΠ½ΠΎ Π»Π΅Π³ΠΊΠΎ ΠΏΠΎΠ½ΡΡ‚ΡŒ, Ссли Π²ΡΠΏΠΎΠΌΠ½ΠΈΡ‚ΡŒ ΠΏΡ€ΠΎ ΠΏΠΈΡ†Ρ†Ρƒ, которая Ρ€Π°Π·Π΄Π΅Π»Π΅Π½Π° Π½Π° Ρ‚Ρ€ΠΈ части. Если ΠΎΡ‚Β  ΠΏΠΈΡ†Ρ†Ρ‹ ΠΎΡ‚Ρ€Π΅Π·Π°Ρ‚ΡŒΒ Β  ΠΏΠΈΡ†Ρ†Ρ‹, Ρ‚ΠΎ получится  ΠΏΠΈΡ†Ρ†Ρ‹:


ΠŸΡ€ΠΈΠΌΠ΅Ρ€ 3. Найти Π·Π½Π°Ρ‡Π΅Π½ΠΈΠ΅ выраТСния

Π­Ρ‚ΠΎΡ‚ ΠΏΡ€ΠΈΠΌΠ΅Ρ€ Ρ€Π΅ΡˆΠ°Π΅Ρ‚ΡΡ Ρ‚ΠΎΡ‡Π½ΠΎ Ρ‚Π°ΠΊΠΆΠ΅, ΠΊΠ°ΠΊ ΠΈ ΠΏΡ€Π΅Π΄Ρ‹Π΄ΡƒΡ‰ΠΈΠ΅. Из числитСля ΠΏΠ΅Ρ€Π²ΠΎΠΉ Π΄Ρ€ΠΎΠ±ΠΈ Π½ΡƒΠΆΠ½ΠΎ Π²Ρ‹Ρ‡Π΅ΡΡ‚ΡŒ числитСли ΠΎΡΡ‚Π°Π»ΡŒΠ½Ρ‹Ρ… Π΄Ρ€ΠΎΠ±Π΅ΠΉ:

Π’ ΠΎΡ‚Π²Π΅Ρ‚Π΅ ΠΏΠΎΠ»ΡƒΡ‡ΠΈΠ»Π°ΡΡŒ Π½Π΅ΠΏΡ€Π°Π²ΠΈΠ»ΡŒΠ½Π°Ρ Π΄Ρ€ΠΎΠ±ΡŒ. Π’Ρ‹Π΄Π΅Π»ΠΈΠΌ Π² Π½Π΅ΠΉ Ρ†Π΅Π»ΡƒΡŽ Ρ‡Π°ΡΡ‚ΡŒ:

Как Π²ΠΈΠ΄ΠΈΡ‚Π΅ Π² Π²Ρ‹Ρ‡ΠΈΡ‚Π°Π½ΠΈΠΈ Π΄Ρ€ΠΎΠ±Π΅ΠΉ с ΠΎΠ΄ΠΈΠ½Π°ΠΊΠΎΠ²Ρ‹ΠΌΠΈ знамСнатСлями Π½ΠΈΡ‡Π΅Π³ΠΎ слоТного Π½Π΅Ρ‚. Достаточно ΠΏΠΎΠ½ΠΈΠΌΠ°Ρ‚ΡŒ ΡΠ»Π΅Π΄ΡƒΡŽΡ‰ΠΈΠ΅ ΠΏΡ€Π°Π²ΠΈΠ»Π°:

  1. Π§Ρ‚ΠΎΠ±Ρ‹ Π²Ρ‹Ρ‡Π΅ΡΡ‚ΡŒ ΠΈΠ· ΠΎΠ΄Π½ΠΎΠΉ Π΄Ρ€ΠΎΠ±ΠΈ Π΄Ρ€ΡƒΠ³ΡƒΡŽ, Π½ΡƒΠΆΠ½ΠΎ ΠΈΠ· числитСля ΠΏΠ΅Ρ€Π²ΠΎΠΉ Π΄Ρ€ΠΎΠ±ΠΈ Π²Ρ‹Ρ‡Π΅ΡΡ‚ΡŒ Ρ‡ΠΈΡΠ»ΠΈΡ‚Π΅Π»ΡŒ Π²Ρ‚ΠΎΡ€ΠΎΠΉ Π΄Ρ€ΠΎΠ±ΠΈ, Π° Π·Π½Π°ΠΌΠ΅Π½Π°Ρ‚Π΅Π»ΡŒ ΠΎΡΡ‚Π°Π²ΠΈΡ‚ΡŒ Π±Π΅Π· измСнСния;
  2. Если Π² ΠΎΡ‚Π²Π΅Ρ‚Π΅ ΠΏΠΎΠ»ΡƒΡ‡ΠΈΠ»Π°ΡΡŒ Π½Π΅ΠΏΡ€Π°Π²ΠΈΠ»ΡŒΠ½Π°Ρ Π΄Ρ€ΠΎΠ±ΡŒ, Ρ‚ΠΎ Π½ΡƒΠΆΠ½ΠΎ Π²Ρ‹Π΄Π΅Π»ΠΈΡ‚ΡŒ Π²Β  Π½Π΅ΠΉ Ρ†Π΅Π»ΡƒΡŽ Ρ‡Π°ΡΡ‚ΡŒ.

Π’Ρ‹Ρ‡ΠΈΡ‚Π°Π½ΠΈΠ΅ Π΄Ρ€ΠΎΠ±Π΅ΠΉ с Ρ€Π°Π·Π½Ρ‹ΠΌΠΈ знамСнатСлями

Π’Π΅ΠΏΠ΅Ρ€ΡŒ научимся Π²Ρ‹Ρ‡ΠΈΡ‚Π°Ρ‚ΡŒ Π΄Ρ€ΠΎΠ±ΠΈ Ρƒ ΠΊΠΎΡ‚ΠΎΡ€Ρ‹Ρ… Ρ€Π°Π·Π½Ρ‹Π΅ Π·Π½Π°ΠΌΠ΅Π½Π°Ρ‚Π΅Π»ΠΈ. Когда Π²Ρ‹Ρ‡ΠΈΡ‚Π°ΡŽΡ‚ Π΄Ρ€ΠΎΠ±ΠΈ ΠΈΡ… Π·Π½Π°ΠΌΠ΅Π½Π°Ρ‚Π΅Π»ΠΈ Π΄ΠΎΠ»ΠΆΠ½Ρ‹ Π±Ρ‹Ρ‚ΡŒ ΠΎΠ΄ΠΈΠ½Π°ΠΊΠΎΠ²Ρ‹ΠΌΠΈ. Но ΠΎΠ΄ΠΈΠ½Π°ΠΊΠΎΠ²Ρ‹ΠΌΠΈ ΠΎΠ½ΠΈ Π±Ρ‹Π²Π°ΡŽΡ‚ Π½Π΅ всСгда.

НапримСр, ΠΎΡ‚ Π΄Ρ€ΠΎΠ±ΠΈΒ  ΠΌΠΎΠΆΠ½ΠΎ Π²Ρ‹Ρ‡Π΅ΡΡ‚ΡŒ Π΄Ρ€ΠΎΠ±ΡŒ ,Β ΠΏΠΎΡΠΊΠΎΠ»ΡŒΠΊΡƒ Ρƒ этих Π΄Ρ€ΠΎΠ±Π΅ΠΉΒ  ΠΎΠ΄ΠΈΠ½Π°ΠΊΠΎΠ²Ρ‹Π΅ Π·Π½Π°ΠΌΠ΅Π½Π°Ρ‚Π΅Π»ΠΈ. А Π²ΠΎΡ‚ ΠΎΡ‚ Π΄Ρ€ΠΎΠ±ΠΈΒ Β Π½Π΅Π»ΡŒΠ·Ρ Π²Ρ‹Ρ‡Π΅ΡΡ‚ΡŒ Π΄Ρ€ΠΎΠ±ΡŒ , ΠΏΠΎΡΠΊΠΎΠ»ΡŒΠΊΡƒ Ρƒ этих Π΄Ρ€ΠΎΠ±Π΅ΠΉ Ρ€Π°Π·Π½Ρ‹Π΅ Π·Π½Π°ΠΌΠ΅Π½Π°Ρ‚Π΅Π»ΠΈ. Π’ Ρ‚Π°ΠΊΠΈΡ… случаях Π΄Ρ€ΠΎΠ±ΠΈ Π½ΡƒΠΆΠ½ΠΎ ΠΏΡ€ΠΈΠ²ΠΎΠ΄ΠΈΡ‚ΡŒ ΠΊ ΠΎΠ΄ΠΈΠ½Π°ΠΊΠΎΠ²ΠΎΠΌΡƒ (ΠΎΠ±Ρ‰Π΅ΠΌΡƒ) Π·Π½Π°ΠΌΠ΅Π½Π°Ρ‚Π΅Π»ΡŽ.

ΠžΠ±Ρ‰ΠΈΠΉ Π·Π½Π°ΠΌΠ΅Π½Π°Ρ‚Π΅Π»ΡŒ находят ΠΏΠΎ Ρ‚ΠΎΠΌΡƒ ΠΆΠ΅ ΠΏΡ€ΠΈΠ½Ρ†ΠΈΠΏΡƒ, ΠΊΠΎΡ‚ΠΎΡ€Ρ‹ΠΌ ΠΌΡ‹ пользовались ΠΏΡ€ΠΈ слоТСнии Π΄Ρ€ΠΎΠ±Π΅ΠΉ с Ρ€Π°Π·Π½Ρ‹ΠΌΠΈ знамСнатСлями. Π’ ΠΏΠ΅Ρ€Π²ΡƒΡŽ ΠΎΡ‡Π΅Ρ€Π΅Π΄ΡŒ находят НОК Π·Π½Π°ΠΌΠ΅Π½Π°Ρ‚Π΅Π»Π΅ΠΉ ΠΎΠ±Π΅ΠΈΡ… Π΄Ρ€ΠΎΠ±Π΅ΠΉ. Π—Π°Ρ‚Π΅ΠΌ НОК дСлят Π½Π° Π·Π½Π°ΠΌΠ΅Π½Π°Ρ‚Π΅Π»ΡŒ ΠΏΠ΅Ρ€Π²ΠΎΠΉ Π΄Ρ€ΠΎΠ±ΠΈ ΠΈ ΠΏΠΎΠ»ΡƒΡ‡Π°ΡŽΡ‚ ΠΏΠ΅Ρ€Π²Ρ‹ΠΉ Π΄ΠΎΠΏΠΎΠ»Π½ΠΈΡ‚Π΅Π»ΡŒΠ½Ρ‹ΠΉ ΠΌΠ½ΠΎΠΆΠΈΡ‚Π΅Π»ΡŒ, ΠΊΠΎΡ‚ΠΎΡ€Ρ‹ΠΉ записываСтся Π½Π°Π΄ ΠΏΠ΅Ρ€Π²ΠΎΠΉ Π΄Ρ€ΠΎΠ±ΡŒΡŽ. Аналогично НОК дСлят Π½Π° Π·Π½Π°ΠΌΠ΅Π½Π°Ρ‚Π΅Π»ΡŒ Π²Ρ‚ΠΎΡ€ΠΎΠΉ Π΄Ρ€ΠΎΠ±ΠΈ ΠΈ ΠΏΠΎΠ»ΡƒΡ‡Π°ΡŽΡ‚ Π²Ρ‚ΠΎΡ€ΠΎΠΉ Π΄ΠΎΠΏΠΎΠ»Π½ΠΈΡ‚Π΅Π»ΡŒΠ½Ρ‹ΠΉ ΠΌΠ½ΠΎΠΆΠΈΡ‚Π΅Π»ΡŒ, ΠΊΠΎΡ‚ΠΎΡ€Ρ‹ΠΉ записываСтся Π½Π°Π΄ Π²Ρ‚ΠΎΡ€ΠΎΠΉ Π΄Ρ€ΠΎΠ±ΡŒΡŽ.

Π—Π°Ρ‚Π΅ΠΌ Π΄Ρ€ΠΎΠ±ΠΈ ΡƒΠΌΠ½ΠΎΠΆΠ°ΡŽΡ‚ΡΡ Π½Π° свои Π΄ΠΎΠΏΠΎΠ»Π½ΠΈΡ‚Π΅Π»ΡŒΠ½Ρ‹Π΅ ΠΌΠ½ΠΎΠΆΠΈΡ‚Π΅Π»ΠΈ. Π’ Ρ€Π΅Π·ΡƒΠ»ΡŒΡ‚Π°Ρ‚Π΅ этих ΠΎΠΏΠ΅Ρ€Π°Ρ†ΠΈΠΉ, Π΄Ρ€ΠΎΠ±ΠΈ Ρƒ ΠΊΠΎΡ‚ΠΎΡ€Ρ‹Ρ… Π±Ρ‹Π»ΠΈ Ρ€Π°Π·Π½Ρ‹Π΅ Π·Π½Π°ΠΌΠ΅Π½Π°Ρ‚Π΅Π»ΠΈ, ΠΎΠ±Ρ€Π°Ρ‰Π°ΡŽΡ‚ΡΡ Π² Π΄Ρ€ΠΎΠ±ΠΈ, Ρƒ ΠΊΠΎΡ‚ΠΎΡ€Ρ‹Ρ… ΠΎΠ΄ΠΈΠ½Π°ΠΊΠΎΠ²Ρ‹Π΅ Π·Π½Π°ΠΌΠ΅Π½Π°Ρ‚Π΅Π»ΠΈ. А ΠΊΠ°ΠΊ Π²Ρ‹Ρ‡ΠΈΡ‚Π°Ρ‚ΡŒ Ρ‚Π°ΠΊΠΈΠ΅ Π΄Ρ€ΠΎΠ±ΠΈ ΠΌΡ‹ ΡƒΠΆΠ΅ Π·Π½Π°Π΅ΠΌ.

ΠŸΡ€ΠΈΠΌΠ΅Ρ€ 1. Найти Π·Π½Π°Ρ‡Π΅Π½ΠΈΠ΅ выраТСния:

Π£ этих Π΄Ρ€ΠΎΠ±Π΅ΠΉ Ρ€Π°Π·Π½Ρ‹Π΅ Π·Π½Π°ΠΌΠ΅Π½Π°Ρ‚Π΅Π»ΠΈ, поэтому Π½ΡƒΠΆΠ½ΠΎ привСсти ΠΈΡ… ΠΊ ΠΎΠ΄ΠΈΠ½Π°ΠΊΠΎΠ²ΠΎΠΌΡƒ (ΠΎΠ±Ρ‰Π΅ΠΌΡƒ) Π·Π½Π°ΠΌΠ΅Π½Π°Ρ‚Π΅Π»ΡŽ.

Π‘Π½Π°Ρ‡Π°Π»Π° Π½Π°Ρ…ΠΎΠ΄ΠΈΠΌ НОК Π·Π½Π°ΠΌΠ΅Π½Π°Ρ‚Π΅Π»Π΅ΠΉ ΠΎΠ±Π΅ΠΈΡ… Π΄Ρ€ΠΎΠ±Π΅ΠΉ. Π—Π½Π°ΠΌΠ΅Π½Π°Ρ‚Π΅Π»ΡŒ ΠΏΠ΅Ρ€Π²ΠΎΠΉ Π΄Ρ€ΠΎΠ±ΠΈ это число 3, Π° Π·Π½Π°ΠΌΠ΅Π½Π°Ρ‚Π΅Π»ΡŒ Π²Ρ‚ΠΎΡ€ΠΎΠΉ Π΄Ρ€ΠΎΠ±ΠΈ β€” число 4. НаимСньшСС ΠΎΠ±Ρ‰Π΅Π΅ ΠΊΡ€Π°Ρ‚Π½ΠΎΠ΅ этих чисСл Ρ€Π°Π²Π½ΠΎ 12

НОК (3 и 4) = 12

Π’Π΅ΠΏΠ΅Ρ€ΡŒ возвращаСмся ΠΊ дробям  ΠΈ

Найдём Π΄ΠΎΠΏΠΎΠ»Π½ΠΈΡ‚Π΅Π»ΡŒΠ½Ρ‹ΠΉ ΠΌΠ½ΠΎΠΆΠΈΡ‚Π΅Π»ΡŒ для ΠΏΠ΅Ρ€Π²ΠΎΠΉ Π΄Ρ€ΠΎΠ±ΠΈ. Для этого Ρ€Π°Π·Π΄Π΅Π»ΠΈΠΌ НОК Π½Π° Π·Π½Π°ΠΌΠ΅Π½Π°Ρ‚Π΅Π»ΡŒ ΠΏΠ΅Ρ€Π²ΠΎΠΉ Π΄Ρ€ΠΎΠ±ΠΈ. НОК это число 12, Π° Π·Π½Π°ΠΌΠ΅Π½Π°Ρ‚Π΅Π»ΡŒ ΠΏΠ΅Ρ€Π²ΠΎΠΉ Π΄Ρ€ΠΎΠ±ΠΈ β€” число 3. Π”Π΅Π»ΠΈΠΌ 12 Π½Π° 3, ΠΏΠΎΠ»ΡƒΡ‡Π°Π΅ΠΌ 4. ЗаписываСм Ρ‡Π΅Ρ‚Π²Ρ‘Ρ€ΠΊΡƒ Π½Π°Π΄ ΠΏΠ΅Ρ€Π²ΠΎΠΉ Π΄Ρ€ΠΎΠ±ΡŒΡŽ:

Аналогично поступаСм ΠΈ со Π²Ρ‚ΠΎΡ€ΠΎΠΉ Π΄Ρ€ΠΎΠ±ΡŒΡŽ. Π”Π΅Π»ΠΈΠΌ НОК Π½Π° Π·Π½Π°ΠΌΠ΅Π½Π°Ρ‚Π΅Π»ΡŒ Π²Ρ‚ΠΎΡ€ΠΎΠΉ Π΄Ρ€ΠΎΠ±ΠΈ. НОК это число 12, Π° Π·Π½Π°ΠΌΠ΅Π½Π°Ρ‚Π΅Π»ΡŒ Π²Ρ‚ΠΎΡ€ΠΎΠΉ Π΄Ρ€ΠΎΠ±ΠΈ β€” число 4. Π”Π΅Π»ΠΈΠΌ 12 Π½Π° 4, ΠΏΠΎΠ»ΡƒΡ‡Π°Π΅ΠΌ 3. ЗаписываСм Ρ‚Ρ€ΠΎΠΉΠΊΡƒ Π½Π°Π΄ Π²Ρ‚ΠΎΡ€ΠΎΠΉ Π΄Ρ€ΠΎΠ±ΡŒΡŽ:

Π’Π΅ΠΏΠ΅Ρ€ΡŒ Ρƒ нас всё Π³ΠΎΡ‚ΠΎΠ²ΠΎ для вычитания. ΠžΡΡ‚Π°Π»ΠΎΡΡŒ ΡƒΠΌΠ½ΠΎΠΆΠΈΡ‚ΡŒ Π΄Ρ€ΠΎΠ±ΠΈ Π½Π° свои Π΄ΠΎΠΏΠΎΠ»Π½ΠΈΡ‚Π΅Π»ΡŒΠ½Ρ‹Π΅ ΠΌΠ½ΠΎΠΆΠΈΡ‚Π΅Π»ΠΈ:

ΠœΡ‹ ΠΏΡ€ΠΈΡˆΠ»ΠΈ ΠΊ Ρ‚ΠΎΠΌΡƒ, Ρ‡Ρ‚ΠΎ Π΄Ρ€ΠΎΠ±ΠΈ Ρƒ ΠΊΠΎΡ‚ΠΎΡ€Ρ‹Ρ… Π±Ρ‹Π»ΠΈ Ρ€Π°Π·Π½Ρ‹Π΅ Π·Π½Π°ΠΌΠ΅Π½Π°Ρ‚Π΅Π»ΠΈ, ΠΏΡ€Π΅Π²Ρ€Π°Ρ‚ΠΈΠ»ΠΈΡΡŒ Π² Π΄Ρ€ΠΎΠ±ΠΈ Ρƒ ΠΊΠΎΡ‚ΠΎΡ€Ρ‹Ρ… ΠΎΠ΄ΠΈΠ½Π°ΠΊΠΎΠ²Ρ‹Π΅ Π·Π½Π°ΠΌΠ΅Π½Π°Ρ‚Π΅Π»ΠΈ. А ΠΊΠ°ΠΊ Π²Ρ‹Ρ‡ΠΈΡ‚Π°Ρ‚ΡŒ Ρ‚Π°ΠΊΠΈΠ΅ Π΄Ρ€ΠΎΠ±ΠΈ ΠΌΡ‹ ΡƒΠΆΠ΅ Π·Π½Π°Π΅ΠΌ. Π”Π°Π²Π°ΠΉΡ‚Π΅ Π΄ΠΎΡ€Π΅ΡˆΠ°Π΅ΠΌ этот ΠΏΡ€ΠΈΠΌΠ΅Ρ€ Π΄ΠΎ ΠΊΠΎΠ½Ρ†Π°:

ΠŸΠΎΠ»ΡƒΡ‡ΠΈΠ»ΠΈ ΠΎΡ‚Π²Π΅Ρ‚

ΠŸΠΎΠΏΡ€ΠΎΠ±ΡƒΠ΅ΠΌ ΠΈΠ·ΠΎΠ±Ρ€Π°Π·ΠΈΡ‚ΡŒ нашС Ρ€Π΅ΡˆΠ΅Π½ΠΈΠ΅ с ΠΏΠΎΠΌΠΎΡ‰ΡŒΡŽ рисунка. Если ΠΎΡ‚Β Β ΠΏΠΈΡ†Ρ†Ρ‹ ΠΎΡ‚Ρ€Π΅Π·Π°Ρ‚ΡŒΒ Β ΠΏΠΈΡ†Ρ†Ρ‹, Ρ‚ΠΎ получится  пиццы

Π­Ρ‚ΠΎ подробная вСрсия Ρ€Π΅ΡˆΠ΅Π½ΠΈΡ. ΠΠ°Ρ…ΠΎΠ΄ΡΡΡŒ Π² школС, Π½Π°ΠΌ ΠΏΡ€ΠΈΡˆΠ»ΠΎΡΡŒ Π±Ρ‹ Ρ€Π΅ΡˆΠΈΡ‚ΡŒ этот ΠΏΡ€ΠΈΠΌΠ΅Ρ€ ΠΏΠΎΠΊΠΎΡ€ΠΎΡ‡Π΅. ВыглядСло Π±Ρ‹ Ρ‚Π°ΠΊΠΎΠ΅ Ρ€Π΅ΡˆΠ΅Π½ΠΈΠ΅ ΡΠ»Π΅Π΄ΡƒΡŽΡ‰ΠΈΠΌ ΠΎΠ±Ρ€Π°Π·ΠΎΠΌ:

ΠŸΡ€ΠΈΠ²Π΅Π΄Π΅Π½ΠΈΠ΅ Π΄Ρ€ΠΎΠ±Π΅ΠΉΒ Β ΠΈΒ Β ΠΊ ΠΎΠ±Ρ‰Π΅ΠΌΡƒ Π·Π½Π°ΠΌΠ΅Π½Π°Ρ‚Π΅Π»ΡŽ Ρ‚Π°ΠΊΠΆΠ΅ ΠΌΠΎΠΆΠ΅Ρ‚ Π±Ρ‹Ρ‚ΡŒ ΠΈΠ·ΠΎΠ±Ρ€Π°ΠΆΠ΅Π½ΠΎ с ΠΏΠΎΠΌΠΎΡ‰ΡŒΡŽ рисунка. ΠŸΡ€ΠΈΠ²Π΅Π΄Ρ эти Π΄Ρ€ΠΎΠ±ΠΈ ΠΊ ΠΎΠ±Ρ‰Π΅ΠΌΡƒ Π·Π½Π°ΠΌΠ΅Π½Π°Ρ‚Π΅Π»ΡŽ, ΠΌΡ‹ ΠΏΠΎΠ»ΡƒΡ‡ΠΈΠ»ΠΈ Π΄Ρ€ΠΎΠ±ΠΈΒ Β ΠΈ . Π­Ρ‚ΠΈ Π΄Ρ€ΠΎΠ±ΠΈ Π±ΡƒΠ΄ΡƒΡ‚ ΠΈΠ·ΠΎΠ±Ρ€Π°ΠΆΠ°Ρ‚ΡŒΡΡ Ρ‚Π΅ΠΌΠΈ ΠΆΠ΅ кусочками ΠΏΠΈΡ†Ρ†, Π½ΠΎ Π² этот Ρ€Π°Π· ΠΎΠ½ΠΈ Π±ΡƒΠ΄ΡƒΡ‚ Ρ€Π°Π·Π΄Π΅Π»Π΅Π½Ρ‹ Π½Π° ΠΎΠ΄ΠΈΠ½Π°ΠΊΠΎΠ²Ρ‹Π΅ Π΄ΠΎΠ»ΠΈ (ΠΏΡ€ΠΈΠ²Π΅Π΄Π΅Π½Ρ‹ ΠΊ ΠΎΠ΄ΠΈΠ½Π°ΠΊΠΎΠ²ΠΎΠΌΡƒ Π·Π½Π°ΠΌΠ΅Π½Π°Ρ‚Π΅Π»ΡŽ):

ΠŸΠ΅Ρ€Π²Ρ‹ΠΉ рисунок ΠΈΠ·ΠΎΠ±Ρ€Π°ΠΆΠ°Π΅Ρ‚ Π΄Ρ€ΠΎΠ±ΡŒ Β (восСмь кусочков ΠΈΠ· Π΄Π²Π΅Π½Π°Π΄Ρ†Π°Ρ‚ΠΈ), Π° Π²Ρ‚ΠΎΡ€ΠΎΠΉ рисунок β€” Π΄Ρ€ΠΎΠ±ΡŒ Β (Ρ‚Ρ€ΠΈ кусочка ΠΈΠ· Π΄Π²Π΅Π½Π°Π΄Ρ†Π°Ρ‚ΠΈ). ΠžΡ‚Ρ€Π΅Π·Π°Π² ΠΎΡ‚ восьми кусочков Ρ‚Ρ€ΠΈ кусочка ΠΌΡ‹ ΠΏΠΎΠ»ΡƒΡ‡Π°Π΅ΠΌ ΠΏΡΡ‚ΡŒ кусочков ΠΈΠ· Π΄Π²Π΅Π½Π°Π΄Ρ†Π°Ρ‚ΠΈ. Π”Ρ€ΠΎΠ±ΡŒΒ Β ΠΈ описываСт эти ΠΏΡΡ‚ΡŒ кусочков.


ΠŸΡ€ΠΈΠΌΠ΅Ρ€ 2. Найти Π·Π½Π°Ρ‡Π΅Π½ΠΈΠ΅ выраТСния

Π£ этих Π΄Ρ€ΠΎΠ±Π΅ΠΉ Ρ€Π°Π·Π½Ρ‹Π΅ Π·Π½Π°ΠΌΠ΅Π½Π°Ρ‚Π΅Π»ΠΈ, поэтому сначала Π½ΡƒΠΆΠ½ΠΎ привСсти ΠΈΡ… ΠΊ ΠΎΠ΄ΠΈΠ½Π°ΠΊΠΎΠ²ΠΎΠΌΡƒ (ΠΎΠ±Ρ‰Π΅ΠΌΡƒ) Π·Π½Π°ΠΌΠ΅Π½Π°Ρ‚Π΅Π»ΡŽ.

Найдём НОК Π·Π½Π°ΠΌΠ΅Π½Π°Ρ‚Π΅Π»Π΅ΠΉ этих Π΄Ρ€ΠΎΠ±Π΅ΠΉ.

Π—Π½Π°ΠΌΠ΅Π½Π°Ρ‚Π΅Π»ΠΈ Π΄Ρ€ΠΎΠ±Π΅ΠΉ это числа 10, 3 ΠΈ 5. НаимСньшСС ΠΎΠ±Ρ‰Π΅Π΅ ΠΊΡ€Π°Ρ‚Π½ΠΎΠ΅ этих чисСл Ρ€Π°Π²Π½ΠΎ 30

НОК (10, 3, 5) = 30

Π’Π΅ΠΏΠ΅Ρ€ΡŒ Π½Π°Ρ…ΠΎΠ΄ΠΈΠΌ Π΄ΠΎΠΏΠΎΠ»Π½ΠΈΡ‚Π΅Π»ΡŒΠ½Ρ‹Π΅ ΠΌΠ½ΠΎΠΆΠΈΡ‚Π΅Π»ΠΈ для ΠΊΠ°ΠΆΠ΄ΠΎΠΉ Π΄Ρ€ΠΎΠ±ΠΈ. Для этого Ρ€Π°Π·Π΄Π΅Π»ΠΈΠΌ НОК Π½Π° Π·Π½Π°ΠΌΠ΅Π½Π°Ρ‚Π΅Π»ΡŒ ΠΊΠ°ΠΆΠ΄ΠΎΠΉ Π΄Ρ€ΠΎΠ±ΠΈ.

Найдём Π΄ΠΎΠΏΠΎΠ»Π½ΠΈΡ‚Π΅Π»ΡŒΠ½Ρ‹ΠΉ ΠΌΠ½ΠΎΠΆΠΈΡ‚Π΅Π»ΡŒ для ΠΏΠ΅Ρ€Π²ΠΎΠΉ Π΄Ρ€ΠΎΠ±ΠΈ. НОК это число 30, Π° Π·Π½Π°ΠΌΠ΅Π½Π°Ρ‚Π΅Π»ΡŒ ΠΏΠ΅Ρ€Π²ΠΎΠΉ Π΄Ρ€ΠΎΠ±ΠΈ β€” число 10. Π”Π΅Π»ΠΈΠΌ 30 Π½Π° 10, ΠΏΠΎΠ»ΡƒΡ‡Π°Π΅ΠΌ ΠΏΠ΅Ρ€Π²Ρ‹ΠΉ Π΄ΠΎΠΏΠΎΠ»Π½ΠΈΡ‚Π΅Π»ΡŒΠ½Ρ‹ΠΉ ΠΌΠ½ΠΎΠΆΠΈΡ‚Π΅Π»ΡŒ 3. ЗаписываСм Π΅Π³ΠΎ Π½Π°Π΄ ΠΏΠ΅Ρ€Π²ΠΎΠΉ Π΄Ρ€ΠΎΠ±ΡŒΡŽ:

Π’Π΅ΠΏΠ΅Ρ€ΡŒ Π½Π°Ρ…ΠΎΠ΄ΠΈΠΌ Π΄ΠΎΠΏΠΎΠ»Π½ΠΈΡ‚Π΅Π»ΡŒΠ½Ρ‹ΠΉ ΠΌΠ½ΠΎΠΆΠΈΡ‚Π΅Π»ΡŒ для Π²Ρ‚ΠΎΡ€ΠΎΠΉ Π΄Ρ€ΠΎΠ±ΠΈ. Π Π°Π·Π΄Π΅Π»ΠΈΠΌ НОК Π½Π° Π·Π½Π°ΠΌΠ΅Π½Π°Ρ‚Π΅Π»ΡŒ Π²Ρ‚ΠΎΡ€ΠΎΠΉ Π΄Ρ€ΠΎΠ±ΠΈ. НОК это число 30, Π° Π·Π½Π°ΠΌΠ΅Π½Π°Ρ‚Π΅Π»ΡŒ Π²Ρ‚ΠΎΡ€ΠΎΠΉ Π΄Ρ€ΠΎΠ±ΠΈ β€” число 3. Π”Π΅Π»ΠΈΠΌ 30 Π½Π° 3, ΠΏΠΎΠ»ΡƒΡ‡Π°Π΅ΠΌ Π²Ρ‚ΠΎΡ€ΠΎΠΉ Π΄ΠΎΠΏΠΎΠ»Π½ΠΈΡ‚Π΅Π»ΡŒΠ½Ρ‹ΠΉ ΠΌΠ½ΠΎΠΆΠΈΡ‚Π΅Π»ΡŒ 10. ЗаписываСм Π΅Π³ΠΎ Π½Π°Π΄ Π²Ρ‚ΠΎΡ€ΠΎΠΉ Π΄Ρ€ΠΎΠ±ΡŒΡŽ:

Π’Π΅ΠΏΠ΅Ρ€ΡŒ Π½Π°Ρ…ΠΎΠ΄ΠΈΠΌ Π΄ΠΎΠΏΠΎΠ»Π½ΠΈΡ‚Π΅Π»ΡŒΠ½Ρ‹ΠΉ ΠΌΠ½ΠΎΠΆΠΈΡ‚Π΅Π»ΡŒ для Ρ‚Ρ€Π΅Ρ‚ΡŒΠ΅ΠΉ Π΄Ρ€ΠΎΠ±ΠΈ. Π Π°Π·Π΄Π΅Π»ΠΈΠΌ НОК Π½Π° Π·Π½Π°ΠΌΠ΅Π½Π°Ρ‚Π΅Π»ΡŒ Ρ‚Ρ€Π΅Ρ‚ΡŒΠ΅ΠΉ Π΄Ρ€ΠΎΠ±ΠΈ. НОК это число 30, Π° Π·Π½Π°ΠΌΠ΅Π½Π°Ρ‚Π΅Π»ΡŒ Ρ‚Ρ€Π΅Ρ‚ΡŒΠ΅ΠΉ Π΄Ρ€ΠΎΠ±ΠΈ β€” число 5. Π”Π΅Π»ΠΈΠΌ 30 Π½Π° 5, ΠΏΠΎΠ»ΡƒΡ‡Π°Π΅ΠΌ Ρ‚Ρ€Π΅Ρ‚ΠΈΠΉ Π΄ΠΎΠΏΠΎΠ»Π½ΠΈΡ‚Π΅Π»ΡŒΠ½Ρ‹ΠΉ ΠΌΠ½ΠΎΠΆΠΈΡ‚Π΅Π»ΡŒ 6. ЗаписываСм Π΅Π³ΠΎ Π½Π°Π΄ Ρ‚Ρ€Π΅Ρ‚ΡŒΠ΅ΠΉ Π΄Ρ€ΠΎΠ±ΡŒΡŽ:

Π’Π΅ΠΏΠ΅Ρ€ΡŒ всё Π³ΠΎΡ‚ΠΎΠ²ΠΎ для вычитания. ΠžΡΡ‚Π°Π»ΠΎΡΡŒ ΡƒΠΌΠ½ΠΎΠΆΠΈΡ‚ΡŒ Π΄Ρ€ΠΎΠ±ΠΈ Π½Π° свои Π΄ΠΎΠΏΠΎΠ»Π½ΠΈΡ‚Π΅Π»ΡŒΠ½Ρ‹Π΅ ΠΌΠ½ΠΎΠΆΠΈΡ‚Π΅Π»ΠΈ:

ΠœΡ‹ ΠΏΡ€ΠΈΡˆΠ»ΠΈΒ  ΠΊ Ρ‚ΠΎΠΌΡƒ, Ρ‡Ρ‚ΠΎ Π΄Ρ€ΠΎΠ±ΠΈ Ρƒ ΠΊΠΎΡ‚ΠΎΡ€Ρ‹Ρ… Π±Ρ‹Π»ΠΈ Ρ€Π°Π·Π½Ρ‹Π΅ Π·Π½Π°ΠΌΠ΅Π½Π°Ρ‚Π΅Π»ΠΈ, ΠΏΡ€Π΅Π²Ρ€Π°Ρ‚ΠΈΠ»ΠΈΡΡŒ Π² Π΄Ρ€ΠΎΠ±ΠΈ Ρƒ ΠΊΠΎΡ‚ΠΎΡ€Ρ‹Ρ… ΠΎΠ΄ΠΈΠ½Π°ΠΊΠΎΠ²Ρ‹Π΅ (ΠΎΠ±Ρ‰ΠΈΠ΅) Π·Π½Π°ΠΌΠ΅Π½Π°Ρ‚Π΅Π»ΠΈ. А ΠΊΠ°ΠΊ Π²Ρ‹Ρ‡ΠΈΡ‚Π°Ρ‚ΡŒ Ρ‚Π°ΠΊΠΈΠ΅ Π΄Ρ€ΠΎΠ±ΠΈ ΠΌΡ‹ ΡƒΠΆΠ΅ Π·Π½Π°Π΅ΠΌ. Π”Π°Π²Π°ΠΉΡ‚Π΅ Π΄ΠΎΡ€Π΅ΡˆΠ°Π΅ΠΌ этот ΠΏΡ€ΠΈΠΌΠ΅Ρ€.

ΠŸΡ€ΠΎΠ΄ΠΎΠ»ΠΆΠ΅Π½ΠΈΠ΅ ΠΏΡ€ΠΈΠΌΠ΅Ρ€Π° Π½Π΅ помСстится Π½Π° ΠΎΠ΄Π½ΠΎΠΉ строкС, поэтому пСрСносим ΠΏΡ€ΠΎΠ΄ΠΎΠ»ΠΆΠ΅Π½ΠΈΠ΅ Π½Π° ΡΠ»Π΅Π΄ΡƒΡŽΡ‰ΡƒΡŽ строку. НС Π·Π°Π±Ρ‹Π²Π°Π΅ΠΌ ΠΏΡ€ΠΎ Π·Π½Π°ΠΊ равСнства (=) Π½Π° Π½ΠΎΠ²ΠΎΠΉ строкС:

Π’ ΠΎΡ‚Π²Π΅Ρ‚Π΅ ΠΏΠΎΠ»ΡƒΡ‡ΠΈΠ»Π°ΡΡŒ ΠΏΡ€Π°Π²ΠΈΠ»ΡŒΠ½Π°Ρ Π΄Ρ€ΠΎΠ±ΡŒ, ΠΈ Π²Ρ€ΠΎΠ΄Π΅ Π±Ρ‹ нас всё устраиваСт, Π½ΠΎ ΠΎΠ½Π° слишком Π³Ρ€ΠΎΠΌΠΎΠ·Π΄ΠΊΠ° ΠΈ нСкрасива. Надо Π±Ρ‹ ΡΠ΄Π΅Π»Π°Ρ‚ΡŒ Π΅Ρ‘ ΠΏΡ€ΠΎΡ‰Π΅. А Ρ‡Ρ‚ΠΎ ΠΌΠΎΠΆΠ½ΠΎ ΡΠ΄Π΅Π»Π°Ρ‚ΡŒ? МоТно ΡΠΎΠΊΡ€Π°Ρ‚ΠΈΡ‚ΡŒ эту Π΄Ρ€ΠΎΠ±ΡŒ.

Π§Ρ‚ΠΎΠ±Ρ‹ ΡΠΎΠΊΡ€Π°Ρ‚ΠΈΡ‚ΡŒ Π΄Ρ€ΠΎΠ±ΡŒ ,Β Π½ΡƒΠΆΠ½ΠΎ Ρ€Π°Π·Π΄Π΅Π»ΠΈΡ‚ΡŒ Π΅Ρ‘ Ρ‡ΠΈΡΠ»ΠΈΡ‚Π΅Π»ΡŒ ΠΈ Π·Π½Π°ΠΌΠ΅Π½Π°Ρ‚Π΅Π»ΡŒ Π½Π° наибольший ΠΎΠ±Ρ‰ΠΈΠΉ Π΄Π΅Π»ΠΈΡ‚Π΅Π»ΡŒ (ΠΠžΠ”) чисСл 20 ΠΈ 30.

Π˜Ρ‚Π°ΠΊ, Π½Π°Ρ…ΠΎΠ΄ΠΈΠΌ ΠΠžΠ” чисСл 20 ΠΈ 30:

Π’Π΅ΠΏΠ΅Ρ€ΡŒ возвращаСмся ΠΊ Π½Π°ΡˆΠ΅ΠΌΡƒ ΠΏΡ€ΠΈΠΌΠ΅Ρ€Ρƒ ΠΈ Π΄Π΅Π»ΠΈΠΌ Ρ‡ΠΈΡΠ»ΠΈΡ‚Π΅Π»ΡŒ ΠΈ Π·Π½Π°ΠΌΠ΅Π½Π°Ρ‚Π΅Π»ΡŒ Π΄Ρ€ΠΎΠ±ΠΈΒ  Π½Π° Π½Π°ΠΉΠ΄Π΅Π½Π½Ρ‹ΠΉ ΠΠžΠ”, Ρ‚ΠΎ Π΅ΡΡ‚ΡŒ Π½Π° 10

ΠŸΠΎΠ»ΡƒΡ‡ΠΈΠ»ΠΈ ΠΎΡ‚Π²Π΅Ρ‚


Π£ΠΌΠ½ΠΎΠΆΠ΅Π½ΠΈΠ΅ Π΄Ρ€ΠΎΠ±ΠΈ Π½Π° число

Π§Ρ‚ΠΎΠ±Ρ‹ ΡƒΠΌΠ½ΠΎΠΆΠΈΡ‚ΡŒ Π΄Ρ€ΠΎΠ±ΡŒ Π½Π° число, Π½ΡƒΠΆΠ½ΠΎ Ρ‡ΠΈΡΠ»ΠΈΡ‚Π΅Π»ΡŒ Π΄Π°Π½Π½ΠΎΠΉ Π΄Ρ€ΠΎΠ±ΠΈ ΡƒΠΌΠ½ΠΎΠΆΠΈΡ‚ΡŒ Π½Π° это число, Π° Π·Π½Π°ΠΌΠ΅Π½Π°Ρ‚Π΅Π»ΡŒ ΠΎΡΡ‚Π°Π²ΠΈΡ‚ΡŒ Π±Π΅Π· ΠΈΠ·ΠΌΠ΅Π½Π΅Π½ΠΈΠΉ.

ΠŸΡ€ΠΈΠΌΠ΅Ρ€ 1. Π£ΠΌΠ½ΠΎΠΆΠΈΡ‚ΡŒ Π΄Ρ€ΠΎΠ±ΡŒΒ  Π½Π° число 1.

Π£ΠΌΠ½ΠΎΠΆΠΈΠΌ Ρ‡ΠΈΡΠ»ΠΈΡ‚Π΅Π»ΡŒ Π΄Ρ€ΠΎΠ±ΠΈ Π½Π° число 1

Π—Π°ΠΏΠΈΡΡŒ ΠΌΠΎΠΆΠ½ΠΎ ΠΏΠΎΠ½ΠΈΠΌΠ°Ρ‚ΡŒ, ΠΊΠ°ΠΊ Π²Π·ΡΡ‚ΡŒΒ ΠΏΠΎΠ»ΠΎΠ²ΠΈΠ½Ρƒ 1Β Ρ€Π°Π·. К ΠΏΡ€ΠΈΠΌΠ΅Ρ€Ρƒ, Ссли ΠΏΠΈΡ†Ρ†Ρ‹ Π²Π·ΡΡ‚ΡŒ 1 Ρ€Π°Π·, Ρ‚ΠΎ получится Β ΠΏΠΈΡ†Ρ†Ρ‹

Из Π·Π°ΠΊΠΎΠ½ΠΎΠ² умноТСния ΠΌΡ‹ Π·Π½Π°Π΅ΠΌ, Ρ‡Ρ‚ΠΎ Ссли ΠΌΠ½ΠΎΠΆΠΈΠΌΠΎΠ΅ ΠΈ ΠΌΠ½ΠΎΠΆΠΈΡ‚Π΅Π»ΡŒ ΠΏΠΎΠΌΠ΅Π½ΡΡ‚ΡŒ мСстами, Ρ‚ΠΎ ΠΏΡ€ΠΎΠΈΠ·Π²Π΅Π΄Π΅Π½ΠΈΠ΅ Π½Π΅ измСнится. Если Π²Ρ‹Ρ€Π°ΠΆΠ΅Π½ΠΈΠ΅Β , Π·Π°ΠΏΠΈΡΠ°Ρ‚ΡŒ ΠΊΠ°ΠΊΒ , Ρ‚ΠΎ ΠΏΡ€ΠΎΠΈΠ·Π²Π΅Π΄Π΅Π½ΠΈΠ΅ ΠΏΠΎ ΠΏΡ€Π΅ΠΆΠ½Π΅ΠΌΡƒ Π±ΡƒΠ΄Π΅Ρ‚ Ρ€Π°Π²Π½ΠΎΒ . ΠžΠΏΡΡ‚ΡŒ ΠΆΠ΅ срабатываСт ΠΏΡ€Π°Π²ΠΈΠ»ΠΎ пСрСмноТСния Ρ†Π΅Π»ΠΎΠ³ΠΎ числа ΠΈ Π΄Ρ€ΠΎΠ±ΠΈ:

Π­Ρ‚Ρƒ запись ΠΌΠΎΠΆΠ½ΠΎ ΠΏΠΎΠ½ΠΈΠΌΠ°Ρ‚ΡŒ, ΠΊΠ°ΠΊ взятиС ΠΏΠΎΠ»ΠΎΠ²ΠΈΠ½Ρ‹ ΠΎΡ‚ Π΅Π΄ΠΈΠ½ΠΈΡ†Ρ‹. К ΠΏΡ€ΠΈΠΌΠ΅Ρ€Ρƒ, Ссли имССтся 1 цСлая ΠΏΠΈΡ†Ρ†Π° ΠΈ ΠΌΡ‹ возьмСм ΠΎΡ‚ Π½Π΅Ρ‘ ΠΏΠΎΠ»ΠΎΠ²ΠΈΠ½Ρƒ, Ρ‚ΠΎ Ρƒ нас окаТСтся  пиццы:


ΠŸΡ€ΠΈΠΌΠ΅Ρ€ 2. Найти Π·Π½Π°Ρ‡Π΅Π½ΠΈΠ΅ выраТСния

Π£ΠΌΠ½ΠΎΠΆΠΈΠΌ Ρ‡ΠΈΡΠ»ΠΈΡ‚Π΅Π»ΡŒ Π΄Ρ€ΠΎΠ±ΠΈ Π½Π° 4

Π’ ΠΎΡ‚Π²Π΅Ρ‚Π΅ ΠΏΠΎΠ»ΡƒΡ‡ΠΈΠ»Π°ΡΡŒ Π½Π΅ΠΏΡ€Π°Π²ΠΈΠ»ΡŒΠ½Π°Ρ Π΄Ρ€ΠΎΠ±ΡŒ. Π’Ρ‹Π΄Π΅Π»ΠΈΠΌ Π² Π½Π΅ΠΉ Ρ†Π΅Π»ΡƒΡŽ Ρ‡Π°ΡΡ‚ΡŒ:

Π’Ρ‹Ρ€Π°ΠΆΠ΅Π½ΠΈΠ΅ Β ΠΌΠΎΠΆΠ½ΠΎ ΠΏΠΎΠ½ΠΈΠΌΠ°Ρ‚ΡŒ, ΠΊΠ°ΠΊ взятиС Π΄Π²ΡƒΡ… Ρ‡Π΅Ρ‚Π²Π΅Ρ€Ρ‚Π΅ΠΉ 4 Ρ€Π°Π·Π°. К ΠΏΡ€ΠΈΠΌΠ΅Ρ€Ρƒ, Ссли Β ΠΏΠΈΡ†Ρ†Ρ‹Β Π²Π·ΡΡ‚ΡŒ 4 Ρ€Π°Π·Π°, Ρ‚ΠΎ получится Π΄Π²Π΅ Ρ†Π΅Π»Ρ‹Π΅ ΠΏΠΈΡ†Ρ†Ρ‹

А Ссли ΠΏΠΎΠΌΠ΅Π½ΡΡ‚ΡŒ ΠΌΠ½ΠΎΠΆΠΈΠΌΠΎΠ΅ ΠΈ ΠΌΠ½ΠΎΠΆΠΈΡ‚Π΅Π»ΡŒ мСстами, Ρ‚ΠΎ ΠΏΠΎΠ»ΡƒΡ‡ΠΈΠΌ Π²Ρ‹Ρ€Π°ΠΆΠ΅Π½ΠΈΠ΅Β . Оно Ρ‚ΠΎΠΆΠ΅ Π±ΡƒΠ΄Π΅Ρ‚ Ρ€Π°Π²Π½ΠΎ 2. Π­Ρ‚ΠΎ Π²Ρ‹Ρ€Π°ΠΆΠ΅Π½ΠΈΠ΅ ΠΌΠΎΠΆΠ½ΠΎ ΠΏΠΎΠ½ΠΈΠΌΠ°Ρ‚ΡŒ, ΠΊΠ°ΠΊ взятиС Π΄Π²ΡƒΡ… ΠΏΠΈΡ†Ρ† ΠΎΡ‚ Ρ‡Π΅Ρ‚Ρ‹Ρ€Π΅Ρ… Ρ†Π΅Π»Ρ‹Ρ… ΠΏΠΈΡ†Ρ†:

Число, ΠΊΠΎΡ‚ΠΎΡ€ΠΎΠ΅ умноТаСтся Π½Π° Π΄Ρ€ΠΎΠ±ΡŒ, ΠΈ Π·Π½Π°ΠΌΠ΅Π½Π°Ρ‚Π΅Π»ΡŒ Π΄Ρ€ΠΎΠ±ΠΈ Ρ€Π°Π·Ρ€Π΅ΡˆΠ°Π΅Ρ‚ΡΡ ΡΠΎΠΊΡ€Π°Ρ‰Π°Ρ‚ΡŒ, Ссли ΠΎΠ½ΠΈ ΠΈΠΌΠ΅ΡŽΡ‚ ΠΎΠ±Ρ‰ΠΈΠΉ Π΄Π΅Π»ΠΈΡ‚Π΅Π»ΡŒ, бóльший Π΅Π΄ΠΈΠ½ΠΈΡ†Ρ‹.

НапримСр, Π²Ρ‹Ρ€Π°ΠΆΠ΅Π½ΠΈΠ΅ ΠΌΠΎΠΆΠ½ΠΎ Π²Ρ‹Ρ‡ΠΈΡΠ»ΠΈΡ‚ΡŒ двумя способами.

ΠŸΠ΅Ρ€Π²Ρ‹ΠΉ способ. Π£ΠΌΠ½ΠΎΠΆΠΈΡ‚ΡŒ число 4 Π½Π° Ρ‡ΠΈΡΠ»ΠΈΡ‚Π΅Π»ΡŒ Π΄Ρ€ΠΎΠ±ΠΈ, Π° Π·Π½Π°ΠΌΠ΅Π½Π°Ρ‚Π΅Π»ΡŒ Π΄Ρ€ΠΎΠ±ΠΈ ΠΎΡΡ‚Π°Π²ΠΈΡ‚ΡŒ Π±Π΅Π· ΠΈΠ·ΠΌΠ΅Π½Π΅Π½ΠΈΠΉ:

Π’Ρ‚ΠΎΡ€ΠΎΠΉ способ. Π£ΠΌΠ½ΠΎΠΆΠ°Π΅ΠΌΡƒΡŽ Ρ‡Π΅Ρ‚Π²Ρ‘Ρ€ΠΊΡƒ ΠΈ Ρ‡Π΅Ρ‚Π²Ρ‘Ρ€ΠΊΡƒ, Π½Π°Ρ…ΠΎΠ΄ΡΡ‰ΡƒΡŽΡΡ Π² Π·Π½Π°ΠΌΠ΅Π½Π°Ρ‚Π΅Π»Π΅ Π΄Ρ€ΠΎΠ±ΠΈ , ΠΌΠΎΠΆΠ½ΠΎ ΡΠΎΠΊΡ€Π°Ρ‚ΠΈΡ‚ΡŒ. Π‘ΠΎΠΊΡ€Π°Ρ‚ΠΈΡ‚ΡŒ эти Ρ‡Π΅Ρ‚Π²Ρ‘Ρ€ΠΊΠΈ ΠΌΠΎΠΆΠ½ΠΎ Π½Π° 4, ΠΏΠΎΡΠΊΠΎΠ»ΡŒΠΊΡƒ наибольший ΠΎΠ±Ρ‰ΠΈΠΉ Π΄Π΅Π»ΠΈΡ‚Π΅Π»ΡŒ для Π΄Π²ΡƒΡ… Ρ‡Π΅Ρ‚Π²Ρ‘Ρ€ΠΎΠΊ Π΅ΡΡ‚ΡŒ сама Ρ‡Π΅Ρ‚Π²Ρ‘Ρ€ΠΊΠ°:

ΠŸΠΎΠ»ΡƒΡ‡ΠΈΠ»ΡΡ Ρ‚ΠΎΡ‚ ΠΆΠ΅ Ρ€Π΅Π·ΡƒΠ»ΡŒΡ‚Π°Ρ‚ 3. ПослС сокращСния Ρ‡Π΅Ρ‚Π²Ρ‘Ρ€ΠΎΠΊ, Π½Π° ΠΈΡ… мСстС ΠΎΠ±Ρ€Π°Π·ΡƒΡŽΡ‚ΡΡ Π½ΠΎΠ²Ρ‹Π΅ числа: Π΄Π²Π΅ Π΅Π΄ΠΈΠ½ΠΈΡ†Ρ‹. Но ΠΏΠ΅Ρ€Π΅ΠΌΠ½ΠΎΠΆΠ΅Π½ΠΈΠ΅ Π΅Π΄ΠΈΠ½ΠΈΡ†Ρ‹ с Ρ‚Ρ€ΠΎΠΉΠΊΠΎΠΉ, ΠΈ Π΄Π°Π»Π΅Π΅ Π΄Π΅Π»Π΅Π½ΠΈΠ΅ Π½Π° Π΅Π΄ΠΈΠ½ΠΈΡ†Ρƒ Π½ΠΈΡ‡Π΅Π³ΠΎ Π½Π΅ мСняСт. ΠŸΠΎΡΡ‚ΠΎΠΌΡƒ Ρ€Π΅ΡˆΠ΅Π½ΠΈΠ΅ ΠΌΠΎΠΆΠ½ΠΎ Π·Π°ΠΏΠΈΡΠ°Ρ‚ΡŒ ΠΏΠΎΠΊΠΎΡ€ΠΎΡ‡Π΅:

Π‘ΠΎΠΊΡ€Π°Ρ‰Π΅Π½ΠΈΠ΅ ΠΌΠΎΠΆΠ΅Ρ‚ Π±Ρ‹Ρ‚ΡŒ Π²Ρ‹ΠΏΠΎΠ»Π½Π΅Π½ΠΎ Π΄Π°ΠΆΠ΅ Ρ‚ΠΎΠ³Π΄Π°, ΠΊΠΎΠ³Π΄Π° ΠΌΡ‹ Ρ€Π΅ΡˆΠΈΠ»ΠΈ Π²ΠΎΡΠΏΠΎΠ»ΡŒΠ·ΠΎΠ²Π°Ρ‚ΡŒΡΡ ΠΏΠ΅Ρ€Π²Ρ‹ΠΌ способом, Π½ΠΎ Π½Π° этапС пСрСмноТСния числа 4 ΠΈ числитСля 3 Ρ€Π΅ΡˆΠΈΠ»ΠΈ Π²ΠΎΡΠΏΠΎΠ»ΡŒΠ·ΠΎΠ²Π°Ρ‚ΡŒΡΡ сокращСниСм:

А Π²ΠΎΡ‚ ΠΊ ΠΏΡ€ΠΈΠΌΠ΅Ρ€Ρƒ Π²Ρ‹Ρ€Π°ΠΆΠ΅Π½ΠΈΠ΅ ΠΌΠΎΠΆΠ½ΠΎ Π²Ρ‹Ρ‡ΠΈΡΠ»ΠΈΡ‚ΡŒ Ρ‚ΠΎΠ»ΡŒΠΊΠΎ ΠΏΠ΅Ρ€Π²Ρ‹ΠΌ способом β€” ΡƒΠΌΠ½ΠΎΠΆΠΈΡ‚ΡŒ число 7 Π½Π° Ρ‡ΠΈΡΠ»ΠΈΡ‚Π΅Π»ΡŒ Π΄Ρ€ΠΎΠ±ΠΈ , Π° Π·Π½Π°ΠΌΠ΅Π½Π°Ρ‚Π΅Π»ΡŒ ΠΎΡΡ‚Π°Π²ΠΈΡ‚ΡŒ Π±Π΅Π· ΠΈΠ·ΠΌΠ΅Π½Π΅Π½ΠΈΠΉ:

Бвязано это с Ρ‚Π΅ΠΌ, Ρ‡Ρ‚ΠΎ число 7 ΠΈ Π·Π½Π°ΠΌΠ΅Π½Π°Ρ‚Π΅Π»ΡŒ Π΄Ρ€ΠΎΠ±ΠΈ Π½Π΅ ΠΈΠΌΠ΅ΡŽΡ‚ ΠΎΠ±Ρ‰Π΅Π³ΠΎ дСлитСля, бóльшСго Π΅Π΄ΠΈΠ½ΠΈΡ†Ρ‹, ΠΈ соотвСтствСнно Π½Π΅ ΡΠΎΠΊΡ€Π°Ρ‰Π°ΡŽΡ‚ΡΡ.

НСкоторыС ΡƒΡ‡Π΅Π½ΠΈΠΊΠΈ ΠΏΠΎ ошибкС ΡΠΎΠΊΡ€Π°Ρ‰Π°ΡŽΡ‚ ΡƒΠΌΠ½ΠΎΠΆΠ°Π΅ΠΌΠΎΠ΅ число ΠΈ Ρ‡ΠΈΡΠ»ΠΈΡ‚Π΅Π»ΡŒ Π΄Ρ€ΠΎΠ±ΠΈ. Π”Π΅Π»Π°Ρ‚ΡŒ этого нСльзя. НапримСр, ΡΠ»Π΅Π΄ΡƒΡŽΡ‰Π°Ρ запись Π½Π΅ являСтся ΠΏΡ€Π°Π²ΠΈΠ»ΡŒΠ½ΠΎΠΉ:

Π‘ΠΎΠΊΡ€Π°Ρ‰Π΅Π½ΠΈΠ΅ Π΄Ρ€ΠΎΠ±ΠΈ ΠΏΠΎΠ΄Ρ€Π°Π·ΡƒΠΌΠ΅Π²Π°Π΅Ρ‚, Ρ‡Ρ‚ΠΎ ΠΈ Ρ‡ΠΈΡΠ»ΠΈΡ‚Π΅Π»ΡŒ ΠΈ Π·Π½Π°ΠΌΠ΅Π½Π°Ρ‚Π΅Π»ΡŒ Π±ΡƒΠ΄Π΅Ρ‚ Ρ€Π°Π·Π΄Π΅Π»Ρ‘Π½ Π½Π° ΠΎΠ΄Π½ΠΎ ΠΈ Ρ‚ΠΎΠΆΠ΅ число. Π’ ситуации с Π²Ρ‹Ρ€Π°ΠΆΠ΅Π½ΠΈΠ΅ΠΌ Π΄Π΅Π»Π΅Π½ΠΈΠ΅ Π²Ρ‹ΠΏΠΎΠ»Π½Π΅Π½ΠΎ Ρ‚ΠΎΠ»ΡŒΠΊΠΎ Π² числитСлС, ΠΏΠΎΡΠΊΠΎΠ»ΡŒΠΊΡƒ Π·Π°ΠΏΠΈΡΠ°Ρ‚ΡŒ  это всё Ρ€Π°Π²Π½ΠΎ, Ρ‡Ρ‚ΠΎ Π·Π°ΠΏΠΈΡΠ°Ρ‚ΡŒ . Π’ΠΈΠ΄ΠΈΠΌ, Ρ‡Ρ‚ΠΎ Π΄Π΅Π»Π΅Π½ΠΈΠ΅ Π²Ρ‹ΠΏΠΎΠ»Π½Π΅Π½ΠΎ Ρ‚ΠΎΠ»ΡŒΠΊΠΎ Π² числитСлС, Π° Π² Π·Π½Π°ΠΌΠ΅Π½Π°Ρ‚Π΅Π»Π΅ Π½ΠΈΠΊΠ°ΠΊΠΎΠ³ΠΎ дСлСния Π½Π΅ происходит.


Π£ΠΌΠ½ΠΎΠΆΠ΅Π½ΠΈΠ΅ Π΄Ρ€ΠΎΠ±Π΅ΠΉ

Π§Ρ‚ΠΎΠ±Ρ‹ ΠΏΠ΅Ρ€Π΅ΠΌΠ½ΠΎΠΆΠΈΡ‚ΡŒ Π΄Ρ€ΠΎΠ±ΠΈ, Π½ΡƒΠΆΠ½ΠΎ ΠΏΠ΅Ρ€Π΅ΠΌΠ½ΠΎΠΆΠΈΡ‚ΡŒ ΠΈΡ… числитСли ΠΈ Π·Π½Π°ΠΌΠ΅Π½Π°Ρ‚Π΅Π»ΠΈ. Если Π² ΠΎΡ‚Π²Π΅Ρ‚Π΅ получится Π½Π΅ΠΏΡ€Π°Π²ΠΈΠ»ΡŒΠ½Π°Ρ Π΄Ρ€ΠΎΠ±ΡŒ, Π½ΡƒΠΆΠ½ΠΎ Π²Ρ‹Π΄Π΅Π»ΠΈΡ‚ΡŒ Π² Π½Π΅ΠΉ Ρ†Π΅Π»ΡƒΡŽ Ρ‡Π°ΡΡ‚ΡŒ.

ΠŸΡ€ΠΈΠΌΠ΅Ρ€ 1. Найти Π·Π½Π°Ρ‡Π΅Π½ΠΈΠ΅ выраТСния .

Π£ΠΌΠ½ΠΎΠΆΠ°Π΅ΠΌ Ρ‡ΠΈΡΠ»ΠΈΡ‚Π΅Π»ΡŒ ΠΏΠ΅Ρ€Π²ΠΎΠΉ Π΄Ρ€ΠΎΠ±ΠΈ Π½Π° Ρ‡ΠΈΡΠ»ΠΈΡ‚Π΅Π»ΡŒ Π²Ρ‚ΠΎΡ€ΠΎΠΉ Π΄Ρ€ΠΎΠ±ΠΈ, Π° Π·Π½Π°ΠΌΠ΅Π½Π°Ρ‚Π΅Π»ΡŒ ΠΏΠ΅Ρ€Π²ΠΎΠΉ Π΄Ρ€ΠΎΠ±ΠΈ Π½Π° Π·Π½Π°ΠΌΠ΅Π½Π°Ρ‚Π΅Π»ΡŒ Π²Ρ‚ΠΎΡ€ΠΎΠΉ Π΄Ρ€ΠΎΠ±ΠΈ:

ΠŸΠΎΠ»ΡƒΡ‡ΠΈΠ»ΠΈ ΠΎΡ‚Π²Π΅Ρ‚Β . Π–Π΅Π»Π°Ρ‚Π΅Π»ΡŒΠ½ΠΎ ΡΠΎΠΊΡ€Π°Ρ‚ΠΈΡ‚ΡŒ Π΄Π°Π½Π½ΡƒΡŽ Π΄Ρ€ΠΎΠ±ΡŒ. Π”Ρ€ΠΎΠ±ΡŒ ΠΌΠΎΠΆΠ½ΠΎ ΡΠΎΠΊΡ€Π°Ρ‚ΠΈΡ‚ΡŒ Π½Π° 2. Π’ΠΎΠ³Π΄Π° ΠΎΠΊΠΎΠ½Ρ‡Π°Ρ‚Π΅Π»ΡŒΠ½ΠΎΠ΅ Ρ€Π΅ΡˆΠ΅Π½ΠΈΠ΅ ΠΏΡ€ΠΈΠΌΠ΅Ρ‚ ΡΠ»Π΅Π΄ΡƒΡŽΡ‰ΠΈΠΉ Π²ΠΈΠ΄:

Π’Ρ‹Ρ€Π°ΠΆΠ΅Π½ΠΈΠ΅Β Β ΠΌΠΎΠΆΠ½ΠΎ ΠΏΠΎΠ½ΠΈΠΌΠ°Ρ‚ΡŒ, ΠΊΠ°ΠΊ взятиС  пиццы ΠΎΡ‚ ΠΏΠΎΠ»ΠΎΠ²ΠΈΠ½Ρ‹ ΠΏΠΈΡ†Ρ†Ρ‹. Допустим, Ρƒ нас Π΅ΡΡ‚ΡŒ ΠΏΠΎΠ»ΠΎΠ²ΠΈΠ½Π° ΠΏΠΈΡ†Ρ†Ρ‹:

Как Π²Π·ΡΡ‚ΡŒ ΠΎΡ‚ этой ΠΏΠΎΠ»ΠΎΠ²ΠΈΠ½Ρ‹ Π΄Π²Π΅ Ρ‚Ρ€Π΅Ρ‚ΡŒΠΈΡ…? Π‘Π½Π°Ρ‡Π°Π»Π° Π½ΡƒΠΆΠ½ΠΎ ΠΏΠΎΠ΄Π΅Π»ΠΈΡ‚ΡŒ эту ΠΏΠΎΠ»ΠΎΠ²ΠΈΠ½Ρƒ Π½Π° Ρ‚Ρ€ΠΈ Ρ€Π°Π²Π½Ρ‹Π΅ части:

И Π²Π·ΡΡ‚ΡŒ ΠΎΡ‚ этих Ρ‚Ρ€Π΅Ρ… кусочков Π΄Π²Π°:

Π£ нас получится  пиццы. ВспомнитС, ΠΊΠ°ΠΊ выглядит ΠΏΠΈΡ†Ρ†Π°, раздСлСнная Π½Π° Ρ‚Ρ€ΠΈ части:

Один кусок ΠΎΡ‚ этой ΠΏΠΈΡ†Ρ†Ρ‹ ΠΈ взятыС Π½Π°ΠΌΠΈ Π΄Π²Π° кусочка Π±ΡƒΠ΄ΡƒΡ‚ ΠΈΠΌΠ΅Ρ‚ΡŒ ΠΎΠ΄ΠΈΠ½Π°ΠΊΠΎΠ²Ρ‹Π΅ Ρ€Π°Π·ΠΌΠ΅Ρ€Ρ‹:

Π”Ρ€ΡƒΠ³ΠΈΠΌΠΈ словами, Ρ€Π΅Ρ‡ΡŒ ΠΈΠ΄Π΅Ρ‚ ΠΎΠ± ΠΎΠ΄Π½ΠΎΠΌ ΠΈ Ρ‚ΠΎΠΌ ΠΆΠ΅ Ρ€Π°Π·ΠΌΠ΅Ρ€Π΅ ΠΏΠΈΡ†Ρ†Ρ‹. ΠŸΠΎΡΡ‚ΠΎΠΌΡƒ Π·Π½Π°Ρ‡Π΅Π½ΠΈΠ΅ выраТСния  равно 


ΠŸΡ€ΠΈΠΌΠ΅Ρ€ 2. Найти Π·Π½Π°Ρ‡Π΅Π½ΠΈΠ΅ выраТСния

Π£ΠΌΠ½ΠΎΠΆΠ°Π΅ΠΌ Ρ‡ΠΈΡΠ»ΠΈΡ‚Π΅Π»ΡŒ ΠΏΠ΅Ρ€Π²ΠΎΠΉ Π΄Ρ€ΠΎΠ±ΠΈ Π½Π° Ρ‡ΠΈΡΠ»ΠΈΡ‚Π΅Π»ΡŒ Π²Ρ‚ΠΎΡ€ΠΎΠΉ Π΄Ρ€ΠΎΠ±ΠΈ, Π° Π·Π½Π°ΠΌΠ΅Π½Π°Ρ‚Π΅Π»ΡŒ ΠΏΠ΅Ρ€Π²ΠΎΠΉ Π΄Ρ€ΠΎΠ±ΠΈ Π½Π° Π·Π½Π°ΠΌΠ΅Π½Π°Ρ‚Π΅Π»ΡŒ Π²Ρ‚ΠΎΡ€ΠΎΠΉ Π΄Ρ€ΠΎΠ±ΠΈ:

Π’ ΠΎΡ‚Π²Π΅Ρ‚Π΅ ΠΏΠΎΠ»ΡƒΡ‡ΠΈΠ»Π°ΡΡŒ Π½Π΅ΠΏΡ€Π°Π²ΠΈΠ»ΡŒΠ½Π°Ρ Π΄Ρ€ΠΎΠ±ΡŒ. Π’Ρ‹Π΄Π΅Π»ΠΈΠΌ Π² Π½Π΅ΠΉ Ρ†Π΅Π»ΡƒΡŽ Ρ‡Π°ΡΡ‚ΡŒ:


ΠŸΡ€ΠΈΠΌΠ΅Ρ€ 3. Найти Π·Π½Π°Ρ‡Π΅Π½ΠΈΠ΅ выраТСния 

Π£ΠΌΠ½ΠΎΠΆΠ°Π΅ΠΌ Ρ‡ΠΈΡΠ»ΠΈΡ‚Π΅Π»ΡŒ ΠΏΠ΅Ρ€Π²ΠΎΠΉ Π΄Ρ€ΠΎΠ±ΠΈ Π½Π° Ρ‡ΠΈΡΠ»ΠΈΡ‚Π΅Π»ΡŒ Π²Ρ‚ΠΎΡ€ΠΎΠΉ Π΄Ρ€ΠΎΠ±ΠΈ, Π° Π·Π½Π°ΠΌΠ΅Π½Π°Ρ‚Π΅Π»ΡŒ ΠΏΠ΅Ρ€Π²ΠΎΠΉ Π΄Ρ€ΠΎΠ±ΠΈ Π½Π° Π·Π½Π°ΠΌΠ΅Π½Π°Ρ‚Π΅Π»ΡŒ Π²Ρ‚ΠΎΡ€ΠΎΠΉ Π΄Ρ€ΠΎΠ±ΠΈ:

Π’ ΠΎΡ‚Π²Π΅Ρ‚Π΅ ΠΏΠΎΠ»ΡƒΡ‡ΠΈΠ»Π°ΡΡŒ ΠΏΡ€Π°Π²ΠΈΠ»ΡŒΠ½Π°Ρ Π΄Ρ€ΠΎΠ±ΡŒ, Π½ΠΎ Π±ΡƒΠ΄Π΅Ρ‚ Ρ…ΠΎΡ€ΠΎΡˆΠΎ, Ссли Π΅Ρ‘ ΡΠΎΠΊΡ€Π°Ρ‚ΠΈΡ‚ΡŒ. Π§Ρ‚ΠΎΠ±Ρ‹ ΡΠΎΠΊΡ€Π°Ρ‚ΠΈΡ‚ΡŒ эту Π΄Ρ€ΠΎΠ±ΡŒ, Π½ΡƒΠΆΠ½ΠΎ Ρ‡ΠΈΡΠ»ΠΈΡ‚Π΅Π»ΡŒ ΠΈ Π·Π½Π°ΠΌΠ΅Π½Π°Ρ‚Π΅Π»ΡŒ Π΄Π°Π½Π½ΠΎΠΉ Π΄Ρ€ΠΎΠ±ΠΈ Ρ€Π°Π·Π΄Π΅Π»ΠΈΡ‚ΡŒ Π½Π° наибольший ΠΎΠ±Ρ‰ΠΈΠΉ Π΄Π΅Π»ΠΈΡ‚Π΅Π»ΡŒ (ΠΠžΠ”) чисСл 105 ΠΈ 450.

Π˜Ρ‚Π°ΠΊ, Π½Π°ΠΉΠ΄Ρ‘ΠΌ ΠΠžΠ” чисСл 105 ΠΈ 450:

Π’Π΅ΠΏΠ΅Ρ€ΡŒ Π΄Π΅Π»ΠΈΠΌ Ρ‡ΠΈΡΠ»ΠΈΡ‚Π΅Π»ΡŒ ΠΈ Π·Π½Π°ΠΌΠ΅Π½Π°Ρ‚Π΅Π»ΡŒ нашСго ΠΎΡ‚Π²Π΅Ρ‚Π° Π½Π° ΠΠžΠ”, ΠΊΠΎΡ‚ΠΎΡ€Ρ‹ΠΉ ΠΌΡ‹ сСйчас нашли, Ρ‚ΠΎ Π΅ΡΡ‚ΡŒ Π½Π° 15


ΠŸΡ€Π΅Π΄ΡΡ‚Π°Π²Π»Π΅Π½ΠΈΠ΅ Ρ†Π΅Π»ΠΎΠ³ΠΎ числа Π² Π²ΠΈΠ΄Π΅ Π΄Ρ€ΠΎΠ±ΠΈ

Π›ΡŽΠ±ΠΎΠ΅ Ρ†Π΅Π»ΠΎΠ΅ число ΠΌΠΎΠΆΠ½ΠΎ ΠΏΡ€Π΅Π΄ΡΡ‚Π°Π²ΠΈΡ‚ΡŒ Π² Π²ΠΈΠ΄Π΅ Π΄Ρ€ΠΎΠ±ΠΈ. НапримСр, число 5 ΠΌΠΎΠΆΠ½ΠΎ ΠΏΡ€Π΅Π΄ΡΡ‚Π°Π²ΠΈΡ‚ΡŒ ΠΊΠ°ΠΊΒ  . ΠžΡ‚ этого пятёрка своСго значСния Π½Π΅ помСняСт, ΠΏΠΎΡΠΊΠΎΠ»ΡŒΠΊΡƒ Π²Ρ‹Ρ€Π°ΠΆΠ΅Π½ΠΈΠ΅Β  Β  ΠΎΠ·Π½Π°Ρ‡Π°Π΅Ρ‚ «число ΠΏΡΡ‚ΡŒ Ρ€Π°Π·Π΄Π΅Π»ΠΈΡ‚ΡŒ Π½Π° Π΅Π΄ΠΈΠ½ΠΈΡ†ΡƒΒ», Π° это, ΠΊΠ°ΠΊ извСстно Ρ€Π°Π²Π½ΠΎ пятёркС:


ΠžΠ±Ρ€Π°Ρ‚Π½Ρ‹Π΅ числа

БСйчас ΠΌΡ‹ познакомимся с ΠΎΡ‡Π΅Π½ΡŒ интСрСсной Ρ‚Π΅ΠΌΠΎΠΉ Π² ΠΌΠ°Ρ‚Π΅ΠΌΠ°Ρ‚ΠΈΠΊΠ΅. Она называСтся Β«ΠΎΠ±Ρ€Π°Ρ‚Π½Ρ‹Π΅ числа».

ΠžΠΏΡ€Π΅Π΄Π΅Π»Π΅Π½ΠΈΠ΅. ΠžΠ±Ρ€Π°Ρ‚Π½Ρ‹ΠΌ ΠΊ числу a называСтся число, ΠΊΠΎΡ‚ΠΎΡ€ΠΎΠ΅ ΠΏΡ€ΠΈ ΡƒΠΌΠ½ΠΎΠΆΠ΅Π½ΠΈΠΈ Π½Π° a Π΄Π°Ρ‘Ρ‚ Π΅Π΄ΠΈΠ½ΠΈΡ†Ρƒ.

Π”Π°Π²Π°ΠΉΡ‚Π΅ подставим Π² это ΠΎΠΏΡ€Π΅Π΄Π΅Π»Π΅Π½ΠΈΠ΅ вмСсто ΠΏΠ΅Ρ€Π΅ΠΌΠ΅Π½Π½ΠΎΠΉ a число 5 ΠΈ ΠΏΠΎΠΏΡ€ΠΎΠ±ΡƒΠ΅ΠΌ ΠΏΡ€ΠΎΡ‡ΠΈΡ‚Π°Ρ‚ΡŒ ΠΎΠΏΡ€Π΅Π΄Π΅Π»Π΅Π½ΠΈΠ΅:

ΠžΠ±Ρ€Π°Ρ‚Π½Ρ‹ΠΌ ΠΊ числу 5 называСтся число, ΠΊΠΎΡ‚ΠΎΡ€ΠΎΠ΅ ΠΏΡ€ΠΈ ΡƒΠΌΠ½ΠΎΠΆΠ΅Π½ΠΈΠΈ Π½Π° 5 Π΄Π°Ρ‘Ρ‚ Π΅Π΄ΠΈΠ½ΠΈΡ†Ρƒ.

МоТно Π»ΠΈ Π½Π°ΠΉΡ‚ΠΈ Ρ‚Π°ΠΊΠΎΠ΅ число, ΠΊΠΎΡ‚ΠΎΡ€ΠΎΠ΅ ΠΏΡ€ΠΈ ΡƒΠΌΠ½ΠΎΠΆΠ΅Π½ΠΈΠΈ Π½Π° 5, Π΄Π°Ρ‘Ρ‚ Π΅Π΄ΠΈΠ½ΠΈΡ†Ρƒ? ΠžΠΊΠ°Π·Ρ‹Π²Π°Π΅Ρ‚ΡΡ ΠΌΠΎΠΆΠ½ΠΎ. ΠŸΡ€Π΅Π΄ΡΡ‚Π°Π²ΠΈΠΌ пятёрку Π² Π²ΠΈΠ΄Π΅ Π΄Ρ€ΠΎΠ±ΠΈ:

Π—Π°Ρ‚Π΅ΠΌ ΡƒΠΌΠ½ΠΎΠΆΠΈΡ‚ΡŒ эту Π΄Ρ€ΠΎΠ±ΡŒ Π½Π° саму сСбя, Ρ‚ΠΎΠ»ΡŒΠΊΠΎ помСняСм мСстами Ρ‡ΠΈΡΠ»ΠΈΡ‚Π΅Π»ΡŒ ΠΈ Π·Π½Π°ΠΌΠ΅Π½Π°Ρ‚Π΅Π»ΡŒ. Π”Ρ€ΡƒΠ³ΠΈΠΌΠΈ словами, ΡƒΠΌΠ½ΠΎΠΆΠΈΠΌ Π΄Ρ€ΠΎΠ±ΡŒ Π½Π° саму сСбя, Ρ‚ΠΎΠ»ΡŒΠΊΠΎ ΠΏΠ΅Ρ€Π΅Π²Ρ‘Ρ€Π½ΡƒΡ‚ΡƒΡŽ:

Π§Ρ‚ΠΎ получится Π² Ρ€Π΅Π·ΡƒΠ»ΡŒΡ‚Π°Ρ‚Π΅ этого? Если ΠΌΡ‹ ΠΏΡ€ΠΎΠ΄ΠΎΠ»ΠΆΠΈΠΌ Ρ€Π΅ΡˆΠ°Ρ‚ΡŒ этот ΠΏΡ€ΠΈΠΌΠ΅Ρ€, Ρ‚ΠΎ ΠΏΠΎΠ»ΡƒΡ‡ΠΈΠΌ Π΅Π΄ΠΈΠ½ΠΈΡ†Ρƒ:

Π—Π½Π°Ρ‡ΠΈΡ‚ ΠΎΠ±Ρ€Π°Ρ‚Π½Ρ‹ΠΌ ΠΊ числу 5, являСтся число , ΠΏΠΎΡΠΊΠΎΠ»ΡŒΠΊΡƒ ΠΏΡ€ΠΈ ΡƒΠΌΠ½ΠΎΠΆΠ΅Π½ΠΈΠΈ 5 Π½Π° получаСтся Π΅Π΄ΠΈΠ½ΠΈΡ†Π°.

ΠžΠ±Ρ€Π°Ρ‚Π½ΠΎΠ΅ число ΠΌΠΎΠΆΠ½ΠΎ Π½Π°ΠΉΡ‚ΠΈ Ρ‚Π°ΠΊΠΆΠ΅ для любого Π΄Ρ€ΡƒΠ³ΠΎΠ³ΠΎ Ρ†Π΅Π»ΠΎΠ³ΠΎ числа.

ΠŸΡ€ΠΈΠΌΠ΅Ρ€Ρ‹:

  • ΠΎΠ±Ρ€Π°Ρ‚Π½Ρ‹ΠΌ числа 2 являСтся Π΄Ρ€ΠΎΠ±ΡŒ
  • ΠΎΠ±Ρ€Π°Ρ‚Π½Ρ‹ΠΌΒ  числа 3 являСтся Π΄Ρ€ΠΎΠ±ΡŒ
  • ΠΎΠ±Ρ€Π°Ρ‚Π½Ρ‹ΠΌ числа 4 являСтся Π΄Ρ€ΠΎΠ±ΡŒ

Найти ΠΎΠ±Ρ€Π°Ρ‚Π½ΠΎΠ΅ число ΠΌΠΎΠΆΠ½ΠΎ Ρ‚Π°ΠΊΠΆΠ΅ для любой Π΄Ρ€ΡƒΠ³ΠΎΠΉ Π΄Ρ€ΠΎΠ±ΠΈ. Для этого достаточно ΠΏΠ΅Ρ€Π΅Π²Π΅Ρ€Π½ΡƒΡ‚ΡŒ Π΅Ρ‘.

ΠŸΡ€ΠΈΠΌΠ΅Ρ€Ρ‹:


Π”Π΅Π»Π΅Π½ΠΈΠ΅ Π΄Ρ€ΠΎΠ±ΠΈ Π½Π° число

Допустим, Ρƒ нас имССтся ΠΏΠΎΠ»ΠΎΠ²ΠΈΠ½Π° ΠΏΠΈΡ†Ρ†Ρ‹:

Π Π°Π·Π΄Π΅Π»ΠΈΠΌ Π΅Ρ‘ ΠΏΠΎΡ€ΠΎΠ²Π½Ρƒ Π½Π° Π΄Π²ΠΎΠΈΡ…. Бколько ΠΏΠΈΡ†Ρ†Ρ‹ достанСтся ΠΊΠ°ΠΆΠ΄ΠΎΠΌΡƒ?

Π’ΠΈΠ΄Π½ΠΎ, Ρ‡Ρ‚ΠΎ послС раздСлСния ΠΏΠΎΠ»ΠΎΠ²ΠΈΠ½Ρ‹ ΠΏΠΈΡ†Ρ†Ρ‹ ΠΏΠΎΠ»ΡƒΡ‡ΠΈΠ»ΠΎΡΡŒ Π΄Π²Π° Ρ€Π°Π²Π½Ρ‹Ρ… кусочка, ΠΊΠ°ΠΆΠ΄Ρ‹ΠΉ ΠΈΠ· ΠΊΠΎΡ‚ΠΎΡ€Ρ‹Ρ… составляСт  пиццы. Π—Π½Π°Ρ‡ΠΈΡ‚ ΠΊΠ°ΠΆΠ΄ΠΎΠΌΡƒ достанСтся ΠΏΠΎΒ Β ΠΏΠΈΡ†Ρ†Ρ‹.

Π”Π΅Π»Π΅Π½ΠΈΠ΅ Π΄Ρ€ΠΎΠ±Π΅ΠΉ выполняСтся с ΠΏΠΎΠΌΠΎΡ‰ΡŒΡŽ ΠΎΠ±Ρ€Π°Ρ‚Π½Ρ‹Ρ… чисСл. ΠžΠ±Ρ€Π°Ρ‚Π½Ρ‹Π΅ числа ΠΏΠΎΠ·Π²ΠΎΠ»ΡΡŽΡ‚ Π·Π°ΠΌΠ΅Π½ΠΈΡ‚ΡŒ Π΄Π΅Π»Π΅Π½ΠΈΠ΅ ΡƒΠΌΠ½ΠΎΠΆΠ΅Π½ΠΈΠ΅ΠΌ.

Π§Ρ‚ΠΎΠ±Ρ‹ Ρ€Π°Π·Π΄Π΅Π»ΠΈΡ‚ΡŒ Π΄Ρ€ΠΎΠ±ΡŒ Π½Π° число, Π½ΡƒΠΆΠ½ΠΎ эту Π΄Ρ€ΠΎΠ±ΡŒ ΡƒΠΌΠ½ΠΎΠΆΠΈΡ‚ΡŒ Π½Π° число, ΠΎΠ±Ρ€Π°Ρ‚Π½ΠΎΠ΅ Π΄Π΅Π»ΠΈΡ‚Π΅Π»ΡŽ.

ΠŸΠΎΠ»ΡŒΠ·ΡƒΡΡΡŒ этим ΠΏΡ€Π°Π²ΠΈΠ»ΠΎΠΌ, запишСм Π΄Π΅Π»Π΅Π½ΠΈΠ΅ нашСй ΠΏΠΎΠ»ΠΎΠ²ΠΈΠ½Ρ‹ ΠΏΠΈΡ†Ρ†Ρ‹ Π½Π° Π΄Π²Π΅ части.

Π˜Ρ‚Π°ΠΊ, трСбуСтся Ρ€Π°Π·Π΄Π΅Π»ΠΈΡ‚ΡŒ Π΄Ρ€ΠΎΠ±ΡŒΒ Β Π½Π° число 2. Π—Π΄Π΅ΡΡŒ Π΄Π΅Π»ΠΈΠΌΡ‹ΠΌ являСтся Π΄Ρ€ΠΎΠ±ΡŒΒ , Π° Π΄Π΅Π»ΠΈΡ‚Π΅Π»Π΅ΠΌ число 2.

Π§Ρ‚ΠΎΠ±Ρ‹ Ρ€Π°Π·Π΄Π΅Π»ΠΈΡ‚ΡŒ Π΄Ρ€ΠΎΠ±ΡŒΒ Β Π½Π° число 2, Π½ΡƒΠΆΠ½ΠΎ эту Π΄Ρ€ΠΎΠ±ΡŒ ΡƒΠΌΠ½ΠΎΠΆΠΈΡ‚ΡŒ Π½Π° число, ΠΎΠ±Ρ€Π°Ρ‚Π½ΠΎΠ΅ Π΄Π΅Π»ΠΈΡ‚Π΅Π»ΡŽ 2. ΠžΠ±Ρ€Π°Ρ‚Π½ΠΎΠ΅ Π΄Π΅Π»ΠΈΡ‚Π΅Π»ΡŽ 2 это Π΄Ρ€ΠΎΠ±ΡŒΒ . Π—Π½Π°Ρ‡ΠΈΡ‚ Π½ΡƒΠΆΠ½ΠΎ ΡƒΠΌΠ½ΠΎΠΆΠΈΡ‚ΡŒΒ Β Π½Π°Β 

ΠŸΠΎΠ»ΡƒΡ‡ΠΈΠ»ΠΈ ΠΎΡ‚Π²Π΅Ρ‚Β . Π—Π½Π°Ρ‡ΠΈΡ‚ ΠΏΡ€ΠΈ Π΄Π΅Π»Π΅Π½ΠΈΠΈ ΠΏΠΎΠ»ΠΎΠ²ΠΈΠ½Ρ‹ Π½Π° Π΄Π²Π΅ части получаСтся Ρ‡Π΅Ρ‚Π²Π΅Ρ€Ρ‚ΡŒ.

ΠŸΠΎΠΏΡ€ΠΎΠ±ΡƒΠ΅ΠΌ ΠΏΠΎΠ½ΡΡ‚ΡŒ ΠΌΠ΅Ρ…Π°Π½ΠΈΠ·ΠΌ этого ΠΏΡ€Π°Π²ΠΈΠ»Π°. Для этого рассмотрим ΡΠ»Π΅Π΄ΡƒΡŽΡ‰ΠΈΠΉ ΠΏΡ€ΠΎΡΡ‚Π΅ΠΉΡˆΠΈΠΉ ΠΏΡ€ΠΈΠΌΠ΅Ρ€. ΠŸΡƒΡΡ‚ΡŒ Ρƒ нас имССтся ΠΎΠ΄Π½Π° цСлая ΠΏΠΈΡ†Ρ†Π°:

Π£ΠΌΠ½ΠΎΠΆΠΈΠΌ Π΅Ρ‘ Π½Π° 2. Π’ΠΎ Π΅ΡΡ‚ΡŒ ΠΏΠΎΠ²Ρ‚ΠΎΡ€ΠΈΠΌ Π΅Ρ‘ Π΄Π²Π° Ρ€Π°Π·Π° (ΠΈΠ»ΠΈ Π²ΠΎΠ·ΡŒΠΌΡ‘ΠΌ Π΄Π²Π° Ρ€Π°Π·Π°). Π’ Ρ€Π΅Π·ΡƒΠ»ΡŒΡ‚Π°Ρ‚Π΅ Π±ΡƒΠ΄Π΅ΠΌ ΠΈΠΌΠ΅Ρ‚ΡŒ Π΄Π²Π΅ ΠΏΠΈΡ†Ρ†Ρ‹:

Π’Π΅ΠΏΠ΅Ρ€ΡŒ угостим этими ΠΏΠΈΡ†Ρ†Π°ΠΌΠΈ Π΄Π²ΠΎΠΈΡ… Π΄Ρ€ΡƒΠ·Π΅ΠΉ. Π’ΠΎ Π΅ΡΡ‚ΡŒ Ρ€Π°Π·Π΄Π΅Π»ΠΈΠΌ Π΄Π²Π΅ ΠΏΠΈΡ†Ρ†Ρ‹ Π½Π° 2. Π’ΠΎΠ³Π΄Π° ΠΊΠ°ΠΆΠ΄ΠΎΠΌΡƒ достанСтся ΠΏΠΎ ΠΎΠ΄Π½ΠΎΠΉ ΠΏΠΈΡ†Ρ†Π΅:

Π Π°Π·Π΄Π΅Π»ΠΈΡ‚ΡŒ Π΄Π²Π΅ ΠΏΠΈΡ†Ρ†Ρ‹ Π½Π° 2 это всё Ρ€Π°Π²Π½ΠΎ, Ρ‡Ρ‚ΠΎ Π²Π·ΡΡ‚ΡŒ ΠΏΠΎΠ»ΠΎΠ²ΠΈΠ½Ρƒ ΠΎΡ‚ этих ΠΏΠΈΡ†Ρ†, Ρ‚ΠΎ Π΅ΡΡ‚ΡŒ ΡƒΠΌΠ½ΠΎΠΆΠΈΡ‚ΡŒ число 2 Π½Π° Π΄Ρ€ΠΎΠ±ΡŒΒ 

Π’ ΠΎΠ±ΠΎΠΈΡ… случаях получился ΠΎΠ΄ΠΈΠ½ ΠΈ Ρ‚ΠΎΡ‚ ΠΆΠ΅ Ρ€Π΅Π·ΡƒΠ»ΡŒΡ‚Π°Ρ‚.

Π’ΠΎΠΆΠ΅ самоС происходило, ΠΊΠΎΠ³Π΄Π° ΠΌΡ‹ Π΄Π΅Π»ΠΈΠ»ΠΈ ΠΏΠΎΠ»ΠΎΠ²ΠΈΠ½Ρƒ ΠΏΠΈΡ†Ρ†Ρ‹ Π½Π° Π΄Π²Π΅ части. Π§Ρ‚ΠΎΠ±Ρ‹ Ρ€Π°Π·Π΄Π΅Π»ΠΈΡ‚ΡŒΒ Β Π½Π° 2, ΠΌΡ‹ ΡƒΠΌΠ½ΠΎΠΆΠΈΠ»ΠΈ эту Π΄Ρ€ΠΎΠ±ΡŒ Π½Π° число, ΠΎΠ±Ρ€Π°Ρ‚Π½ΠΎΠ΅ Π΄Π΅Π»ΠΈΡ‚Π΅Π»ΡŽ 2. А ΠΎΠ±Ρ€Π°Ρ‚Π½ΠΎΠ΅ Π΄Π΅Π»ΠΈΡ‚Π΅Π»ΡŽ 2 это Π΄Ρ€ΠΎΠ±ΡŒΒ 


ΠŸΡ€ΠΈΠΌΠ΅Ρ€ 2. Найти Π·Π½Π°Ρ‡Π΅Π½ΠΈΠ΅ выраТСния 

Π£ΠΌΠ½ΠΎΠΆΠΈΠΌ ΠΏΠ΅Ρ€Π²ΡƒΡŽ Π΄Ρ€ΠΎΠ±ΡŒ Π½Π° число, ΠΎΠ±Ρ€Π°Ρ‚Π½ΠΎΠ΅ Π΄Π΅Π»ΠΈΡ‚Π΅Π»ΡŽ:

Допустим, имССтся Ρ‡Π΅Ρ‚Π²Π΅Ρ€Ρ‚ΡŒ ΠΏΠΈΡ†Ρ†Ρ‹ ΠΈ Π½ΡƒΠΆΠ½ΠΎ Ρ€Π°Π·Π΄Π΅Π»ΠΈΡ‚ΡŒ Π΅Ρ‘ Π½Π° Π΄Π²ΠΎΠΈΡ…:

Если Ρ€Π°Π·Π΄Π΅Π»ΠΈΡ‚ΡŒ эту Ρ‡Π΅Ρ‚Π²Π΅Ρ€Ρ‚ΡŒ Π½Π° Π΄Π²Π΅ части, Ρ‚ΠΎ каТдая ΠΏΠΎΠ»ΡƒΡ‡ΠΈΠ²ΡˆΠ°ΡΡΡ Ρ‡Π°ΡΡ‚ΡŒ Π±ΡƒΠ΄Π΅Ρ‚ ΠΎΠ΄Π½ΠΎΠΉ восьмой Ρ‡Π°ΡΡ‚ΡŒΡŽ Ρ†Π΅Π»ΠΎΠΉ ΠΏΠΈΡ†Ρ†Ρ‹:


Π—Π°ΠΌΠ΅Π½ΡΡ‚ΡŒ Π΄Π΅Π»Π΅Π½ΠΈΠ΅ ΡƒΠΌΠ½ΠΎΠΆΠ΅Π½ΠΈΠ΅ΠΌ ΠΌΠΎΠΆΠ½ΠΎ Π½Π΅ Ρ‚ΠΎΠ»ΡŒΠΊΠΎ ΠΏΡ€ΠΈ Ρ€Π°Π±ΠΎΡ‚Π΅ с дробями, Π½ΠΎ ΠΈ с ΠΎΠ±Ρ‹Ρ‡Π½Ρ‹ΠΌΠΈ числами. НапримСр, всС ΠΌΡ‹ Π·Π½Π°Π΅ΠΌ, Ρ‡Ρ‚ΠΎ 10 Ρ€Π°Π·Π΄Π΅Π»ΠΈΡ‚ΡŒ Π½Π° 2 Π±ΡƒΠ΄Π΅Ρ‚ 5

10 : 2 = 5

Π—Π°ΠΌΠ΅Π½ΠΈΠΌ Π² этом ΠΏΡ€ΠΈΠΌΠ΅Ρ€Π΅ Π΄Π΅Π»Π΅Π½ΠΈΠ΅ ΡƒΠΌΠ½ΠΎΠΆΠ΅Π½ΠΈΠ΅ΠΌ. Π§Ρ‚ΠΎΠ±Ρ‹ Ρ€Π°Π·Π΄Π΅Π»ΠΈΡ‚ΡŒ число 10 Π½Π° число 2, ΠΌΠΎΠΆΠ½ΠΎ ΡƒΠΌΠ½ΠΎΠΆΠΈΡ‚ΡŒ число 10 Π½Π° число, ΠΎΠ±Ρ€Π°Ρ‚Π½ΠΎΠ΅ числу 2. А ΠΎΠ±Ρ€Π°Ρ‚Π½ΠΎΠ΅ числу 2 это Π΄Ρ€ΠΎΠ±ΡŒΒ 

Как Π²ΠΈΠ΄Π½ΠΎ Ρ€Π΅Π·ΡƒΠ»ΡŒΡ‚Π°Ρ‚ Π½Π΅ измСнился. ΠœΡ‹ снова ΠΏΠΎΠ»ΡƒΡ‡ΠΈΠ»ΠΈ ΠΎΡ‚Π²Π΅Ρ‚ 5.

МоТно ΡΠ΄Π΅Π»Π°Ρ‚ΡŒ Π²Ρ‹Π²ΠΎΠ΄, Ρ‡Ρ‚ΠΎ Π΄Π΅Π»Π΅Π½ΠΈΠ΅ ΠΌΠΎΠΆΠ½ΠΎ Π·Π°ΠΌΠ΅Π½ΡΡ‚ΡŒ ΡƒΠΌΠ½ΠΎΠΆΠ΅Π½ΠΈΠ΅ΠΌ ΠΏΡ€ΠΈ условии, Ρ‡Ρ‚ΠΎ вмСсто дСлитСля Π±ΡƒΠ΄Π΅Ρ‚ подставлСно ΠΎΠ±Ρ€Π°Ρ‚Π½ΠΎΠ΅ Π΅ΠΌΡƒ число.

ΠŸΡ€ΠΈΠΌΠ΅Ρ€ 3. Найти Π·Π½Π°Ρ‡Π΅Π½ΠΈΠ΅ выраТСния

Π£ΠΌΠ½ΠΎΠΆΠΈΠΌ ΠΏΠ΅Ρ€Π²ΡƒΡŽ Π΄Ρ€ΠΎΠ±ΡŒ Π½Π° число, ΠΎΠ±Ρ€Π°Ρ‚Π½ΠΎΠ΅ Π΄Π΅Π»ΠΈΡ‚Π΅Π»ΡŽ. ΠžΠ±Ρ€Π°Ρ‚Π½ΠΎΠ΅ Π΄Π΅Π»ΠΈΡ‚Π΅Π»ΡŽ число это Π΄Ρ€ΠΎΠ±ΡŒΒ 

Допустим, имСлось ΠΏΠΈΡ†Ρ†Ρ‹:

Как Ρ€Π°Π·Π΄Π΅Π»ΠΈΡ‚ΡŒ Ρ‚Π°ΠΊΡƒΡŽ ΠΏΠΈΡ†Ρ†Ρƒ Π½Π° ΡˆΠ΅ΡΡ‚Π΅Ρ€Ρ‹Ρ…? Если ΠΊΠ°ΠΆΠ΄Ρ‹ΠΉ ΠΈΠ· Ρ‚Ρ€Π΅Ρ… кусков Ρ€Π°Π·Π΄Π΅Π»ΠΈΡ‚ΡŒ ΠΏΠΎΠΏΠΎΠ»Π°ΠΌ, Ρ‚ΠΎ ΠΌΠΎΠΆΠ½ΠΎ ΠΏΠΎΠ»ΡƒΡ‡ΠΈΡ‚ΡŒ 6 Ρ€Π°Π²Π½Ρ‹Ρ… кусков

Π­Ρ‚ΠΈ ΡˆΠ΅ΡΡ‚ΡŒ кусков ΡΠ²Π»ΡΡŽΡ‚ΡΡ ΡˆΠ΅ΡΡ‚ΡŒΡŽ кусками ΠΈΠ· Π΄Π²Π΅Π½Π°Π΄Ρ†Π°Ρ‚ΠΈ. А ΠΎΠ΄ΠΈΠ½ ΠΈΠ· этих кусков составляСт . ΠŸΠΎΡΡ‚ΠΎΠΌΡƒ ΠΏΡ€ΠΈ Π΄Π΅Π»Π΅Π½ΠΈΠΈΒ Β Π½Π° 6 получаСтся 


Π”Π΅Π»Π΅Π½ΠΈΠ΅ числа Π½Π° Π΄Ρ€ΠΎΠ±ΡŒ

ΠŸΡ€Π°Π²ΠΈΠ»ΠΎ дСлСния числа Π½Π° Π΄Ρ€ΠΎΠ±ΡŒ Ρ‚Π°ΠΊΠΎΠ΅ ΠΆΠ΅, ΠΊΠ°ΠΊ ΠΈ ΠΏΡ€Π°Π²ΠΈΠ»ΠΎ дСлСния Π΄Ρ€ΠΎΠ±ΠΈ Π½Π° число.

Π§Ρ‚ΠΎΠ±Ρ‹ Ρ€Π°Π·Π΄Π΅Π»ΠΈΡ‚ΡŒ число Π½Π° Π΄Ρ€ΠΎΠ±ΡŒ, Π½ΡƒΠΆΠ½ΠΎ ΡƒΠΌΠ½ΠΎΠΆΠΈΡ‚ΡŒ это число Π½Π° Π΄Ρ€ΠΎΠ±ΡŒ, ΠΎΠ±Ρ€Π°Ρ‚Π½ΡƒΡŽ Π΄Π΅Π»ΠΈΡ‚Π΅Π»ΡŽ.

НапримСр, Ρ€Π°Π·Π΄Π΅Π»ΠΈΠΌ число 1 Π½Π° .

Π§Ρ‚ΠΎΠ±Ρ‹ Ρ€Π°Π·Π΄Π΅Π»ΠΈΡ‚ΡŒ число 1 Π½Π° , Π½ΡƒΠΆΠ½ΠΎ это число 1 ΡƒΠΌΠ½ΠΎΠΆΠΈΡ‚ΡŒ Π½Π° Π΄Ρ€ΠΎΠ±ΡŒ, ΠΎΠ±Ρ€Π°Ρ‚Π½ΡƒΡŽ Π΄Ρ€ΠΎΠ±ΠΈΒ . А обратная дроби  это Π΄Ρ€ΠΎΠ±ΡŒΒ 

Π’Ρ‹Ρ€Π°ΠΆΠ΅Π½ΠΈΠ΅Β  ΠΌΠΎΠΆΠ½ΠΎ ΠΏΠΎΠ½ΠΈΠΌΠ°Ρ‚ΡŒ, ΠΊΠ°ΠΊ ΠΎΠΏΡ€Π΅Π΄Π΅Π»Π΅Π½ΠΈΠ΅ количСства ΠΏΠΎΠ»ΠΎΠ²ΠΈΠ½Β Π² ΠΎΠ΄Π½ΠΎΠΉ Ρ†Π΅Π»ΠΎΠΉ ΠΏΠΈΡ†Ρ†Π΅. Допустим, имССтся ΠΎΠ΄Π½Π° цСлая ΠΏΠΈΡ†Ρ†Π°:

Если Π·Π°Π΄Π°Π΄ΠΈΠΌ вопрос «сколько Ρ€Π°Π· ΠΏΠΎΠ»ΠΎΠ²ΠΈΠ½Π° содСрТится Π² этой ΠΏΠΈΡ†Ρ†Π΅Β», Ρ‚ΠΎ ΠΎΡ‚Π²Π΅Ρ‚ΠΎΠΌ Π±ΡƒΠ΄Π΅Ρ‚ 2. Π”Π΅ΠΉΡΡ‚Π²ΠΈΡ‚Π΅Π»ΡŒΠ½ΠΎ, ΠΏΠΎΠ»ΠΎΠ²ΠΈΠ½Π° содСрТится Π² ΠΎΠ΄Π½ΠΎΠΉ Ρ†Π΅Π»ΠΎΠΉ ΠΏΠΈΡ†Ρ†Π΅ Π΄Π²Π° Ρ€Π°Π·Π°

Β 


ΠŸΡ€ΠΈΠΌΠ΅Ρ€ 2. Найти Π·Π½Π°Ρ‡Π΅Π½ΠΈΠ΅ Π²Ρ‹Ρ€Π°ΠΆΠ΅Π½ΠΈΠ΅Β 

Π£ΠΌΠ½ΠΎΠΆΠΈΠΌ число 2 Π½Π° Π΄Ρ€ΠΎΠ±ΡŒ, ΠΎΠ±Ρ€Π°Ρ‚Π½ΡƒΡŽ Π΄Π΅Π»ΠΈΡ‚Π΅Π»ΡŽ. А обратная Π΄Π΅Π»ΠΈΡ‚Π΅Π»ΡŽ Π΄Ρ€ΠΎΠ±ΡŒ это Π΄Ρ€ΠΎΠ±ΡŒΒ 

Допустим, Ρƒ нас ΠΈΠΌΠ΅ΡŽΡ‚ΡΡ Π΄Π²Π΅ Ρ†Π΅Π»Ρ‹Π΅ ΠΏΠΈΡ†Ρ†Ρ‹:

Если Π·Π°Π΄Π°Π΄ΠΈΠΌ вопрос «сколько Ρ€Π°Π· ΠΏΠΎΠ»ΠΎΠ²ΠΈΠ½Π° содСрТится Π² Π΄Π²ΡƒΡ… ΠΏΠΈΡ†Ρ†Π°Ρ…Β», Ρ‚ΠΎ ΠΎΡ‚Π²Π΅Ρ‚ΠΎΠΌ Π±ΡƒΠ΄Π΅Ρ‚ 4. Π”Π΅ΠΉΡΡ‚Π²ΠΈΡ‚Π΅Π»ΡŒΠ½ΠΎ, ΠΏΠΎΠ»ΠΎΠ²ΠΈΠ½Π° содСрТится Π² Π΄Π²ΡƒΡ… ΠΏΠΈΡ†Ρ†Π°Ρ… Ρ‡Π΅Ρ‚Ρ‹Ρ€Π΅ Ρ€Π°Π·Π°:


Π”Π΅Π»Π΅Π½ΠΈΠ΅ Π΄Ρ€ΠΎΠ±Π΅ΠΉ

Π§Ρ‚ΠΎΠ±Ρ‹ Ρ€Π°Π·Π΄Π΅Π»ΠΈΡ‚ΡŒ Π΄Ρ€ΠΎΠ±ΡŒ Π½Π° Π΄Ρ€ΠΎΠ±ΡŒ, Π½ΡƒΠΆΠ½ΠΎ ΠΏΠ΅Ρ€Π²ΡƒΡŽ Π΄Ρ€ΠΎΠ±ΡŒ ΡƒΠΌΠ½ΠΎΠΆΠΈΡ‚ΡŒ Π½Π° Π΄Ρ€ΠΎΠ±ΡŒ, ΠΎΠ±Ρ€Π°Ρ‚Π½ΡƒΡŽ Π²Ρ‚ΠΎΡ€ΠΎΠΉ.

НапримСр, Ρ€Π°Π·Π΄Π΅Π»ΠΈΠΌΒ Β Π½Π°Β 

Π§Ρ‚ΠΎΠ±Ρ‹ Ρ€Π°Π·Π΄Π΅Π»ΠΈΡ‚ΡŒΒ Β Π½Π°Β , Π½ΡƒΠΆΠ½ΠΎΒ Β ΡƒΠΌΠ½ΠΎΠΆΠΈΡ‚ΡŒ Π½Π° Π΄Ρ€ΠΎΠ±ΡŒ, ΠΎΠ±Ρ€Π°Ρ‚Π½ΡƒΡŽ Π΄Ρ€ΠΎΠ±ΠΈΒ . А обратная дроби  это Π΄Ρ€ΠΎΠ±ΡŒΒ 

Допустим, имССтся ΠΏΠΎΠ»ΠΎΠ²ΠΈΠ½Π° ΠΏΠΈΡ†Ρ†Ρ‹:

Если Π·Π°Π΄Π°Π΄ΠΈΠΌ вопрос «сколько Ρ€Π°Π· Ρ‡Π΅Ρ‚Π²Π΅Ρ€Ρ‚ΡŒ ΠΏΠΈΡ†Ρ†Ρ‹ содСрТится Π² этой ΠΏΠΎΠ»ΠΎΠ²ΠΈΠ½Π΅Β», Ρ‚ΠΎ ΠΎΡ‚Π²Π΅Ρ‚ΠΎΠΌ Π±ΡƒΠ΄Π΅Ρ‚ 2. Π”Π΅ΠΉΡΡ‚Π²ΠΈΡ‚Π΅Π»ΡŒΠ½ΠΎ, Ρ‡Π΅Ρ‚Π²Π΅Ρ€Ρ‚ΡŒ ΠΏΠΈΡ†Ρ†Ρ‹ содСрТится Π² ΠΏΠΎΠ»ΠΎΠ²ΠΈΠ½Π΅ ΠΏΠΈΡ†Ρ†Ρ‹ Π΄Π²Π° Ρ€Π°Π·Π°:


ΠŸΡ€ΠΈΠΌΠ΅Ρ€ 1. Найти Π·Π½Π°Ρ‡Π΅Π½ΠΈΠ΅ выраТСния 

Π£ΠΌΠ½ΠΎΠΆΠ°Π΅ΠΌ ΠΏΠ΅Ρ€Π²ΡƒΡŽ Π΄Ρ€ΠΎΠ±ΡŒ Π½Π° Π΄Ρ€ΠΎΠ±ΡŒ, ΠΎΠ±Ρ€Π°Ρ‚Π½ΡƒΡŽ Π²Ρ‚ΠΎΡ€ΠΎΠΉ. Π“Ρ€ΡƒΠ±ΠΎ говоря, ΡƒΠΌΠ½ΠΎΠΆΠ°Π΅ΠΌ ΠΏΠ΅Ρ€Π²ΡƒΡŽ Π΄Ρ€ΠΎΠ±ΡŒ Π½Π° ΠΏΠ΅Ρ€Π΅Π²Ρ‘Ρ€Π½ΡƒΡ‚ΡƒΡŽ Π²Ρ‚ΠΎΡ€ΡƒΡŽ:


ΠŸΡ€ΠΈΠΌΠ΅Ρ€ 2. Найти Π·Π½Π°Ρ‡Π΅Π½ΠΈΠ΅ выраТСния

Π£ΠΌΠ½ΠΎΠΆΠ°Π΅ΠΌ ΠΏΠ΅Ρ€Π²ΡƒΡŽ Π΄Ρ€ΠΎΠ±ΡŒ Π½Π° Π΄Ρ€ΠΎΠ±ΡŒ ΠΎΠ±Ρ€Π°Ρ‚Π½ΡƒΡŽ Π²Ρ‚ΠΎΡ€ΠΎΠΉ:


Π—Π΄Π΅ΡΡŒ совСтуСм ΠΎΡΡ‚Π°Π½ΠΎΠ²ΠΈΡ‚ΡŒΡΡ ΠΈ ΠΏΠΎΡ‚Ρ€Π΅Π½ΠΈΡ€ΠΎΠ²Π°Ρ‚ΡŒΡΡ. Π Π΅ΡˆΠΈΡ‚Π΅ нСсколько ΠΏΡ€ΠΈΠΌΠ΅Ρ€ΠΎΠ², ΠΏΡ€ΠΈΠ²Π΅Π΄Π΅Π½Π½Ρ‹Ρ… Π½ΠΈΠΆΠ΅. ΠœΠΎΠΆΠ΅Ρ‚Π΅ ΠΈΡΠΏΠΎΠ»ΡŒΠ·ΠΎΠ²Π°Ρ‚ΡŒ ΠΌΠ°Ρ‚Π΅Ρ€ΠΈΠ°Π»Ρ‹ сайта, ΠΊΠ°ΠΊ справочник. Π­Ρ‚ΠΎ ΠΏΠΎΠ·Π²ΠΎΠ»ΠΈΡ‚ Π²Π°ΠΌ Π½Π°ΡƒΡ‡ΠΈΡ‚ΡŒΡΡ Ρ€Π°Π±ΠΎΡ‚Π°Ρ‚ΡŒ с Π»ΠΈΡ‚Π΅Ρ€Π°Ρ‚ΡƒΡ€ΠΎΠΉ.

КаТдая ΡΠ»Π΅Π΄ΡƒΡŽΡ‰Π°Ρ Ρ‚Π΅ΠΌΠ° Π±ΡƒΠ΄Π΅Ρ‚ Π±ΠΎΠ»Π΅Π΅ слоТной, поэтому Π½ΡƒΠΆΠ½ΠΎ Ρ‚Ρ€Π΅Π½ΠΈΡ€ΠΎΠ²Π°Ρ‚ΡŒΡΡ.

Задания для ΡΠ°ΠΌΠΎΡΡ‚ΠΎΡΡ‚Π΅Π»ΡŒΠ½ΠΎΠ³ΠΎ Ρ€Π΅ΡˆΠ΅Π½ΠΈΡ:

Π—Π°Π΄Π°Π½ΠΈΠ΅ 1. НайдитС Π·Π½Π°Ρ‡Π΅Π½ΠΈΠ΅ выраТСния:

РСшСниС:

Π—Π°Π΄Π°Π½ΠΈΠ΅ 2. НайдитС Π·Π½Π°Ρ‡Π΅Π½ΠΈΠ΅ выраТСния:

РСшСниС:

Π—Π°Π΄Π°Π½ΠΈΠ΅ 3. НайдитС Π·Π½Π°Ρ‡Π΅Π½ΠΈΠ΅ выраТСния:

РСшСниС:

Π—Π°Π΄Π°Π½ΠΈΠ΅ 4. НайдитС Π·Π½Π°Ρ‡Π΅Π½ΠΈΠ΅ выраТСния:

РСшСниС:

Π—Π°Π΄Π°Π½ΠΈΠ΅ 5. НайдитС Π·Π½Π°Ρ‡Π΅Π½ΠΈΠ΅ выраТСния:

РСшСниС:

Π—Π°Π΄Π°Π½ΠΈΠ΅ 6. НайдитС Π·Π½Π°Ρ‡Π΅Π½ΠΈΠ΅ выраТСния:

РСшСниС:

Π—Π°Π΄Π°Π½ΠΈΠ΅ 7. НайдитС Π·Π½Π°Ρ‡Π΅Π½ΠΈΠ΅ выраТСния:

РСшСниС:

Π—Π°Π΄Π°Π½ΠΈΠ΅ 8. НайдитС Π·Π½Π°Ρ‡Π΅Π½ΠΈΠ΅ выраТСния:

РСшСниС:

Π—Π°Π΄Π°Π½ΠΈΠ΅ 9. НайдитС Π·Π½Π°Ρ‡Π΅Π½ΠΈΠ΅ выраТСния:

РСшСниС:

Π—Π°Π΄Π°Π½ΠΈΠ΅ 10. НайдитС Π·Π½Π°Ρ‡Π΅Π½ΠΈΠ΅ выраТСния:

РСшСниС:

Π—Π°Π΄Π°Π½ΠΈΠ΅ 11. НайдитС Π·Π½Π°Ρ‡Π΅Π½ΠΈΠ΅ выраТСния:

РСшСниС:

Π—Π°Π΄Π°Π½ΠΈΠ΅ 12. НайдитС Π·Π½Π°Ρ‡Π΅Π½ΠΈΠ΅ выраТСния:

РСшСниС:

Π—Π°Π΄Π°Π½ΠΈΠ΅ 13. НайдитС Π·Π½Π°Ρ‡Π΅Π½ΠΈΠ΅ выраТСния:

РСшСниС:

Π—Π°Π΄Π°Π½ΠΈΠ΅ 14. НайдитС Π·Π½Π°Ρ‡Π΅Π½ΠΈΠ΅ выраТСния:

РСшСниС:


ΠŸΠΎΠ½Ρ€Π°Π²ΠΈΠ»ΡΡ ΡƒΡ€ΠΎΠΊ?
Вступай Π² Π½Π°ΡˆΡƒ Π½ΠΎΠ²ΡƒΡŽ Π³Ρ€ΡƒΠΏΠΏΡƒ Π’ΠΊΠΎΠ½Ρ‚Π°ΠΊΡ‚Π΅ ΠΈ Π½Π°Ρ‡Π½ΠΈ ΠΏΠΎΠ»ΡƒΡ‡Π°Ρ‚ΡŒ увСдомлСния ΠΎ Π½ΠΎΠ²Ρ‹Ρ… ΡƒΡ€ΠΎΠΊΠ°Ρ…

Π’ΠΎΠ·Π½ΠΈΠΊΠ»ΠΎ ΠΆΠ΅Π»Π°Π½ΠΈΠ΅ ΠΏΠΎΠ΄Π΄Π΅Ρ€ΠΆΠ°Ρ‚ΡŒ ΠΏΡ€ΠΎΠ΅ΠΊΡ‚?
Π˜ΡΠΏΠΎΠ»ΡŒΠ·ΡƒΠΉ ΠΊΠ½ΠΎΠΏΠΊΡƒ Π½ΠΈΠΆΠ΅

Навигация ΠΏΠΎ записям

ΠœΠ°Ρ‚Π΅ΠΌΠ°Ρ‚ΠΈΠΊΠ°: ΡƒΡ€ΠΎΠΊΠΈ, тСсты, задания.

ΠœΠ°Ρ‚Π΅ΠΌΠ°Ρ‚ΠΈΠΊΠ°: ΡƒΡ€ΠΎΠΊΠΈ, тСсты, задания.
    1. Π‘Ρ€Π°Π²Π½Π΅Π½ΠΈΠ΅ ΠΏΡ€Π΅Π΄ΠΌΠ΅Ρ‚ΠΎΠ²
    2. Π’ΠΎΡ‡ΠΊΠ°, прямая линия, кривая ΠΈ ΠΎΡ‚Ρ€Π΅Π·ΠΎΠΊ
    3. ΠžΡΠΎΠ±Π΅Π½Π½ΠΎΡΡ‚ΠΈ ΠΌΠ½ΠΎΠ³ΠΎΡƒΠ³ΠΎΠ»ΡŒΠ½ΠΈΠΊΠΎΠ²
    4. ΠŸΡ€ΠΎΡΡ‚Ρ€Π°Π½ΡΡ‚Π²Π΅Π½Π½Ρ‹Π΅ ΠΈ Π²Ρ€Π΅ΠΌΠ΅Π½Π½Ρ‹Π΅ прСдставлСния
    5. ОбъСдинСниС ΠΏΡ€Π΅Π΄ΠΌΠ΅Ρ‚ΠΎΠ² Π² Π³Ρ€ΡƒΠΏΠΏΡ‹ ΠΈ ΠΏΠ°Ρ€Ρ‹
    6. Π‘Ρ€Π°Π²Π½Π΅Π½ΠΈΠ΅ (большС, мСньшС, ΡΡ‚ΠΎΠ»ΡŒΠΊΠΎ ΠΆΠ΅)
    7. Π—Π½Π°ΠΊΠΈ сравнСния ΠΈ Π·Π½Π°ΠΊΠΈ дСйствий
    1. НумСрация.
      Бколько? ΠžΡ‚ 1 Π΄ΠΎ 5
    2. ΠŸΡ€ΠΈΠΌΠ΅Ρ€Ρ‹ Π½Π° слоТСниС ΠΈ Π²Ρ‹Ρ‡ΠΈΡ‚Π°Π½ΠΈΠ΅ ΠΎΡ‚ 1 Π΄ΠΎ 5
    3. Π‘Ρ€Π°Π²Π½Π΅Π½ΠΈΠ΅ чисСл ΠΎΡ‚ 1 Π΄ΠΎ 5
    4. ВСкстовыС Π·Π°Π΄Π°Ρ‡ΠΈ (ΠΎΡ‚ 1 Π΄ΠΎ 5)
    5. Π—Π°Π΄Π°Ρ‡ΠΈ Π½Π° смСкалку (ΠΎΡ‚ 1 Π΄ΠΎ 5)
    1. ΠŸΡ€ΠΈΠΌΠ΅Ρ€Ρ‹ Π½Π° сумму
    2. ВСкстовыС Π·Π°Π΄Π°Ρ‡ΠΈ (сумма)
  1. ΠŸΠ΅Ρ€Π΅ΠΌΠ΅ΡΡ‚ΠΈΡ‚Π΅Π»ΡŒΠ½Ρ‹ΠΉ Π·Π°ΠΊΠΎΠ½ слоТСния

    1. ΠŸΡ€ΠΈΠΌΠ΅Ρ€Ρ‹ Π½Π° Ρ€Π°Π·Π½ΠΎΡΡ‚ΡŒ
    2. ВСкстовыС Π·Π°Π΄Π°Ρ‡ΠΈ (Ρ€Π°Π·Π½ΠΎΡΡ‚ΡŒ)
  2. Π’Π°Π±Π»ΠΈΡ†Π° слоТСния.

    Числа ΠΎΡ‚ 1 Π΄ΠΎ 9
    1. НумСрация. Бколько? ΠžΡ‚ 0 Π΄ΠΎ 10
    2. ΠŸΡ€ΠΈΠΌΠ΅Ρ€Ρ‹ ΠΎΡ‚ 0 Π΄ΠΎ 10
    3. Π‘Ρ€Π°Π²Π½Π΅Π½ΠΈΠ΅ чисСл ΠΎΡ‚ 0 Π΄ΠΎ 10 ΠΈ Π²Ρ‹Ρ€Π°ΠΆΠ΅Π½ΠΈΠΉ
    4. ВСкстовыС Π·Π°Π΄Π°Ρ‡ΠΈ (ΠΎΡ‚ 0 Π΄ΠΎ 10)
    5. Π—Π°Π΄Π°Ρ‡ΠΈ Π½Π° смСкалку (ΠΎΡ‚ 0 Π΄ΠΎ 10)
  3. Π£Π²Π΅Π»ΠΈΡ‡ΠΈΡ‚ΡŒ/ΡƒΠΌΠ΅Π½ΡŒΡˆΠΈΡ‚ΡŒ на…

    1. ΠœΠ΅Ρ€Π° Π΄Π»ΠΈΠ½Ρ‹ β€” сантимСтр
    2. ΠœΠ΅Ρ€Π° Π΄Π»ΠΈΠ½Ρ‹ β€” Π΄Π΅Ρ†ΠΈΠΌΠ΅Ρ‚Ρ€
  4. На сколько большС? На сколько мСньшС?

    1. Π‘Ρ‡Ρ‘Ρ‚ дСсятками
    2. Π‘Ρ‡Ρ‘Ρ‚ ΠΊΡ€ΡƒΠ³Π»Ρ‹Ρ… чисСл
    1. НумСрация.
      Бколько? ΠžΡ‚ 11 Π΄ΠΎ 20
    2. ΠŸΡ€ΠΈΠΌΠ΅Ρ€Ρ‹ ΠΎΡ‚ 11 Π΄ΠΎ 20
    3. БравнСния чисСл ΠΎΡ‚ 11 Π΄ΠΎ 20
    4. ВСкстовыС Π·Π°Π΄Π°Ρ‡ΠΈ (ΠΎΡ‚ 11 Π΄ΠΎ 20)
    5. Π—Π°Π΄Π°Ρ‡ΠΈ Π½Π° смСкалку (ΠΎΡ‚ 11 Π΄ΠΎ 20)
  1. Числа ΠΎΡ‚ 20 Π΄ΠΎ 100.

    НумСрация. Числа ΠΈ Ρ†ΠΈΡ„Ρ€Ρ‹
    1. Π‘ΠΎΡ‡Π΅Ρ‚Π°Ρ‚Π΅Π»ΡŒΠ½Ρ‹ΠΉ Π·Π°ΠΊΠΎΠ½ слоТСния. Π‘ΠΊΠΎΠ±ΠΊΠΈ
    2. Π’Π°Π±Π»ΠΈΡ†Π° слоТСния. Числа ΠΎΡ‚ 0 Π΄ΠΎ 18
    3. Π’Ρ‹Ρ‡ΠΈΡ‚Π°Π΅ΠΌ сумму ΠΈΠ· числа
    4. ΠŸΡ€Π°Π²ΠΈΠ»Π° слоТСния ΠΈ вычитания чисСл Π² ΠΏΡ€Π΅Π΄Π΅Π»Π°Ρ… 20 с ΠΏΠ΅Ρ€Π΅Ρ…ΠΎΠ΄ΠΎΠΌ Ρ‡Π΅Ρ€Π΅Π· дСсяток
    5. Π‘Π»ΠΎΠΆΠ΅Π½ΠΈΠ΅ ΠΈ Π²Ρ‹Ρ‡ΠΈΡ‚Π°Π½ΠΈΠ΅ чисСл Π² ΠΏΡ€Π΅Π΄Π΅Π»Π°Ρ… 100 Π±Π΅Π· ΠΏΠ΅Ρ€Π΅Ρ…ΠΎΠ΄Π° Ρ‡Π΅Ρ€Π΅Π· дСсяток
    6. ΠŸΡ€Π°Π²ΠΈΠ»Π° слоТСния ΠΈ вычитания чисСл Π² ΠΏΡ€Π΅Π΄Π΅Π»Π°Ρ… 100 с ΠΏΠ΅Ρ€Π΅Ρ…ΠΎΠ΄ΠΎΠΌ Ρ‡Π΅Ρ€Π΅Π· дСсяток
    7. ΠŸΡ€Π°Π²ΠΈΠ»Π° слоТСния ΠΈ вычитания чисСл Π² ΠΏΡ€Π΅Π΄Π΅Π»Π°Ρ… 100
    1. Находим ΠΏΠ΅Ρ€ΠΈΠΌΠ΅Ρ‚Ρ€
    2. РСшСниС Π·Π°Π΄Π°Ρ‡ Π² Π΄Π²Π° дСйствия
    1. ΠœΠ΅Ρ€Π° Π΄Π»ΠΈΠ½Ρ‹ β€” ΠΌΠ΅Ρ‚Ρ€
    2. ΠšΠΈΠ»ΠΎΠ³Ρ€Π°ΠΌΠΌ
    3. Π›ΠΈΡ‚Ρ€
    1. Π£Ρ€Π°Π²Π½Π΅Π½ΠΈΠ΅ (сумма)
    2. Π£Ρ€Π°Π²Π½Π΅Π½ΠΈΠ΅ (Ρ€Π°Π·Π½ΠΎΡΡ‚ΡŒ)
    1. ΠŸΠΎΠ½ΡΡ‚ΠΈΠ΅ умноТСния
    2. ΠŸΠ΅Ρ€Π΅ΠΌΠ΅ΡΡ‚ΠΈΡ‚Π΅Π»ΡŒΠ½Ρ‹ΠΉ Π·Π°ΠΊΠΎΠ½ умноТСния
    3. Π£ΠΌΠ½ΠΎΠΆΠ΅Π½ΠΈΠ΅ Π½Π° 2 (Ρ‚Π°Π±Π»ΠΈΡ†Π°)
    4. Π£ΠΌΠ½ΠΎΠΆΠ΅Π½ΠΈΠ΅ Π½Π° 3 (Ρ‚Π°Π±Π»ΠΈΡ†Π°)
    5. Π£ΠΌΠ½ΠΎΠΆΠ΅Π½ΠΈΠ΅ Π½Π° 4 (Ρ‚Π°Π±Π»ΠΈΡ†Π°)
    6. Π£ΠΌΠ½ΠΎΠΆΠ΅Π½ΠΈΠ΅ Π½Π° 5 (Ρ‚Π°Π±Π»ΠΈΡ†Π°)
  2. Π”Π΅Π»Π΅Π½ΠΈΠ΅

  3. Π§Ρ‘Ρ‚Π½Ρ‹Π΅ ΠΈ Π½Π΅Ρ‡Ρ‘Ρ‚Π½Ρ‹Π΅ числа

    1. ВыраТСния Π±Π΅Π· скобок
    2. ВыраТСния со скобками
    1. Π£Π·Π½Π°Ρ‘ΠΌ ΠΎ Π»ΡƒΡ‡Π΅
    2. Π€ΠΈΠ³ΡƒΡ€Π° ΡƒΠ³ΠΎΠ» ΠΈ Π΅Π³ΠΎ характСристики
    3. Π₯арактСристики прямого, Ρ‚ΡƒΠΏΠΎΠ³ΠΎ ΠΈ острого ΡƒΠ³Π»ΠΎΠ²
    1. Π£Π²Π΅Π»ΠΈΡ‡ΠΈΡ‚ΡŒ Π½Π°.
      .. Π£Π²Π΅Π»ΠΈΡ‡ΠΈΡ‚ΡŒ Π²… Π£ΠΌΠ΅Π½ΡŒΡˆΠΈΡ‚ΡŒ Π½Π°… Π£ΠΌΠ΅Π½ΡŒΡˆΠΈΡ‚ΡŒ Π²…
    2. Π‘ΠΎΠ»ΡŒΡˆΠ΅ Π½Π°… Π‘ΠΎΠ»ΡŒΡˆΠ΅ Π²… МСньшС Π½Π°… МСньшС Π²…
    1. Π£ΠΌΠ½ΠΎΠΆΠ΅Π½ΠΈΠ΅ Π½Π° 6 (Ρ‚Π°Π±Π»ΠΈΡ†Π°)
    2. Π£ΠΌΠ½ΠΎΠΆΠ΅Π½ΠΈΠ΅ Π½Π° 7 (Ρ‚Π°Π±Π»ΠΈΡ†Π°)
    3. Π£ΠΌΠ½ΠΎΠΆΠ΅Π½ΠΈΠ΅ Π½Π° 8 (Ρ‚Π°Π±Π»ΠΈΡ†Π°)
    4. Π£ΠΌΠ½ΠΎΠΆΠ΅Π½ΠΈΠ΅ Π½Π° 9 (Ρ‚Π°Π±Π»ΠΈΡ†Π°)
    1. НахоТдСниС нСизвСстного мноТитСля
    2. НахоТдСниС нСизвСстного Π΄Π΅Π»ΠΈΠΌΠΎΠ³ΠΎ
    3. НахоТдСниС нСизвСстного дСлитСля
    1. Бвойства Π»ΠΎΠΌΠ°Π½ΠΎΠΉ Π»ΠΈΠ½ΠΈΠΈ
    2. Π’Ρ€Π΅ΡƒΠ³ΠΎΠ»ΡŒΠ½ΠΈΠΊΠΈ.
      Π’ΠΈΠ΄Ρ‹ Ρ‚Ρ€Π΅ΡƒΠ³ΠΎΠ»ΡŒΠ½ΠΈΠΊΠΎΠ²
    1. Π£ΠΌΠ½ΠΎΠΆΠ΅Π½ΠΈΠ΅ ΠΈ Π΄Π΅Π»Π΅Π½ΠΈΠ΅ Π½Π° 0, 1, 10. Π”Π΅Π»Π΅Π½ΠΈΠ΅ числа Π½Π° само сСбя
    2. ВыполняСм ΡƒΠΌΠ½ΠΎΠΆΠ΅Π½ΠΈΠ΅ ΠΈ Π΄Π΅Π»Π΅Π½ΠΈΠ΅ ΠΊΡ€ΡƒΠ³Π»ΠΎΠ³ΠΎ числа Π½Π° ΠΎΠ΄Π½ΠΎΠ·Π½Π°Ρ‡Π½ΠΎΠ΅ число
    3. ΠŸΡ€Π°Π²ΠΈΠ»Π° дСлСния ΠΊΡ€ΡƒΠ³Π»ΠΎΠ³ΠΎ числа Π½Π° ΠΊΡ€ΡƒΠ³Π»ΠΎΠ΅ число
    1. Π£ΠΌΠ½ΠΎΠΆΠ°Π΅ΠΌ сумму Π½Π° число
    2. Π£ΠΌΠ½ΠΎΠΆΠ°Π΅ΠΌ Π΄Π²ΡƒΠ·Π½Π°Ρ‡Π½ΠΎΠ΅ число Π½Π° ΠΎΠ΄Π½ΠΎΠ·Π½Π°Ρ‡Π½ΠΎΠ΅ число
    1. ΠŸΡ€Π°Π²ΠΈΠ»Π° дСлСния суммы Π½Π° число
    2. ΠŸΡ€Π°Π²ΠΈΠ»Π° дСлСния Π΄Π²ΡƒΠ·Π½Π°Ρ‡Π½ΠΎΠ³ΠΎ числа Π½Π° ΠΎΠ΄Π½ΠΎΠ·Π½Π°Ρ‡Π½ΠΎΠ΅
    3. ΠŸΡ€Π°Π²ΠΈΠ»Π° дСлСния Π΄Π²ΡƒΠ·Π½Π°Ρ‡Π½ΠΎΠ³ΠΎ числа Π½Π° Π΄Π²ΡƒΠ·Π½Π°Ρ‡Π½ΠΎΠ΅
    4. ΠŸΡ€Π°Π²ΠΈΠ»Π° дСлСния с остатком
    1. Находим долю ΠΎΡ‚ числа
    2. Π‘Ρ€Π°Π²Π½ΠΈΠ²Π°Π΅ΠΌ Π΄ΠΎΠ»ΠΈ
    3. Находим число ΠΏΠΎ Π΄ΠΎΠ»Π΅
    1. Π’Ρ€Ρ‘Ρ…Π·Π½Π°Ρ‡Π½Ρ‹Π΅ числа.
      НумСрация
    2. Π‘Π»ΠΎΠΆΠ΅Π½ΠΈΠ΅ ΠΈ Π²Ρ‹Ρ‡ΠΈΡ‚Π°Π½ΠΈΠ΅ Ρ‚Ρ€Ρ‘Ρ…Π·Π½Π°Ρ‡Π½Ρ‹Ρ… чисСл
    3. ВыполняСм ΡƒΠΌΠ½ΠΎΠΆΠ΅Π½ΠΈΠ΅ ΠΈ Π΄Π΅Π»Π΅Π½ΠΈΠ΅ Ρ‚Ρ€Ρ‘Ρ…Π·Π½Π°Ρ‡Π½ΠΎΠ³ΠΎ числа Π½Π° ΠΎΠ΄Π½ΠΎΠ·Π½Π°Ρ‡Π½ΠΎΠ΅ число
    4. Бвязь ΠΌΠ΅ΠΆΠ΄Ρƒ Π²Π΅Π»ΠΈΡ‡ΠΈΠ½Π°ΠΌΠΈ
  1. ΠšΠ°Π»Π΅Π½Π΄Π°Ρ€ΡŒ

    1. НумСрация
    2. ΠŸΡ€Π°Π²ΠΈΠ»Π° слоТСния ΠΈ вычитания ΠΌΠ½ΠΎΠ³ΠΎΠ·Π½Π°Ρ‡Π½Ρ‹Ρ… чисСл
    3. ΠŸΡ€Π°Π²ΠΈΠ»Π° ΡΠΎΡ‡Π΅Ρ‚Π°Ρ‚Π΅Π»ΡŒΠ½ΠΎΠ³ΠΎ Π·Π°ΠΊΠΎΠ½Π° умноТСния
    4. Π£ΠΌΠ½ΠΎΠΆΠ°Π΅ΠΌ ΠΈ Π΄Π΅Π»ΠΈΠΌ числа Π½Π° 10, 100, 1000
    5. ΠšΡ€ΡƒΠ³Π»Ρ‹Π΅ числа (ΡƒΠΌΠ½ΠΎΠΆΠ΅Π½ΠΈΠ΅ ΠΈ Π΄Π΅Π»Π΅Π½ΠΈΠ΅)
    1. Π•Π΄ΠΈΠ½ΠΈΡ†Ρ‹ измСрСния Π²Ρ€Π΅ΠΌΠ΅Π½ΠΈ (час, ΠΌΠΈΠ½ΡƒΡ‚Π°, сутки)
    2. ΠœΠΈΠ»Π»ΠΈΠΌΠ΅Ρ‚Ρ€
    3. ΠšΠΈΠ»ΠΎΠΌΠ΅Ρ‚Ρ€
    1. НахоТдСниС ΠΏΠ»ΠΎΡ‰Π°Π΄ΠΈ Ρ„ΠΈΠ³ΡƒΡ€Ρ‹, ΠΏΡ€ΡΠΌΠΎΡƒΠ³ΠΎΠ»ΡŒΠ½ΠΈΠΊΠ°
    2. Π•Π΄ΠΈΠ½ΠΈΡ†Ρ‹ измСрСния ΠΏΠ»ΠΎΡ‰Π°Π΄ΠΈ
    1. Π£ΠΌΠ½ΠΎΠΆΠ΅Π½ΠΈΠ΅ Π½Π° ΠΎΠ΄Π½ΠΎΠ·Π½Π°Ρ‡Π½ΠΎΠ΅ число.
      Π Π°ΡΠΏΡ€Π΅Π΄Π΅Π»ΠΈΡ‚Π΅Π»ΡŒΠ½Ρ‹ΠΉ Π·Π°ΠΊΠΎΠ½ умноТСния ΠΎΡ‚Π½ΠΎΡΠΈΡ‚Π΅Π»ΡŒΠ½ΠΎ слоТСния
    2. Π£ΠΌΠ½ΠΎΠΆΠ°Π΅ΠΌ ΠΊΡ€ΡƒΠ³Π»ΠΎΠ΅ число Π½Π° ΠΎΠ΄Π½ΠΎΠ·Π½Π°Ρ‡Π½ΠΎΠ΅ число
    3. ВыполняСм ΡƒΠΌΠ½ΠΎΠΆΠ΅Π½ΠΈΠ΅ Π½Π° ΠΊΡ€ΡƒΠ³Π»ΠΎΠ΅ число
    4. ВыполняСм ΡƒΠΌΠ½ΠΎΠΆΠ΅Π½ΠΈΠ΅ ΠΊΡ€ΡƒΠ³Π»Ρ‹Ρ… чисСл
    5. ВыполняСм ΡƒΠΌΠ½ΠΎΠΆΠ΅Π½ΠΈΠ΅ Π½Π° Π΄Π²ΡƒΠ·Π½Π°Ρ‡Π½ΠΎΠ΅ число
    6. ВыполняСм ΡƒΠΌΠ½ΠΎΠΆΠ΅Π½ΠΈΠ΅ Π½Π° Ρ‚Ρ€Ρ‘Ρ…Π·Π½Π°Ρ‡Π½ΠΎΠ΅ число
    1. Π”Π΅Π»Π΅Π½ΠΈΠ΅ ΠΌΠ½ΠΎΠ³ΠΎΠ·Π½Π°Ρ‡Π½ΠΎΠ³ΠΎ числа Π½Π° ΠΎΠ΄Π½ΠΎΠ·Π½Π°Ρ‡Π½ΠΎΠ΅ число
    2. Π”Π΅Π»Π΅Π½ΠΈΠ΅ ΠΊΡ€ΡƒΠ³Π»ΠΎΠ³ΠΎ ΠΌΠ½ΠΎΠ³ΠΎΠ·Π½Π°Ρ‡Π½ΠΎΠ³ΠΎ числа Π½Π° ΠΎΠ΄Π½ΠΎΠ·Π½Π°Ρ‡Π½ΠΎΠ΅
    3. Π”Π΅Π»Π΅Π½ΠΈΠ΅ ΠΌΠ½ΠΎΠ³ΠΎΠ·Π½Π°Ρ‡Π½ΠΎΠ³ΠΎ числа Π½Π° 10, 100, 1000 с остатком
    4. Π”Π΅Π»Π΅Π½ΠΈΠ΅ ΠΌΠ½ΠΎΠ³ΠΎΠ·Π½Π°Ρ‡Π½ΠΎΠ³ΠΎ числа с остатком Π½Π° ΠΎΠ΄Π½ΠΎΠ·Π½Π°Ρ‡Π½ΠΎΠ΅ число
    5. ВыполняСм Π΄Π΅Π»Π΅Π½ΠΈΠ΅ Ρ‚Ρ€Ρ‘Ρ…Π·Π½Π°Ρ‡Π½ΠΎΠ³ΠΎ числа Π½Π° Π΄Π²ΡƒΠ·Π½Π°Ρ‡Π½ΠΎΠ΅ число
    6. Π”Π΅Π»Π΅Π½ΠΈΠ΅ с остатком Ρ‚Ρ€Ρ‘Ρ…Π·Π½Π°Ρ‡Π½ΠΎΠ³ΠΎ числа Π½Π° Π΄Π²ΡƒΠ·Π½Π°Ρ‡Π½ΠΎΠ΅ число
    7. Π”Π΅Π»Π΅Π½ΠΈΠ΅ ΠΌΠ½ΠΎΠ³ΠΎΠ·Π½Π°Ρ‡Π½ΠΎΠ³ΠΎ числа Π½Π° Π΄Π²ΡƒΠ·Π½Π°Ρ‡Π½ΠΎΠ΅ число
    8. Π”Π΅Π»Π΅Π½ΠΈΠ΅ с остатком Π½Π° Π΄Π²ΡƒΠ·Π½Π°Ρ‡Π½ΠΎΠ΅ число
    9. ВыполняСм Π΄Π΅Π»Π΅Π½ΠΈΠ΅ Π½Π° Ρ‚Ρ€Ρ‘Ρ…Π·Π½Π°Ρ‡Π½ΠΎΠ΅ число
    10. Π”Π΅Π»Π΅Π½ΠΈΠ΅ с остатком Π½Π° Ρ‚Ρ€Ρ‘Ρ…Π·Π½Π°Ρ‡Π½ΠΎΠ΅ число
    11. Π”Π΅Π»Π΅Π½ΠΈΠ΅ ΠΊΡ€ΡƒΠ³Π»ΠΎΠ³ΠΎ ΠΌΠ½ΠΎΠ³ΠΎΠ·Π½Π°Ρ‡Π½ΠΎΠ³ΠΎ числа Π½Π° ΠΊΡ€ΡƒΠ³Π»ΠΎΠ΅ число
    1. Π•Π΄ΠΈΠ½ΠΈΡ†Ρ‹ Π²Ρ€Π΅ΠΌΠ΅Π½ΠΈ.
      ΠœΠΈΠ½ΡƒΡ‚Π°. Π‘Π΅ΠΊΡƒΠ½Π΄Π°
    2. Π•Π΄ΠΈΠ½ΠΈΡ†Ρ‹ массы ΠΈ ΠΏΠ»ΠΎΡ‰Π°Π΄ΠΈ. Π“Π΅ΠΊΡ‚Π°Ρ€. Π¦Π΅Π½Ρ‚Π½Π΅Ρ€. Π’ΠΎΠ½Π½Π°
    1. ΠŸΠΎΠ½ΡΡ‚ΠΈΠ΅ Π΄Ρ€ΠΎΠ±ΠΈ
    2. Π‘Ρ€Π°Π²Π½ΠΈΠ²Π°Π΅ΠΌ Π΄Ρ€ΠΎΠ±ΠΈ
    3. Π”Ρ€ΠΎΠ±ΠΈ.
      НахоТдСниС части числа
    4. Π”Ρ€ΠΎΠ±ΠΈ. НахоТдСниС числа ΠΏΠΎ Π΅Π³ΠΎ части
    1. РСшСниС Π·Π°Π΄Π°Ρ‡ Π½Π° Π½Π°Ρ…ΠΎΠΆΠ΄Π΅Π½ΠΈΠ΅ скорости, Π²Ρ€Π΅ΠΌΠ΅Π½ΠΈ, расстояния
    2. РСшСниС Π·Π°Π΄Π°Ρ‡ Π½Π° Π½Π°Ρ…ΠΎΠΆΠ΄Π΅Π½ΠΈΠ΅ Ρ€Π°Π±ΠΎΡ‚Ρ‹, Π²Ρ€Π΅ΠΌΠ΅Π½ΠΈ, ΠΏΡ€ΠΎΠΈΠ·Π²ΠΎΠ΄ΠΈΡ‚Π΅Π»ΡŒΠ½ΠΎΡΡ‚ΠΈ
    3. РСшСниС Π·Π°Π΄Π°Ρ‡ Π½Π° Π½Π°Ρ…ΠΎΠΆΠ΄Π΅Π½ΠΈΠ΅ Ρ†Π΅Π½Ρ‹, количСства, стоимости
    1. ДСсятичная систСма счислСния.
      Римская нумСрация
    2. ЧисловыС ΠΈ Π±ΡƒΠΊΠ²Π΅Π½Π½Ρ‹Π΅ выраТСния
    3. ΠΠ°Ρ‡Π°Π»ΡŒΠ½Ρ‹Π΅ гСомСтричСскиС понятия: прямая, ΠΎΡ‚Ρ€Π΅Π·ΠΎΠΊ, Π»ΡƒΡ‡, ломаная, ΠΏΡ€ΡΠΌΠΎΡƒΠ³ΠΎΠ»ΡŒΠ½ΠΈΠΊ
    4. ΠžΠΏΡ€Π΅Π΄Π΅Π»Π΅Π½ΠΈΠ΅ ΠΊΠΎΠΎΡ€Π΄ΠΈΠ½Π°Ρ‚Π½ΠΎΠ³ΠΎ Π»ΡƒΡ‡Π°
    5. ΠžΠΊΡ€ΡƒΠ³Π»Π΅Π½ΠΈΠ΅ чисСл.
      ΠŸΡ€ΠΈΠΊΠΈΠ΄ΠΊΠ° ΠΈ ΠΎΡ†Π΅Π½ΠΊΠ° Ρ€Π΅Π·ΡƒΠ»ΡŒΡ‚Π°Ρ‚ΠΎΠ² вычислСний
    6. Π—Π°ΠΊΠΎΠ½Ρ‹ арифмСтичСских дСйствий. ВычислСния с ΠΌΠ½ΠΎΠ³ΠΎΠ·Π½Π°Ρ‡Π½Ρ‹ΠΌΠΈ числами
    7. РСшСниС тСкстовых Π·Π°Π΄Π°Ρ‡ арифмСтичСским способом
    8. Π€ΠΎΡ€ΠΌΡƒΠ»Ρ‹. УравнСния. Π£ΠΏΡ€ΠΎΡ‰Π΅Π½ΠΈΠ΅ Π²Ρ‹Ρ€Π°ΠΆΠ΅Π½ΠΈΠΉ
    9. ΠœΠ°Ρ‚Π΅ΠΌΠ°Ρ‚ΠΈΡ‡Π΅ΡΠΊΠΈΠΉ язык ΠΈ матСматичСская модСль
    1. Π”Π΅Π»Π΅Π½ΠΈΠ΅ с остатком.
      ΠŸΠΎΠ½ΡΡ‚ΠΈΠ΅ ΠΎΠ±Ρ‹ΠΊΠ½ΠΎΠ²Π΅Π½Π½ΠΎΠΉ Π΄Ρ€ΠΎΠ±ΠΈ
    2. ОсновноС свойство Π΄Ρ€ΠΎΠ±ΠΈ. Π‘ΠΎΠΊΡ€Π°Ρ‰Π΅Π½ΠΈΠ΅ ΠΈ Ρ€Π°ΡΡˆΠΈΡ€Π΅Π½ΠΈΠ΅ Π΄Ρ€ΠΎΠ±Π΅ΠΉ
    3. ΠŸΡ€Π°Π²ΠΈΠ»ΡŒΠ½Ρ‹Π΅ ΠΈ Π½Π΅ΠΏΡ€Π°Π²ΠΈΠ»ΡŒΠ½Ρ‹Π΅ Π΄Ρ€ΠΎΠ±ΠΈ. Π‘ΠΌΠ΅ΡˆΠ°Π½Π½Ρ‹Π΅ числа. ΠŸΠΎΠ½ΡΡ‚ΠΈΠ΅, запись ΠΈ Ρ‡Ρ‚Π΅Π½ΠΈΠ΅
    4. Π‘Ρ€Π°Π²Π½Π΅Π½ΠΈΠ΅ ΠΎΠ±Ρ‹ΠΊΠ½ΠΎΠ²Π΅Π½Π½Ρ‹Ρ… Π΄Ρ€ΠΎΠ±Π΅ΠΉ
    5. Π‘Π»ΠΎΠΆΠ΅Π½ΠΈΠ΅ ΠΈ Π²Ρ‹Ρ‡ΠΈΡ‚Π°Π½ΠΈΠ΅ ΠΎΠ±Ρ‹ΠΊΠ½ΠΎΠ²Π΅Π½Π½Ρ‹Ρ… Π΄Ρ€ΠΎΠ±Π΅ΠΉ ΠΈ ΡΠΌΠ΅ΡˆΠ°Π½Π½Ρ‹Ρ… чисСл
    6. Π£ΠΌΠ½ΠΎΠΆΠ΅Π½ΠΈΠ΅ ΠΈ Π΄Π΅Π»Π΅Π½ΠΈΠ΅ ΠΎΠ±Ρ‹ΠΊΠ½ΠΎΠ²Π΅Π½Π½ΠΎΠΉ Π΄Ρ€ΠΎΠ±ΠΈ Π½Π° Π½Π°Ρ‚ΡƒΡ€Π°Π»ΡŒΠ½ΠΎΠ΅ число
    7. НахоТдСниС части ΠΎΡ‚ Ρ†Π΅Π»ΠΎΠ³ΠΎ ΠΈ числа ΠΏΠΎ Π΅Π³ΠΎ части
    8. ГСомСтричСскиС понятия: ΠΎΠΊΡ€ΡƒΠΆΠ½ΠΎΡΡ‚ΡŒ ΠΈ ΠΊΡ€ΡƒΠ³
    1. Π£Π³ΠΎΠ».
      Π˜Π·ΠΌΠ΅Ρ€Π΅Π½ΠΈΠ΅ ΡƒΠ³Π»ΠΎΠ²
    2. БиссСктриса ΡƒΠ³Π»Π°. Бвойство биссСктрисы ΡƒΠ³Π»Π°
    3. Π’Ρ€Π΅ΡƒΠ³ΠΎΠ»ΡŒΠ½ΠΈΠΊ. ΠŸΠ»ΠΎΡ‰Π°Π΄ΡŒ Ρ‚Ρ€Π΅ΡƒΠ³ΠΎΠ»ΡŒΠ½ΠΈΠΊΠ°
    4. Бвойство ΡƒΠ³Π»ΠΎΠ² Ρ‚Ρ€Π΅ΡƒΠ³ΠΎΠ»ΡŒΠ½ΠΈΠΊΠ°. Π Π°Π·ΠΌΠ΅Ρ€Ρ‹ ΠΎΠ±ΡŠΠ΅ΠΊΡ‚ΠΎΠ² ΠΎΠΊΡ€ΡƒΠΆΠ°ΡŽΡ‰Π΅Π³ΠΎ ΠΌΠΈΡ€Π° (ΠΌΠ°ΡΡˆΡ‚Π°Π±)
    5. Расстояния ΠΌΠ΅ΠΆΠ΄Ρƒ двумя Ρ‚ΠΎΡ‡ΠΊΠ°ΠΌΠΈ.
      ΠœΠ°ΡΡˆΡ‚Π°Π±. Π’ΠΈΠ΄Ρ‹ ΠΌΠ°ΡΡˆΡ‚Π°Π±Π°
    6. ΠŸΠ΅Ρ€ΠΏΠ΅Π½Π΄ΠΈΠΊΡƒΠ»ΡΡ€Π½ΠΎΡΡ‚ΡŒ прямых. РасстояниС ΠΎΡ‚ Ρ‚ΠΎΡ‡ΠΊΠΈ Π΄ΠΎ прямой. Π‘Π΅Ρ€Π΅Π΄ΠΈΠ½Π½Ρ‹ΠΉ пСрпСндикуляр
    1. ΠŸΠΎΠ½ΡΡ‚ΠΈΠ΅ дСсятичной Π΄Ρ€ΠΎΠ±ΠΈ. ΠŸΡ€Π΅Π΄ΡΡ‚Π°Π²Π»Π΅Π½ΠΈΠ΅ дСсятичной Π΄Ρ€ΠΎΠ±ΠΈ Π² Π²ΠΈΠ΄Π΅ ΠΎΠ±Ρ‹ΠΊΠ½ΠΎΠ²Π΅Π½Π½ΠΎΠΉ Π΄Ρ€ΠΎΠ±ΠΈ ΠΈ Π½Π°ΠΎΠ±ΠΎΡ€ΠΎΡ‚
    2. ДСсятичныС Π΄Ρ€ΠΎΠ±ΠΈ.
      Π‘Ρ€Π°Π²Π½Π΅Π½ΠΈΠ΅
    3. ДСсятичныС Π΄Ρ€ΠΎΠ±ΠΈ. Π‘Π»ΠΎΠΆΠ΅Π½ΠΈΠ΅ ΠΈ Π²Ρ‹Ρ‡ΠΈΡ‚Π°Π½ΠΈΠ΅
    4. ДСсятичныС Π΄Ρ€ΠΎΠ±ΠΈ. Π£ΠΌΠ½ΠΎΠΆΠ΅Π½ΠΈΠ΅
    5. Π‘Ρ‚Π΅ΠΏΠ΅Π½ΡŒ с Π½Π°Ρ‚ΡƒΡ€Π°Π»ΡŒΠ½Ρ‹ΠΌ ΠΏΠΎΠΊΠ°Π·Π°Ρ‚Π΅Π»Π΅ΠΌ
    6. ДСсятичныС Π΄Ρ€ΠΎΠ±ΠΈ.
      Π‘Ρ€Π΅Π΄Π½Π΅Π΅ арифмСтичСскоС, Π΄Π΅Π»Π΅Π½ΠΈΠ΅ Π½Π° Π½Π°Ρ‚ΡƒΡ€Π°Π»ΡŒΠ½ΠΎΠ΅ число
    7. ДСсятичныС Π΄Ρ€ΠΎΠ±ΠΈ. Π”Π΅Π»Π΅Π½ΠΈΠ΅ Π½Π° Π΄Π΅ΡΡΡ‚ΠΈΡ‡Π½ΡƒΡŽ Π΄Ρ€ΠΎΠ±ΡŒ
    8. ΠŸΡ€ΠΎΡ†Π΅Π½Ρ‚Ρ‹. Π—Π°Π΄Π°Ρ‡ΠΈ Π½Π° ΠΏΡ€ΠΎΡ†Π΅Π½Ρ‚Ρ‹: Π½Π°Ρ…ΠΎΠΆΠ΄Π΅Π½ΠΈΠ΅ ΠΏΡ€ΠΎΡ†Π΅Π½Ρ‚Π° ΠΎΡ‚ Π²Π΅Π»ΠΈΡ‡ΠΈΠ½Ρ‹ ΠΈ Π²Π΅Π»ΠΈΡ‡ΠΈΠ½Ρ‹ ΠΏΠΎ Π΅Ρ‘ ΠΏΡ€ΠΎΡ†Π΅Π½Ρ‚Ρƒ
    1. ΠŸΡ€ΡΠΌΠΎΡƒΠ³ΠΎΠ»ΡŒΠ½Ρ‹ΠΉ ΠΏΠ°Ρ€Π°Π»Π»Π΅Π»Π΅ΠΏΠΈΠΏΠ΅Π΄.
      ΠžΠΏΡ€Π΅Π΄Π΅Π»Π΅Π½ΠΈΠ΅, свойства
    2. ΠŸΡ€ΡΠΌΠΎΡƒΠ³ΠΎΠ»ΡŒΠ½Ρ‹ΠΉ ΠΏΠ°Ρ€Π°Π»Π»Π΅Π»Π΅ΠΏΠΈΠΏΠ΅Π΄. Π Π°Π·Π²Ρ‘Ρ€Ρ‚ΠΊΠ°
    3. ΠŸΡ€ΡΠΌΠΎΡƒΠ³ΠΎΠ»ΡŒΠ½Ρ‹ΠΉ ΠΏΠ°Ρ€Π°Π»Π»Π΅Π»Π΅ΠΏΠΈΠΏΠ΅Π΄. ΠžΠ±ΡŠΡ‘ΠΌ
    1. Π”Π΅Π»ΠΈΠΌΠΎΡΡ‚ΡŒ Π½Π°Ρ‚ΡƒΡ€Π°Π»ΡŒΠ½Ρ‹Ρ… чисСл
    2. ΠŸΡ€ΠΈΠ·Π½Π°ΠΊΠΈ дСлимости Π½Π° 2, 3, 5, 9, 10
    3. ΠŸΡ€ΠΎΡΡ‚Ρ‹Π΅ ΠΈ составныС числа.
      Π Π°Π·Π»ΠΎΠΆΠ΅Π½ΠΈΠ΅ Π½Π°Ρ‚ΡƒΡ€Π°Π»ΡŒΠ½ΠΎΠ³ΠΎ числа Π½Π° простыС ΠΌΠ½ΠΎΠΆΠΈΡ‚Π΅Π»ΠΈ
    4. Наибольший ΠΎΠ±Ρ‰ΠΈΠΉ Π΄Π΅Π»ΠΈΡ‚Π΅Π»ΡŒ ΠΈ наимСньшСС ΠΎΠ±Ρ‰Π΅Π΅ ΠΊΡ€Π°Ρ‚Π½ΠΎΠ΅
    1. ΠŸΠΎΠ»ΠΎΠΆΠΈΡ‚Π΅Π»ΡŒΠ½Ρ‹Π΅ ΠΈ ΠΎΡ‚Ρ€ΠΈΡ†Π°Ρ‚Π΅Π»ΡŒΠ½Ρ‹Π΅ числа. ΠžΠΏΡ€Π΅Π΄Π΅Π»Π΅Π½ΠΈΠ΅ ΠΊΠΎΠΎΡ€Π΄ΠΈΠ½Π°Ρ‚Π½ΠΎΠΉ прямой
    2. ΠŸΡ€ΠΎΡ‚ΠΈΠ²ΠΎΠΏΠΎΠ»ΠΎΠΆΠ½Ρ‹Π΅ числа.
      ΠœΠΎΠ΄ΡƒΠ»ΡŒ числа. Π¦Π΅Π»Ρ‹Π΅ ΠΈ Ρ€Π°Ρ†ΠΈΠΎΠ½Π°Π»ΡŒΠ½Ρ‹Π΅ числа
    3. Π‘Ρ€Π°Π²Π½Π΅Π½ΠΈΠ΅ Ρ€Π°Ρ†ΠΈΠΎΠ½Π°Π»ΡŒΠ½Ρ‹Ρ… чисСл
    4. Π‘Π»ΠΎΠΆΠ΅Π½ΠΈΠ΅ Ρ€Π°Ρ†ΠΈΠΎΠ½Π°Π»ΡŒΠ½Ρ‹Ρ… чисСл с ΠΏΠΎΠΌΠΎΡ‰ΡŒΡŽ ΠΊΠΎΠΎΡ€Π΄ΠΈΠ½Π°Ρ‚Π½ΠΎΠΉ прямой
    5. АлгСбраичСская сумма. Бвойства
    6. АлгСбраичСская сумма Ρ€Π°Ρ†ΠΈΠΎΠ½Π°Π»ΡŒΠ½Ρ‹Ρ… чисСл с ΠΎΠ΄ΠΈΠ½Π°ΠΊΠΎΠ²Ρ‹ΠΌΠΈ Π·Π½Π°ΠΊΠ°ΠΌΠΈ
    7. АлгСбраичСская сумма Ρ€Π°Ρ†ΠΈΠΎΠ½Π°Π»ΡŒΠ½Ρ‹Ρ… чисСл с Ρ€Π°Π·Π½Ρ‹ΠΌΠΈ Π·Π½Π°ΠΊΠ°ΠΌΠΈ
    8. Π£ΠΌΠ½ΠΎΠΆΠ΅Π½ΠΈΠ΅ ΠΈ Π΄Π΅Π»Π΅Π½ΠΈΠ΅ Ρ€Π°Ρ†ΠΈΠΎΠ½Π°Π»ΡŒΠ½Ρ‹Ρ… чисСл
    9. Π£ΠΌΠ½ΠΎΠΆΠ΅Π½ΠΈΠ΅ ΠΈ Π΄Π΅Π»Π΅Π½ΠΈΠ΅ ΠΎΠ±Ρ‹ΠΊΠ½ΠΎΠ²Π΅Π½Π½Ρ‹Ρ… Π΄Ρ€ΠΎΠ±Π΅ΠΉ
    10. Π”Ρ€ΠΎΠ±Π½Ρ‹Π΅ выраТСния
    11. ΠšΠΎΠΎΡ€Π΄ΠΈΠ½Π°Ρ‚Ρ‹.
      ΠšΠΎΠΎΡ€Π΄ΠΈΠ½Π°Ρ‚Π½Π°Ρ ΠΏΠ»ΠΎΡΠΊΠΎΡΡ‚ΡŒ, ΠΊΠΎΠΎΡ€Π΄ΠΈΠ½Π°Ρ‚Ρ‹ Ρ‚ΠΎΡ‡ΠΊΠΈ
    1. ΠžΡ‚Π½ΠΎΡˆΠ΅Π½ΠΈΠ΅ Π΄Π²ΡƒΡ… чисСл
    2. ΠŸΡ€ΠΎΠΏΠΎΡ€Ρ†ΠΈΡ. ОсновноС свойство ΠΏΡ€ΠΎΠΏΠΎΡ€Ρ†ΠΈΠΈ
    3. ΠŸΡ€ΡΠΌΠ°Ρ ΠΈ обратная ΠΏΡ€ΠΎΠΏΠΎΡ€Ρ†ΠΈΠΎΠ½Π°Π»ΡŒΠ½ΠΎΡΡ‚ΡŒ
    4. РСшСниС Π·Π°Π΄Π°Ρ‡ с ΠΏΠΎΠΌΠΎΡ‰ΡŒΡŽ ΠΏΡ€ΠΎΠΏΠΎΡ€Ρ†ΠΈΠΉ
    5. Π Π°Π·Π½Ρ‹Π΅ Π·Π°Π΄Π°Ρ‡ΠΈ
    1. Π£ΠΏΡ€ΠΎΡ‰Π΅Π½ΠΈΠ΅ Π²Ρ‹Ρ€Π°ΠΆΠ΅Π½ΠΈΠΉ, раскрытиС скобок
    2. РСшСниС Π»ΠΈΠ½Π΅ΠΉΠ½Ρ‹Ρ… ΡƒΡ€Π°Π²Π½Π΅Π½ΠΈΠΉ
    3. Π­Ρ‚Π°ΠΏΡ‹ Ρ€Π΅ΡˆΠ΅Π½ΠΈΡ Π»ΠΈΠ½Π΅ΠΉΠ½Ρ‹Ρ… ΡƒΡ€Π°Π²Π½Π΅Π½ΠΈΠΉ
    1. ΠΠ°Ρ‡Π°Π»ΡŒΠ½Ρ‹Π΅ понятия ΠΈ Ρ„Π°ΠΊΡ‚Ρ‹ курса Π³Π΅ΠΎΠΌΠ΅Ρ‚Ρ€ΠΈΠΈ
    2. ΠŸΠ°Ρ€Π°Π»Π»Π΅Π»ΡŒΠ½ΠΎΡΡ‚ΡŒ прямых
    3. Π¦Π΅Π½Ρ‚Ρ€Π°Π»ΡŒΠ½Π°Ρ ΠΈ осСвая симмСтрия
    4. ΠžΠΊΡ€ΡƒΠΆΠ½ΠΎΡΡ‚ΡŒ ΠΈ ΠΊΡ€ΡƒΠ³.
      Число Пи. Π”Π»ΠΈΠ½Π° окруТности. ΠŸΠ»ΠΎΡ‰Π°Π΄ΡŒ ΠΊΡ€ΡƒΠ³Π°
    5. НаглядныС прСдставлСния ΠΎ ΡˆΠ°Ρ€Π΅, сфСрС. Π€ΠΎΡ€ΠΌΡƒΠ»Ρ‹ ΠΏΠ»ΠΎΡ‰Π°Π΄ΠΈ повСрхности сфСры ΠΈ ΠΎΠ±ΡŠΡ‘ΠΌΠ° ΡˆΠ°Ρ€Π°
  1. ΠšΠΎΠ»Π»Π΅ΠΊΡ†ΠΈΡ ΠΈΠ½Ρ‚Π΅Ρ€Π°ΠΊΡ‚ΠΈΠ²Π½Ρ‹Ρ… ΠΌΠΎΠ΄Π΅Π»Π΅ΠΉ

Π’Π΅ΠΌΠ° Π΄Ρ€ΠΎΠ±ΠΈ 5 класс, ΡΡƒΡ‚ΡŒ Π΄Ρ€ΠΎΠ±ΠΈ, слоТСниС, Π²Ρ‹Ρ‡ΠΈΡ‚Π°Π½ΠΈΠ΅, Π΄Π΅Π»Π΅Π½ΠΈΠ΅, ΡƒΠΌΠ½ΠΎΠΆΠ΅Π½ΠΈΠ΅, ΠΏΡ€ΠΈΠΌΠ΅Ρ€Ρ‹ с объяснСниями.

Как ΠΏΠΎΠ½ΡΡ‚ΡŒ Π΄Ρ€ΠΎΠ±ΠΈ

ΠŸΡ€Π°ΠΊΡ‚ΠΈΡ‡Π΅ΡΠΊΠΈ ΠΊΠ°ΠΆΠ΄Ρ‹ΠΉ пятиклассник послС ΠΏΠ΅Ρ€Π²ΠΎΠ³ΠΎ знакомства с ΠΎΠ±Ρ‹ΠΊΠ½ΠΎΠ²Π΅Π½Π½Ρ‹ΠΌΠΈ дробями находится Π² нСбольшом шокС. Мало Ρ‚ΠΎΠ³ΠΎ, Ρ‡Ρ‚ΠΎ Π½ΡƒΠΆΠ½ΠΎ Π΅Ρ‰Π΅ ΠΏΠΎΠ½ΡΡ‚ΡŒ ΡΡƒΡ‚ΡŒ Π΄Ρ€ΠΎΠ±ΠΈ, Ρ‚Π°ΠΊ с Π½ΠΈΠΌΠΈ Π΅Ρ‰Π΅ придСтся Π²Ρ‹ΠΏΠΎΠ»Π½ΡΡ‚ΡŒ арифмСтичСскиС дСйствия. ПослС этого малСнькиС ΡƒΡ‡Π΅Π½ΠΈΠΊΠΈ Π±ΡƒΠ΄ΡƒΡ‚ систСматичСски Π΄ΠΎΠΏΡ€Π°ΡˆΠΈΠ²Π°Ρ‚ΡŒ своСго учитСля, Ρ€Π°Π·ΡƒΠ·Π½Π°Π²Π°Ρ‚ΡŒ ΠΊΠΎΠ³Π΄Π° ΠΆΠ΅ эти Π΄Ρ€ΠΎΠ±ΠΈ кончатся.

Π‘ΠΎΠ΄Π΅Ρ€ΠΆΠ°Π½ΠΈΠ΅ ΡΡ‚Π°Ρ‚ΡŒΠΈ

Π§Ρ‚ΠΎΠ±Ρ‹ ΠΈΠ·Π±Π΅ΠΆΠ°Ρ‚ΡŒ ΠΏΠΎΠ΄ΠΎΠ±Π½Ρ‹Ρ… ситуаций, достаточно всСго лишь ΠΊΠ°ΠΊ ΠΌΠΎΠΆΠ½ΠΎ ΠΏΡ€ΠΎΡ‰Π΅ ΠΎΠ±ΡŠΡΡΠ½ΠΈΡ‚ΡŒ дСтям эту Π½Π΅Π»Π΅Π³ΠΊΡƒΡŽ Ρ‚Π΅ΠΌΡƒ, Π° Π»ΡƒΡ‡ΡˆΠ΅ Π² ΠΈΠ³Ρ€ΠΎΠ²ΠΎΠΉ Ρ„ΠΎΡ€ΠΌΠ΅.

Π‘ΡƒΡ‚ΡŒ Π΄Ρ€ΠΎΠ±ΠΈ

ΠŸΠ΅Ρ€Π΅Π΄ Ρ‚Π΅ΠΌ, ΠΊΠ°ΠΊ ΡƒΠ·Π½Π°Ρ‚ΡŒ Ρ‡Ρ‚ΠΎ Ρ‚Π°ΠΊΠΎΠ΅ Π΄Ρ€ΠΎΠ±ΡŒ, Ρ€Π΅Π±Π΅Π½ΠΎΠΊ Π΄ΠΎΠ»ΠΆΠ΅Π½ ΠΏΠΎΠ·Π½Π°ΠΊΠΎΠΌΠΈΡ‚ΡŒΡΡ с понятиСм доля.Β Π—Π΄Π΅ΡΡŒ Π»ΡƒΡ‡ΡˆΠ΅ всСго ΠΏΠΎΠ΄ΠΎΠΉΠ΄Π΅Ρ‚ ассоциативный ΠΌΠ΅Ρ‚ΠΎΠ΄.

ΠŸΡ€Π΅Π΄ΡΡ‚Π°Π²ΡŒΡ‚Π΅ Ρ†Π΅Π»Ρ‹ΠΉ Ρ‚ΠΎΡ€Ρ‚, ΠΊΠΎΡ‚ΠΎΡ€Ρ‹ΠΉ ΠΏΠΎΠ΄Π΅Π»ΠΈΠ»ΠΈ Π½Π° нСсколько Ρ€Π°Π²Π½Ρ‹Ρ… частСй, допустим Π½Π° Ρ‡Π΅Ρ‚Ρ‹Ρ€Π΅. Π’ΠΎΠ³Π΄Π° ΠΊΠ°ΠΆΠ΄Ρ‹ΠΉ кусочСк Ρ‚ΠΎΡ€Ρ‚Π°, ΠΌΠΎΠΆΠ½ΠΎ Π½Π°Π·Π²Π°Ρ‚ΡŒ Π΄ΠΎΠ»Π΅ΠΉ. Если Π²Π·ΡΡ‚ΡŒ ΠΎΠ΄ΠΈΠ½ ΠΈΠ· Ρ‡Π΅Ρ‚Ρ‹Ρ€Π΅Ρ… кусков Ρ‚ΠΎΡ€Ρ‚Π°, Ρ‚ΠΎ ΠΎΠ½ Π±ΡƒΠ΄Π΅Ρ‚ ΠΎΠ΄Π½ΠΎΠΉ Ρ‡Π΅Ρ‚Π²Π΅Ρ€Ρ‚ΠΎΠΉ Π΄ΠΎΠ»Π΅ΠΉ.

Π”ΠΎΠ»ΠΈ Π±Ρ‹Π²Π°ΡŽΡ‚ Ρ€Π°Π·Π½Ρ‹Π΅, ΠΏΠΎΡ‚ΠΎΠΌΡƒ Ρ‡Ρ‚ΠΎ, Ρ†Π΅Π»ΠΎΠ΅ ΠΌΠΎΠΆΠ½ΠΎ ΠΏΠΎΠ΄Π΅Π»ΠΈΡ‚ΡŒ Π½Π° ΡΠΎΠ²Π΅Ρ€ΡˆΠ΅Π½Π½ΠΎ Ρ€Π°Π·Π½ΠΎΠ΅ количСство частСй. Π§Π΅ΠΌ большС Π΄ΠΎΠ»Π΅ΠΉ Π² Ρ†Π΅Π»ΠΎΠΌ, Ρ‚Π΅ΠΌ ΠΎΠ½ΠΈ мСньшС, ΠΈ Π½Π°ΠΎΠ±ΠΎΡ€ΠΎΡ‚.

Π§Ρ‚ΠΎΠ±Ρ‹ Π΄ΠΎΠ»ΠΈ ΠΌΠΎΠΆΠ½ΠΎ Π±Ρ‹Π»ΠΎ ΠΎΠ±ΠΎΠ·Π½Π°Ρ‡ΠΈΡ‚ΡŒ, ΠΏΡ€ΠΈΠ΄ΡƒΠΌΠ°Π»ΠΈ Ρ‚Π°ΠΊΠΎΠ΅ матСматичСскоС понятиС, ΠΊΠ°ΠΊ обыкновСнная Π΄Ρ€ΠΎΠ±ΡŒ. Π”Ρ€ΠΎΠ±ΡŒ ΠΏΠΎΠ·Π²ΠΎΠ»ΠΈΡ‚ Π½Π°ΠΌ Π·Π°ΠΏΠΈΡΠ°Ρ‚ΡŒ ΡΡ‚ΠΎΠ»ΡŒΠΊΠΎ Π΄ΠΎΠ»Π΅ΠΉ, сколько потрСбуСтся.

Боставными частями Π΄Ρ€ΠΎΠ±ΠΈ ΡΠ²Π»ΡΡŽΡ‚ΡΡ Ρ‡ΠΈΡΠ»ΠΈΡ‚Π΅Π»ΡŒ ΠΈ Π·Π½Π°ΠΌΠ΅Π½Π°Ρ‚Π΅Π»ΡŒ, ΠΊΠΎΡ‚ΠΎΡ€Ρ‹Π΅ Ρ€Π°Π·Π΄Π΅Π»Π΅Π½Ρ‹ Π΄Ρ€ΠΎΠ±Π½ΠΎΠΉ Ρ‡Π΅Ρ€Ρ‚ΠΎΠΉ Π»ΠΈΠ±ΠΎ Π½Π°ΠΊΠ»ΠΎΠ½Π½ΠΎΠΉ Ρ‡Π΅Ρ€Ρ‚ΠΎΠΉ. МногиС Π΄Π΅Ρ‚ΠΈ Π½Π΅ ΠΏΠΎΠ½ΠΈΠΌΠ°ΡŽΡ‚ ΠΈΡ… смысла, поэтому ΠΈ ΡΡƒΡ‚ΡŒ Π΄Ρ€ΠΎΠ±ΠΈ ΠΈΠΌ Π½Π΅ понятна. Дробная Ρ‡Π΅Ρ€Ρ‚Π° ΠΎΠ±ΠΎΠ·Π½Π°Ρ‡Π°Π΅Ρ‚ Π΄Π΅Π»Π΅Π½ΠΈΠ΅, здСсь Π½Π΅Ρ‚ Π½ΠΈΡ‡Π΅Π³ΠΎ слоТного.

Π—Π½Π°ΠΌΠ΅Π½Π°Ρ‚Π΅Π»ΡŒ принято Π·Π°ΠΏΠΈΡΡ‹Π²Π°Ρ‚ΡŒ снизу, ΠΏΠΎΠ΄ Π΄Ρ€ΠΎΠ±Π½ΠΎΠΉ Ρ‡Π΅Ρ€Ρ‚ΠΎΠΉ ΠΈΠ»ΠΈ справа ΠΎΡ‚ Π½Π°ΠΊΠ».Ρ‡Π΅Ρ€Ρ‚Ρ‹. Он ΠΏΠΎΠΊΠ°Π·Ρ‹Π²Π°Π΅Ρ‚ количСство Π΄ΠΎΠ»Π΅ΠΉ Ρ†Π΅Π»ΠΎΠ³ΠΎ. Π§ΠΈΡΠ»ΠΈΡ‚Π΅Π»ΡŒ, ΠΎΠ½ записываСтся свСрху Π½Π°Π΄ Π΄Ρ€ΠΎΠ±Π½ΠΎΠΉ Ρ‡Π΅Ρ€Ρ‚ΠΎΠΉ ΠΈΠ»ΠΈ слСва ΠΎΡ‚ Π½Π°ΠΊΠ».Ρ‡Π΅Ρ€Ρ‚Ρ‹, опрСдСляСт сколько Π΄ΠΎΠ»Π΅ΠΉ взяли.К ΠΏΡ€ΠΈΠΌΠ΅Ρ€Ρƒ Π΄Ρ€ΠΎΠ±ΡŒ 4/7. Π’ Π΄Π°Π½Π½ΠΎΠΌ случаС 7-это Π·Π½Π°ΠΌΠ΅Π½Π°Ρ‚Π΅Π»ΡŒ, ΠΏΠΎΠΊΠ°Π·Ρ‹Π²Π°Π΅Ρ‚, Ρ‡Ρ‚ΠΎ Π΅ΡΡ‚ΡŒ всСго 7 Π΄ΠΎΠ»Π΅ΠΉ, Π° Ρ‡ΠΈΡΠ»ΠΈΡ‚Π΅Π»ΡŒ 4 ΡƒΠΊΠ°Π·Ρ‹Π²Π°Π΅Ρ‚ Π½Π° Ρ‚ΠΎ, Ρ‡Ρ‚ΠΎ ΠΈΠ· сСми Π΄ΠΎΠ»Π΅ΠΉ взяли Ρ‡Π΅Ρ‚Ρ‹Ρ€Π΅.

ΠžΡΠ½ΠΎΠ²Π½Ρ‹Π΅ Π΄ΠΎΠ»ΠΈ ΠΈ ΠΈΡ… запись Π² дробях:

Помимо ΠΎΠ±Ρ‹ΠΊΠ½ΠΎΠ²Π΅Π½ΠΎΠΉ, сущСствуСт Π΅Ρ‰Π΅ ΠΈ дСсятичная Π΄Ρ€ΠΎΠ±ΡŒ.

ДСйствия с дробями 5 класс

Π’ пятом классС учатся Π²Ρ‹ΠΏΠΎΠ»Π½ΡΡ‚ΡŒ всС арифмСтичСскиС дСйствия с дробями.

ВсС дСйствия с дробями Π²Ρ‹ΠΏΠΎΠ»Π½ΡΡŽΡ‚ΡΡ ΠΏΠΎ ΠΏΡ€Π°Π²ΠΈΠ»Π°ΠΌ, ΠΈ Π½Π°Π΄Π΅ΡΡ‚ΡŒΡΡ Π½Π° Ρ‚ΠΎ, Ρ‡Ρ‚ΠΎ Π½Π΅ Π²Ρ‹ΡƒΡ‡ΠΈΠ² ΠΏΡ€Π°Π²ΠΈΠ»ΠΎ всС получится само сабой Π½Π΅ стоит. ΠŸΠΎΡΡ‚ΠΎΠΌΡƒ Π½Π΅ стоит ΠΏΡ€Π΅Π½Π΅Π±Ρ€Π΅Π³Π°Ρ‚ΡŒ устной Ρ‡Π°ΡΡ‚ΡŒΡŽ домашнСго задания ΠΏΠΎ ΠΌΠ°Ρ‚Π΅ΠΌΠ°Ρ‚ΠΈΠΊΠ΅.

ΠœΡ‹ ΡƒΠΆΠ΅ поняли, Ρ‡Ρ‚ΠΎ запись дСсятичной ΠΈ ΠΎΠ±Ρ‹ΠΊΠ½ΠΎΠ²Π΅Π½Π½ΠΎΠΉ Π΄Ρ€ΠΎΠ±ΠΈ Ρ€Π°Π·Π»ΠΈΡ‡Π½Ρ‹, ΡΠ»Π΅Π΄ΠΎΠ²Π°Ρ‚Π΅Π»ΡŒΠ½ΠΎ ΠΈ арифмСтичСскиС дСйствия Π±ΡƒΠ΄ΡƒΡ‚ Π²Ρ‹ΠΏΠΎΠ»Π½ΡΡ‚ΡŒΡΡ ΠΏΠΎ-Ρ€Π°Π·Π½ΠΎΠΌΡƒ. ДСйствия с ΠΎΠ±Ρ‹ΠΊΠ½ΠΎΠ²Π΅Π½Π½Ρ‹ΠΌΠΈ дробями зависят ΠΎΡ‚ Ρ‚Π΅Ρ… чисСл, ΠΊΠΎΡ‚ΠΎΡ€Ρ‹Π΅ стоят Π² Π·Π½Π°ΠΌΠ΅Π½Π°Ρ‚Π΅Π»Π΅, Π° Π² дСсятичной-послС запятой справа.

Для Π΄Ρ€ΠΎΠ±Π΅ΠΉ, Ρƒ ΠΊΠΎΡ‚ΠΎΡ€Ρ‹Ρ… Π·Π½Π°ΠΌΠ΅Π½Π°Ρ‚Π΅Π»ΠΈ ΠΎΠ΄ΠΈΠ½Π°ΠΊΠΎΠ²Ρ‹Π΅, Π°Π»Π³ΠΎΡ€ΠΈΡ‚ΠΌ слоТСния ΠΈ вычитания ΠΎΡ‡Π΅Π½ΡŒ прост. ДСйствия выполняСм Ρ‚ΠΎΠ»ΡŒΠΊΠΎ с числитСлями.

ΠŸΡ€ΠΈΠΌΠ΅Ρ€:

Для Π΄Ρ€ΠΎΠ±Π΅ΠΉ с Ρ€Π°Π·Π½Ρ‹ΠΌΠΈ знамСнатСлями Π½ΡƒΠΆΠ½ΠΎ Π½Π°ΠΉΡ‚ΠΈ НаимСньший ΠžΠ±Ρ‰ΠΈΠΉ Π—Π½Π°ΠΌΠ΅Π½Π°Ρ‚Π΅Π»ΡŒ ( ΠΠžΠ—).Β Π­Ρ‚ΠΎ Ρ‚ΠΎ число, ΠΊΠΎΡ‚ΠΎΡ€ΠΎΠ΅ Π±ΡƒΠ΄Π΅Ρ‚ Π΄Π΅Π»ΠΈΡ‚ΡŒΡΡ Π±Π΅Π· остатка Π½Π° всС Π·Π½Π°ΠΌΠ΅Π½Π°Ρ‚Π΅Π»ΠΈ, ΠΈ Π±ΡƒΠ΄Π΅Ρ‚ наимСньшим ΠΈΠ· Ρ‚Π°ΠΊΠΈΡ… чисСл, Ссли ΠΈΡ… нСсколько.

ΠŸΡ€ΠΈΠΌΠ΅Ρ€:

Для слоТСния Π»ΠΈΠ±ΠΎ вычитания дСсятичных Π΄Ρ€ΠΎΠ±Π΅ΠΉ, Π½ΡƒΠΆΠ½ΠΎ Π·Π°ΠΏΠΈΡΠ°Ρ‚ΡŒ ΠΈΡ… Π² столбик, запятая ΠΏΠΎΠ΄ запятой, ΠΈ ΡƒΡ€Π°Π²Π½ΠΈΡ‚ΡŒ количСство дСсятичных Π·Π½Π°ΠΊΠΎΠ² Ссли это трСбуСтся.

ΠŸΡ€ΠΈΠΌΠ΅Ρ€:

Π§Ρ‚ΠΎΠ±Ρ‹ ΠΏΠ΅Ρ€Π΅ΠΌΠ½ΠΎΠΆΠΈΡ‚ΡŒ ΠΎΠ±Ρ‹ΠΊΠ½ΠΎΠ²Π΅Π½Π½Ρ‹Π΅ Π΄Ρ€ΠΎΠ±ΠΈ просто Π½Π°ΠΉΠ΄ΠΈ ΠΏΡ€ΠΎΠΈΠ·Π²Π΅Π΄Π΅Π½ΠΈΠ΅ числитСлСй ΠΈ Π·Π½Π°ΠΌΠ΅Π½Π°Ρ‚Π΅Π»Π΅ΠΉ. ΠžΡ‡Π΅Π½ΡŒ простоС ΠΏΡ€Π°Π²ΠΈΠ»ΠΎ.

ΠŸΡ€ΠΈΠΌΠ΅Ρ€:

Π”Π΅Π»Π΅Π½ΠΈΠ΅Β  выполняСтся ΠΏΠΎ ΡΠ»Π΅Π΄ΡƒΡŽΡ‰Π΅ΠΌΡƒ Π°Π»Π³ΠΎΡ€ΠΈΡ‚ΠΌΡƒ:

  1. Π”Π΅Π»ΠΈΠΌΠΎΠ΅ Π·Π°ΠΏΠΈΡΠ°Ρ‚ΡŒ Π±Π΅Π· измСнСния
  2. Π”Π΅Π»Π΅Π½ΠΈΠ΅ ΠΏΡ€Π΅Π²Ρ€Π°Ρ‚ΠΈΡ‚ΡŒ Π²Β Β ΡƒΠΌΠ½ΠΎΠΆΠ΅Π½ΠΈΠ΅
  3. Π”Π΅Π»ΠΈΡ‚Π΅Π»ΡŒ ΠΏΠ΅Ρ€Π΅Π²Π΅Ρ€Π½ΡƒΡ‚ΡŒ (Π·Π°ΠΏΠΈΡΠ°Ρ‚ΡŒ ΠΎΠ±Ρ€Π°Ρ‚Π½ΡƒΡŽ Π΄Ρ€ΠΎΠ±ΡŒ Π΄Π΅Π»ΠΈΡ‚Π΅Π»ΡŽ)
  4. Π’Ρ‹ΠΏΠΎΠ»Π½ΠΈΡ‚ΡŒ ΡƒΠΌΠ½ΠΎΠΆΠ΅Π½ΠΈΠ΅

ΠŸΡ€ΠΈΠΌΠ΅Ρ€:

Π‘Π»ΠΎΠΆΠ΅Π½ΠΈΠ΅ Π΄Ρ€ΠΎΠ±Π΅ΠΉ, объяснСниС

Π”Π°Π²Π°ΠΉΡ‚Π΅ Π±ΠΎΠ»Π΅Π΅ ΠΏΠΎΠ΄Ρ€ΠΎΠ±Π½ΠΎ Ρ€Π°Π·Π±Π΅Ρ€Π΅ΠΌ, ΠΊΠ°ΠΊ ΡΠΊΠ»Π°Π΄Ρ‹Π²Π°Ρ‚ΡŒ ΠΎΠ±Ρ‹ΠΊΠ½ΠΎΠ²Π΅Π½Π½Ρ‹Π΅ ΠΈ дСсятичныС Π΄Ρ€ΠΎΠ±ΠΈ.

Как Π²ΠΈΠ΄Π½ΠΎ Π½Π° ΠΈΠ·ΠΎΠ±Ρ€Π°ΠΆΠ΅Π½ΠΈΠΈ Π²Ρ‹ΡˆΠ΅, Ρƒ Π΄Ρ€ΠΎΠ±ΠΈ ΠΎΠ΄Π½Π° Ρ‚Ρ€Π΅Ρ‚ΡŒΡ ΠΈ Π΄Π²Π΅ Ρ‚Ρ€Π΅Ρ‚ΡŒΠΈΡ… ΠΎΠ±Ρ‰ΠΈΠΉ Π·Π½Π°ΠΌΠ΅Π½Π°Ρ‚Π΅Π»ΡŒ Ρ‚Ρ€ΠΈ. Π—Π½Π°Ρ‡ΠΈΡ‚ трСбуСтся ΡΠ»ΠΎΠΆΠΈΡ‚ΡŒ Ρ‚ΠΎΠ»ΡŒΠΊΠΎ числитСли Π΅Π΄ΠΈΠ½ΠΈΡ†Ρƒ ΠΈ Π΄Π²Π°, Π° Π·Π½Π°ΠΌΠ΅Π½Π°Ρ‚Π΅Π»ΡŒ ΠΎΡΡ‚Π°Π²ΠΈΡ‚ΡŒ Π±Π΅Π· измСнСния. Π’ ΠΈΡ‚ΠΎΠ³Π΅ получаСтся сумма Ρ‚Ρ€ΠΈ Ρ‚Ρ€Π΅Ρ‚ΡŒΠΈΡ…. Π’Π°ΠΊΠΎΠΉ ΠΎΡ‚Π²Π΅Ρ‚, ΠΊΠΎΠ³Π΄Π° Ρ‡ΠΈΡΠ»ΠΈΡ‚Π΅Π»ΡŒ ΠΈ Π·Π½Π°ΠΌΠ΅Π½Π°Ρ‚Π΅Π»ΡŒ Π΄Ρ€ΠΎΠ±ΠΈ Ρ€Π°Π²Π½Ρ‹, ΠΌΠΎΠΆΠ½ΠΎ Π·Π°ΠΏΠΈΡΠ°Ρ‚ΡŒ ΠΊΠ°ΠΊ 1, Ρ‚Π°ΠΊ ΠΊΠ°ΠΊ 3:3 = 1.

ВрСбуСтся Π½Π°ΠΉΡ‚ΠΈ сумму Π΄Ρ€ΠΎΠ±Π΅ΠΉ Π΄Π²Π΅ Ρ‚Ρ€Π΅Ρ‚ΡŒΠΈΡ… ΠΈ Π΄Π²Π΅ дСвятых. Π’ этом случаС Π·Π½Π°ΠΌΠ΅Π½Π°Ρ‚Π΅Π»ΠΈ Ρ€Π°Π·Π»ΠΈΡ‡Π½Ρ‹, 3 ΠΈ 9. Π§Ρ‚ΠΎΠ±Ρ‹ Π²Ρ‹ΠΏΠΎΠ»Π½ΠΈΡ‚ΡŒ слоТСниС, Π½ΡƒΠΆΠ½ΠΎ ΠΏΠΎΠ΄ΠΎΠ±Ρ€Π°Ρ‚ΡŒ ΠΎΠ±Ρ‰ΠΈΠΉ. Π•ΡΡ‚ΡŒ ΠΎΡ‡Π΅Π½ΡŒ простой способ. Π’Ρ‹Π±ΠΈΡ€Π°Π΅ΠΌ наибольший Π·Π½Π°ΠΌΠ΅Π½Π°Ρ‚Π΅Π»ΡŒ, это 9. ΠŸΡ€ΠΎΠ²Π΅Ρ€ΡΠ΅ΠΌ дСлится Π»ΠΈ ΠΎΠ½ Π½Π° 3. Π’Π°ΠΊ ΠΊΠ°ΠΊ 9:3 = 3 Π±Π΅Π· остатка, ΡΠ»Π΅Π΄ΠΎΠ²Π°Ρ‚Π΅Π»ΡŒΠ½ΠΎ 9 ΠΏΠΎΠ΄Ρ…ΠΎΠ΄ΠΈΡ‚ ΠΊΠ°ΠΊ ΠΎΠ±Ρ‰ΠΈΠΉ Π·Π½Π°ΠΌΠ΅Π½Π°Ρ‚Π΅Π»ΡŒ.

Π‘Π»Π΅Π΄ΡƒΡŽΡ‰ΠΈΠΌ шагом Π½Π°Ρ…ΠΎΠ΄ΠΈΠΌ Π΄ΠΎΠΏΠΎΠ»Π½ΠΈΡ‚Π΅Π»ΡŒΠ½Ρ‹Π΅ ΠΌΠ½ΠΎΠΆΠΈΡ‚Π΅Π»ΠΈ для ΠΊΠ°ΠΆΠ΄ΠΎΠ³ΠΎ числитСля. Для этого ΠΎΠ±Ρ‰ΠΈΠΉ Π·Π½Π°ΠΌΠ΅Π½Π°Ρ‚Π΅Π»ΡŒ 9 Π΄Π΅Π»ΠΈΠΌ ΠΏΠΎΠΎΡ‡Π΅Ρ€Π΅Π΄Π½ΠΎ Π½Π° Π·Π½Π°ΠΌΠ΅Π½Π°Ρ‚Π΅Π»ΡŒ ΠΊΠ°ΠΆΠ΄ΠΎΠΉ Π΄Ρ€ΠΎΠ±ΠΈ, ΠΏΠΎΠ»ΡƒΡ‡Π΅Π½Π½Ρ‹Π΅ числа ΠΈ Π±ΡƒΠ΄ΡƒΡ‚ Π΄ΠΎΠΏΠΎΠ». ΠΌΠ½ΠΎΠΆ. Для ΠΏΠ΅Ρ€Π²ΠΎΠΉ Π΄Ρ€ΠΎΠ±ΠΈ: 9:3 = 3, дописываСм ΠΊ Ρ‡ΠΈΡΠ»ΠΈΡ‚Π΅Π»ΡŽ ΠΏΠ΅Ρ€Π²ΠΎΠΉ Π΄Ρ€ΠΎΠ±ΠΈ 3. Для Π²Ρ‚ΠΎΡ€ΠΎΠΉ Π΄Ρ€ΠΎΠ±ΠΈ: 9:9 = 1, Π΅Π΄ΠΈΠ½ΠΈΡ†Ρƒ ΠΌΠΎΠΆΠ½ΠΎ Π½Π΅ Π΄ΠΎΠΏΠΈΡΡ‹Π²Π°Ρ‚ΡŒ, Ρ‚Π°ΠΊ ΠΊΠ°ΠΊ ΠΏΡ€ΠΈ ΡƒΠΌΠ½ΠΎΠΆΠ΅Π½ΠΈΠΈ Π½Π° Π½Π΅Π΅ получится Ρ‚ΠΎ ΠΆΠ΅ самоС число.

Π’Π΅ΠΏΠ΅Ρ€ΡŒ ΡƒΠΌΠ½ΠΎΠΆΠ°Π΅ΠΌ числитСли Π½Π° ΠΈΡ… Π΄ΠΎΠΏΠΎΠ»Π½ΠΈΡ‚Π΅Π»ΡŒΠ½Ρ‹Π΅ ΠΌΠ½ΠΎΠΆΠΈΡ‚Π΅Π»ΠΈ ΠΈ складываСм Ρ€Π΅Π·ΡƒΠ»ΡŒΡ‚Π°Ρ‚Ρ‹. ΠŸΠΎΠ»ΡƒΡ‡Π΅Π½Π½Π°Ρ сумма Π΄Ρ€ΠΎΠ±ΡŒ восСмь дСвятых.

Π‘Π»ΠΎΠΆΠ΅Π½ΠΈΠ΅ дСсятичных Π΄Ρ€ΠΎΠ±Π΅ΠΉ выполняСтся ΠΏΠΎ Ρ‚ΠΎΠΌΡƒ ΠΆΠ΅ ΠΏΡ€Π°Π²ΠΈΠ»Ρƒ, Ρ‡Ρ‚ΠΎ ΠΈ слоТСниС Π½Π°Ρ‚ΡƒΡ€Π°Π»ΡŒΠ½Ρ‹Ρ… чисСл. Π’ столбик, разряд записываСтся ΠΏΠΎΠ΄ разрядом. ЕдинствСнноС ΠΎΡ‚Π»ΠΈΡ‡ΠΈΠ΅ Π² Ρ‚ΠΎΠΌ, Ρ‡Ρ‚ΠΎ Π² дСсятичных дробях Π½ΡƒΠΆΠ½ΠΎ ΠΏΡ€Π°Π²ΠΈΠ»ΡŒΠ½ΠΎ ΠΏΠΎΡΡ‚Π°Π²ΠΈΡ‚ΡŒ Π·Π°ΠΏΡΡ‚ΡƒΡŽ Π² Ρ€Π΅Π·ΡƒΠ»ΡŒΡ‚Π°Ρ‚Π΅. Для этого Π΄Ρ€ΠΎΠ±ΠΈ Π·Π°ΠΏΠΈΡΡ‹Π²Π°ΡŽΡ‚ΡΡ запятая ΠΏΠΎΠ΄ запятой, ΠΈ Π² суммС трСбуСтся лишь снСсти Π·Π°ΠΏΡΡ‚ΡƒΡŽ Π²Π½ΠΈΠ·.

НайдСм сумму Π΄Ρ€ΠΎΠ±Π΅ΠΉ 38, 251 ΠΈ 1, 56. Π§Ρ‚ΠΎΠ±Ρ‹ Π±Ρ‹Π»ΠΎ ΡƒΠ΄ΠΎΠ±Π½Π΅Π΅ Π²Ρ‹ΠΏΠΎΠ»Π½ΡΡ‚ΡŒ дСйствия, ΠΌΡ‹ уровняли количСство дСсятичных Π·Π½Π°ΠΊΠΎΠ² справа, Π΄ΠΎΠ±Π°Π²ΠΈΠ² 0.

Π‘ΠΊΠ»Π°Π΄Ρ‹Π²Π°Π΅ΠΌ Π΄Ρ€ΠΎΠ±ΠΈ Π½Π΅ обращая внимания Π½Π° Π·Π°ΠΏΡΡ‚ΡƒΡŽ. А Π² ΠΏΠΎΠ»ΡƒΡ‡Π΅Π½Π½ΠΎΠΉ суммС просто опускаСм Π·Π°ΠΏΡΡ‚ΡƒΡŽ Π²Π½ΠΈΠ·. ΠžΡ‚Π²Π΅Ρ‚: 39, 811.

Π’Ρ‹Ρ‡ΠΈΡ‚Π°Π½ΠΈΠ΅ Π΄Ρ€ΠΎΠ±Π΅ΠΉ, объяснСниС

Π§Ρ‚ΠΎΠ±Ρ‹ Π½Π°ΠΉΡ‚ΠΈ Ρ€Π°Π·Π½ΠΎΡΡ‚ΡŒ Π΄Ρ€ΠΎΠ±Π΅ΠΉ Π΄Π²Π΅ Ρ‚Ρ€Π΅Ρ‚ΡŒΠΈΡ… ΠΈ ΠΎΠ΄Π½Π° Ρ‚Ρ€Π΅Ρ‚ΡŒΡ, Π½ΡƒΠΆΠ½ΠΎ Π²Ρ‹Ρ‡ΠΈΡΠ»ΠΈΡ‚ΡŒ Ρ€Π°Π·Π½ΠΎΡΡ‚ΡŒ числитСлСй 2-1 = 1, Π° Π·Π½Π°ΠΌΠ΅Π½Π°Ρ‚Π΅Π»ΡŒ ΠΎΡΡ‚Π°Π²ΠΈΡ‚ΡŒ Π±Π΅Π· измСнСния. Π’ ΠΎΡ‚Π²Π΅Ρ‚Π΅ ΠΏΠΎΠ»ΡƒΡ‡Π°Π΅ΠΌ Ρ€Π°Π·Π½ΠΎΡΡ‚ΡŒ ΠΎΠ΄Π½Ρƒ Ρ‚Ρ€Π΅Ρ‚ΡŒΡŽ.

НайдСм Ρ€Π°Π·Π½ΠΎΡΡ‚ΡŒ Π΄Ρ€ΠΎΠ±Π΅ΠΉ ΠΏΡΡ‚ΡŒ ΡˆΠ΅ΡΡ‚Ρ‹Ρ… ΠΈ сСмь дСсятых. Находим ΠΎΠ±Ρ‰ΠΈΠΉ Π·Π½Π°ΠΌΠ΅Π½Π°Ρ‚Π΅Π»ΡŒ. Π˜ΡΠΏΠΎΠ»ΡŒΠ·ΡƒΠ΅ΠΌ способ ΠΏΠΎΠ΄Π±ΠΎΡ€Π°, ΠΈΠ· 6 ΠΈ 10 наибольший 10. ΠŸΡ€ΠΎΠ²Π΅Ρ€ΡΠ΅ΠΌ: 10 : 6 Π±Π΅Π· остатка Π½Π΅ дСлится. ДобавляСм Π΅Ρ‰Π΅ 10, получаСтся 20:6, Ρ‚ΠΎΠΆΠ΅ Π±Π΅Π· остатка Π½Π΅ дСлится. Π‘Π½ΠΎΠ²Π° ΡƒΠ²Π΅Π»ΠΈΡ‡ΠΈΠ²Π°Π΅ΠΌ Π½Π° 10, ΠΏΠΎΠ»ΡƒΡ‡ΠΈΠ»ΠΈ 30:6 = 5. ΠžΠ±Ρ‰ΠΈΠΉ Π·Π½Π°ΠΌΠ΅Π½Π°Ρ‚Π΅Π»ΡŒ 30. Π’Π°ΠΊ ΠΆΠ΅ ΠΠžΠ— ΠΌΠΎΠΆΠ½ΠΎ Π½Π°ΠΉΡ‚ΠΈ ΠΏΠΎ Ρ‚Π°Π±Π»ΠΈΡ†Π΅ умноТСния.

Находим Π΄ΠΎΠΏΠΎΠ»Π½ΠΈΡ‚Π΅Π»ΡŒΠ½Ρ‹Π΅ ΠΌΠ½ΠΎΠΆΠΈΡ‚Π΅Π»ΠΈ. 30:6 = 5 β€” для ΠΏΠ΅Ρ€Π²ΠΎΠΉ Π΄Ρ€ΠΎΠ±ΠΈ. 30:10 = 3 β€” для Π²Ρ‚ΠΎΡ€ΠΎΠΉ. ΠŸΠ΅Ρ€Π΅ΠΌΠ½ΠΎΠΆΠ°Π΅ΠΌ числитСли ΠΈ ΠΈΡ… Π΄ΠΎΠΏ.ΠΌΠ½ΠΎΠΆ. ΠŸΠΎΠ»ΡƒΡ‡Π°Π΅ΠΌ ΡƒΠΌΠ΅Π½ΡŒΡˆΠ°Π΅ΠΌΠΎΠ΅ 25/30 ΠΈ Π²Ρ‹Ρ‡ΠΈΡ‚Π°Π΅ΠΌΠΎΠ΅ 21/30. Π”Π°Π»Π΅Π΅ выполняСм Π²Ρ‹Ρ‡ΠΈΡ‚Π°Π½ΠΈΠ΅ числитСлСй, Π° Π·Π½Π°ΠΌΠ΅Π½Π°Ρ‚Π΅Π»ΡŒ оставляСм Π±Π΅Π· измСнСния.

Π’ Ρ€Π΅Π·ΡƒΠ»ΡŒΡ‚Π°Ρ‚Π΅ ΠΏΠΎΠ»ΡƒΡ‡ΠΈΠ»Π°ΡΡŒ Ρ€Π°Π·Π½ΠΎΡΡ‚ΡŒ 4/30. Π”Ρ€ΠΎΠ±ΡŒ сократимая. Π Π°Π·Π΄Π΅Π»ΠΈΠΌ Π΅Π΅ Π½Π° 2. Π’ ΠΎΡ‚Π²Π΅Ρ‚Π΅ 2/15.

Π”Π΅Π»Π΅Π½ΠΈΠ΅ дСсятичных Π΄Ρ€ΠΎΠ±Π΅ΠΉ 5 класс

Π’ этой Ρ‚Π΅ΠΌΠ΅ рассматриваСтся Π΄Π²Π° Π²Π°Ρ€ΠΈΠ°Π½Ρ‚Π° дСйствий:

Π£ΠΌΠ½ΠΎΠΆΠ΅Π½ΠΈΠ΅ дСсятичных Π΄Ρ€ΠΎΠ±Π΅ΠΉ 5 класс

ВспомнитС, ΠΊΠ°ΠΊ Π²Ρ‹ ΡƒΠΌΠ½ΠΎΠΆΠ°Π΅Ρ‚Π΅ Π½Π°Ρ‚ΡƒΡ€Π°Π»ΡŒΠ½Ρ‹Π΅ числа, Ρ‚ΠΎΡ‡Π½ΠΎ Ρ‚Π°ΠΊΠΈΠΌ ΠΆΠ΅ способом ΠΈ находят ΠΏΡ€ΠΎΠΈΠ·Π²Π΅Π΄Π΅Π½ΠΈΠ΅ дСсятичных Π΄Ρ€ΠΎΠ±Π΅ΠΉ. Π‘Π½Π°Ρ‡Π°Π»Π° разбСрСмся, ΠΊΠ°ΠΊ ΡƒΠΌΠ½ΠΎΠΆΠΈΡ‚ΡŒ Π΄Π΅ΡΡΡ‚ΠΈΡ‡Π½ΡƒΡŽ Π΄Ρ€ΠΎΠ±ΡŒ Π½Π° Π½Π°Ρ‚ΡƒΡ€Π°Π»ΡŒΠ½ΠΎΠ΅ число. Для этого:

Β ΠŸΡ€ΠΈ ΡƒΠΌΠ½ΠΎΠΆΠ΅Π½ΠΈΠΈ дСсятичной Π΄Ρ€ΠΎΠ±ΠΈ Π½Π° Π΄Π΅ΡΡΡ‚ΠΈΡ‡Π½ΡƒΡŽ, дСйствуСм Ρ‚ΠΎΡ‡Π½ΠΎ Ρ‚Π°ΠΊΠΆΠ΅.

Π‘ΠΌΠ΅ΡˆΠ°Π½Π½Ρ‹Π΅ Π΄Ρ€ΠΎΠ±ΠΈ 5 класс

ΠŸΡΡ‚ΠΈΠΊΠ»Π°ΡˆΠΊΠΈ Π»ΡŽΠ±ΡΡ‚ Π½Π°Π·Ρ‹Π²Π°Ρ‚ΡŒ Ρ‚Π°ΠΊΠΈΠ΅ Π΄Ρ€ΠΎΠ±ΠΈ Π½Π΅ ΡΠΌΠ΅ΡˆΠ°Π½Π½Ρ‹Π΅, Π° <<ΡΠΌΠ΅ΡˆΠ½Ρ‹Π΅>>, Π½Π°Π²Π΅Ρ€Π½ΠΎΠ΅ Ρ‚Π°ΠΊ Π»Π΅Π³Ρ‡Π΅ Π·Π°ΠΏΠΎΠΌΠ½ΠΈΡ‚ΡŒ. Π‘ΠΌΠ΅ΡˆΠ°Π½Π½Ρ‹Π΅ Π΄Ρ€ΠΎΠ±ΠΈ Π½Π°Π·Ρ‹Π²Π°ΡŽΡ‚ΡΡ Ρ‚Π°ΠΊ ΠΎΡ‚ Ρ‚ΠΎΠ³ΠΎ, Ρ‡Ρ‚ΠΎ ΠΎΠ½ΠΈ ΠΏΠΎΠ»ΡƒΡ‡ΠΈΠ»ΠΈΡΡŒ ΠΏΡƒΡ‚Π΅ΠΌ соСдинСния Ρ†Π΅Π»ΠΎΠ³ΠΎ Π½Π°Ρ‚ΡƒΡ€Π°Π»ΡŒΠ½ΠΎΠ³ΠΎ числа  ΠΈ ΠΎΠ±Ρ‹ΠΊΠ½ΠΎΠ²Π΅Π½Π½ΠΎΠΉ Π΄Ρ€ΠΎΠ±ΠΈ.

БмСшанная Π΄Ρ€ΠΎΠ±ΡŒ состоит ΠΈΠ· Ρ†Π΅Π»ΠΎΠΉ ΠΈ Π΄Ρ€ΠΎΠ±Π½ΠΎΠΉ части.

ΠŸΡ€ΠΈ Ρ‡Ρ‚Π΅Π½ΠΈΠΈ Ρ‚Π°ΠΊΠΈΡ… Π΄Ρ€ΠΎΠ±Π΅ΠΉ сначала Π½Π°Π·Ρ‹Π²Π°ΡŽΡ‚ Ρ†Π΅Π»ΡƒΡŽ Ρ‡Π°ΡΡ‚ΡŒ, Π·Π°Ρ‚Π΅ΠΌ Π΄Ρ€ΠΎΠ±Π½ΡƒΡŽ: ΠΎΠ΄Π½Π° цСлая Π΄Π²Π΅ Ρ‚Ρ€Π΅Ρ‚ΡŒΠΈΡ…, Π΄Π²Π΅ Ρ†Π΅Π»Ρ‹Ρ… ΠΎΠ΄Π½Π° пятая, Ρ‚Ρ€ΠΈ Ρ†Π΅Π»Ρ‹Ρ… Π΄Π²Π΅ пятых, Ρ‡Π΅Ρ‚Ρ‹Ρ€Π΅ Ρ†Π΅Π»Ρ‹Ρ… Ρ‚Ρ€ΠΈ Ρ‡Π΅Ρ‚Π²Π΅Ρ€Ρ‚Ρ‹Ρ….

Как ΠΆΠ΅ ΠΎΠ½ΠΈ ΠΏΠΎΠ»ΡƒΡ‡Π°ΡŽΡ‚ΡΡ, эти ΡΠΌΠ΅ΡˆΠ°Π½Π½Ρ‹Π΅ Π΄Ρ€ΠΎΠ±ΠΈ? ВсС довольно просто. Когда ΠΌΡ‹ ΠΏΠΎΠ»ΡƒΡ‡Π°Π΅ΠΌ Π² ΠΎΡ‚Π²Π΅Ρ‚Π΅ Π½Π΅ΠΏΡ€Π°Π²ΠΈΠ»ΡŒΠ½ΡƒΡŽ Π΄Ρ€ΠΎΠ±ΡŒ ( Π΄Ρ€ΠΎΠ±ΡŒ Ρƒ ΠΊΠΎΡ‚ΠΎΡ€ΠΎΠΉ Ρ‡ΠΈΡΠ»ΠΈΡ‚Π΅Π»ΡŒ большС знамСнатСля), ΠΌΡ‹ Π΅Π΅ Π΄ΠΎΠ»ΠΆΠ½Ρ‹ всСгда ΠΏΠ΅Ρ€Π΅Π²ΠΎΠ΄ΠΈΡ‚ΡŒ Π² ΡΠΌΠ΅ΡˆΠ°Π½Π½ΡƒΡŽ. Достаточно Ρ€Π°Π·Π΄Π΅Π»ΠΈΡ‚ΡŒ Ρ‡ΠΈΡΠ»ΠΈΡ‚Π΅Π»ΡŒ Π½Π° Π·Π½Π°ΠΌΠ΅Π½Π°Ρ‚Π΅Π»ΡŒ. Π­Ρ‚ΠΎ дСйствиС называСтся Π²Ρ‹Π΄Π΅Π»Π΅Π½ΠΈΠ΅ΠΌ Ρ†Π΅Π»ΠΎΠΉ части:

ΠŸΠ΅Ρ€Π΅Π²Π΅ΡΡ‚ΠΈ ΡΠΌΠ΅ΡˆΠ°Π½Π½ΡƒΡŽ Π΄Ρ€ΠΎΠ±ΡŒ ΠΎΠ±Ρ€Π°Ρ‚Π½ΠΎ Π² Π½Π΅ΠΏΡ€Π°Π²ΠΈΠ»ΡŒΠ½ΡƒΡŽ Ρ‚ΠΎΠΆΠ΅ нСслоТно:

ΠŸΡ€ΠΈΠΌΠ΅Ρ€Ρ‹ с дСсятичными дробями 5 класс с объяснСниСм

Много вопросов Ρƒ Π΄Π΅Ρ‚Π΅ΠΉ Π²Ρ‹Π·Ρ‹Π²Π°ΡŽΡ‚ ΠΏΡ€ΠΈΠΌΠ΅Ρ€Ρ‹ Π½Π° нСсколько дСйствий. Π Π°Π·Π±Π΅Ρ€Π΅ΠΌ ΠΏΠ°Ρ€Ρƒ Ρ‚Π°ΠΊΠΈΡ… ΠΏΡ€ΠΈΠΌΠ΅Ρ€ΠΎΠ².

ΠŸΡ€ΠΈΠΌΠ΅Ρ€ 1.

( 0,4Β Β· 8,25 β€” 2,025 ) : 0,5 =Β 

ΠŸΠ΅Ρ€Π²Ρ‹ΠΌ дСйствиСм Π½Π°Ρ…ΠΎΠ΄ΠΈΠΌ ΠΏΡ€ΠΎΠΈΠ·Π²Π΅Π΄Π΅Π½ΠΈΠ΅ чисСл 8,25 ΠΈ 0,4. ВыполняСм ΡƒΠΌΠ½ΠΎΠΆΠ΅Π½ΠΈΠ΅ ΠΏΠΎ ΠΏΡ€Π°Π²ΠΈΠ»Ρƒ. Π’ ΠΎΡ‚Π²Π΅Ρ‚Π΅ отсчитываСм справа Π½Π°Π»Π΅Π²ΠΎ Ρ‚Ρ€ΠΈ Π·Π½Π°ΠΊΠ° ΠΈ ставим Π·Π°ΠΏΡΡ‚ΡƒΡŽ.

Π’Ρ‚ΠΎΡ€ΠΎΠ΅ дСйствиС находится Ρ‚Π°ΠΌ ΠΆΠ΅ Π² скобках, это Ρ€Π°Π·Π½ΠΎΡΡ‚ΡŒ. ΠžΡ‚ 3,300 Π²Ρ‹Ρ‡ΠΈΡ‚Π°Π΅ΠΌ 2,025. ЗаписываСм дСйствиС Π² столбик, запятая ΠΏΠΎΠ΄ запятой.

Π’Ρ€Π΅Ρ‚ΡŒΠ΅ дСйствиС-Π΄Π΅Π»Π΅Π½ΠΈΠ΅. ΠŸΠΎΠ»ΡƒΡ‡Π΅Π½Π½ΡƒΡŽ Ρ€Π°Π·Π½ΠΎΡΡ‚ΡŒ Π²ΠΎ Π²Ρ‚ΠΎΡ€ΠΎΠΌ дСйствии Π΄Π΅Π»ΠΈΠΌ Π½Π° 0,5. Запятая пСрСносится Π½Π° ΠΎΠ΄ΠΈΠ½ Π·Π½Π°ΠΊ. Π Π΅Π·ΡƒΠ»ΡŒΡ‚Π°Ρ‚Β  2,55.

ΠžΡ‚Π²Π΅Ρ‚: 2,55.

ΠŸΡ€ΠΈΠΌΠ΅Ρ€ 2.

( 0, 93 + 0, 07 ) : ( 0, 93 β€” 0, 805 ) =

ΠŸΠ΅Ρ€Π²ΠΎΠ΅ дСйствиС сумма Π² скобках.Π‘ΠΊΠ»Π°Π΄Ρ‹Π²Π°Π΅ΠΌ Π² столбик, ΠΏΠΎΠΌΠ½ΠΈΠΌ, Ρ‡Ρ‚ΠΎ запятая ΠΏΠΎΠ΄ запятой. ΠŸΠΎΠ»ΡƒΡ‡Π°Π΅ΠΌ ΠΎΡ‚Π²Π΅Ρ‚ 1,00.

Π’Ρ‚ΠΎΡ€ΠΎΠ΅ дСйствиС Ρ€Π°Π·Π½ΠΎΡΡ‚ΡŒ ΠΈΠ· Π²Ρ‚ΠΎΡ€ΠΎΠΉ скобки. Π’Π°ΠΊ ΠΊΠ°ΠΊ Ρƒ ΡƒΠΌΠ΅Π½ΡŒΡˆΠ°Π΅ΠΌΠΎΠ³ΠΎ мСньшС Π·Π½Π°ΠΊΠΎΠ² послС запятой, Ρ‡Π΅ΠΌ Ρƒ Π²Ρ‹Ρ‡ΠΈΡ‚Π°Π΅ΠΌΠΎΠ³ΠΎ, добавляСм Π½Π΅Π΄ΠΎΡΡ‚Π°ΡŽΡ‰ΠΈΠΉ. Π Π΅Π·ΡƒΠ»ΡŒΡ‚Π°Ρ‚ вычитания 0 ,125.

Π’Ρ€Π΅Ρ‚ΡŒΠΈΠΌ дСйствиС Π΄Π΅Π»ΠΈΠΌ сумму Π½Π° Ρ€Π°Π·Π½ΠΎΡΡ‚ΡŒ. Запятая пСрСносится Π½Π° Ρ‚Ρ€ΠΈ Π·Π½Π°ΠΊΠ°. ΠŸΠΎΠ»ΡƒΡ‡ΠΈΠ»ΠΎΡΡŒ Π΄Π΅Π»Π΅Π½ΠΈΠ΅ 1000 Π½Π° 125.

ΠžΡ‚Π²Π΅Ρ‚: 8.

ΠŸΡ€ΠΈΠΌΠ΅Ρ€Ρ‹ с ΠΎΠ±Ρ‹ΠΊΠ½ΠΎΠ²Π΅Π½Π½Ρ‹ΠΌΠΈ дробями с Ρ€Π°Π·Π½Ρ‹ΠΌΠΈ знамСнатСлями 5 класс с объяснСниСм

Π’ ΠΏΠ΅Ρ€Π²ΠΎΠΌ ΠΏΡ€ΠΈΠΌΠ΅Ρ€Π΅ Π½Π°Ρ…ΠΎΠ΄ΠΈΠΌ сумму Π΄Ρ€ΠΎΠ±Π΅ΠΉ 5/8 ΠΈ 3/7. ΠžΠ±Ρ‰ΠΈΠΌ Π·Π½Π°ΠΌΠ΅Π½Π°Ρ‚Π΅Π»Π΅ΠΌ Π±ΡƒΠ΄Π΅Ρ‚ число 56. Находим Π΄ΠΎΠΏΠΎΠ»Π½ΠΈΡ‚Π΅Π»ΡŒΠ½Ρ‹Π΅ ΠΌΠ½ΠΎΠΆ., Ρ€Π°Π·Π΄Π΅Π»ΠΈΠΌ 56:8 = 7 ΠΈ 56:7 = 8. ДописываСм ΠΈΡ… ΠΊ ΠΏΠ΅Ρ€Π²ΠΎΠΉ ΠΈ Π²Ρ‚ΠΎΡ€ΠΎΠΉ Π΄Ρ€ΠΎΠ±ΠΈ соотвСтствСнно. ΠŸΠ΅Ρ€Π΅ΠΌΠ½ΠΎΠΆΠ°Π΅ΠΌ числитСли ΠΈ ΠΈΡ… ΠΌΠ½ΠΎΠΆΠΈΡ‚Π΅Π»ΠΈ, ΠΏΠΎΠ»ΡƒΡ‡Π°Π΅ΠΌ сумму Π΄Ρ€ΠΎΠ±Π΅ΠΉ 35/56 ΠΈ 24/56.Β  ΠŸΠΎΠ»ΡƒΡ‡ΠΈΠ»ΠΈ сумму 59/56. Π”Ρ€ΠΎΠ±ΡŒ Π½Π΅ΠΏΡ€Π°Π²ΠΈΠ»ΡŒΠ½Π°Ρ, ΠΏΠ΅Ρ€Π΅Π²ΠΎΠ΄ΠΈΠΌ Π΅Π΅ Π² смСшанноС число. ΠžΡΡ‚Π°Π»ΡŒΠ½Ρ‹Π΅ ΠΏΡ€ΠΈΠΌΠ΅Ρ€Ρ‹ Ρ€Π΅ΡˆΠ°ΡŽΡ‚ΡΡ Π°Π½Π°Π»ΠΎΠ³ΠΈΡ‡Π½ΠΎ.

ΠŸΡ€ΠΈΠΌΠ΅Ρ€Ρ‹ с дробями 5 класс для Ρ‚Ρ€Π΅Π½ΠΈΡ€ΠΎΠ²ΠΊΠΈ

Для удобства ΠΏΠ΅Ρ€Π΅Π²Π΅Π΄ΠΈΡ‚Π΅ ΡΠΌΠ΅ΡˆΠ°Π½Π½Ρ‹Π΅ Π΄Ρ€ΠΎΠ±ΠΈ Π² Π½Π΅ΠΏΡ€Π°Π²ΠΈΠ»ΡŒΠ½Ρ‹Π΅ ΠΈ выполняйтС дСйствия.

Как Π½Π°ΡƒΡ‡ΠΈΡ‚ΡŒ Ρ€Π΅Π±Π΅Π½ΠΊΠ° Π»Π΅Π³ΠΊΠΎ Ρ€Π΅ΡˆΠ°Ρ‚ΡŒ Π΄Ρ€ΠΎΠ±ΠΈ с ΠΏΠΎΠΌΠΎΡ‰ΡŒΡŽ Π»Π΅Π³ΠΎ

Π‘ ΠΏΠΎΠΌΠΎΡ‰ΡŒΡŽ Ρ‚Π°ΠΊΠΎΠ³ΠΎ конструктора ΠΌΠΎΠΆΠ½ΠΎ Π½Π΅ Ρ‚ΠΎΠ»ΡŒΠΊΠΎ Ρ…ΠΎΡ€ΠΎΡˆΠΎ Ρ€Π°Π·Π²ΠΈΠ²Π°Ρ‚ΡŒ Π²ΠΎΠΎΠ±Ρ€Π°ΠΆΠ΅Π½ΠΈΠ΅ Ρ€Π΅Π±Π΅Π½ΠΊΠ°, Π½ΠΎ ΠΈ ΠΎΠ±ΡŠΡΡΠ½ΠΈΡ‚ΡŒ наглядно Π² ΠΈΠ³Ρ€ΠΎΠ²ΠΎΠΉ Ρ„ΠΎΡ€ΠΌΠ΅, Ρ‡Ρ‚ΠΎ Ρ‚Π°ΠΊΠΎΠ΅ доля ΠΈ Π΄Ρ€ΠΎΠ±ΡŒ.

На ΠΊΠ°Ρ€Ρ‚ΠΈΠ½ΠΊΠ΅ Π½ΠΈΠΆΠ΅ ΠΏΠΎΠΊΠ°Π·Π°Π½ΠΎ, Ρ‡Ρ‚ΠΎ ΠΎΠ΄Π½Π° Ρ‡Π°ΡΡ‚ΡŒ с восСмью ΠΊΡ€ΡƒΠΆΠΊΠ°ΠΌΠΈ это Ρ†Π΅Π»ΠΎΠ΅. Π—Π½Π°Ρ‡ΠΈΡ‚, взяв ΠΏΠ°Π·Π» с Ρ‡Π΅Ρ‚Ρ‹Ρ€ΡŒΠΌΡ ΠΊΡ€ΡƒΠΆΠΊΠ°ΠΌΠΈ, получаСтся ΠΏΠΎΠ»ΠΎΠ²ΠΈΠ½Π°, ΠΈΠ»ΠΈ 1/2. На ΠΊΠ°Ρ€Ρ‚ΠΈΠ½ΠΊΠ΅ наглядно ΠΏΠΎΠΊΠ°Π·Π°Π½ΠΎ, ΠΊΠ°ΠΊ Ρ€Π΅ΡˆΠ°Ρ‚ΡŒ ΠΏΡ€ΠΈΠΌΠ΅Ρ€Ρ‹ с Π»Π΅Π³ΠΎ, Ссли ΡΡ‡ΠΈΡ‚Π°Ρ‚ΡŒ ΠΊΡ€ΡƒΠΆΠΊΠΈ Π½Π° дСталях.

Π’Ρ‹ ΠΌΠΎΠΆΠ΅Ρ‚Π΅ ΠΏΠΎΡΡ‚Ρ€ΠΎΠΈΡ‚ΡŒ башСнки ΠΈΠ· ΠΎΠΏΡ€Π΅Π΄Π΅Π»Π΅Π½Π½ΠΎΠ³ΠΎ количСства частСй ΠΈ ΠΏΠΎΠ΄ΠΏΠΈΡΠ°Ρ‚ΡŒ ΠΊΠ°ΠΆΠ΄ΡƒΡŽ ΠΈΠ· Π½ΠΈΡ…, ΠΊΠ°ΠΊ Π½Π° ΠΊΠ°Ρ€Ρ‚ΠΈΠ½ΠΊΠ΅ Π½ΠΈΠΆΠ΅. НапримСр возьмСм Π±Π°ΡˆΠ΅Π½ΠΊΡƒ ΠΈΠ· сСми частСй. КаТдая Ρ‡Π°ΡΡ‚ΡŒ Π·Π΅Π»Π΅Π½ΠΎΠ³ΠΎ конструктора Π±ΡƒΠ΄Π΅Ρ‚ 1/7. Если Π²Ρ‹ ΠΊ ΠΎΠ΄Π½ΠΎΠΉ Ρ‚Π°ΠΊΠΎΠΉ части Π΄ΠΎΠ±Π°Π²ΠΈΡ‚Π΅ Π΅Ρ‰Π΅ Π΄Π²Π΅, Ρ‚ΠΎ получится 3/7. НаглядноС объяснСниС ΠΏΡ€ΠΈΠΌΠ΅Ρ€Π° 1/7+2/7 = 3/7.

Π§Ρ‚ΠΎΠ±Ρ‹ ΠΏΠΎΠ»ΡƒΡ‡Π°Ρ‚ΡŒ пятСрки ΠΏΠΎ ΠΌΠ°Ρ‚Π΅ΠΌΠ°Ρ‚ΠΈΠΊΠ΅ Π½Π΅ Π·Π°Π±Ρ‹Π²Π°ΠΉΡ‚Π΅ ΡƒΡ‡ΠΈΡ‚ΡŒ ΠΏΡ€Π°Π²ΠΈΠ»Π° ΠΈ ΠΎΡ‚Ρ€Π°Π±Π°Ρ‚Ρ‹Π²Π°Ρ‚ΡŒ ΠΈΡ… Π½Π° ΠΏΡ€Π°ΠΊΡ‚ΠΈΠΊΠ΅.

Π­Π»Π΅ΠΊΡ‚Ρ€ΠΎΠ½Π½Ρ‹ΠΉ справочник ΠΏΠΎ ΠΌΠ°Ρ‚Π΅ΠΌΠ°Ρ‚ΠΈΠΊΠ΅ для школьников Π°Ρ€ΠΈΡ„ΠΌΠ΅Ρ‚ΠΈΠΊΠ° слоТСниС Π²Ρ‹Ρ‡ΠΈΡ‚Π°Π½ΠΈΠ΅ ΡƒΠΌΠ½ΠΎΠΆΠ΅Π½ΠΈΠ΅ Π΄Π΅Π»Π΅Π½ΠΈΠ΅ Π΄Ρ€ΠΎΠ±Π΅ΠΉ дСйствия со ΡΠΌΠ΅ΡˆΠ°Π½Π½Ρ‹ΠΌΠΈ числами

ДСйствия с дробями ΠΈ ΡΠΌΠ΅ΡˆΠ°Π½Π½Ρ‹ΠΌΠΈ числами

Π‘ΠΎΠ΄Π΅Ρ€ΠΆΠ°Π½ΠΈΠ΅

Π‘Π»ΠΎΠΆΠ΅Π½ΠΈΠ΅ ΠΈ Π²Ρ‹Ρ‡ΠΈΡ‚Π°Π½ΠΈΠ΅ Π΄Ρ€ΠΎΠ±Π΅ΠΉ

ΠŸΡ€ΠΈ слоТСнии (Π²Ρ‹Ρ‡ΠΈΡ‚Π°Π½ΠΈΠΈ) Π΄Ρ€ΠΎΠ±Π΅ΠΉ с ΠΎΠ΄ΠΈΠ½Π°ΠΊΠΎΠ²Ρ‹ΠΌΠΈ знамСнатСлями получаСтся Π΄Ρ€ΠΎΠ±ΡŒ с Ρ‚Π΅ΠΌ ΠΆΠ΅ Π·Π½Π°ΠΌΠ΅Π½Π°Ρ‚Π΅Π»Π΅ΠΌ, Π° Π΅Ρ‘ Ρ‡ΠΈΡΠ»ΠΈΡ‚Π΅Π»ΡŒ Ρ€Π°Π²Π΅Π½ суммС (разности) числитСлСй рассматриваСмых Π΄Ρ€ΠΎΠ±Π΅ΠΉ.

НапримСр,

ΠŸΡ€ΠΈ слоТСнии (Π²Ρ‹Ρ‡ΠΈΡ‚Π°Π½ΠΈΠΈ) Π΄Ρ€ΠΎΠ±Π΅ΠΉ с Ρ€Π°Π·Π½Ρ‹ΠΌΠΈ знамСнатСлями ΠΏΡ€Π΅Π΄Π²Π°Ρ€ΠΈΡ‚Π΅Π»ΡŒΠ½ΠΎ Π½ΡƒΠΆΠ½ΠΎ привСсти ΠΈΡ… ΠΊ ΠΎΠ±Ρ‰Π΅ΠΌΡƒ Π·Π½Π°ΠΌΠ΅Π½Π°Ρ‚Π΅Π»ΡŽ. Для упрощСния вычислСний ΠΆΠ΅Π»Π°Ρ‚Π΅Π»ΡŒΠ½ΠΎ ΠΏΡ€ΠΈΠ²ΠΎΠ΄ΠΈΡ‚ΡŒ Π΄Ρ€ΠΎΠ±ΠΈ ΠΊ Π½Π°ΠΈΠΌΠ΅Π½ΡŒΡˆΠ΅ΠΌΡƒ ΠΎΠ±Ρ‰Π΅ΠΌΡƒ Π·Π½Π°ΠΌΠ΅Π½Π°Ρ‚Π΅Π»ΡŽ, хотя это Π½Π΅ являСтся ΠΎΠ±ΡΠ·Π°Ρ‚Π΅Π»ΡŒΠ½Ρ‹ΠΌ.

НапримСр,

(Π² ΡƒΠ³ΠΎΠ»ΠΊΠ°Ρ… свСрху здСсь ΠΎΠ±ΠΎΠ·Π½Π°Ρ‡Π΅Π½Ρ‹ Π΄ΠΎΠΏΠΎΠ»Π½ΠΈΡ‚Π΅Π»ΡŒΠ½Ρ‹Π΅ ΠΌΠ½ΠΎΠΆΠΈΡ‚Π΅Π»ΠΈ).

Π£ΠΌΠ½ΠΎΠΆΠ΅Π½ΠΈΠ΅ ΠΈ Π΄Π΅Π»Π΅Π½ΠΈΠ΅ Π΄Ρ€ΠΎΠ±Π΅ΠΉ

ΠŸΡ€ΠΈ ΡƒΠΌΠ½ΠΎΠΆΠ΅Π½ΠΈΠΈ Π΄Ρ€ΠΎΠ±Π΅ΠΉ получаСтся Π΄Ρ€ΠΎΠ±ΡŒ, Ρ‡ΠΈΡΠ»ΠΈΡ‚Π΅Π»ΡŒ ΠΊΠΎΡ‚ΠΎΡ€ΠΎΠΉ Ρ€Π°Π²Π΅Π½ ΠΏΡ€ΠΎΠΈΠ·Π²Π΅Π΄Π΅Π½ΠΈΡŽ числитСлСй, Π° Π·Π½Π°ΠΌΠ΅Π½Π°Ρ‚Π΅Π»ΡŒ – ΠΏΡ€ΠΎΠΈΠ·Π²Π΅Π΄Π΅Π½ΠΈΡŽ Π·Π½Π°ΠΌΠ΅Π½Π°Ρ‚Π΅Π»Π΅ΠΉ.

НапримСр,

Π”Π΅Π»Π΅Π½ΠΈΠ΅ Π΄Ρ€ΠΎΠ±Π΅ΠΉ осущСствляСтся Π² соотвСтствии со ΡΠ»Π΅Π΄ΡƒΡŽΡ‰ΠΈΠΌ ΠΏΡ€Π°Π²ΠΈΠ»ΠΎΠΌ:

Иногда это ΠΏΡ€Π°Π²ΠΈΠ»ΠΎ Ρ„ΠΎΡ€ΠΌΡƒΠ»ΠΈΡ€ΡƒΡŽΡ‚ Ρ‚Π°ΠΊ: для Ρ‚ΠΎΠ³ΠΎ, Ρ‡Ρ‚ΠΎΠ±Ρ‹ Ρ€Π°Π·Π΄Π΅Π»ΠΈΡ‚ΡŒ ΠΏΠ΅Ρ€Π²ΡƒΡŽ Π΄Ρ€ΠΎΠ±ΡŒ Π½Π° Π²Ρ‚ΠΎΡ€ΡƒΡŽ, Π½ΡƒΠΆΠ½ΠΎ ΠΏΠ΅Ρ€Π²ΡƒΡŽ Π΄Ρ€ΠΎΠ±ΡŒ ΡƒΠΌΠ½ΠΎΠΆΠΈΡ‚ΡŒ Π½Π° ΠΏΠ΅Ρ€Π΅Π²Π΅Ρ€Π½ΡƒΡ‚ΡƒΡŽ Π²Ρ‚ΠΎΡ€ΡƒΡŽ.

Π’ частности,

ДСйствия со ΡΠΌΠ΅ΡˆΠ°Π½Π½Ρ‹ΠΌΠΈ числами

Для Ρ‚ΠΎΠ³ΠΎ, Ρ‡Ρ‚ΠΎΠ±Ρ‹ ΠΈΠ·Π±Π΅ΠΆΠ°Ρ‚ΡŒ ошибок ΠΏΡ€ΠΈ Π²Ρ‹ΠΏΠΎΠ»Π½Π΅Π½ΠΈΠΈ арифмСтичСских дСйствий со ΡΠΌΠ΅ΡˆΠ°Π½Π½Ρ‹ΠΌΠΈ числами, рСкомСндуСтся сначала ΠΎΠ±Ρ€Π°Ρ‚ΠΈΡ‚ΡŒ ΡΠΌΠ΅ΡˆΠ°Π½Π½Ρ‹Π΅ числа Π² Π½Π΅ΠΏΡ€Π°Π²ΠΈΠ»ΡŒΠ½Ρ‹Π΅ Π΄Ρ€ΠΎΠ±ΠΈ, Π·Π°Ρ‚Π΅ΠΌ Π²Ρ‹ΠΏΠΎΠ»Π½ΠΈΡ‚ΡŒ Π½ΡƒΠΆΠ½Ρ‹Π΅ арифмСтичСскиС дСйствия, Π° ΠΏΠΎΡ‚ΠΎΠΌ, Ссли это трСбуСтся, ΠΎΠ±Ρ€Π°Ρ‚ΠΈΡ‚ΡŒ Ρ€Π΅Π·ΡƒΠ»ΡŒΡ‚Π°Ρ‚ Π² смСшанноС число.

ΠŸΠ Π˜ΠœΠ•Π . Найти сумму, Ρ€Π°Π·Π½ΠΎΡΡ‚ΡŒ, ΠΏΡ€ΠΎΠΈΠ·Π²Π΅Π΄Π΅Π½ΠΈΠ΅ ΠΈ частноС ΡΠΌΠ΅ΡˆΠ°Π½Π½Ρ‹Ρ… чисСл

Β  ΠΈ Β 

Π Π•Π¨Π•ΠΠ˜Π•. ΠŸΡ€Π΅ΠΎΠ±Ρ€Π°Π·ΡƒΠ΅ΠΌ эти числа Π² Π½Π΅ΠΏΡ€Π°Π²ΠΈΠ»ΡŒΠ½Ρ‹Π΅ Π΄Ρ€ΠΎΠ±ΠΈ:

Π”Π°Π»Π΅Π΅ ΠΏΠΎΠ»ΡƒΡ‡Π°Π΅ΠΌ:

ДСйствия с дробями: ΠΏΡ€Π°Π²ΠΈΠ»Π°, ΠΏΡ€ΠΈΠΌΠ΅Ρ€Ρ‹, Ρ€Π΅ΡˆΠ΅Π½ΠΈΡ

Данная ΡΡ‚Π°Ρ‚ΡŒΡ рассматриваСт дСйствия Π½Π°Π΄ дробями. Π‘ΡƒΠ΄ΡƒΡ‚Β  сформированы ΠΈ обоснованы ΠΏΡ€Π°Π²ΠΈΠ»Π° слоТСния, вычитания, умноТСния, дСлСния ΠΈΠ»ΠΈ возвСдСния Π² ΡΡ‚Π΅ΠΏΠ΅Π½ΡŒ Π΄Ρ€ΠΎΠ±Π΅ΠΉ Π²ΠΈΠ΄Π° AB, Π³Π΄Π΅ A ΠΈ B ΠΌΠΎΠ³ΡƒΡ‚ Π±Ρ‹Ρ‚ΡŒ числами, числовыми выраТСниями ΠΈΠ»ΠΈ выраТСниями с ΠΏΠ΅Ρ€Π΅ΠΌΠ΅Π½Π½Ρ‹ΠΌΠΈ. Π’ Π·Π°ΠΊΠ»ΡŽΡ‡Π΅Π½ΠΈΠΈ Π±ΡƒΠ΄ΡƒΡ‚ рассмотрСны ΠΏΡ€ΠΈΠΌΠ΅Ρ€Ρ‹ Ρ€Π΅ΡˆΠ΅Π½ΠΈΡ с ΠΏΠΎΠ΄Ρ€ΠΎΠ±Π½Ρ‹ΠΌ описаниСм.

ΠŸΡ€Π°Π²ΠΈΠ»Π° выполнСния дСйствий с числовыми дробями ΠΎΠ±Ρ‰Π΅Π³ΠΎ Π²ΠΈΠ΄Π°

ЧисловыС Π΄Ρ€ΠΎΠ±ΠΈ ΠΎΠ±Ρ‰Π΅Π³ΠΎ Π²ΠΈΠ΄Π° ΠΈΠΌΠ΅ΡŽΡ‚ Ρ‡ΠΈΡΠ»ΠΈΡ‚Π΅Π»ΡŒ ΠΈ Π·Π½Π°ΠΌΠ΅Π½Π°Ρ‚Π΅Π»ΡŒ, Π² ΠΊΠΎΡ‚ΠΎΡ€Ρ‹Ρ… ΠΈΠΌΠ΅ΡŽΡ‚ΡΡ Π½Π°Ρ‚ΡƒΡ€Π°Π»ΡŒΠ½Ρ‹Π΅ числа ΠΈΠ»ΠΈ числовыС выраТСния. Если Ρ€Π°ΡΡΠΌΠΎΡ‚Ρ€Π΅Ρ‚ΡŒ Ρ‚Π°ΠΊΠΈΠ΅ Π΄Ρ€ΠΎΠ±ΠΈ, ΠΊΠ°ΠΊ 35,Β 2,84,Β 1+2Β·34Β·(5-2),Β 34+782,3-0,8,Β 12Β·2,Β Ο€1-23+Ο€,Β 20,5lnΒ 3, Ρ‚ΠΎ Π²ΠΈΠ΄Π½ΠΎ, Ρ‡Ρ‚ΠΎ Ρ‡ΠΈΡΠ»ΠΈΡ‚Π΅Π»ΡŒ ΠΈ Π·Π½Π°ΠΌΠ΅Π½Π°Ρ‚Π΅Π»ΡŒ ΠΌΠΎΠΆΠ΅Ρ‚ ΠΈΠΌΠ΅Ρ‚ΡŒ Π½Π΅ Ρ‚ΠΎΠ»ΡŒΠΊΠΎ числа, Π½ΠΎ ΠΈ выраТСния Ρ€Π°Π·Π»ΠΈΡ‡Π½ΠΎΠ³ΠΎ ΠΏΠ»Π°Π½Π°.

ΠžΠΏΡ€Π΅Π΄Π΅Π»Π΅Π½ΠΈΠ΅ 1

Π‘ΡƒΡ‰Π΅ΡΡ‚Π²ΡƒΡŽΡ‚ ΠΏΡ€Π°Π²ΠΈΠ»Π°, ΠΏΠΎ ΠΊΠΎΡ‚ΠΎΡ€Ρ‹ΠΌ ΠΈΠ΄Π΅Ρ‚ Π²Ρ‹ΠΏΠΎΠ»Π½Π΅Π½ΠΈΠ΅ дСйствий с ΠΎΠ±Ρ‹ΠΊΠ½ΠΎΠ²Π΅Π½Π½Ρ‹ΠΌΠΈ дробями. Оно ΠΏΠΎΠ΄Ρ…ΠΎΠ΄ΠΈΡ‚ ΠΈ для Π΄Ρ€ΠΎΠ±Π΅ΠΉ ΠΎΠ±Ρ‰Π΅Π³ΠΎ Π²ΠΈΠ΄Π°:

  • ΠŸΡ€ΠΈ Π²Ρ‹Ρ‡ΠΈΡ‚Π°Π½ΠΈΠΈ Π΄Ρ€ΠΎΠ±Π΅ΠΉ с ΠΎΠ΄ΠΈΠ½Π°ΠΊΠΎΠ²Ρ‹ΠΌΠΈ знамСнатСлями ΡΠΊΠ»Π°Π΄Ρ‹Π²Π°ΡŽΡ‚ΡΡ Ρ‚ΠΎΠ»ΡŒΠΊΠΎ числитСли, Π° Π·Π½Π°ΠΌΠ΅Π½Π°Ρ‚Π΅Π»ΡŒ остаСтся ΠΏΡ€Π΅ΠΆΠ½ΠΈΠΌ, Π° ΠΈΠΌΠ΅Π½Π½ΠΎ: adΒ±cd=aΒ±cd, значСния a,Β cΒ ΠΈΒ dβ‰ 0 ΡΠ²Π»ΡΡŽΡ‚ΡΡ Π½Π΅ΠΊΠΎΡ‚ΠΎΡ€Ρ‹ΠΌΠΈ числами ΠΈΠ»ΠΈ числовыми выраТСниями.
  • ΠŸΡ€ΠΈ слоТСнии ΠΈΠ»ΠΈ Π²Ρ‹Ρ‡ΠΈΡ‚Π°Π½ΠΈΠΈ Π΄Ρ€ΠΎΠ±ΠΈ ΠΏΡ€ΠΈ Ρ€Π°Π·Π½Ρ‹Ρ… знамСнатСлях, Π½Π΅ΠΎΠ±Ρ…ΠΎΠ΄ΠΈΠΌΠΎ произвСсти ΠΏΡ€ΠΈΠ²Π΅Π΄Π΅Π½ΠΈΠ΅ ΠΊ ΠΎΠ±Ρ‰Π΅ΠΌΡƒ, послС Ρ‡Π΅Π³ΠΎ произвСсти слоТСниС ΠΈΠ»ΠΈ Π²Ρ‹Ρ‡ΠΈΡ‚Π°Π½ΠΈΠ΅ ΠΏΠΎΠ»ΡƒΡ‡Π΅Π½Π½Ρ‹Ρ… Π΄Ρ€ΠΎΠ±Π΅ΠΉ с ΠΎΠ΄ΠΈΠ½Π°ΠΊΠΎΠ²Ρ‹ΠΌΠΈ показатСлями.Β  Π‘ΡƒΠΊΠ²Π΅Π½Π½ΠΎ это выглядит Ρ‚Π°ΠΊΠΎΠΌ ΠΎΠ±Ρ€Π°Π·ΠΎΠΌ abΒ±cd=aΒ·pΒ±cΒ·rs, Π³Π΄Π΅ значСния a,Β bβ‰ 0,Β c,Β dβ‰ 0,Β pβ‰ 0,Β rβ‰ 0,Β sβ‰ 0 ΡΠ²Π»ΡΡŽΡ‚ΡΡ Π΄Π΅ΠΉΡΡ‚Π²ΠΈΡ‚Π΅Π»ΡŒΠ½Ρ‹ΠΌΠΈ числами, Π°Β bΒ·p=dΒ·r=s. Когда p=dΒ ΠΈΒ r=b, Ρ‚ΠΎΠ³Π΄Π° abΒ±cd=aΒ·dΒ±cΒ·dbΒ·d.
  • ΠŸΡ€ΠΈ ΡƒΠΌΠ½ΠΎΠΆΠ΅Π½ΠΈΠΈ Π΄Ρ€ΠΎΠ±Π΅ΠΉ выполняСтся дСйствиС с числитСлями, послС Ρ‡Π΅Π³ΠΎ со знамСнатСлями, Ρ‚ΠΎΠ³Π΄Π° ΠΏΠΎΠ»ΡƒΡ‡ΠΈΠΌ abΒ·cd=aΒ·cbΒ·d, Π³Π΄Π΅ a,Β bβ‰ 0,Β c,Β dβ‰ 0Β Π²Ρ‹ΡΡ‚ΡƒΠΏΠ°ΡŽΡ‚ Π² Ρ€ΠΎΠ»ΠΈ Π΄Π΅ΠΉΡΡ‚Π²ΠΈΡ‚Π΅Π»ΡŒΠ½Ρ‹Ρ… чисСл.
  • ΠŸΡ€ΠΈ Π΄Π΅Π»Π΅Π½ΠΈΠΈ Π΄Ρ€ΠΎΠ±ΠΈ Π½Π° Π΄Ρ€ΠΎΠ±ΡŒ ΠΏΠ΅Ρ€Π²ΡƒΡŽ ΡƒΠΌΠ½ΠΎΠΆΠ°Π΅ΠΌ Π½Π° Π²Ρ‚ΠΎΡ€ΡƒΡŽ ΠΎΠ±Ρ€Π°Ρ‚Π½ΡƒΡŽ, Ρ‚ΠΎ Π΅ΡΡ‚ΡŒ ΠΏΡ€ΠΎΠΈΠ·Π²ΠΎΠ΄ΠΈΠΌ Π·Π°ΠΌΠ΅Π½Ρƒ мСстами числитСля ΠΈ знамСнатСля: ab:cd=abΒ·dc.

ОбоснованиС ΠΏΡ€Π°Π²ΠΈΠ»

ΠžΠΏΡ€Π΅Π΄Π΅Π»Π΅Π½ΠΈΠ΅ 2

Π‘ΡƒΡ‰Π΅ΡΡ‚Π²ΡƒΡŽΡ‚ ΡΠ»Π΅Π΄ΡƒΡŽΡ‰ΠΈΠ΅ матСматичСскиС ΠΌΠΎΠΌΠ΅Π½Ρ‚Ρ‹, Π½Π° ΠΊΠΎΡ‚ΠΎΡ€Ρ‹Π΅ слСдуСт ΠΎΠΏΠΈΡ€Π°Ρ‚ΡŒΡΡ ΠΏΡ€ΠΈ вычислСнии:

  • дробная Ρ‡Π΅Ρ€Ρ‚Π° ΠΎΠ·Π½Π°Ρ‡Π°Π΅Ρ‚ Π·Π½Π°ΠΊ дСлСния;
  • Π΄Π΅Π»Π΅Π½ΠΈΠ΅ Π½Π° число рассматриваСтся ΠΊΠ°ΠΊ ΡƒΠΌΠ½ΠΎΠΆΠ΅Π½ΠΈΠ΅ Π½Π° Π΅Π³ΠΎ ΠΎΠ±Ρ€Π°Ρ‚Π½ΠΎΠ΅ Π·Π½Π°Ρ‡Π΅Π½ΠΈΠ΅;
  • ΠΏΡ€ΠΈΠΌΠ΅Π½Π΅Π½ΠΈΠ΅ свойства дСйствий с Π΄Π΅ΠΉΡΡ‚Π²ΠΈΡ‚Π΅Π»ΡŒΠ½Ρ‹ΠΌΠΈ числами;
  • ΠΏΡ€ΠΈΠΌΠ΅Π½Π΅Π½ΠΈΠ΅ основного свойства Π΄Ρ€ΠΎΠ±ΠΈ ΠΈ числовых нСравСнств.

Π‘ ΠΈΡ… ΠΏΠΎΠΌΠΎΡ‰ΡŒΡŽ ΠΌΠΎΠΆΠ½ΠΎ ΠΏΡ€ΠΎΠΈΠ·Π²ΠΎΠ΄ΠΈΡ‚ΡŒ прСобразования Π²ΠΈΠ΄Π°:

adΒ±cd=aΒ·d-1Β±cΒ·d-1=aΒ±cΒ·d-1=aΒ±cd;abΒ±cd=aΒ·pbΒ·pΒ±cΒ·rdΒ·r=aΒ·psΒ±cΒ·es=aΒ·pΒ±cΒ·rs;abΒ·cd=aΒ·dbΒ·dΒ·bΒ·cbΒ·d=aΒ·dΒ·aΒ·d-1Β·bΒ·cΒ·bΒ·d-1==aΒ·dΒ·bΒ·cΒ·bΒ·d-1Β·bΒ·d-1=aΒ·dΒ·bΒ·cbΒ·dΒ·bΒ·d-1==(aΒ·c)Β·(bΒ·d)-1=aΒ·cbΒ·d

ΠŸΡ€ΠΈΠΌΠ΅Ρ€Ρ‹

Π’ ΠΏΡ€Π΅Π΄Ρ‹Π΄ΡƒΡ‰Π΅ΠΌ ΠΏΡƒΠ½ΠΊΡ‚Π΅ Π±Ρ‹Π»ΠΎ сказано ΠΏΡ€ΠΎ дСйствия с дробями. ИмСнно послС этого Π΄Ρ€ΠΎΠ±ΡŒ нуТдаСтся Π² ΡƒΠΏΡ€ΠΎΡ‰Π΅Π½ΠΈΠΈ. ΠŸΠΎΠ΄Ρ€ΠΎΠ±Π½ΠΎ эта Ρ‚Π΅ΠΌΠ° Π±Ρ‹Π»Π° рассмотрСна Π² ΠΏΡƒΠ½ΠΊΡ‚Π΅ ΠΎ ΠΏΡ€Π΅ΠΎΠ±Ρ€Π°Π·ΠΎΠ²Π°Π½ΠΈΠΈ Π΄Ρ€ΠΎΠ±Π΅ΠΉ.

Для Π½Π°Ρ‡Π°Π»Π° рассмотрим ΠΏΡ€ΠΈΠΌΠ΅Ρ€ слоТСния ΠΈ вычитания Π΄Ρ€ΠΎΠ±Π΅ΠΉ с ΠΎΠ΄ΠΈΠ½Π°ΠΊΠΎΠ²Ρ‹ΠΌ Π·Π½Π°ΠΌΠ΅Π½Π°Ρ‚Π΅Π»Π΅ΠΌ.

ΠŸΡ€ΠΈΠΌΠ΅Ρ€ 1

Π”Π°Π½Ρ‹ Π΄Ρ€ΠΎΠ±ΠΈ 82,7Β ΠΈ 12,7, Ρ‚ΠΎ ΠΏΠΎ ΠΏΡ€Π°Π²ΠΈΠ»Ρƒ Π½Π΅ΠΎΠ±Ρ…ΠΎΠ΄ΠΈΠΌΠΎ Ρ‡ΠΈΡΠ»ΠΈΡ‚Π΅Π»ΡŒ ΡΠ»ΠΎΠΆΠΈΡ‚ΡŒ, Π° Π·Π½Π°ΠΌΠ΅Π½Π°Ρ‚Π΅Π»ΡŒ ΠΏΠ΅Ρ€Π΅ΠΏΠΈΡΠ°Ρ‚ΡŒ.

РСшСниС

Π’ΠΎΠ³Π΄Π° ΠΏΠΎΠ»ΡƒΡ‡Π°Π΅ΠΌ Π΄Ρ€ΠΎΠ±ΡŒ Π²ΠΈΠ΄Π° 8+12,7. ПослС выполнСния слоТСния ΠΏΠΎΠ»ΡƒΡ‡Π°Π΅ΠΌ Π΄Ρ€ΠΎΠ±ΡŒ Π²ΠΈΠ΄Π° 8+12,7=92,7=9027=313. Π—Π½Π°Ρ‡ΠΈΡ‚, 82,7+12,7=8+12,7=92,7=9027=313.

ΠžΡ‚Π²Π΅Ρ‚:Β 82,7+12,7=313

Π˜ΠΌΠ΅Π΅Ρ‚ΡΡ Π΄Ρ€ΡƒΠ³ΠΎΠΉ способ Ρ€Π΅ΡˆΠ΅Π½ΠΈΡ. Для Π½Π°Ρ‡Π°Π»Π° производится ΠΏΠ΅Ρ€Π΅Ρ…ΠΎΠ΄ ΠΊ Π²ΠΈΠ΄Ρƒ ΠΎΠ±Ρ‹ΠΊΠ½ΠΎΠ²Π΅Π½Π½ΠΎΠΉ Π΄Ρ€ΠΎΠ±ΠΈ, послС Ρ‡Π΅Π³ΠΎ выполняСм ΡƒΠΏΡ€ΠΎΡ‰Π΅Π½ΠΈΠ΅. Π­Ρ‚ΠΎ выглядит Ρ‚Π°ΠΊΠΈΠΌ ΠΎΠ±Ρ€Π°Π·ΠΎΠΌ:

82,7+12,7=8027+1027=9027=313

ΠŸΡ€ΠΈΠΌΠ΅Ρ€ 2

ΠŸΡ€ΠΎΠΈΠ·Π²Π΅Π΄Π΅ΠΌ Π²Ρ‹Ρ‡ΠΈΡ‚Π°Π½ΠΈΠ΅ ΠΈΠ· 1-23Β·log23Β·log25+1Β Π΄Ρ€ΠΎΠ±ΠΈ Π²ΠΈΠ΄Π° 233Β·log23Β·log25+1.

Π’Π°ΠΊ ΠΊΠ°ΠΊ Π΄Π°Π½Ρ‹ Ρ€Π°Π²Π½Ρ‹Π΅ Π·Π½Π°ΠΌΠ΅Π½Π°Ρ‚Π΅Π»ΠΈ, Π·Π½Π°Ρ‡ΠΈΡ‚, Ρ‡Ρ‚ΠΎ ΠΌΡ‹ выполняСм вычислСниС Π΄Ρ€ΠΎΠ±ΠΈ ΠΏΡ€ΠΈ ΠΎΠ΄ΠΈΠ½Π°ΠΊΠΎΠ²ΠΎΠΌ Π·Π½Π°ΠΌΠ΅Π½Π°Ρ‚Π΅Π»Π΅. ΠŸΠΎΠ»ΡƒΡ‡ΠΈΠΌ, Ρ‡Ρ‚ΠΎ

1-23Β·log23Β·log25+1-233Β·log23Β·log25+1=1-2-233Β·log23Β·log25+1

Π˜ΠΌΠ΅ΡŽΡ‚ΡΡ ΠΏΡ€ΠΈΠΌΠ΅Ρ€Ρ‹ вычислСния Π΄Ρ€ΠΎΠ±Π΅ΠΉ с Ρ€Π°Π·Π½Ρ‹ΠΌΠΈ знамСнатСлями. Π’Π°ΠΆΠ½Ρ‹ΠΉ ΠΏΡƒΠ½ΠΊΡ‚ – это ΠΏΡ€ΠΈΠ²Π΅Π΄Π΅Π½ΠΈΠ΅ ΠΊ ΠΎΠ±Ρ‰Π΅ΠΌΡƒ Π·Π½Π°ΠΌΠ΅Π½Π°Ρ‚Π΅Π»ΡŽ. Π‘Π΅Π· этого ΠΌΡ‹ Π½Π΅ смоТСм Π²Ρ‹ΠΏΠΎΠ»Π½ΡΡ‚ΡŒ дальнСйшиС дСйствия с дробями.

ΠŸΡ€ΠΎΡ†Π΅ΡΡ ΠΎΡ‚Π΄Π°Π»Π΅Π½Π½ΠΎ Π½Π°ΠΏΠΎΠΌΠΈΠ½Π°Π΅Ρ‚ ΠΏΡ€ΠΈΠ²Π΅Π΄Π΅Π½ΠΈΠ΅ ΠΊ ΠΎΠ±Ρ‰Π΅ΠΌΡƒ Π·Π½Π°ΠΌΠ΅Π½Π°Ρ‚Π΅Π»ΡŽ. Π’ΠΎ Π΅ΡΡ‚ΡŒ производится поиск наимСньшСго ΠΎΠ±Ρ‰Π΅Π³ΠΎ дСлитСля Π² Π·Π½Π°ΠΌΠ΅Π½Π°Ρ‚Π΅Π»Π΅, послС Ρ‡Π΅Π³ΠΎ Π΄ΠΎΠ±Π°Π²Π»ΡΡŽΡ‚ΡΡ Π½Π΅Π΄ΠΎΡΡ‚Π°ΡŽΡ‰ΠΈΠ΅ ΠΌΠ½ΠΎΠΆΠΈΡ‚Π΅Π»ΠΈ ΠΊ дробям.

Если складываСмыС Π΄Ρ€ΠΎΠ±ΠΈ Π½Π΅ ΠΈΠΌΠ΅ΡŽΡ‚ ΠΎΠ±Ρ‰ΠΈΡ… ΠΌΠ½ΠΎΠΆΠΈΡ‚Π΅Π»Π΅ΠΉ, Ρ‚ΠΎΠ³Π΄Π° ΠΈΠΌ ΠΌΠΎΠΆΠ΅Ρ‚ ΡΡ‚Π°Ρ‚ΡŒ ΠΈΡ… ΠΏΡ€ΠΎΠΈΠ·Π²Π΅Π΄Π΅Π½ΠΈΠ΅.

ΠŸΡ€ΠΈΠΌΠ΅Ρ€ 3

Рассмотрим Π½Π° ΠΏΡ€ΠΈΠΌΠ΅Ρ€Π΅ слоТСния Π΄Ρ€ΠΎΠ±Π΅ΠΉ 235+1Β ΠΈ 12.Β 

РСшСниС

Π’ Π΄Π°Π½Π½ΠΎΠΌ случаС ΠΎΠ±Ρ‰ΠΈΠΌ Π·Π½Π°ΠΌΠ΅Π½Π°Ρ‚Π΅Π»Π΅ΠΌ выступаСт ΠΏΡ€ΠΎΠΈΠ·Π²Π΅Π΄Π΅Π½ΠΈΠ΅ Π·Π½Π°ΠΌΠ΅Π½Π°Ρ‚Π΅Π»Π΅ΠΉ. Π’ΠΎΠ³Π΄Π° ΠΏΠΎΠ»ΡƒΡ‡Π°Π΅ΠΌ, Ρ‡Ρ‚ΠΎ 2Β·35+1. Π’ΠΎΠ³Π΄Π° ΠΏΡ€ΠΈ выставлСнии Π΄ΠΎΠΏΠΎΠ»Π½ΠΈΡ‚Π΅Π»ΡŒΠ½Ρ‹Ρ… ΠΌΠ½ΠΎΠΆΠΈΡ‚Π΅Π»Π΅ΠΉ ΠΈΠΌΠ΅Π΅ΠΌ, Ρ‡Ρ‚ΠΎ ΠΊ ΠΏΠ΅Ρ€Π²ΠΎΠΉ Π΄Ρ€ΠΎΠ±ΠΈ ΠΎΠ½ Ρ€Π°Π²Π΅Π½ 2, Π° ΠΊΠΎ Π²Ρ‚ΠΎΡ€ΠΎΠΉ 35+1. ПослС пСрСмноТСния Π΄Ρ€ΠΎΠ±ΠΈ приводятся ΠΊ Π²ΠΈΠ΄Ρƒ 42Β·35+1. ΠžΠ±Ρ‰Π΅Π΅ ΠΏΡ€ΠΈΠ²Π΅Π΄Π΅Π½ΠΈΠ΅ 12 Π±ΡƒΠ΄Π΅Ρ‚ ΠΈΠΌΠ΅Ρ‚ΡŒ Π²ΠΈΠ΄ 35+12Β·35+1. ΠŸΠΎΠ»ΡƒΡ‡Π΅Π½Π½Ρ‹Π΅ Π΄Ρ€ΠΎΠ±Π½Ρ‹Π΅ выраТСния складываСм ΠΈ ΠΏΠΎΠ»ΡƒΡ‡Π°Π΅ΠΌ, Ρ‡Ρ‚ΠΎ

235+1+12=2Β·22Β·35+1+1Β·35+12Β·35+1==42Β·35+1+35+12Β·35+1=4+35+12Β·35+1=5+352Β·35+1

ΠžΡ‚Π²Π΅Ρ‚:Β 235+1+12=5+352Β·35+1

Когда ΠΈΠΌΠ΅Π΅ΠΌ Π΄Π΅Π»ΠΎ с дробями ΠΎΠ±Ρ‰Π΅Π³ΠΎ Π²ΠΈΠ΄Π°, Ρ‚ΠΎΠ³Π΄Π° ΠΎ наимСньшСм ΠΎΠ±Ρ‰Π΅ΠΌ Π·Π½Π°ΠΌΠ΅Π½Π°Ρ‚Π΅Π»Π΅ ΠΎΠ±Ρ‹Ρ‡Π½ΠΎ Π΄Π΅Π»ΠΎ Π½Π΅ ΠΈΠ΄Π΅Ρ‚.Β  Π’ качСствС знамСнатСля Π½Π΅Ρ€Π΅Π½Ρ‚Π°Π±Π΅Π»ΡŒΠ½ΠΎ ΠΏΡ€ΠΈΠ½ΠΈΠΌΠ°Ρ‚ΡŒ ΠΏΡ€ΠΎΠΈΠ·Π²Π΅Π΄Π΅Π½ΠΈΠ΅ числитСлСй. Для Π½Π°Ρ‡Π°Π»Π° Π½Π΅ΠΎΠ±Ρ…ΠΎΠ΄ΠΈΠΌΠΎ ΠΏΡ€ΠΎΠ²Π΅Ρ€ΠΈΡ‚ΡŒ, имССтся Π»ΠΈ число, ΠΊΠΎΡ‚ΠΎΡ€ΠΎΠ΅ мСньшС ΠΏΠΎ Π·Π½Π°Ρ‡Π΅Π½ΠΈΡŽ, Ρ‡Π΅ΠΌ ΠΈΡ… ΠΏΡ€ΠΎΠΈΠ·Π²Π΅Π΄Π΅Π½ΠΈΠ΅.

ΠŸΡ€ΠΈΠΌΠ΅Ρ€ 4

Рассмотрим Π½Π° ΠΏΡ€ΠΈΠΌΠ΅Ρ€Π΅ 16Β·215Β ΠΈ 14Β·235, ΠΊΠΎΠ³Π΄Π° ΠΈΡ… ΠΏΡ€ΠΎΠΈΠ·Π²Π΅Π΄Π΅Π½ΠΈΠ΅ Π±ΡƒΠ΄Π΅Ρ‚ Ρ€Π°Π²Π½ΠΎ 6Β·215Β·4Β·235=24Β·245. Π’ΠΎΠ³Π΄Π°Β  Π² качСствС ΠΎΠ±Ρ‰Π΅Π³ΠΎ знамСнатСля Π±Π΅Ρ€Π΅ΠΌ 12Β·235.

Рассмотрим ΠΏΡ€ΠΈΠΌΠ΅Ρ€Ρ‹ ΡƒΠΌΠ½ΠΎΠΆΠ΅Π½ΠΈΠΉ Π΄Ρ€ΠΎΠ±Π΅ΠΉ ΠΎΠ±Ρ‰Π΅Π³ΠΎ Π²ΠΈΠ΄Π°.

ΠŸΡ€ΠΈΠΌΠ΅Ρ€ 5

Для этого Π½Π΅ΠΎΠ±Ρ…ΠΎΠ΄ΠΈΠΌΠΎ произвСсти ΡƒΠΌΠ½ΠΎΠΆΠ΅Π½ΠΈΠ΅ 2+16Β ΠΈ 2Β·53Β·2+1.Β 

РСшСниС

Π‘Π»Π΅Π΄ΡƒΡŽ ΠΏΡ€Π°Π²ΠΈΠ»Ρƒ, Π½Π΅ΠΎΠ±Ρ…ΠΎΠ΄ΠΈΠΌΠΎ ΠΏΠ΅Ρ€Π΅ΠΏΠΈΡΠ°Ρ‚ΡŒ ΠΈ Π² Π²ΠΈΠ΄Π΅ знамСнатСля Π½Π°ΠΏΠΈΡΠ°Ρ‚ΡŒ ΠΏΡ€ΠΎΠΈΠ·Π²Π΅Π΄Π΅Π½ΠΈΠ΅ числитСлСй. ΠŸΠΎΠ»ΡƒΡ‡Π°Π΅ΠΌ, Ρ‡Ρ‚ΠΎ 2+16Β·2Β·53Β·2+12+1Β·2Β·56Β·3Β·2+1.Β  Когда Π΄Ρ€ΠΎΠ±ΡŒ Π±ΡƒΠ΄Π΅Ρ‚ ΡƒΠΌΠ½ΠΎΠΆΠ΅Π½Π°, ΠΌΠΎΠΆΠ½ΠΎ ΠΏΡ€ΠΎΠΈΠ·Π²ΠΎΠ΄ΠΈΡ‚ΡŒ сокращСния для Π΅Π΅ упрощСния. Π’ΠΎΠ³Π΄Π°Β 5Β·332+1:1093=5Β·332+1Β·9310.

Π˜ΡΠΏΠΎΠ»ΡŒΠ·ΡƒΡ ΠΏΡ€Π°Π²ΠΈΠ»ΠΎ ΠΏΠ΅Ρ€Π΅Ρ…ΠΎΠ΄Π° ΠΎΡ‚ дСлСния ΠΊ ΡƒΠΌΠ½ΠΎΠΆΠ΅Π½ΠΈΡŽ Π½Π° ΠΎΠ±Ρ€Π°Ρ‚Π½ΡƒΡŽ Π΄Ρ€ΠΎΠ±ΡŒ, ΠΏΠΎΠ»ΡƒΡ‡ΠΈΠΌ Π΄Ρ€ΠΎΠ±ΡŒ, ΠΎΠ±Ρ€Π°Ρ‚Π½ΡƒΡŽ Π΄Π°Π½Π½ΠΎΠΉ. Для этого Ρ‡ΠΈΡΠ»ΠΈΡ‚Π΅Π»ΡŒ ΠΈ Π·Π½Π°ΠΌΠ΅Π½Π°Ρ‚Π΅Π»ΡŒ ΠΌΠ΅Π½ΡΡŽΡ‚ΡΡ мСстами. Рассмотрим Π½Π° ΠΏΡ€ΠΈΠΌΠ΅Ρ€Π΅:

5Β·332+1:1093=5Β·332+1Β·9310

 ПослС Ρ‡Π΅Π³ΠΎ Π΄ΠΎΠ»ΠΆΠ½Ρ‹ Π²Ρ‹ΠΏΠΎΠ»Π½ΠΈΡ‚ΡŒ ΡƒΠΌΠ½ΠΎΠΆΠ΅Π½ΠΈΠ΅ ΠΈ ΡƒΠΏΡ€ΠΎΡΡ‚ΠΈΡ‚ΡŒ ΠΏΠΎΠ»ΡƒΡ‡Π΅Π½Π½ΡƒΡŽ Π΄Ρ€ΠΎΠ±ΡŒ. Если Π½Π΅ΠΎΠ±Ρ…ΠΎΠ΄ΠΈΠΌΠΎ, Ρ‚ΠΎ ΠΈΠ·Π±Π°Π²ΠΈΡ‚ΡŒΡΡ ΠΎΡ‚ ΠΈΡ€Ρ€Π°Ρ†ΠΈΠΎΠ½Π°Π»ΡŒΠ½ΠΎΡΡ‚ΠΈ Π² Π·Π½Π°ΠΌΠ΅Π½Π°Ρ‚Π΅Π»Π΅. ΠŸΠΎΠ»ΡƒΡ‡Π°Π΅ΠΌ, Ρ‡Ρ‚ΠΎ

5Β·332+1:1093=5Β·33Β·9310Β·2+1=5Β·210Β·2+1=32Β·2+1==3Β·2-12Β·2+1Β·2-1=3Β·2-12Β·22-12=3Β·2-12

ΠžΡ‚Π²Π΅Ρ‚:Β 5Β·332+1:1093=3Β·2-12

Π”Π°Π½Π½Ρ‹ΠΉ ΠΏΡƒΠ½ΠΊΡ‚ ΠΏΡ€ΠΈΠΌΠ΅Π½ΠΈΠΌ, ΠΊΠΎΠ³Π΄Π° число ΠΈΠ»ΠΈ числовоС Π²Ρ‹Ρ€Π°ΠΆΠ΅Π½ΠΈΠ΅ ΠΌΠΎΠΆΠ΅Ρ‚ Π±Ρ‹Ρ‚ΡŒ прСдставлСно Π² Π²ΠΈΠ΄Π΅ Π΄Ρ€ΠΎΠ±ΠΈ, ΠΈΠΌΠ΅ΡŽΡ‰ΡƒΡŽ Π·Π½Π°ΠΌΠ΅Π½Π°Ρ‚Π΅Π»ΡŒ, Ρ€Π°Π²Π½Ρ‹ΠΉ 1, Ρ‚ΠΎΠ³Π΄Π° ΠΈ дСйствиС с Ρ‚Π°ΠΊΠΎΠΉ Π΄Ρ€ΠΎΠ±ΡŒΡŽ рассматриваСтся ΠΎΡ‚Π΄Π΅Π»ΡŒΠ½Ρ‹ΠΌ ΠΏΡƒΠ½ΠΊΡ‚ΠΎΠΌ. НапримСр, Π²Ρ‹Ρ€Π°ΠΆΠ΅Π½ΠΈΠ΅ 16Β·74-1Β·3Β Π²ΠΈΠ΄Π½ΠΎ, Ρ‡Ρ‚ΠΎ ΠΊΠΎΡ€Π΅Π½ΡŒ ΠΈΠ· 3 ΠΌΠΎΠΆΠ΅Ρ‚ Π±Ρ‹Ρ‚ΡŒ Π·Π°ΠΌΠ΅Π½Π΅Π½ Π΄Ρ€ΡƒΠ³ΠΈΠΌ 31Β Π²Ρ‹Ρ€Π°ΠΆΠ΅Π½ΠΈΠ΅ΠΌ. Π’ΠΎΠ³Π΄Π° эта запись Π±ΡƒΠ΄Π΅Ρ‚ Π²Ρ‹Π³Π»ΡΠ΄Π΅Ρ‚ΡŒ ΠΊΠ°ΠΊ ΡƒΠΌΠ½ΠΎΠΆΠ΅Π½ΠΈΠ΅ Π΄Π²ΡƒΡ… Π΄Ρ€ΠΎΠ±Π΅ΠΉ Π²ΠΈΠ΄Π° 16Β·74-1Β·3=16Β·74-1Β·31.

Π’Ρ‹ΠΏΠΎΠ»Π½Π΅Π½ΠΈΠ΅ дСйствиС с дробями, содСрТащими ΠΏΠ΅Ρ€Π΅ΠΌΠ΅Π½Π½Ρ‹Π΅

ΠŸΡ€Π°Π²ΠΈΠ»Π°, рассмотрСнныС Π² ΠΏΠ΅Ρ€Π²ΠΎΠΉ ΡΡ‚Π°Ρ‚ΡŒΠ΅ , ΠΏΡ€ΠΈΠΌΠ΅Π½ΠΈΠΌΡ‹ для дСйствий с дробями, содСрТащими ΠΏΠ΅Ρ€Π΅ΠΌΠ΅Π½Π½Ρ‹Π΅. Рассмотрим ΠΏΡ€Π°Π²ΠΈΠ»ΠΎ вычитания, ΠΊΠΎΠ³Π΄Π° Π·Π½Π°ΠΌΠ΅Π½Π°Ρ‚Π΅Π»ΠΈ ΠΎΠ΄ΠΈΠ½Π°ΠΊΠΎΠ²Ρ‹Π΅.

НСобходимо Π΄ΠΎΠΊΠ°Π·Π°Ρ‚ΡŒ, Ρ‡Ρ‚ΠΎ A,Β CΒ ΠΈΒ DΒ (D Π½Π΅ Ρ€Π°Π²Π½ΠΎΠ΅ Π½ΡƒΠ»ΡŽ) ΠΌΠΎΠ³ΡƒΡ‚ Π±Ρ‹Ρ‚ΡŒ Π»ΡŽΠ±Ρ‹ΠΌΠΈ выраТСниями, ΠΏΡ€ΠΈΡ‡Π΅ΠΌ равСнство ADΒ±CD=AΒ±CDΒ Ρ€Π°Π²Π½ΠΎΡ†Π΅Π½Π½ΠΎ с Π΅Π³ΠΎ ΠΎΠ±Π»Π°ΡΡ‚ΡŒΡŽ допустимых Π·Π½Π°Ρ‡Π΅Π½ΠΈΠΉ.

НСобходимо Π²Π·ΡΡ‚ΡŒ Π½Π°Π±ΠΎΡ€ ΠΏΠ΅Ρ€Π΅ΠΌΠ΅Π½Π½Ρ‹Ρ… ΠžΠ”Π—. Π’ΠΎΠ³Π΄Π°Β  А,Β Π‘,Β D Π΄ΠΎΠ»ΠΆΠ½Ρ‹ ΠΏΡ€ΠΈΠ½ΠΈΠΌΠ°Ρ‚ΡŒ соотвСтствСнныС значСния a0,Β c0Β ΠΈΒ d0. Β ΠŸΠΎΠ΄ΡΡ‚Π°Π½ΠΎΠ²ΠΊΠ° Π²ΠΈΠ΄Π°Β ADΒ±CD ΠΏΡ€ΠΈΠ²ΠΎΠ΄ΠΈΡ‚ Ρ€Π°Π·Π½ΠΎΡΡ‚ΡŒΒ  Π²ΠΈΠ΄Π° a0d0Β±c0d0, Π³Π΄Π΅ ΠΏΠΎ ΠΏΡ€Π°Π²ΠΈΠ»Ρƒ слоТСния ΠΏΠΎΠ»ΡƒΡ‡Π°Π΅ΠΌ Ρ„ΠΎΡ€ΠΌΡƒΠ»Ρƒ Π²ΠΈΠ΄Π° a0Β±c0d0. Если ΠΏΠΎΠ΄ΡΡ‚Π°Π²ΠΈΡ‚ΡŒ Π²Ρ‹Ρ€Π°ΠΆΠ΅Π½ΠΈΠ΅ AΒ±CD, Ρ‚ΠΎΠ³Π΄Π° ΠΏΠΎΠ»ΡƒΡ‡Π°Π΅ΠΌ Ρ‚Ρƒ ΠΆΠ΅ Π΄Ρ€ΠΎΠ±ΡŒ Π²ΠΈΠ΄Π° a0Β±c0d0. ΠžΡ‚ΡΡŽΠ΄Π° Π΄Π΅Π»Π°Π΅ΠΌ Π²Ρ‹Π²ΠΎΠ΄, Ρ‡Ρ‚ΠΎ Π²Ρ‹Π±Ρ€Π°Π½Π½ΠΎΠ΅ Π·Π½Π°Ρ‡Π΅Π½ΠΈΠ΅, ΡƒΠ΄ΠΎΠ²Π»Π΅Ρ‚Π²ΠΎΡ€ΡΡŽΡ‰Π΅Π΅ ΠžΠ”Π—, AΒ±CDΒ ΠΈ ADΒ±CDΒ ΡΡ‡ΠΈΡ‚Π°ΡŽΡ‚ΡΡ Ρ€Π°Π²Π½Ρ‹ΠΌΠΈ.

ΠŸΡ€ΠΈ любом Π·Π½Π°Ρ‡Π΅Π½ΠΈΠΈ ΠΏΠ΅Ρ€Π΅ΠΌΠ΅Π½Π½Ρ‹Ρ… Π΄Π°Π½Π½Ρ‹Π΅ выраТСния Π±ΡƒΠ΄ΡƒΡ‚ Ρ€Π°Π²Π½Ρ‹, Ρ‚ΠΎ Π΅ΡΡ‚ΡŒ ΠΈΡ… Π½Π°Π·Ρ‹Π²Π°ΡŽΡ‚ тоТдСствСнно Ρ€Π°Π²Π½Ρ‹ΠΌΠΈ. Π—Π½Π°Ρ‡ΠΈΡ‚ это Π²Ρ‹Ρ€Π°ΠΆΠ΅Π½ΠΈΠ΅ считаСтся Π΄ΠΎΠΊΠ°Π·Ρ‹Π²Π°Π΅ΠΌΡ‹ΠΌ равСнством Π²ΠΈΠ΄Π° ADΒ±CD=AΒ±CD.

ΠŸΡ€ΠΈΠΌΠ΅Ρ€Ρ‹ слоТСния ΠΈ вычитания Π΄Ρ€ΠΎΠ±Π΅ΠΉ с ΠΏΠ΅Ρ€Π΅ΠΌΠ΅Π½Π½Ρ‹ΠΌΠΈ

Когда ΠΈΠΌΠ΅ΡŽΡ‚ΡΡ ΠΎΠ΄ΠΈΠ½Π°ΠΊΠΎΠ²Ρ‹Π΅ Π·Π½Π°ΠΌΠ΅Π½Π°Ρ‚Π΅Π»ΠΈ, Π½Π΅ΠΎΠ±Ρ…ΠΎΠ΄ΠΈΠΌΠΎ Ρ‚ΠΎΠ»ΡŒΠΊΠΎ ΡΠΊΠ»Π°Π΄Ρ‹Π²Π°Ρ‚ΡŒ ΠΈΠ»ΠΈ Π²Ρ‹Ρ‡ΠΈΡ‚Π°Ρ‚ΡŒ числитСли. Вакая Π΄Ρ€ΠΎΠ±ΡŒ ΠΌΠΎΠΆΠ΅Ρ‚ Π±Ρ‹Ρ‚ΡŒ ΡƒΠΏΡ€ΠΎΡ‰Π΅Π½Π°. Иногда приходится Ρ€Π°Π±ΠΎΡ‚Π°Ρ‚ΡŒ с дробями, ΠΊΠΎΡ‚ΠΎΡ€Ρ‹Π΅ ΡΠ²Π»ΡΡŽΡ‚ΡΡ тоТдСствСнно Ρ€Π°Π²Π½Ρ‹ΠΌΠΈ, Π½ΠΎ ΠΏΡ€ΠΈ ΠΏΠ΅Ρ€Π²ΠΎΠΌ взглядС это  Π½Π΅Π·Π°ΠΌΠ΅Ρ‚Π½ΠΎ, Ρ‚Π°ΠΊ ΠΊΠ°ΠΊ Π½Π΅ΠΎΠ±Ρ…ΠΎΠ΄ΠΈΠΌΠΎ Π²Ρ‹ΠΏΠΎΠ»Π½ΡΡ‚ΡŒ Π½Π΅ΠΊΠΎΡ‚ΠΎΡ€Ρ‹Π΅ прСобразования. НапримСр, x23Β·x13+1Β ΠΈΒ x13+12 ΠΈΠ»ΠΈ 12Β·sinΒ 2Ξ±Β ΠΈ sinΒ aΒ·cosΒ a. Π§Π°Ρ‰Π΅ всСго трСбуСтся ΡƒΠΏΡ€ΠΎΡ‰Π΅Π½ΠΈΠ΅ исходного выраТСния для Ρ‚ΠΎΠ³ΠΎ, Ρ‡Ρ‚ΠΎΠ±Ρ‹ ΡƒΠ²ΠΈΠ΄Π΅Ρ‚ΡŒ ΠΎΠ΄ΠΈΠ½Π°ΠΊΠΎΠ²Ρ‹Π΅ Π·Π½Π°ΠΌΠ΅Π½Π°Ρ‚Π΅Π»ΠΈ.

НуТна ΠΏΠΎΠΌΠΎΡ‰ΡŒ прСподаватСля?

Опиши Π·Π°Π΄Π°Π½ΠΈΠ΅Β β€” и наши экспСрты Ρ‚Π΅Π±Π΅ ΠΏΠΎΠΌΠΎΠ³ΡƒΡ‚!

ΠžΠΏΠΈΡΠ°Ρ‚ΡŒ Π·Π°Π΄Π°Π½ΠΈΠ΅ ΠŸΡ€ΠΈΠΌΠ΅Ρ€ 6

Π’Ρ‹Ρ‡ΠΈΡΠ»ΠΈΡ‚ΡŒ:1)Β x2+1x+x-2-5-xx+x-2, 2)lg2x+4xΒ·(lgΒ x+2)+4Β·lgΒ xxΒ·(lgΒ x+2), x-1x-1+xx+1.

РСшСниС

  1. Π§Ρ‚ΠΎΠ±Ρ‹ произвСсти вычислСниС, Π½Π΅ΠΎΠ±Ρ…ΠΎΠ΄ΠΈΠΌΠΎ Π²Ρ‹Ρ‡Π΅ΡΡ‚ΡŒ Π΄Ρ€ΠΎΠ±ΠΈ, ΠΊΠΎΡ‚ΠΎΡ€Ρ‹ΠΌ ΠΈΠΌΠ΅ΡŽΡ‚ ΠΎΠ΄ΠΈΠ½Π°ΠΊΠΎΠ²Ρ‹Π΅ Π·Π½Π°ΠΌΠ΅Π½Π°Ρ‚Π΅Π»ΠΈ. Π’ΠΎΠ³Π΄Π° ΠΏΠΎΠ»ΡƒΡ‡Π°Π΅ΠΌ, Ρ‡Ρ‚ΠΎ x2+1x+x-2-5-xx+x-2=x2+1-5-xx+x-2. Β  ПослС Ρ‡Π΅Π³ΠΎ ΠΌΠΎΠΆΠ½ΠΎ Π²Ρ‹ΠΏΠΎΠ»Π½ΡΡ‚ΡŒ раскрытиС скобок с ΠΏΡ€ΠΈΠ²Π΅Π΄Π΅Π½ΠΈΠ΅ΠΌ ΠΏΠΎΠ΄ΠΎΠ±Π½Ρ‹Ρ… слагаСмых. ΠŸΠΎΠ»ΡƒΡ‡Π°Π΅ΠΌ, Ρ‡Ρ‚ΠΎx2+1-5-xx+x-2=x2+1-5+xx+x-2=x2+x-4x+x-2
  2. Π’Π°ΠΊ ΠΊΠ°ΠΊ Π·Π½Π°ΠΌΠ΅Π½Π°Ρ‚Π΅Π»ΠΈ ΠΎΠ΄ΠΈΠ½Π°ΠΊΠΎΠ²Ρ‹Π΅, Ρ‚ΠΎ остаСтся Ρ‚ΠΎΠ»ΡŒΠΊΠΎ ΡΠ»ΠΎΠΆΠΈΡ‚ΡŒ числитСли, оставив Π·Π½Π°ΠΌΠ΅Π½Π°Ρ‚Π΅Π»ΡŒ:​​​​​​lg2x+4xΒ·(lgΒ x+2)+4Β·lgΒ xxΒ·(lgΒ x+2)=lg2x+4+4xΒ·(lgΒ x+2)
    Π‘Π»ΠΎΠΆΠ΅Π½ΠΈΠ΅ Π±Ρ‹Π»ΠΎ Π²Ρ‹ΠΏΠΎΠ»Π½Π΅Π½ΠΎ. Π’ΠΈΠ΄Π½ΠΎ, Ρ‡Ρ‚ΠΎ ΠΌΠΎΠΆΠ½ΠΎ произвСсти сокращСниС Π΄Ρ€ΠΎΠ±ΠΈ. Β Π•Π΅ Ρ‡ΠΈΡΠ»ΠΈΡ‚Π΅Π»ΡŒ ΠΌΠΎΠΆΠ΅Ρ‚ Π±Ρ‹Ρ‚ΡŒ свСрнут ΠΏΠΎ Ρ„ΠΎΡ€ΠΌΡƒΠ»Π΅ ΠΊΠ²Π°Π΄Ρ€Π°Ρ‚Π° суммы, Ρ‚ΠΎΠ³Π΄Π° ΠΏΠΎΠ»ΡƒΡ‡ΠΈΠΌ (lgΒ x+2)2ΠΈΠ· Ρ„ΠΎΡ€ΠΌΡƒΠ» сокращСнного умноТСния. Π’ΠΎΠ³Π΄Π° ΠΏΠΎΠ»ΡƒΡ‡Π°Π΅ΠΌ, Ρ‡Ρ‚ΠΎ
    lg2x+4+2Β·lgΒ xxΒ·(lgΒ x+2)=(lgΒ x+2)2xΒ·(lgΒ x+2)=lgΒ x+2x
  3. Β Π—Π°Π΄Π°Π½Π½Ρ‹Π΅ Π΄Ρ€ΠΎΠ±ΠΈ Π²ΠΈΠ΄Π° x-1x-1+xx+1 с Ρ€Π°Π·Π½Ρ‹ΠΌΠΈ знамСнатСлями. ПослС прСобразования ΠΌΠΎΠΆΠ½ΠΎ ΠΏΠ΅Ρ€Π΅ΠΉΡ‚ΠΈ ΠΊ слоТСнию.

Рассмотрим двоякий способ Ρ€Π΅ΡˆΠ΅Π½ΠΈΡ.

ΠŸΠ΅Ρ€Π²Ρ‹ΠΉ способ Π·Π°ΠΊΠ»ΡŽΡ‡Π°Π΅Ρ‚ΡΡ Π² Ρ‚ΠΎΠΌ, Ρ‡Ρ‚ΠΎ Π·Π½Π°ΠΌΠ΅Π½Π°Ρ‚Π΅Π»ΡŒ ΠΏΠ΅Ρ€Π²ΠΎΠΉ Π΄Ρ€ΠΎΠ±ΠΈ подвСргаСтся Ρ€Π°Π·Π»ΠΎΠΆΠ΅Π½ΠΈΡŽ Π½Π° ΠΌΠ½ΠΎΠΆΠΈΡ‚Π΅Π»ΠΈ ΠΏΡ€ΠΈ ΠΏΠΎΠΌΠΎΡ‰ΠΈ ΠΊΠ²Π°Π΄Ρ€Π°Ρ‚ΠΎΠ², ΠΏΡ€ΠΈΡ‡Π΅ΠΌ с Π΅Π΅ ΠΏΠΎΡΠ»Π΅Π΄ΡƒΡŽΡ‰ΠΈΠΌ сокращСниСм. ΠŸΠΎΠ»ΡƒΡ‡ΠΈΠΌ Π΄Ρ€ΠΎΠ±ΡŒ Π²ΠΈΠ΄Π°

x-1x-1=x-1(x-1)Β·x+1=1x+1

Π—Π½Π°Ρ‡ΠΈΡ‚, x-1x-1+xx+1=1x+1+xx+1=1+xx+1.

Π’ Ρ‚Π°ΠΊΠΎΠΌ случаС Π½Π΅ΠΎΠ±Ρ…ΠΎΠ΄ΠΈΠΌΠΎ ΠΈΠ·Π±Π°Π²Π»ΡΡ‚ΡŒΡΡ ΠΎΡ‚ ΠΈΡ€Ρ€Π°Ρ†ΠΈΠΎΠ½Π°Π»ΡŒΠ½ΠΎΡΡ‚ΠΈ Π² Π·Π½Π°ΠΌΠ΅Π½Π°Ρ‚Π΅Π»Π΅.

ΠŸΠΎΠ»ΡƒΡ‡ΠΈΠΌ:

1+xx+1=1+xΒ·x-1x+1Β·x-1=x-1+xΒ·x-xx-1

Π’Ρ‚ΠΎΡ€ΠΎΠΉ способ Π·Π°ΠΊΠ»ΡŽΡ‡Π°Π΅Ρ‚ΡΡ Π² ΡƒΠΌΠ½ΠΎΠΆΠ΅Π½ΠΈΠΈ числитСля ΠΈ знамСнатСля Π²Ρ‚ΠΎΡ€ΠΎΠΉ Π΄Ρ€ΠΎΠ±ΠΈ Π½Π° Π²Ρ‹Ρ€Π°ΠΆΠ΅Π½ΠΈΠ΅ x-1. Π’Π°ΠΊΠΈΠΌ ΠΎΠ±Ρ€Π°Π·ΠΎΠΌ, ΠΌΡ‹ избавляСмся ΠΎΡ‚ ΠΈΡ€Ρ€Π°Ρ†ΠΈΠΎΠ½Π°Π»ΡŒΠ½ΠΎΡΡ‚ΠΈ ΠΈ ΠΏΠ΅Ρ€Π΅Ρ…ΠΎΠ΄ΠΈΠΌ ΠΊ слоТСнию Π΄Ρ€ΠΎΠ±ΠΈ ΠΏΡ€ΠΈ Π½Π°Π»ΠΈΡ‡ΠΈΠΈ ΠΎΠ΄ΠΈΠ½Π°ΠΊΠΎΠ²ΠΎΠ³ΠΎ знамСнатСля. Π’ΠΎΠ³Π΄Π°

x-1x-1+xx+1=x-1x-1+xΒ·x-1x+1Β·x-1==x-1x-1+xΒ·x-xx-1=x-1+xΒ·x-xx-1

ΠžΡ‚Π²Π΅Ρ‚: 1)Β x2+1x+x-2-5-xx+x-2=x2+x-4x+x-2, 2)lg2x+4xΒ·(lgΒ x+2)+4Β·lgΒ xxΒ·(lgΒ x+2)=lgΒ x+2x, 3)x-1x-1+xx+1=x-1+xΒ·x-xx-1.

Π’ послСднСм ΠΏΡ€ΠΈΠΌΠ΅Ρ€Π΅ ΠΏΠΎΠ»ΡƒΡ‡ΠΈΠ»ΠΈ, Ρ‡Ρ‚ΠΎ ΠΏΡ€ΠΈΠ²Π΅Π΄Π΅Π½ΠΈΠ΅ ΠΊ ΠΎΠ±Ρ‰Π΅ΠΌΡƒ Π·Π½Π°ΠΌΠ΅Π½Π°Ρ‚Π΅Π»ΡŽ Π½Π΅ΠΈΠ·Π±Π΅ΠΆΠ½ΠΎ. Для этого Π½Π΅ΠΎΠ±Ρ…ΠΎΠ΄ΠΈΠΌΠΎ ΡƒΠΏΡ€ΠΎΡ‰Π°Ρ‚ΡŒ Π΄Ρ€ΠΎΠ±ΠΈ. Для слоТСния ΠΈΠ»ΠΈ вычитая всСгда Π½Π΅ΠΎΠ±Ρ…ΠΎΠ΄ΠΈΠΌΠΎ ΠΈΡΠΊΠ°Ρ‚ΡŒ ΠΎΠ±Ρ‰ΠΈΠΉ Π·Π½Π°ΠΌΠ΅Π½Π°Ρ‚Π΅Π»ΡŒ, ΠΊΠΎΡ‚ΠΎΡ€Ρ‹ΠΉ выглядит ΠΊΠ°ΠΊ ΠΏΡ€ΠΎΠΈΠ·Π²Π΅Π΄Π΅Π½ΠΈΠ΅ Π·Π½Π°ΠΌΠ΅Π½Π°Ρ‚Π΅Π»Π΅ΠΉΒ  с Π΄ΠΎΠ±Π°Π²Π»Π΅Π½ΠΈΠ΅ΠΌ Π΄ΠΎΠΏΠΎΠ»Π½ΠΈΠ΅Ρ‚Π»ΡŒΠ½Ρ‹Ρ… ΠΌΠ½ΠΎΠΆΠΈΡ‚Π΅Π»Π΅ΠΉ ΠΊ числитСлям.

ΠŸΡ€ΠΈΠΌΠ΅Ρ€ 7

Π’Ρ‹Ρ‡ΠΈΡΠ»ΠΈΡ‚ΡŒ значСния Π΄Ρ€ΠΎΠ±Π΅ΠΉ: 1)Β x3+1×7+2Β·2, 2)Β x+1xΒ·ln2(x+1)Β·(2x-4)-sinΒ xx5Β·ln(x+1)Β·(2x-4), 3)Β 1cos2x-x+1cos2x+2Β·cosΒ xΒ·x+x

РСшСниС

  1.  Никаких слоТных вычислСний Π·Π½Π°ΠΌΠ΅Π½Π°Ρ‚Π΅Π»ΡŒ Π½Π΅ Ρ‚Ρ€Π΅Π±ΡƒΠ΅Ρ‚, поэтому Π½ΡƒΠΆΠ½ΠΎ Π²Ρ‹Π±Ρ€Π°Ρ‚ΡŒ ΠΈΡ… ΠΏΡ€ΠΎΠΈΠ·Π²Π΅Π΄Π΅Π½ΠΈΠ΅ Π²ΠΈΠ΄Π° 3Β·x7+2Β·2, Ρ‚ΠΎΠ³Π΄Π° ΠΊ ΠΏΠ΅Ρ€Π²ΠΎΠΉ Π΄Ρ€ΠΎΠ±ΠΈ x7+2Β·2Β Π²Ρ‹Π±ΠΈΡ€Π°ΡŽΡ‚ ΠΊΠ°ΠΊ Π΄ΠΎΠΏΠΎΠ»Π½ΠΈΡ‚Π΅Π»ΡŒΠ½Ρ‹ΠΉ ΠΌΠ½ΠΎΠΆΠΈΡ‚Π΅Π»ΡŒ, Π° 3 ΠΊΠΎ Π²Ρ‚ΠΎΡ€ΠΎΠΉ. ΠŸΡ€ΠΈ ΠΏΠ΅Ρ€Π΅ΠΌΠ½ΠΎΠΆΠ΅Π½ΠΈΠΈ ΠΏΠΎΠ»ΡƒΡ‡Π°Π΅ΠΌ Π΄Ρ€ΠΎΠ±ΡŒ Π²ΠΈΠ΄Π° x3+1×7+2Β·2=xΒ·x7+2Β·23Β·x7+2Β·2+3Β·13Β·x7+2Β·2==xΒ·x7+2Β·2+33Β·x7+2Β·2=xΒ·x7+2Β·2Β·x+33Β·x7+2Β·2
  2. Π’ΠΈΠ΄Π½ΠΎ, Ρ‡Ρ‚ΠΎ Π·Π½Π°ΠΌΠ΅Π½Π°Ρ‚Π΅Π»ΠΈ прСдставлСны Π² Π²ΠΈΠ΄Π΅ произвСдСния, Ρ‡Ρ‚ΠΎ ΠΎΠ·Π½Π°Ρ‡Π°Π΅Ρ‚ Π½Π΅Π½ΡƒΠΆΠ½ΠΎΡΡ‚ΡŒ Π΄ΠΎΠΏΠΎΠ»Π½ΠΈΡ‚Π΅Π»ΡŒΠ½Ρ‹Ρ… ΠΏΡ€Π΅ΠΎΠ±Ρ€Π°Π·ΠΎΠ²Π°Π½ΠΈΠΉ. ΠžΠ±Ρ‰ΠΈΠΌ Π·Π½Π°ΠΌΠ΅Π½Π°Ρ‚Π΅Π»Π΅ΠΌ Π±ΡƒΠ΄Π΅Ρ‚ ΡΡ‡ΠΈΡ‚Π°Ρ‚ΡŒΡΡ Β ΠΏΡ€ΠΎΠΈΠ·Π²Π΅Π΄Π΅Π½ΠΈΠ΅ Π²ΠΈΠ΄Π° x5Β·ln2x+1Β·2x-4. ΠžΡ‚ΡΡŽΠ΄Π° x4 являСтся Π΄ΠΎΠΏΠΎΠ»Π½ΠΈΡ‚Π΅Π»ΡŒΠ½Ρ‹ΠΌ ΠΌΠ½ΠΎΠΆΠΈΡ‚Π΅Π»Π΅ΠΌ ΠΊ ΠΏΠ΅Ρ€Π²ΠΎΠΉ Π΄Ρ€ΠΎΠ±ΠΈ, Π° ln(x+1) ΠΊΠΎ Π²Ρ‚ΠΎΡ€ΠΎΠΉ. ПослС Ρ‡Π΅Π³ΠΎ ΠΏΡ€ΠΎΠΈΠ·Π²ΠΎΠ΄ΠΈΠΌ Π²Ρ‹Ρ‡ΠΈΡ‚Π°Π½ΠΈΠ΅ ΠΈ ΠΏΠΎΠ»ΡƒΡ‡Π°Π΅ΠΌ, Ρ‡Ρ‚ΠΎ:
    x+1xΒ·ln2(x+1)Β·2x-4-sinΒ xx5Β·ln(x+1)Β·2x-4==x+1Β·x4x5Β·ln2(x+1)Β·2x-4-sinΒ xΒ·lnx+1×5Β·ln2(x+1)Β·(2x-4)==x+1Β·x4-sinΒ xΒ·ln(x+1)x5Β·ln2(x+1)Β·(2x-4)=xΒ·x4+x4-sinΒ xΒ·ln(x+1)x5Β·ln2(x+1)Β·(2x-4)
  3. Β Π”Π°Π½Π½Ρ‹ΠΉ ΠΏΡ€ΠΈΠΌΠ΅Ρ€ ΠΈΠΌΠ΅Π΅Ρ‚ смысл ΠΏΡ€ΠΈ Ρ€Π°Π±ΠΎΡ‚Π΅ со знамСнатСлями дробями. НСобходимо ΠΏΡ€ΠΈΠΌΠ΅Π½ΠΈΡ‚ΡŒ Ρ„ΠΎΡ€ΠΌΡƒΠ»Ρ‹ разности ΠΊΠ²Π°Π΄Ρ€Π°Ρ‚ΠΎΠ² ΠΈ ΠΊΠ²Π°Π΄Ρ€Π°Ρ‚ суммы, Ρ‚Π°ΠΊ ΠΊΠ°ΠΊ ΠΈΠΌΠ΅Π½Π½ΠΎ ΠΎΠ½ΠΈ Π΄Π°Π΄ΡƒΡ‚ Π²ΠΎΠ·ΠΌΠΎΠΆΠ½ΠΎΡΡ‚ΡŒ ΠΏΠ΅Ρ€Π΅ΠΉΡ‚ΠΈ ΠΊ Π²Ρ‹Ρ€Π°ΠΆΠ΅Π½ΠΈΡŽ Π²ΠΈΠ΄Π° 1cosΒ x-xΒ·cosΒ x+x+1(cosΒ x+x)2.Β  Π’ΠΈΠ΄Π½ΠΎ, Ρ‡Ρ‚ΠΎ Π΄Ρ€ΠΎΠ±ΠΈ приводятся ΠΊ ΠΎΠ±Ρ‰Π΅ΠΌΡƒ Π·Π½Π°ΠΌΠ΅Π½Π°Ρ‚Π΅Π»ΡŽ. ΠŸΠΎΠ»ΡƒΡ‡Π°Π΅ΠΌ, Ρ‡Ρ‚ΠΎ cosΒ x-xΒ·cosΒ x+x2.

ПослС Ρ‡Π΅Π³ΠΎ ΠΏΠΎΠ»ΡƒΡ‡Π°Π΅ΠΌ, Ρ‡Ρ‚ΠΎ

1cos2x-x+1cos2x+2Β·cosΒ xΒ·x+x==1cosΒ x-xΒ·cosΒ x+x+1cosΒ x+x2==cosΒ x+xcosΒ x-xΒ·cosΒ x+x2+cosΒ x-xcosΒ x-xΒ·cosΒ x+x2==cosΒ x+x+cosΒ x-xcosΒ x-xΒ·cosΒ x+x2=2Β·cosΒ xcosΒ x-xΒ·cosΒ x+x2

ΠžΡ‚Π²Π΅Ρ‚:

1)Β x3+1×7+2Β·2=xΒ·x7+2Β·2Β·x+33Β·x7+2Β·2, 2)Β x+1xΒ·ln2(x+1)Β·2x-4-sinΒ xx5Β·ln(x+1)Β·2x-4==xΒ·x4+x4-sinΒ xΒ·ln(x+1)x5Β·ln2(x+1)Β·(2x-4), 3)Β 1cos2x-x+1cos2x+2Β·cosΒ xΒ·x+x=2Β·cosΒ xcosΒ x-xΒ·cosΒ x+x2.

ΠŸΡ€ΠΈΠΌΠ΅Ρ€Ρ‹ умноТСния Π΄Ρ€ΠΎΠ±Π΅ΠΉ с ΠΏΠ΅Ρ€Π΅ΠΌΠ΅Π½Π½Ρ‹ΠΌΠΈ

ΠŸΡ€ΠΈ ΡƒΠΌΠ½ΠΎΠΆΠ΅Π½ΠΈΠΈ Π΄Ρ€ΠΎΠ±Π΅ΠΉ Ρ‡ΠΈΡΠ»ΠΈΡ‚Π΅Π»ΡŒ умноТаСтся Π½Π° Ρ‡ΠΈΡΠ»ΠΈΡ‚Π΅Π»ΡŒ, Π° Π·Π½Π°ΠΌΠ΅Π½Π°Ρ‚Π΅Π»ΡŒ Π½Π° Π·Π½Π°ΠΌΠ΅Π½Π°Ρ‚Π΅Π»ΡŒ. Π’ΠΎΠ³Π΄Π° ΠΌΠΎΠΆΠ½ΠΎ ΠΏΡ€ΠΈΠΌΠ΅Π½ΡΡ‚ΡŒ свойство сокращСния.

ΠŸΡ€ΠΈΠΌΠ΅Ρ€ 8

ΠŸΡ€ΠΎΠΈΠ·Π²Π΅ΡΡ‚ΠΈ ΡƒΠΌΠ½ΠΎΠΆΠ΅Π½ΠΈΠ΅ Π΄Ρ€ΠΎΠ±Π΅ΠΉ x+2Β·xx2Β·lnΒ x2Β·lnΒ x+1Β ΠΈ 3Β·x213Β·x+1-2sin2Β·x-x.

РСшСниС

НСобходимо Π²Ρ‹ΠΏΠΎΠ»Π½ΠΈΡ‚ΡŒ ΡƒΠΌΠ½ΠΎΠΆΠ΅Π½ΠΈΠ΅. ΠŸΠΎΠ»ΡƒΡ‡Π°Π΅ΠΌ, Ρ‡Ρ‚ΠΎ

x+2Β·xx2Β·lnΒ x2Β·lnΒ x+1Β·3Β·x213Β·x+1-2sin(2Β·x-x)==x-2Β·xΒ·3Β·x213Β·x+1-2×2Β·lnΒ x2Β·lnΒ x+1Β·sinΒ (2Β·x-x)

Число 3 пСрСносится Π½Π° ΠΏΠ΅Ρ€Π²ΠΎΠ΅ мСсто для удобства подсчСтов, ΠΏΡ€ΠΈΡ‡Π΅ΠΌ ΠΌΠΎΠΆΠ½ΠΎ произвСсти сокращСниС Π΄Ρ€ΠΎΠ±ΠΈ Π½Π° x2, Ρ‚ΠΎΠ³Π΄Π° ΠΏΠΎΠ»ΡƒΡ‡ΠΈΠΌ Π²Ρ‹Ρ€Π°ΠΆΠ΅Π½ΠΈΠ΅ Π²ΠΈΠ΄Π°

3Β·x-2Β·xΒ·x13Β·x+1-2lnΒ x2Β·lnΒ x+1Β·sinΒ (2Β·x-x)

ΠžΡ‚Π²Π΅Ρ‚: x+2Β·xx2Β·lnΒ x2Β·lnΒ x+1Β·3Β·x213Β·x+1-2sin(2Β·x-x)=3Β·x-2Β·xΒ·x13Β·x+1-2lnΒ x2Β·lnΒ x+1Β·sinΒ (2Β·x-x).

Π”Π΅Π»Π΅Π½ΠΈΠ΅

Π”Π΅Π»Π΅Π½ΠΈΠ΅ Ρƒ Π΄Ρ€ΠΎΠ±Π΅ΠΉ Π°Π½Π°Π»ΠΎΠ³ΠΈΡ‡Π½ΠΎ ΡƒΠΌΠ½ΠΎΠΆΠ΅Π½ΠΈΡŽ, Ρ‚Π°ΠΊ ΠΊΠ°ΠΊ ΠΏΠ΅Ρ€Π²ΡƒΡŽ Π΄Ρ€ΠΎΠ±ΡŒ ΡƒΠΌΠ½ΠΎΠΆΠ°ΡŽΡ‚ Π½Π° Π²Ρ‚ΠΎΡ€ΡƒΡŽ ΠΎΠ±Ρ€Π°Ρ‚Π½ΡƒΡŽ. Если Π²Π·ΡΡ‚ΡŒ ΠΊ ΠΏΡ€ΠΈΠΌΠ΅Ρ€Ρƒ Π΄Ρ€ΠΎΠ±ΡŒ x+2Β·xx2Β·lnΒ x2Β·lnΒ x+1Β ΠΈ Ρ€Π°Π·Π΄Π΅Π»ΠΈΡ‚ΡŒ Π½Π° 3Β·x213Β·x+1-2sin2Β·x-x, Ρ‚ΠΎΠ³Π΄Π° это ΠΌΠΎΠΆΠ½ΠΎ Π·Π°ΠΏΠΈΡΠ°Ρ‚ΡŒ Ρ‚Π°ΠΊΠΈΠΌ ΠΎΠ±Ρ€Π°Π·ΠΎΠΌ, ΠΊΠ°ΠΊ

x+2Β·xx2Β·lnΒ x2Β·lnΒ x+1:3Β·x213Β·x+1-2sin(2Β·x-x), послС Ρ‡Π΅Π³ΠΎ Π·Π°ΠΌΠ΅Π½ΠΈΡ‚ΡŒ ΠΏΡ€ΠΎΠΈΠ·Π²Π΅Π΄Π΅Π½ΠΈΠ΅ΠΌ Π²ΠΈΠ΄Π° x+2Β·xx2Β·lnΒ x2Β·lnΒ x+1Β·3Β·x213Β·x+1-2sin(2Β·x-x)

Π’ΠΎΠ·Π²Π΅Π΄Π΅Π½ΠΈΠ΅ Π² ΡΡ‚Π΅ΠΏΠ΅Π½ΡŒ

ΠŸΠ΅Ρ€Π΅ΠΉΠ΄Π΅ΠΌ ΠΊ Ρ€Π°ΡΡΠΌΠΎΡ‚Ρ€Π΅Π½ΠΈΡŽ дСйствия с дробями ΠΎΠ±Ρ‰Π΅Π³ΠΎ Π²ΠΈΠ΄Π° с Π²ΠΎΠ·Π²Π΅Π΄Π΅Π½ΠΈΠ΅ΠΌ Π² ΡΡ‚Π΅ΠΏΠ΅Π½ΡŒ. Если имССтся ΡΡ‚Π΅ΠΏΠ΅Π½ΡŒ с Π½Π°Ρ‚ΡƒΡ€Π°Π»ΡŒΠ½Ρ‹ΠΌ ΠΏΠΎΠΊΠ°Π·Π°Ρ‚Π΅Π»Π΅ΠΌ, Ρ‚ΠΎΠ³Π΄Π° дСйствиС Ρ€Π°ΡΡΠΌΠ°Ρ‚Ρ€ΠΈΠ²Π°ΡŽΡ‚ ΠΊΠ°ΠΊ ΡƒΠΌΠ½ΠΎΠΆΠ΅Π½ΠΈΠ΅ ΠΎΠ΄ΠΈΠ½Π°ΠΊΠΎΠ²Ρ‹Ρ… Π΄Ρ€ΠΎΠ±Π΅ΠΉ. Но Ρ€Π΅ΠΊΠΎΠΌΠ΅Π½Π΄ΠΎΠ²Π°Π½ΠΎ Β ΠΈΡΠΏΠΎΠ»ΡŒΠ·ΠΎΠ²Π°Ρ‚ΡŒ ΠΎΠ±Ρ‰ΠΈΠΉ ΠΏΠΎΠ΄Ρ…ΠΎΠ΄, Π±Π°Π·ΠΈΡ€ΡƒΡŽΡ‰ΠΈΠΉΡΡ Π½Π° свойствах стСпСнСй. Π›ΡŽΠ±Ρ‹Π΅ выраТСния А ΠΈ Π‘, Π³Π΄Π΅ Π‘ тоТдСствСнно Π½Π΅ равняСтся Π½ΡƒΠ»ΡŽ, Π° любоС Π΄Π΅ΠΉΡΡ‚Π²ΠΈΡ‚Π΅Π»ΡŒΠ½ΠΎΠ΅ r Π½Π° ΠžΠ”Π— для выраТСния Π²ΠΈΠ΄Π° ACr справСдливо равСнство ACr=ArCr. Π Π΅Π·ΡƒΠ»ΡŒΡ‚Π°Ρ‚ – Π΄Ρ€ΠΎΠ±ΡŒ, возвСдСнная Π² ΡΡ‚Π΅ΠΏΠ΅Π½ΡŒ. Для примСра рассмотрим:

x0,7-π·ln3x-2-5x+12,5==x0,7-π·ln3x-2-52,5x+12,5

ΠŸΠΎΡ€ΡΠ΄ΠΎΠΊ выполнСния дСйствий с дробями

ДСйствия Π½Π°Π΄ дробями Π²Ρ‹ΠΏΠΎΠ»Π½ΡΡŽΡ‚ΡΡ ΠΏΠΎ ΠΎΠΏΡ€Π΅Π΄Π΅Π»Π΅Π½Π½Ρ‹ΠΌ ΠΏΡ€Π°Π²ΠΈΠ»Π°ΠΌ. На ΠΏΡ€Π°ΠΊΡ‚ΠΈΠΊΠ΅ Π·Π°ΠΌΠ΅Ρ‡Π°Π΅ΠΌ, Ρ‡Ρ‚ΠΎ Π²Ρ‹Ρ€Π°ΠΆΠ΅Π½ΠΈΠ΅ ΠΌΠΎΠΆΠ΅Ρ‚ ΡΠΎΠ΄Π΅Ρ€ΠΆΠ°Ρ‚ΡŒ нСсколько Π΄Ρ€ΠΎΠ±Π΅ΠΉ ΠΈΠ»ΠΈ Π΄Ρ€ΠΎΠ±Π½Ρ‹Ρ… Π²Ρ‹Ρ€Π°ΠΆΠ΅Π½ΠΈΠΉ. Π’ΠΎΠ³Π΄Π° Π½Π΅ΠΎΠ±Ρ…ΠΎΠ΄ΠΈΠΌΠΎ всС дСйствия Π²Ρ‹ΠΏΠΎΠ»Π½ΡΡ‚ΡŒΒ  Π² строгом порядкС: Π²ΠΎΠ·Π²ΠΎΠ΄ΠΈΡ‚ΡŒ Π² ΡΡ‚Π΅ΠΏΠ΅Π½ΡŒ, ΡƒΠΌΠ½ΠΎΠΆΠ°Ρ‚ΡŒ, Π΄Π΅Π»ΠΈΡ‚ΡŒ, послС Ρ‡Π΅Π³ΠΎ ΡΠΊΠ»Π°Π΄Ρ‹Π²Π°Ρ‚ΡŒ ΠΈ Π²Ρ‹Ρ‡ΠΈΡ‚Π°Ρ‚ΡŒ. ΠŸΡ€ΠΈ Π½Π°Π»ΠΈΡ‡ΠΈΠΈ скобок ΠΏΠ΅Ρ€Π²ΠΎΠ΅ дСйствиС выполняСтся ΠΈΠΌΠ΅Π½Π½ΠΎ Π² Π½ΠΈΡ….

ΠŸΡ€ΠΈΠΌΠ΅Ρ€ 9

Π’Ρ‹Ρ‡ΠΈΡΠ»ΠΈΡ‚ΡŒ 1-xcosΒ x-1cosΒ xΒ·1+1x.

РСшСниС

Π’Π°ΠΊ ΠΊΠ°ΠΊ ΠΈΠΌΠ΅Π΅ΠΌ ΠΎΠ΄ΠΈΠ½Π°ΠΊΠΎΠ²Ρ‹ΠΉ Π·Π½Π°ΠΌΠ΅Π½Π°Ρ‚Π΅Π»ΡŒ, Ρ‚ΠΎ 1-xcosΒ xΒ ΠΈ 1cosΒ x, Π½ΠΎ ΠΏΡ€ΠΎΠΈΠ·Π²ΠΎΠ΄ΠΈΡ‚ΡŒ вычитания ΠΏΠΎ ΠΏΡ€Π°Π²ΠΈΠ»Ρƒ нСльзя, сначала Π²Ρ‹ΠΏΠΎΠ»Π½ΡΡŽΡ‚ΡΡ дСйствия Π² скобках, послС Ρ‡Π΅Π³ΠΎ ΡƒΠΌΠ½ΠΎΠΆΠ΅Π½ΠΈΠ΅, Π° ΠΏΠΎΡ‚ΠΎΠΌ слоТСниС. Π’ΠΎΠ³Π΄Π° ΠΏΡ€ΠΈ вычислСнии ΠΏΠΎΠ»ΡƒΡ‡Π°Π΅ΠΌ, Ρ‡Ρ‚ΠΎ

1+1x=11+1x=xx+1x=x+1x

ΠŸΡ€ΠΈ подстановкС выраТСния  Π² исходноС ΠΏΠΎΠ»ΡƒΡ‡Π°Π΅ΠΌ, Ρ‡Ρ‚ΠΎ 1-xcosΒ x-1cosΒ xΒ·x+1x. ΠŸΡ€ΠΈ ΡƒΠΌΠ½ΠΎΠΆΠ΅Π½ΠΈΠΈ Π΄Ρ€ΠΎΠ±Π΅ΠΉ ΠΈΠΌΠ΅Π΅ΠΌ: 1cosΒ xΒ·x+1x=x+1cosΒ xΒ·x. ΠŸΡ€ΠΎΠΈΠ·Π²Π΅Π΄Ρ всС подстановки, ΠΏΠΎΠ»ΡƒΡ‡ΠΈΠΌ 1-xcosΒ x-x+1cosΒ xΒ·x. Π’Π΅ΠΏΠ΅Ρ€ΡŒ Π½Π΅ΠΎΠ±Ρ…ΠΎΠ΄ΠΈΠΌΠΎ Ρ€Π°Π±ΠΎΡ‚Π°Ρ‚ΡŒ с дробями, ΠΊΠΎΡ‚ΠΎΡ€Ρ‹Π΅ ΠΈΠΌΠ΅ΡŽΡ‚ Ρ€Π°Π·Π½Ρ‹Π΅ Π·Π½Π°ΠΌΠ΅Π½Π°Ρ‚Π΅Π»ΠΈ. ΠŸΠΎΠ»ΡƒΡ‡ΠΈΠΌ:

xΒ·1-xcosΒ xΒ·x-x+1cosΒ xΒ·x=xΒ·1-x-1+xcosΒ xΒ·x==x-x-x-1cosΒ xΒ·x=-x+1cosΒ xΒ·x

ΠžΡ‚Π²Π΅Ρ‚: 1-xcosΒ x-1cosΒ xΒ·1+1x=-x+1cosΒ xΒ·x.

Π‘Π»ΠΎΠΆΠ΅Π½ΠΈΠ΅, Π²Ρ‹Ρ‡ΠΈΡ‚Π°Π½ΠΈΠ΅, ΡƒΠΌΠ½ΠΎΠΆΠ΅Π½ΠΈΠ΅ ΠΈ Π΄Π΅Π»Π΅Π½ΠΈΠ΅ дСсятичных Π΄Ρ€ΠΎΠ±Π΅ΠΉ

Π‘Π»ΠΎΠΆΠ΅Π½ΠΈΠ΅ ΠΈ Π²Ρ‹Ρ‡ΠΈΡ‚Π°Π½ΠΈΠ΅ дСсятичных Π΄Ρ€ΠΎΠ±Π΅ΠΉ Π°Π½Π°Π»ΠΎΠ³ΠΈΡ‡Π½ΠΎ слоТСнию ΠΈ Π²Ρ‹Ρ‡ΠΈΡ‚Π°Π½ΠΈΡŽ Π½Π°Ρ‚ΡƒΡ€Π°Π»ΡŒΠ½Ρ‹Ρ… чисСл, Π½ΠΎ с ΠΎΠΏΡ€Π΅Π΄Π΅Π»Π΅Π½Π½Ρ‹ΠΌΠΈ условиями.

ΠŸΡ€Π°Π²ΠΈΠ»ΠΎ. Π‘Π»ΠΎΠΆΠ΅Π½ΠΈΠ΅ ΠΈ Π²Ρ‹Ρ‡ΠΈΡ‚Π°Π½ΠΈΠ΅ дСсятичных Π΄Ρ€ΠΎΠ±Π΅ΠΉ производится ΠΏΠΎ разрядам Ρ†Π΅Π»ΠΎΠΉ ΠΈ Π΄Ρ€ΠΎΠ±Π½ΠΎΠΉ части ΠΊΠ°ΠΊ Π½Π°Ρ‚ΡƒΡ€Π°Π»ΡŒΠ½Ρ‹Ρ… чисСл.

ΠŸΡ€ΠΈ письмСнном слоТСнии ΠΈ Π²Ρ‹Ρ‡ΠΈΡ‚Π°Π½ΠΈΠΈ дСсятичных Π΄Ρ€ΠΎΠ±Π΅ΠΉ запятая, ΠΎΡ‚Π΄Π΅Π»ΡΡŽΡ‰Π°Ρ Ρ†Π΅Π»ΡƒΡŽ Ρ‡Π°ΡΡ‚ΡŒ ΠΎΡ‚ Π΄Ρ€ΠΎΠ±Π½ΠΎΠΉ, Π΄ΠΎΠ»ΠΆΠ½Π° Π½Π°Ρ…ΠΎΠ΄ΠΈΡ‚ΡŒΡΡ Ρƒ слагаСмых ΠΈ суммы ΠΈΠ»ΠΈ Ρƒ ΡƒΠΌΠ΅Π½ΡŒΡˆΠ°Π΅ΠΌΠΎΠ³ΠΎ, Π²Ρ‹Ρ‡ΠΈΡ‚Π°Π΅ΠΌΠΎΠ³ΠΎ ΠΈ разности Π² ΠΎΠ΄Π½ΠΎΠΌ столбцС (запятая ΠΏΠΎΠ΄ запятой ΠΎΡ‚ записи условия Π΄ΠΎ ΠΊΠΎΠ½Ρ†Π° вычислСния).

ΠŸΡ€ΠΈΠΌΠ΅Ρ€Ρ‹.

Π‘Π»ΠΎΠΆΠ΅Π½ΠΈΠ΅ ΠΈ Π²Ρ‹Ρ‡ΠΈΡ‚Π°Π½ΠΈΠ΅ дСсятичных Π΄Ρ€ΠΎΠ±Π΅ΠΉ Π² строку:

243,625 + 24,026 = 200 + 40 + 3 + 0,6 + 0,02 + 0,005 + 20 + 4 + 0,02 + 0,006 = 200 + (40 + 20) + (3 + 4)+ 0,6 + (0,02 + 0,02) + (0,005 + 0,006) = 200 + 60 + 7 + 0,6 + 0,04 + 0,011 = 200 + 60 + 7 + 0,6 + (0,04 + 0,01) + 0,001 = 200 + 60 + 7 + 0,6 + 0,05 + 0,001 = 267,651

843,217Β β€” 700,628 = (800Β β€” 700) + 40 + 3 + (0,2Β β€” 0,6) + (0,01Β β€” 0,02) + (0,007Β β€” 0,008) = 100 + 40 + 2 + (1,2Β β€” 0,6) + (0,01Β β€” 0,02) + (0,007Β β€” 0,008) = 100 + 40 + 2 + 0,5 + (0,11Β β€” 0,02) + (0,007Β β€” 0,008) = 100 + 40 + 2 + 0,5 + 0,09 + (0,007Β β€” 0,008) = 100 + 40 + 2 + 0,5 + 0,08 + (0,017Β β€” 0,008) = 100 + 40 + 2 + 0,5 + 0,08 + 0,009 = 142,589

Π‘Π»ΠΎΠΆΠ΅Π½ΠΈΠ΅ ΠΈ Π²Ρ‹Ρ‡ΠΈΡ‚Π°Π½ΠΈΠ΅ дСсятичных Π΄Ρ€ΠΎΠ±Π΅ΠΉ Π² столбик:

Π‘Π»ΠΎΠΆΠ΅Π½ΠΈΠ΅ дСсятичных Π΄Ρ€ΠΎΠ±Π΅ΠΉ Ρ‚Ρ€Π΅Π±ΡƒΠ΅Ρ‚ Π²Π΅Ρ€Ρ…Π½Π΅ΠΉ Π΄ΠΎΠΏΠΎΠ»Π½ΠΈΡ‚Π΅Π»ΡŒΠ½ΠΎΠΉ строки для записи чисСл, ΠΊΠΎΠ³Π΄Π° сумма разряда ΠΏΠ΅Ρ€Π΅Ρ…ΠΎΠ΄ΠΈΡ‚ Ρ‡Π΅Ρ€Π΅Π· дСсяток. Π’Ρ‹Ρ‡ΠΈΡ‚Π°Π½ΠΈΠ΅ дСсятичных Π΄Ρ€ΠΎΠ±Π΅ΠΉ Ρ‚Ρ€Π΅Π±ΡƒΠ΅Ρ‚ Π²Π΅Ρ€Ρ…Π½Π΅ΠΉ Π΄ΠΎΠΏΠΎΠ»Π½ΠΈΡ‚Π΅Π»ΡŒΠ½ΠΎΠΉ строки для Ρ‚ΠΎΠ³ΠΎ, Ρ‡Ρ‚ΠΎΠ±Ρ‹ ΠΎΡ‚ΠΌΠ΅Ρ‚ΠΈΡ‚ΡŒ разряд, Π² ΠΊΠΎΡ‚ΠΎΡ€ΠΎΠΌ одалТиваСтся 1.

Если справа ΠΎΡ‚ слагаСмого ΠΈΠ»ΠΈ ΡƒΠΌΠ΅Π½ΡŒΡˆΠ°Π΅ΠΌΠΎΠ³ΠΎ Π½Π΅ Ρ…Π²Π°Ρ‚Π°Π΅Ρ‚ разрядов Π΄Ρ€ΠΎΠ±Π½ΠΎΠΉ части, Ρ‚ΠΎ справа Π² Π΄Ρ€ΠΎΠ±Π½ΠΎΠΉ части ΠΌΠΎΠΆΠ½ΠΎ Π΄ΠΎΠΏΠΈΡΡ‹Π²Π°Ρ‚ΡŒ ΡΡ‚ΠΎΠ»ΡŒΠΊΠΎ Π½ΡƒΠ»Π΅ΠΉ (ΡƒΠ²Π΅Π»ΠΈΡ‡ΠΈΠ²Π°Ρ‚ΡŒ Ρ€Π°Π·Ρ€ΡΠ΄Π½ΠΎΡΡ‚ΡŒ Π΄Ρ€ΠΎΠ±Π½ΠΎΠΉ части), сколько разрядов Π² Π΄Ρ€ΡƒΠ³ΠΎΠΌ слагаСмом ΠΈΠ»ΠΈ ΡƒΠΌΠ΅Π½ΡŒΡˆΠ°Π΅ΠΌΠΎΠΌ.

Π£ΠΌΠ½ΠΎΠΆΠ΅Π½ΠΈΠ΅ дСсятичных Π΄Ρ€ΠΎΠ±Π΅ΠΉ производится Ρ‚Π°ΠΊ ΠΆΠ΅, ΠΊΠ°ΠΊ ΠΈ ΡƒΠΌΠ½ΠΎΠΆΠ΅Π½ΠΈΠ΅ Π½Π°Ρ‚ΡƒΡ€Π°Π»ΡŒΠ½Ρ‹Ρ… чисСл, ΠΏΠΎ Ρ‚Π΅ΠΌ ΠΆΠ΅ ΠΏΡ€Π°Π²ΠΈΠ»Π°ΠΌ, Π½ΠΎ Π² ΠΏΡ€ΠΎΠΈΠ·Π²Π΅Π΄Π΅Π½ΠΈΠΈ ставится запятая ΠΏΠΎ суммС разрядов ΠΌΠ½ΠΎΠΆΠΈΡ‚Π΅Π»Π΅ΠΉ Π² Π΄Ρ€ΠΎΠ±Π½ΠΎΠΉ части, считая справа Π½Π°Π»Π΅Π²ΠΎ (сумма разрядов ΠΌΠ½ΠΎΠΆΠΈΡ‚Π΅Π»Π΅ΠΉ β€” это количСство разрядов послС запятой Ρƒ ΠΌΠ½ΠΎΠΆΠΈΡ‚Π΅Π»Π΅ΠΉ, вмСстС взятых).

ΠŸΡ€ΠΈΠΌΠ΅Ρ€:

ΠŸΡ€ΠΈ ΡƒΠΌΠ½ΠΎΠΆΠ΅Π½ΠΈΠΈ дСсятичных Π΄Ρ€ΠΎΠ±Π΅ΠΉ Π² столбик пСрвая справа значащая Ρ†ΠΈΡ„Ρ€Π° подписываСтся ΠΏΠΎΠ΄ ΠΏΠ΅Ρ€Π²ΠΎΠΉ справа Π·Π½Π°Ρ‡Π°Ρ‰Π΅ΠΉ Ρ†ΠΈΡ„Ρ€ΠΎΠΉ, ΠΊΠ°ΠΊ ΠΈ Π² Π½Π°Ρ‚ΡƒΡ€Π°Π»ΡŒΠ½Ρ‹Ρ… числах:

Π—Π°ΠΏΠΈΡΡŒ умноТСния дСсятичных Π΄Ρ€ΠΎΠ±Π΅ΠΉ Π² столбик:

Π—Π°ΠΏΠΈΡΡŒ дСлСния дСсятичных Π΄Ρ€ΠΎΠ±Π΅ΠΉ Π² столбик:

ΠŸΠΎΠ΄Ρ‡Π΅Ρ€ΠΊΠ½ΡƒΡ‚Ρ‹Π΅ Π·Π½Π°ΠΊΠΈ β€” это Π·Π½Π°ΠΊΠΈ, Π·Π° ΠΊΠΎΡ‚ΠΎΡ€Ρ‹Π΅ пСрСносится запятая, ΠΏΠΎΡ‚ΠΎΠΌΡƒ Ρ‡Ρ‚ΠΎ Π΄Π΅Π»ΠΈΡ‚Π΅Π»ΡŒ Π΄ΠΎΠ»ΠΆΠ΅Π½ Π±Ρ‹Ρ‚ΡŒ Ρ†Π΅Π»Ρ‹ΠΌ числом.

ΠŸΡ€Π°Π²ΠΈΠ»ΠΎ. ΠŸΡ€ΠΈ Π΄Π΅Π»Π΅Π½ΠΈΠΈ Π΄Ρ€ΠΎΠ±Π΅ΠΉ Π΄Π΅Π»ΠΈΡ‚Π΅Π»ΡŒ дСсятичной Π΄Ρ€ΠΎΠ±ΠΈ увСличиваСтся Π½Π° ΡΡ‚ΠΎΠ»ΡŒΠΊΠΎ разрядов, сколько разрядов Π² Π΄Ρ€ΠΎΠ±Π½ΠΎΠΉ Π΅Π³ΠΎ части. Π§Ρ‚ΠΎΠ±Ρ‹ Π΄Ρ€ΠΎΠ±ΡŒ Π½Π΅ измСнилась, Π½Π° ΡΡ‚ΠΎΠ»ΡŒΠΊΠΎ ΠΆΠ΅ разрядов увСличиваСтся ΠΈ Π΄Π΅Π»ΠΈΠΌΠΎΠ΅ (Π² Π΄Π΅Π»ΠΈΠΌΠΎΠΌ ΠΈ Π΄Π΅Π»ΠΈΡ‚Π΅Π»Π΅ запятая пСрСносится Π½Π° ΠΎΠ΄Π½ΠΎ ΠΈ Ρ‚ΠΎ ΠΆΠ΅ число Π·Π½Π°ΠΊΠΎΠ²). Запятая ставится Π² частном Π½Π° Ρ‚ΠΎΠΌ этапС дСлСния, ΠΊΠΎΠ³Π΄Π° цСлая Ρ‡Π°ΡΡ‚ΡŒ Π΄Ρ€ΠΎΠ±ΠΈ Ρ€Π°Π·Π΄Π΅Π»Π΅Π½Π°.

Для дСсятичных Π΄Ρ€ΠΎΠ±Π΅ΠΉ, ΠΊΠ°ΠΊ ΠΈ для Π½Π°Ρ‚ΡƒΡ€Π°Π»ΡŒΠ½Ρ‹Ρ… чисСл, сохраняСтся ΠΏΡ€Π°Π²ΠΈΠ»ΠΎ: Π½Π° ноль Π΄Π΅ΡΡΡ‚ΠΈΡ‡Π½ΡƒΡŽ Π΄Ρ€ΠΎΠ±ΡŒ Π΄Π΅Π»ΠΈΡ‚ΡŒ нСльзя!


Π£Ρ€ΠΎΠΊ 65. Π΄Π΅Π»Π΅Π½ΠΈΠ΅ Π΄Ρ€ΠΎΠ±Π΅ΠΉ — ΠœΠ°Ρ‚Π΅ΠΌΠ°Ρ‚ΠΈΠΊΠ° — 5 класс

ΠœΠ°Ρ‚Π΅ΠΌΠ°Ρ‚ΠΈΠΊΠ°

5 класс

Π£Ρ€ΠΎΠΊ β„– 65

Π”Π΅Π»Π΅Π½ΠΈΠ΅ Π΄Ρ€ΠΎΠ±Π΅ΠΉ

ΠŸΠ΅Ρ€Π΅Ρ‡Π΅Π½ΡŒ рассматриваСмых вопросов:

– ΠΏΡ€Π°Π²ΠΈΠ»ΠΎ дСлСния ΠΎΠ±Ρ‹ΠΊΠ½ΠΎΠ²Π΅Π½Π½Ρ‹Ρ… Π΄Ρ€ΠΎΠ±Π΅ΠΉ;

– Π΄Π΅Π»Π΅Π½ΠΈΠ΅ Π΄Ρ€ΠΎΠ±ΠΈ Π½Π° Π½Π°Ρ‚ΡƒΡ€Π°Π»ΡŒΠ½ΠΎΠ΅ число.

ВСзаурус

ЧастноС Π΄Π²ΡƒΡ… Π΄Ρ€ΠΎΠ±Π΅ΠΉ – это Π΄Ρ€ΠΎΠ±ΡŒ, которая ΠΏΡ€ΠΈ ΡƒΠΌΠ½ΠΎΠΆΠ΅Π½ΠΈΠΈ Π½Π° Π΄Π΅Π»ΠΈΡ‚Π΅Π»ΡŒ Π΄Π°Ρ‘Ρ‚ Π΄Π΅Π»ΠΈΠΌΠΎΠ΅.

ΠŸΠ΅Ρ€ΠΈΠΌΠ΅Ρ‚Ρ€ – сумма Π΄Π»ΠΈΠ½ всСх сторон плоской гСомСтричСской Ρ„ΠΈΠ³ΡƒΡ€Ρ‹.

ΠžΠ±ΡΠ·Π°Ρ‚Π΅Π»ΡŒΠ½Π°Ρ Π»ΠΈΡ‚Π΅Ρ€Π°Ρ‚ΡƒΡ€Π°:

1. Никольский Π‘. М. ΠœΠ°Ρ‚Π΅ΠΌΠ°Ρ‚ΠΈΠΊΠ°. 5 класс. Π£Ρ‡Π΅Π±Π½ΠΈΠΊ для ΠΎΠ±Ρ‰Π΅ΠΎΠ±Ρ€Π°Π·ΠΎΠ²Π°Ρ‚Π΅Π»ΡŒΠ½Ρ‹Ρ… ΡƒΡ‡Ρ€Π΅ΠΆΠ΄Π΅Π½ΠΈΠΉ. Π€Π“ΠžΠ‘. / Π‘. М. Никольский, М. К. ΠŸΠΎΡ‚Π°ΠΏΠΎΠ², Н. Н. Π Π΅ΡˆΠ΅Ρ‚Π½ΠΈΠΊΠΎΠ² ΠΈ Π΄Ρ€.– М.: ΠŸΡ€ΠΎΡΠ²Π΅Ρ‰Π΅Π½ΠΈΠ΅, 2017, стр. 272.

ВСорСтичСский ΠΌΠ°Ρ‚Π΅Ρ€ΠΈΠ°Π» для ΡΠ°ΠΌΠΎΡΡ‚ΠΎΡΡ‚Π΅Π»ΡŒΠ½ΠΎΠ³ΠΎ изучСния

БСгодня ΠΌΡ‹ с Π²Π°ΠΌΠΈ познакомимся с послСдним, Ρ‡Π΅Ρ‚Π²Ρ‘Ρ€Ρ‚Ρ‹ΠΌ арифмСтичСским дСйствиСм Π½Π°Π΄ ΠΎΠ±Ρ‹ΠΊΠ½ΠΎΠ²Π΅Π½Π½Ρ‹ΠΌΠΈ дробями – Π΄Π΅Π»Π΅Π½ΠΈΠ΅ΠΌ.

Частным Π΄Π²ΡƒΡ… Π΄Ρ€ΠΎΠ±Π΅ΠΉ Π½Π°Π·Ρ‹Π²Π°ΡŽΡ‚ Π΄Ρ€ΠΎΠ±ΡŒ, которая ΠΏΡ€ΠΈ ΡƒΠΌΠ½ΠΎΠΆΠ΅Π½ΠΈΠΈ Π½Π° Π΄Π΅Π»ΠΈΡ‚Π΅Π»ΡŒ Π΄Π°Ρ‘Ρ‚ Π΄Π΅Π»ΠΈΠΌΠΎΠ΅.

ΠŸΡ€ΠΎΠ²Π΅Ρ€ΠΈΠΌ ΠΏΡ€Π°Π²ΠΈΠ»ΠΎ, ΡƒΠΌΠ½ΠΎΠΆΠΈΠ² частноС Π½Π° Π΄Π΅Π»ΠΈΡ‚Π΅Π»ΡŒ:

Π”Π΅ΠΉΡΡ‚Π²ΠΈΡ‚Π΅Π»ΡŒΠ½ΠΎ, Ссли ΡƒΠΌΠ½ΠΎΠΆΠΈΡ‚ΡŒ частноС Π½Π° Π΄Π΅Π»ΠΈΡ‚Π΅Π»ΡŒ, ΠΌΡ‹ ΠΏΠΎΠ»ΡƒΡ‡ΠΈΠΌ Π΄Π΅Π»ΠΈΠΌΠΎΠ΅.

НапримСр, Π΄Π²Π΅ Ρ‚Ρ€Π΅Ρ‚ΡŒΠΈΡ… Ρ€Π°Π·Π΄Π΅Π»ΠΈΠΌ Π½Π° Ρ‡Π΅Ρ‚Ρ‹Ρ€Π΅ пятых. Π—Π°ΠΌΠ΅Π½ΠΈΠΌ Π΄Π΅Π»Π΅Π½ΠΈΠ΅ ΡƒΠΌΠ½ΠΎΠΆΠ΅Π½ΠΈΠ΅ΠΌ, Π° Π΄Π΅Π»ΠΈΡ‚Π΅Π»ΡŒ Ρ‡Π΅Ρ‚Ρ‹Ρ€Π΅ пятых Π½Π° ΠΎΠ±Ρ€Π°Ρ‚Π½ΡƒΡŽ Π΄Ρ€ΠΎΠ±ΡŒ – ΠΏΡΡ‚ΡŒ Ρ‡Π΅Ρ‚Π²Ρ‘Ρ€Ρ‚Ρ‹Ρ…. Π‘ΠΎΠΊΡ€Π°Ρ‚ΠΈΠΌ Π΄Π²Π° ΠΈ Ρ‡Π΅Ρ‚Ρ‹Ρ€Π΅, ΠΏΠΎΠ»ΡƒΡ‡ΠΈΠΌ Π΄Ρ€ΠΎΠ±ΡŒ ΠΏΡΡ‚ΡŒ ΡˆΠ΅ΡΡ‚Ρ‹Ρ….

ΠŸΡΡ‚ΡŒ ΡΠ΅Π΄ΡŒΠΌΡ‹Ρ… Ρ€Π°Π·Π΄Π΅Π»ΠΈΠΌ Π½Π° сСмь пятых. Π£ΠΌΠ½ΠΎΠΆΠΈΠΌ Π΄Π΅Π»ΠΈΠΌΠΎΠ΅ Π½Π° Π΄Ρ€ΠΎΠ±ΡŒ, ΠΎΠ±Ρ€Π°Ρ‚Π½ΡƒΡŽ Π΄Π΅Π»ΠΈΡ‚Π΅Π»ΡŽ. ΠŸΠΎΠ»ΡƒΡ‡ΠΈΠΌ Π΄Π²Π°Π΄Ρ†Π°Ρ‚ΡŒ ΠΏΡΡ‚ΡŒ сорок дСвятых.

РСшим Π·Π°Π΄Π°Ρ‡Ρƒ.

ΠšΡƒΡΠΎΠΊ Ρ‚ΠΊΠ°Π½ΠΈ Π΄Π»ΠΈΠ½ΠΎΠΉ 4 ΠΌΠ΅Ρ‚Ρ€Π° Ρ€Π°Π·Π΄Π΅Π»ΠΈΠ»ΠΈ Π½Π° нСсколько Ρ€Π°Π²Π½Ρ‹Ρ… частСй Π΄Π»ΠΈΠ½ΠΎΠΉ ΠΌΠ΅Ρ‚Ρ€Π°. Бколько Ρ‚Π°ΠΊΠΈΡ… частСй ΠΏΠΎΠ»ΡƒΡ‡ΠΈΠ»ΠΎΡΡŒ?

РСшСниС. Π˜Ρ‚Π°ΠΊ, прСдставим Π½Π°Ρ‚ΡƒΡ€Π°Π»ΡŒΠ½ΠΎΠ΅ число Ρ‡Π΅Ρ‚Ρ‹Ρ€Π΅ Π² Π²ΠΈΠ΄Π΅ ΠΎΠ±Ρ‹ΠΊΠ½ΠΎΠ²Π΅Π½Π½ΠΎΠΉ Π΄Ρ€ΠΎΠ±ΠΈ – Ρ‡Π΅Ρ‚Ρ‹Ρ€Π΅ ΠΏΠ΅Ρ€Π²Ρ‹Ρ…, ΠΈ ΠΏΡ€ΠΈΠΌΠ΅Π½ΠΈΠΌ ΠΏΡ€Π°Π²ΠΈΠ»ΠΎ дСлСния Π΄Π²ΡƒΡ… ΠΎΠ±Ρ‹ΠΊΠ½ΠΎΠ²Π΅Π½Π½Ρ‹Ρ… Π΄Ρ€ΠΎΠ±Π΅ΠΉ. Π§Ρ‚ΠΎΠ±Ρ‹ ΡƒΠ·Π½Π°Ρ‚ΡŒ, сколько частСй Ρ‚ΠΊΠ°Π½ΠΈ ΠΏΠΎΠ»ΡƒΡ‡ΠΈΠ»ΠΎΡΡŒ, Ρ€Π°Π·Π΄Π΅Π»ΠΈΠΌ Ρ‡Π΅Ρ‚Ρ‹Ρ€Π΅ ΠΌΠ΅Ρ‚Ρ€Π° Ρ‚ΠΊΠ°Π½ΠΈ Π½Π° ΠΎΠ΄Π½Ρƒ Ρ‡Π°ΡΡ‚ΡŒ Π΄Π»ΠΈΠ½ΠΎΠΉ ΠΌΠ΅Ρ‚Ρ€Π°.

ΠžΡ‚Π²Π΅Ρ‚: 32 части Ρ‚ΠΊΠ°Π½ΠΈ.

Π§Ρ‚ΠΎΠ±Ρ‹ Ρ€Π°Π·Π΄Π΅Π»ΠΈΡ‚ΡŒ Π΄Ρ€ΠΎΠ±ΡŒ Π½Π° Π½Π°Ρ‚ΡƒΡ€Π°Π»ΡŒΠ½ΠΎΠ΅ число, ΠΌΠΎΠΆΠ½ΠΎ Π΅Ρ‘ Π·Π½Π°ΠΌΠ΅Π½Π°Ρ‚Π΅Π»ΡŒ ΡƒΠΌΠ½ΠΎΠΆΠΈΡ‚ΡŒ Π½Π° это число.

НапримСр, Ρ€Π°Π·Π΄Π΅Π»ΠΈΠΌ ΡˆΠ΅ΡΡ‚ΡŒ ΠΎΠ΄ΠΈΠ½Π½Π°Π΄Ρ†Π°Ρ‚Ρ‹Ρ… Π½Π° ΠΏΡΡ‚ΡŒ. ΠŸΡ€ΠΈΠΌΠ΅Π½ΡΡ ΠΏΡ€Π°Π²ΠΈΠ»ΠΎ дСлСния Π΄Ρ€ΠΎΠ±ΠΈ Π½Π° Π½Π°Ρ‚ΡƒΡ€Π°Π»ΡŒΠ½ΠΎΠ΅ число, ΠΏΠΎΠ»ΡƒΡ‡Π°Π΅ΠΌ, Ρ‡Ρ‚ΠΎ Π·Π½Π°ΠΌΠ΅Π½Π°Ρ‚Π΅Π»ΡŒ, Ρ€Π°Π²Π½Ρ‹ΠΉ ΠΎΠ΄ΠΈΠ½Π½Π°Π΄Ρ†Π°Ρ‚ΠΈ, Π½ΡƒΠΆΠ½ΠΎ ΡƒΠΌΠ½ΠΎΠΆΠΈΡ‚ΡŒ Π½Π° Π½Π°Ρ‚ΡƒΡ€Π°Π»ΡŒΠ½ΠΎΠ΅ число ΠΏΡΡ‚ΡŒ. ΠŸΠΎΠ»ΡƒΡ‡Π°Π΅ΠΌ:

ΠžΡ‚ΠΌΠ΅Ρ‚ΠΈΠΌ, Ρ‡Ρ‚ΠΎ число 0, Π΄Π΅Π»Ρ‘Π½Π½ΠΎΠ΅ Π½Π° Π»ΡŽΠ±ΡƒΡŽ ΠΎΡ‚Π»ΠΈΡ‡Π½ΡƒΡŽ ΠΎΡ‚ нуля Π΄Ρ€ΠΎΠ±ΡŒ, Π΄Π°Ρ‘Ρ‚ 0.

Π”Π΅Π»ΠΈΡ‚ΡŒ Π½Π° Π½ΡƒΠ»ΡŒ нСльзя!

Для Π»ΡŽΠ±Ρ‹Ρ… Π΄Π²ΡƒΡ… Π½Π°Ρ‚ΡƒΡ€Π°Π»ΡŒΠ½Ρ‹Ρ… чисСл p ΠΈ q всСгда Π΅ΡΡ‚ΡŒ ΠΈΡ… частноС – Π΄Ρ€ΠΎΠ±ΡŒ .

Π§Π΅Ρ€Ρ‚Ρƒ Π² записи Π΄Ρ€ΠΎΠ±ΠΈ ΠΌΠΎΠΆΠ½ΠΎ Ρ€Π°ΡΡΠΌΠ°Ρ‚Ρ€ΠΈΠ²Π°Ρ‚ΡŒ ΠΊΠ°ΠΊ Π·Π½Π°ΠΊ дСлСния числитСля Π½Π° Π·Π½Π°ΠΌΠ΅Π½Π°Ρ‚Π΅Π»ΡŒ. ΠŸΠΎΡΡ‚ΠΎΠΌΡƒ ΠΈΠ½ΠΎΠ³Π΄Π° говорят Π½Π΅ Β«Ρ‚Ρ€ΠΈ пятых», Π° Β«Ρ‚Ρ€ΠΈ, Π΄Π΅Π»Ρ‘Π½Π½ΠΎΠ΅ Π½Π° ΠΏΡΡ‚ΡŒΒ»:

Π˜Ρ‚Π°ΠΊ, Π΅Ρ‰Ρ‘ Ρ€Π°Π· ΠΏΠΎΠ²Ρ‚ΠΎΡ€ΠΈΠΌ Π°Π»Π³ΠΎΡ€ΠΈΡ‚ΠΌ дСлСния ΠΎΠ±Ρ‹ΠΊΠ½ΠΎΠ²Π΅Π½Π½Ρ‹Ρ… Π΄Ρ€ΠΎΠ±Π΅ΠΉ:

1) замСняСм Π΄Π΅Π»Π΅Π½ΠΈΠ΅ ΡƒΠΌΠ½ΠΎΠΆΠ΅Π½ΠΈΠ΅ΠΌ;

2) Β«ΠΏΠ΅Ρ€Π΅Π²ΠΎΡ€Π°Ρ‡ΠΈΠ²Π°Π΅ΠΌΒ» Π΄Π΅Π»ΠΈΡ‚Π΅Π»ΡŒ;

3) примСняСм ΠΏΡ€Π°Π²ΠΈΠ»ΠΎ умноТСния ΠΎΠ±Ρ‹ΠΊΠ½ΠΎΠ²Π΅Π½Π½Ρ‹Ρ… Π΄Ρ€ΠΎΠ±Π΅ΠΉ.

Π Π°Π·Π±ΠΎΡ€ Ρ€Π΅ΡˆΠ΅Π½ΠΈΡ Π·Π°Π΄Π°Π½ΠΈΠΉ Ρ‚Ρ€Π΅Π½ΠΈΡ€ΠΎΠ²ΠΎΡ‡Π½ΠΎΠ³ΠΎ модуля

β„– 1. ВычислитС Π·Π½Π°Ρ‡Π΅Π½ΠΈΠ΅ выраТСния .

Π‘Π½Π°Ρ‡Π°Π»Π° ΠΎΠΏΡ€Π΅Π΄Π΅Π»ΠΈΠΌ порядок дСйствий:

  1. Π²Ρ‹Ρ‡ΠΈΡ‚Π°Π½ΠΈΠ΅, Ρ‚. ΠΊ. ΠΎΠ½ΠΎ находится Π² скобках;
  2. Π΄Π΅Π»Π΅Π½ΠΈΠ΅;
  3. ΡƒΠΌΠ½ΠΎΠΆΠ΅Π½ΠΈΠ΅;
  4. слоТСниС.

ΠžΡ‚Π²Π΅Ρ‚: .

β„– 2. НайдитС частноС

РСшСниС: Ρ€Π°Π·Π΄Π΅Π»ΠΈΠΌ Π΄Ρ€ΠΎΠ±ΠΈ, ΠΏΡ€ΠΈΠΌΠ΅Π½ΠΈΠ² ΠΏΡ€Π°Π²ΠΈΠ»ΠΎ дСлСния Π΄Ρ€ΠΎΠ±Π΅ΠΉ ΠΈ сократим ΠΏΠΎΠ»ΡƒΡ‡Π΅Π½Π½Ρ‹ΠΉ Ρ€Π΅Π·ΡƒΠ»ΡŒΡ‚Π°Ρ‚. ΠŸΠΎΠ»ΡƒΡ‡Π°Π΅ΠΌ:

ΠžΡ‚Π²Π΅Ρ‚: .

ΠžΠΏΠ΅Ρ€Π°Ρ†ΠΈΠΈ с дробями — ΠΌΠ°Ρ‚Π΅ΠΌΠ°Ρ‚ΠΈΠΊΠ° 5-Π³ΠΎ класса

Π‘Π»ΠΎΠΆΠ΅Π½ΠΈΠ΅ Π΄Ρ€ΠΎΠ±Π΅ΠΉ

Π”Ρ€ΠΎΠ±ΠΈ — это части Ρ†Π΅Π»ΠΎΠ³ΠΎ. Π‘Π»ΠΎΠΆΠΈΠ² Π΄Π²Π΅ Π΄Ρ€ΠΎΠ±ΠΈ, Π²Ρ‹ ΠΏΠΎΠ»ΡƒΡ‡ΠΈΡ‚Π΅ Ρ†Π΅Π»ΠΎΠ΅. НапримСр, Ссли Ρƒ вас Π΅ΡΡ‚ΡŒ ΠΏΠΎΠ»ΠΎΠ²ΠΈΠ½Π° Ρ„Π°ΠΉΠ»Π° cookie ΠΈ ΠΎΠ΄Π½Π° Ρ‚Ρ€Π΅Ρ‚ΡŒ Ρ„Π°ΠΉΠ»Π° cookie, Π²Ρ‹ ΠΏΡ€ΠΈΠ±Π»ΠΈΠΆΠ°Π΅Ρ‚Π΅ΡΡŒ ΠΊ созданию Ρ†Π΅Π»ΠΎΠ³ΠΎ Ρ„Π°ΠΉΠ»Π° cookie. Π§Ρ‚ΠΎΠ±Ρ‹ ΡΠ»ΠΎΠΆΠΈΡ‚ΡŒ эти Π΄Ρ€ΠΎΠ±ΠΈ вмСстС, ΠΊΠ°ΠΆΠ΄ΡƒΡŽ Π΄Ρ€ΠΎΠ±ΡŒ Π½Π΅ΠΎΠ±Ρ…ΠΎΠ΄ΠΈΠΌΠΎ Ρ€Π°Π·Π±ΠΈΡ‚ΡŒ Π½Π° ΠΎΠ΄ΠΈΠ½Π°ΠΊΠΎΠ²ΠΎΠ΅ количСство частСй. НиТС ΠΏΡ€ΠΈΠ²Π΅Π΄Π΅Π½Ρ‹ Π½Π΅ΠΊΠΎΡ‚ΠΎΡ€Ρ‹Π΅ стратСгии, ΠΊΠΎΡ‚ΠΎΡ€Ρ‹Π΅ ΠΏΠΎΠΌΠΎΠ³ΡƒΡ‚ Π²Π°ΠΌ ΡΠ»ΠΎΠΆΠΈΡ‚ΡŒ Π΄Ρ€ΠΎΠ±ΠΈ.

БтратСгия: ΠŸΠ»ΠΎΡ‰Π°Π΄ΡŒ ΠΈΠ»ΠΈ ΠΏΡ€ΡΠΌΠΎΡƒΠ³ΠΎΠ»ΡŒΠ½Π°Ρ модСль для отобраТСния слоТСния Π΄Ρ€ΠΎΠ±Π΅ΠΉ

Π¨Π°Π³ 1: НарисуйтС ΠΊΠ°ΠΆΠ΄ΡƒΡŽ Π΄Ρ€ΠΎΠ±ΡŒ (ΠΎΠ΄Π½Ρƒ с Π³ΠΎΡ€ΠΈΠ·ΠΎΠ½Ρ‚Π°Π»ΡŒΠ½Ρ‹ΠΌ Ρ€Π°Π·Π΄Π΅Π»Π΅Π½ΠΈΠ΅ΠΌ, Π° Π΄Ρ€ΡƒΠ³ΡƒΡŽ с Π²Π΅Ρ€Ρ‚ΠΈΠΊΠ°Π»ΡŒΠ½Ρ‹ΠΌ Ρ€Π°Π·Π΄Π΅Π»Π΅Π½ΠΈΠ΅ΠΌ)

Π¨Π°Π³ 2: НарисуйтС ΠΏΡƒΠ½ΠΊΡ‚ΠΈΡ€Π½Ρ‹Π΅ Π»ΠΈΠ½ΠΈΠΈ Π² ΠΊΠ°ΠΆΠ΄ΠΎΠΉ Π΄Ρ€ΠΎΠ±ΠΈ Ρ‚Π°ΠΊ, Ρ‡Ρ‚ΠΎΠ±Ρ‹ Ρƒ Π½ΠΈΡ… Π±Ρ‹Π» ΠΎΠ±Ρ‰ΠΈΠΉ Π·Π½Π°ΠΌΠ΅Π½Π°Ρ‚Π΅Π»ΡŒ.

Π¨Π°Π³ 3: ΠŸΠΎΠ΄ΡΡ‡ΠΈΡ‚Π°ΠΉΡ‚Π΅ всС Π·Π°ΠΊΡ€Π°ΡˆΠ΅Π½Π½Ρ‹Π΅ части ΠΈ Π½Π°Ρ‡Π΅Ρ€Ρ‚ΠΈΡ‚Π΅ сумму Π΄Ρ€ΠΎΠ±Π΅ΠΉ

ΠŸΡ€ΠΈΠΌΠ΅Ρ€ 1:


ΠŸΡ€ΠΈΠΌΠ΅Ρ€ 2:

БтратСгия: Найти ΠΎΠ±Ρ‰Π΅Π΅ Π—Π½Π°ΠΌΠ΅Π½Π°Ρ‚Π΅Π»ΡŒ ΠΈ слоТитС

Π¨Π°Π³ 1. ΠŸΠΎΡΠΌΠΎΡ‚Ρ€ΠΈΡ‚Π΅ Π½Π° Π΄Ρ€ΠΎΠ±ΠΈ

Π¨Π°Π³ 2. ΠŸΠΎΠ΄ΡƒΠΌΠ°ΠΉΡ‚Π΅, ΠΊΠ°ΠΊ ΡΠ΄Π΅Π»Π°Ρ‚ΡŒ Ρ‚Π°ΠΊ, Ρ‡Ρ‚ΠΎΠ±Ρ‹ каТдая Π΄Ρ€ΠΎΠ±ΡŒ ΠΈΠΌΠ΅Π»Π° ΠΎΠ±Ρ‰ΠΈΠΉ Π·Π½Π°ΠΌΠ΅Π½Π°Ρ‚Π΅Π»ΡŒ (ΠžΠ±Ρ‰ΠΈΠΉ коэффициСнт)

Π¨Π°Π³ 3: ΠŸΠ΅Ρ€Π΅ΠΏΠΈΡˆΠΈΡ‚Π΅ Π΄Ρ€ΠΎΠ±ΠΈ Π² ΠΈΡ… ΡΠΊΠ²ΠΈΠ²Π°Π»Π΅Π½Ρ‚Π½ΡƒΡŽ Π΄Ρ€ΠΎΠ±ΡŒ с Ρ‚Π΅ΠΌΠΈ ΠΆΠ΅ знамСнатСлями

Π¨Π°Π³ 4: НайдитС сумму Π΄Ρ€ΠΎΠ±Π΅ΠΉ

Π¨Π°Π³ 5: Π—Π°ΠΏΠΈΡˆΠΈΡ‚Π΅ сумму Π΄Ρ€ΠΎΠ±ΠΈ Π² ΠΌΠ»Π°Π΄ΡˆΠΈΡ… числах


Π‘Π»ΠΎΠΆΠ΅Π½ΠΈΠ΅ Π΄Ρ€ΠΎΠ±Π΅ΠΉ со ΡΠΌΠ΅ΡˆΠ°Π½Π½Ρ‹ΠΌΠΈ числами

БтратСгия 1:

Π¨Π°Π³ 1: Π‘Π½Π°Ρ‡Π°Π»Π° слоТитС Ρ†Π΅Π»Ρ‹Π΅ числа

Π¨Π°Π³ 2: Π—Π°Ρ‚Π΅ΠΌ слоТитС Π΄Ρ€ΠΎΠ±Π½Ρ‹Π΅ части (ΠΈΡΠΏΠΎΠ»ΡŒΠ·ΡƒΠΉΡ‚Π΅ ΠΏΡ€ΠΈΠ²Π΅Π΄Π΅Π½Π½Ρ‹Π΅ Π²Ρ‹ΡˆΠ΅ стратСгии) ​​

Π¨Π°Π³ 3: ΠŸΠΎΡΠΌΠΎΡ‚Ρ€ΠΈΡ‚Π΅ Π½Π° сумму ΠΈ ΠΏΡ€ΠΈ нСобходимости ΠΏΠ΅Ρ€Π΅ΠΏΠΈΡˆΠΈΡ‚Π΅ Π΄Ρ€ΠΎΠ±ΡŒ

ΠœΡ‹ Π·Π½Π°Π΅ΠΌ, Ρ‡Ρ‚ΠΎ это Ρ‚Π°ΠΊΠΆΠ΅ эквивалСнтно 4 ΠΈ 1 / 2

Π’ΠΎΡ‚ Π΅Ρ‰Π΅ ΠΎΠ΄ΠΈΠ½ ΠΏΡ€ΠΈΠΌΠ΅Ρ€:

БтратСгия: Π˜ΡΠΏΠΎΠ»ΡŒΠ·ΡƒΠΉΡ‚Π΅ Π½Π΅ΠΏΡ€Π°Π²ΠΈΠ»ΡŒΠ½Ρ‹Π΅ Π΄Ρ€ΠΎΠ±ΠΈ для слоТСния Π΄Ρ€ΠΎΠ±Π½Ρ‹Ρ… частСй


Π¨Π°Π³ 1: ΠŸΡ€Π΅ΠΎΠ±Ρ€Π°Π·ΡƒΠΉΡ‚Π΅ ΠΈΡ… Π² Π½Π΅ΠΏΡ€Π°Π²ΠΈΠ»ΡŒΠ½Ρ‹Π΅ Π΄Ρ€ΠΎΠ±ΠΈ

Π¨Π°Π³ 2: НайдитС ΠΎΠ±Ρ‰ΠΈΠ΅ Π·Π½Π°ΠΌΠ΅Π½Π°Ρ‚Π΅Π»ΠΈ

Π¨Π°Π³ 3 : НайдитС сумму Π΄Ρ€ΠΎΠ±Π΅ΠΉ

Π¨Π°Π³ 4: Π’Ρ‹Π»ΠΎΠΆΠΈΡ‚Π΅ сумму Π΄Ρ€ΠΎΠ±Π΅ΠΉ Π² ΠΌΠ»Π°Π΄ΡˆΠΈΡ… Ρ‡Π»Π΅Π½Π°Ρ…

Π—Π°Ρ‚Π΅ΠΌ Π½Π΅ Π·Π°Π±ΡƒΠ΄ΡŒΡ‚Π΅ ΠΏΡ€Π΅ΠΎΠ±Ρ€Π°Π·ΠΎΠ²Π°Ρ‚ΡŒ Π½Π΅ΠΏΡ€Π°Π²ΠΈΠ»ΡŒΠ½ΠΎΠ΅ число Π² смСшанноС число:

Π’Ρ‹Ρ‡ΠΈΡ‚Π°Π½ΠΈΠ΅ Π΄Ρ€ΠΎΠ±Π΅ΠΉ

Π”Ρ€ΠΎΠ±ΠΈ ΡΠ²Π»ΡΡŽΡ‚ΡΡ частями вСсь. ΠŸΡ€ΠΈ Π²Ρ‹Ρ‡ΠΈΡ‚Π°Π½ΠΈΠΈ Π΄Ρ€ΠΎΠ±Π΅ΠΉ Π²Ρ‹ ΡƒΠ±ΠΈΡ€Π°Π΅Ρ‚Π΅ Ρ‡Π°ΡΡ‚ΡŒ Π΄Ρ€ΠΎΠ±ΠΈ. НапримСр, Ссли Ρƒ вас Π΅ΡΡ‚ΡŒ ΠΏΠΎΠ»ΠΎΠ²ΠΈΠ½Π° Ρ„Π°ΠΉΠ»Π° cookie, Π° Π²Ρ‹ ΠΏΠ΅Ρ€Π΅Π΄Π°Π»ΠΈ Ρ‚Ρ€Π΅Ρ‚ΡŒ Ρ„Π°ΠΉΠ»Π° cookie, Ρƒ вас останСтся мСньшС ΠΏΠΎΠ»ΠΎΠ²ΠΈΠ½Ρ‹ Ρ‚ΠΎΠ³ΠΎ, Ρ‡Ρ‚ΠΎ Ρƒ вас Π΅ΡΡ‚ΡŒ. Π£ вас останСтся ΠΎΠ΄Π½Π° ΡˆΠ΅ΡΡ‚Π°Ρ Ρ‡Π°ΡΡ‚ΡŒ ΠΎΡΡ‚Π°Π²ΡˆΠ΅Π³ΠΎΡΡ Ρ„Π°ΠΉΠ»Π° cookie.

Для Ρ€Π΅ΡˆΠ΅Π½ΠΈΡ Π·Π°Π΄Π°Ρ‡ вычитания ΠΌΠΎΠΆΠ½ΠΎ ΠΈΡΠΏΠΎΠ»ΡŒΠ·ΠΎΠ²Π°Ρ‚ΡŒ ΠΌΠ½ΠΎΠ³ΠΈΠ΅ ΠΈΠ· Ρ‚Π΅Ρ… ΠΆΠ΅ стратСгий, Ρ‡Ρ‚ΠΎ ΠΈ для Π·Π°Π΄Π°Ρ‡ слоТСния.

Π£ΠΌΠ½ΠΎΠΆΠ΅Π½ΠΈΠ΅ Π΄Ρ€ΠΎΠ±Π΅ΠΉ

Когда ΠΌΡ‹ ΡƒΠΌΠ½ΠΎΠΆΠ°Π΅ΠΌ Π΄Ρ€ΠΎΠ±ΠΈ, ΠΌΡ‹ Π΄ΡƒΠΌΠ°Π΅ΠΌ ΠΎ словС Β«OfΒ».НапримСр:

Π§Ρ‚ΠΎ Ρ‚Π°ΠΊΠΎΠ΅ 1/2 ΠΎΡ‚ 20 ΠΈΠ»ΠΈ 1/4 ΠΎΡ‚ 100. ΠœΡ‹ часто ΠΌΠΎΠΆΠ΅ΠΌ Π΄ΡƒΠΌΠ°Ρ‚ΡŒ, Ρ‡Ρ‚ΠΎ, умноТая Π΄Ρ€ΠΎΠ±ΠΈ, ΠΌΡ‹ Π΄Π΅Π»ΠΈΠΌ ΠΈΡ… Π½Π° Ρ†Π΅Π»ΠΎΠ΅ число.

Π”Π°Π²Π°ΠΉΡ‚Π΅ посмотрим Π½Π° Π½Π΅ΠΊΠΎΡ‚ΠΎΡ€Ρ‹Π΅ закономСрности:

Когда ΠΎΠ±Π° Ρ„Π°ΠΊΡ‚ΠΎΡ€Π° мСньшС, Ρ‡Π΅ΠΌ Π·Π½Π°Ρ‡Π΅Π½ΠΈΠ΅ 1, ΠΏΡ€ΠΎΠΈΠ·Π²Π΅Π΄Π΅Π½ΠΈΠ΅ мСньшС, Ρ‡Π΅ΠΌ ΠΎΠ±Π° Ρ„Π°ΠΊΡ‚ΠΎΡ€Π°.

ΠŸΡ€ΠΈΠΌΠ΅Ρ€Ρ‹:

1/2 x 1/4 —— ΠŸΠΎΠ΄ΡƒΠΌΠ°ΠΉΡ‚Π΅: 1/4 ΠΎΡ‚ 1/2 ΠΈΠ»ΠΈ 1/2 ΠΎΡ‚ 1/4


Π’Π°ΠΊΠΈΠΌ ΠΎΠ±Ρ€Π°Π·ΠΎΠΌ, ΠΎΡ‚Π²Π΅Ρ‚ 1/8 (ΠΏΡ€ΠΎΠ΄ΡƒΠΊΡ‚ мСньшС, Ρ‡Π΅ΠΌ 1/2 ΠΈ 1/4)

Π’ΠΎΡ‚ Π΅Ρ‰Π΅ ΠΎΠ΄ΠΈΠ½:

2/7 x 3/5 —— это ΠΊΠ°ΠΊ Π΄ΡƒΠΌΠ°Ρ‚ΡŒ, Ρ‡Ρ‚ΠΎ ΠΌΠ½Π΅ Π½ΡƒΠΆΠ½ΠΎ 3 / 5 ΠΈΠ· 2/7 — 6/35 мСньшС ΠΎΠ±ΠΎΠΈΡ… Ρ„Π°ΠΊΡ‚ΠΎΡ€ΠΎΠ².

Глядя Π½Π° ΠΌΠΎΠ΄Π΅Π»ΠΈ, ΠΌΡ‹ Π²ΠΈΠ΄ΠΈΠΌ, Ρ‡Ρ‚ΠΎ Π΅ΡΡ‚ΡŒ шаги, ΠΊΠΎΡ‚ΠΎΡ€Ρ‹Π΅ ΠΌΠΎΠΆΠ½ΠΎ ΠΏΡ€Π΅Π΄ΠΏΡ€ΠΈΠ½ΡΡ‚ΡŒ, Ρ‡Ρ‚ΠΎΠ±Ρ‹ Π΄ΠΎΠ±Ρ€Π°Ρ‚ΡŒΡΡ Π΄ΠΎ ΠΏΡ€ΠΎΠ΄ΡƒΠΊΡ‚Π°.

* Π£ΠΌΠ½ΠΎΠΆΠΈΡ‚ΡŒ числитСли

* Π£ΠΌΠ½ΠΎΠΆΠΈΡ‚ΡŒ Π·Π½Π°ΠΌΠ΅Π½Π°Ρ‚Π΅Π»ΠΈ

* Π£ΠΏΡ€ΠΎΡΡ‚ΠΈΡ‚ΡŒ Π΄Ρ€ΠΎΠ±ΡŒ Π΄ΠΎ Π½Π°ΠΈΠΌΠ΅Π½ΡŒΡˆΠΈΡ… Ρ‡Π»Π΅Π½ΠΎΠ² (ΠΏΡ€Π΅ΠΎΠ±Ρ€Π°Π·ΠΎΠ²Π°Ρ‚ΡŒ Π² смСшанноС число)

Когда ΠΎΠ΄ΠΈΠ½ ΠΌΠ½ΠΎΠΆΠΈΡ‚Π΅Π»ΡŒ большС Π΅Π΄ΠΈΠ½ΠΈΡ†Ρ‹, Π° ΠΎΠ΄ΠΈΠ½ мСньшС Π΅Π΄ΠΈΠ½ΠΈΡ†Ρ‹, ΠΌΡ‹ ΠΏΠΎΠ»ΡƒΡ‡Π°Π΅ΠΌ ΠΏΡ€ΠΎΠΈΠ·Π²Π΅Π΄Π΅Π½ΠΈΠ΅, ΠΊΠΎΡ‚ΠΎΡ€ΠΎΠ΅ находится ΠΌΠ΅ΠΆΠ΄Ρƒ ΠΎΠ±ΠΎΠΈΠΌΠΈ этих Ρ„Π°ΠΊΡ‚ΠΎΡ€ΠΎΠ².

ΠŸΡ€ΠΈΠΌΠ΅Ρ€Ρ‹:

1 ΠΈ 2/3 x 1/2

Π’ этом случаС Π½Π°ΠΌ Π½ΡƒΠΆΠ½ΠΎ ΠΏΠΎΠ΄ΡƒΠΌΠ°Ρ‚ΡŒ.(Π’ΠΎ ΠΆΠ΅ самоС, Ссли ΠΌΡ‹ Π΄ΡƒΠΌΠ°Π΅ΠΌ, Ρ‡Ρ‚ΠΎ 23 x 2 = (20 x 2) + (3 x 2)

1 x 1/2 = 1/2

2/3 x 1/2 = (1/2 ΠΈΠ· 2 / 3) Ρ€Π°Π²Π½ΠΎ 2/6 ΠΈΠ»ΠΈ 1/3

1/2 + 1/3 = 3/6 + 2/6 = 5/6

ΠœΡ‹ Π·Π½Π°Π΅ΠΌ, Ρ‡Ρ‚ΠΎ 5/6 большС 1/2, Π½ΠΎ мСньшС 1 ΠΈ 2/3

Когда ΠΎΠ±Π° Ρ„Π°ΠΊΡ‚ΠΎΡ€Π° большС ΠΎΠ΄Π½ΠΎΠ³ΠΎ, наш ΠΏΡ€ΠΎΠ΄ΡƒΠΊΡ‚ Π±ΡƒΠ΄Π΅Ρ‚ большС, Ρ‡Π΅ΠΌ ΠΎΠ±Π° этих Ρ„Π°ΠΊΡ‚ΠΎΡ€Π°.

ΠŸΡ€ΠΈΠΌΠ΅Ρ€:

2 1/2 x 1 2/3 (Π΄Π²Π° ΡΠΌΠ΅ΡˆΠ°Π½Π½Ρ‹Π΅ числа)

На Π΄Π°Π½Π½Ρ‹ΠΉ ΠΌΠΎΠΌΠ΅Π½Ρ‚ ΠΈΠΌΠ΅Π΅Ρ‚ смысл ΡΠ΄Π΅Π»Π°Ρ‚ΡŒ ΠΈΡ… Π½Π΅ΠΏΡ€Π°Π²ΠΈΠ»ΡŒΠ½Ρ‹ΠΌΠΈ дробями.

2 1/2 = 5/2

1 2/3 = 5/3

5/2 x 5/3 = 25/6 = 4 ΠΈ 1/6 (ΠΏΡ€ΠΎΠΈΠ·Π²Π΅Π΄Π΅Π½ΠΈΠ΅ большС ΠΎΠ±ΠΎΠΈΡ… Ρ„Π°ΠΊΡ‚ΠΎΡ€ΠΎΠ²)



Когда ΠΎΠ΄ΠΈΠ½ ΠΈΠ· Ρ„Π°ΠΊΡ‚ΠΎΡ€ΠΎΠ² Ρ€Π°Π²Π΅Π½ 1, Ρ‚ΠΎΠ³Π΄Π° ΠΏΡ€ΠΎΠ΄ΡƒΠΊΡ‚ всСгда Π±ΡƒΠ΄Π΅Ρ‚ Π΄Ρ€ΡƒΠ³ΠΈΠΌ Ρ„Π°ΠΊΡ‚ΠΎΡ€ΠΎΠΌ. (Π˜Π”Π•ΠΠ’Π˜Π§ΠΠžΠ‘Π’Π¬ Π‘Π’ΠžΠ™Π‘Π’Π’Π Π£ΠœΠΠžΠ–Π•ΠΠ˜Π―)

2/3 x 1 = 2/3

242/2313 x 1 = ?????

Когда ΠΎΠ΄ΠΈΠ½ ΠΈΠ· Ρ„Π°ΠΊΡ‚ΠΎΡ€ΠΎΠ² Ρ€Π°Π²Π΅Π½ 0, ΠΏΡ€ΠΎΠ΄ΡƒΠΊΡ‚ всСгда Π±ΡƒΠ΄Π΅Ρ‚ НУЛЬ (ΠΠ£Π›Π•Π’ΠžΠ™ Π‘Π’ΠžΠ™Π‘Π’Π’Π Π£ΠœΠΠžΠ–Π•ΠΠ˜Π―)

2/3 x 0 = 0

1/5 x 0 = 0


Π’Π²Π΅Π΄Π΅Π½ΠΈΠ΅ Π² арифмСтичСскиС ΠΎΠΏΠ΅Ρ€Π°Ρ†ΠΈΠΈ | БСзграничная Π°Π»Π³Π΅Π±Ρ€Π°

ΠžΡΠ½ΠΎΠ²Π½Ρ‹Π΅ ΠΎΠΏΠ΅Ρ€Π°Ρ†ΠΈΠΈ

ΠžΡΠ½ΠΎΠ²Π½Ρ‹ΠΌΠΈ арифмСтичСскими опСрациями с Π΄Π΅ΠΉΡΡ‚Π²ΠΈΡ‚Π΅Π»ΡŒΠ½Ρ‹ΠΌΠΈ числами ΡΠ²Π»ΡΡŽΡ‚ΡΡ слоТСниС, Π²Ρ‹Ρ‡ΠΈΡ‚Π°Π½ΠΈΠ΅, ΡƒΠΌΠ½ΠΎΠΆΠ΅Π½ΠΈΠ΅ ΠΈ Π΄Π΅Π»Π΅Π½ΠΈΠ΅.

Π¦Π΅Π»ΠΈ обучСния

ВычислСниС суммы, Ρ€Π°Π·Π½ΠΈΡ†Ρ‹, произвСдСния ΠΈ частного ΠΏΠΎΠ»ΠΎΠΆΠΈΡ‚Π΅Π»ΡŒΠ½Ρ‹Ρ… Ρ†Π΅Π»Ρ‹Ρ… чисСл

ΠšΠ»ΡŽΡ‡Π΅Π²Ρ‹Π΅ Π²Ρ‹Π²ΠΎΠ΄Ρ‹

ΠšΠ»ΡŽΡ‡Π΅Π²Ρ‹Π΅ ΠΌΠΎΠΌΠ΅Π½Ρ‚Ρ‹
  • ΠžΡΠ½ΠΎΠ²Π½Ρ‹ΠΌΠΈ арифмСтичСскими опСрациями с Π΄Π΅ΠΉΡΡ‚Π²ΠΈΡ‚Π΅Π»ΡŒΠ½Ρ‹ΠΌΠΈ числами ΡΠ²Π»ΡΡŽΡ‚ΡΡ слоТСниС, Π²Ρ‹Ρ‡ΠΈΡ‚Π°Π½ΠΈΠ΅, ΡƒΠΌΠ½ΠΎΠΆΠ΅Π½ΠΈΠ΅ ΠΈ Π΄Π΅Π»Π΅Π½ΠΈΠ΅.
  • ΠžΡΠ½ΠΎΠ²Π½Ρ‹ΠΌΠΈ арифмСтичСскими свойствами ΡΠ²Π»ΡΡŽΡ‚ΡΡ ΠΊΠΎΠΌΠΌΡƒΡ‚Π°Ρ‚ΠΈΠ²Π½Ρ‹Π΅, ассоциативныС ΠΈ дистрибутивныС свойства.
ΠšΠ»ΡŽΡ‡Π΅Π²Ρ‹Π΅ Ρ‚Π΅Ρ€ΠΌΠΈΠ½Ρ‹
  • ассоциативный : ссылка Π½Π° ΠΌΠ°Ρ‚Π΅ΠΌΠ°Ρ‚ΠΈΡ‡Π΅ΡΠΊΡƒΡŽ ΠΎΠΏΠ΅Ρ€Π°Ρ†ΠΈΡŽ, которая Π΄Π°Π΅Ρ‚ ΠΎΠ΄ΠΈΠ½ ΠΈ Ρ‚ΠΎΡ‚ ΠΆΠ΅ Ρ€Π΅Π·ΡƒΠ»ΡŒΡ‚Π°Ρ‚ нСзависимо ΠΎΡ‚ Π³Ρ€ΡƒΠΏΠΏΠΈΡ€ΠΎΠ²ΠΊΠΈ элСмСнтов.
  • ΠΊΠΎΠΌΠΌΡƒΡ‚Π°Ρ‚ΠΈΠ²Π½Ρ‹ΠΉ : ссылка Π½Π° Π΄Π²ΠΎΠΈΡ‡Π½ΡƒΡŽ ΠΎΠΏΠ΅Ρ€Π°Ρ†ΠΈΡŽ, Π² ΠΊΠΎΡ‚ΠΎΡ€ΠΎΠΉ ΠΈΠ·ΠΌΠ΅Π½Π΅Π½ΠΈΠ΅ порядка ΠΎΠΏΠ΅Ρ€Π°Π½Π΄ΠΎΠ² Π½Π΅ мСняСт Ρ€Π΅Π·ΡƒΠ»ΡŒΡ‚Π°Ρ‚ (Π½Π°ΠΏΡ€ΠΈΠΌΠ΅Ρ€, слоТСниС ΠΈ ΡƒΠΌΠ½ΠΎΠΆΠ΅Π½ΠΈΠ΅).
  • ΠΏΡ€ΠΎΠΈΠ·Π²Π΅Π΄Π΅Π½ΠΈΠ΅ : Ρ€Π΅Π·ΡƒΠ»ΡŒΡ‚Π°Ρ‚ умноТСния Π΄Π²ΡƒΡ… Π²Π΅Π»ΠΈΡ‡ΠΈΠ½.
  • частноС : Ρ€Π΅Π·ΡƒΠ»ΡŒΡ‚Π°Ρ‚ дСлСния ΠΎΠ΄Π½ΠΎΠ³ΠΎ количСства Π½Π° Π΄Ρ€ΡƒΠ³ΠΎΠ΅.
  • сумма : Ρ€Π΅Π·ΡƒΠ»ΡŒΡ‚Π°Ρ‚ слоТСния Π΄Π²ΡƒΡ… Π²Π΅Π»ΠΈΡ‡ΠΈΠ½.
  • Ρ€Π°Π·Π½ΠΈΡ†Π° : Ρ€Π΅Π·ΡƒΠ»ΡŒΡ‚Π°Ρ‚ вычитания ΠΎΠ΄Π½ΠΎΠΉ Π²Π΅Π»ΠΈΡ‡ΠΈΠ½Ρ‹ ΠΈΠ· Π΄Ρ€ΡƒΠ³ΠΎΠΉ.
Π§Π΅Ρ‚Ρ‹Ρ€Π΅ арифмСтичСских ΠΎΠΏΠ΅Ρ€Π°Ρ†ΠΈΠΈ

Π”ΠΎΠΏΠΎΠ»Π½Π΅Π½ΠΈΠ΅

Π‘Π»ΠΎΠΆΠ΅Π½ΠΈΠ΅ — это самая основная опСрация Π² Π°Ρ€ΠΈΡ„ΠΌΠ΅Ρ‚ΠΈΠΊΠ΅. Π’ ΠΏΡ€ΠΎΡΡ‚Π΅ΠΉΡˆΠ΅ΠΉ Ρ„ΠΎΡ€ΠΌΠ΅ слоТСниС ΠΎΠ±ΡŠΠ΅Π΄ΠΈΠ½ΡΠ΅Ρ‚ Π΄Π²Π΅ Π²Π΅Π»ΠΈΡ‡ΠΈΠ½Ρ‹ Π² ΠΎΠ΄Π½Ρƒ, ΠΈΠ»ΠΈ , сумма . НапримСр, ΠΏΡ€Π΅Π΄ΠΏΠΎΠ»ΠΎΠΆΠΈΠΌ, Ρ‡Ρ‚ΠΎ Ρƒ вас Π΅ΡΡ‚ΡŒ Π³Ρ€ΡƒΠΏΠΏΠ° ΠΈΠ· 2 ящиков ΠΈ Π΅Ρ‰Π΅ ΠΎΠ΄Π½Π° Π³Ρ€ΡƒΠΏΠΏΠ° ΠΈΠ· 3 ящиков. Если Π²Ρ‹ ΠΎΠ±ΡŠΠ΅Π΄ΠΈΠ½ΠΈΡ‚Π΅ ΠΎΠ±Π΅ Π³Ρ€ΡƒΠΏΠΏΡ‹ вмСстС, Ρƒ вас получится ΠΎΠ΄Π½Π° Π³Ρ€ΡƒΠΏΠΏΠ° ΠΈΠ· 5 ящиков. Π§Ρ‚ΠΎΠ±Ρ‹ ΠΏΡ€Π΅Π΄ΡΡ‚Π°Π²ΠΈΡ‚ΡŒ эту идСю Π² матСматичСских Ρ‚Π΅Ρ€ΠΌΠΈΠ½Π°Ρ…:

[латСкс] 2 + 3 = 5 [/ латСкс]

Π’Ρ‹Ρ‡ΠΈΡ‚Π°Π½ΠΈΠ΅

Π’Ρ‹Ρ‡ΠΈΡ‚Π°Π½ΠΈΠ΅ ΠΏΡ€ΠΎΡ‚ΠΈΠ²ΠΎΠΏΠΎΠ»ΠΎΠΆΠ½ΠΎ слоТСнию.ВмСсто Ρ‚ΠΎΠ³ΠΎ, Ρ‡Ρ‚ΠΎΠ±Ρ‹ ΡΠΊΠ»Π°Π΄Ρ‹Π²Π°Ρ‚ΡŒ количСства вмСстС, ΠΌΡ‹ удаляСм ΠΎΠ΄Π½ΠΎ количСство ΠΈΠ· Π΄Ρ€ΡƒΠ³ΠΎΠ³ΠΎ, Ρ‡Ρ‚ΠΎΠ±Ρ‹ Π½Π°ΠΉΡ‚ΠΈ Ρ€Π°Π·Π½ΠΈΡ†Ρƒ Π² ΠΌΠ΅ΠΆΠ΄Ρƒ Π½ΠΈΠΌΠΈ. ΠŸΡ€ΠΎΠ΄ΠΎΠ»ΠΆΠ°Ρ ΠΏΡ€Π΅Π΄Ρ‹Π΄ΡƒΡ‰ΠΈΠΉ ΠΏΡ€ΠΈΠΌΠ΅Ρ€, ΠΏΡ€Π΅Π΄ΠΏΠΎΠ»ΠΎΠΆΠΈΠΌ, Ρ‡Ρ‚ΠΎ Π²Ρ‹ Π½Π°Ρ‡ΠΈΠ½Π°Π΅Ρ‚Π΅ с Π³Ρ€ΡƒΠΏΠΏΡ‹ ΠΈΠ· 5 Π±Π»ΠΎΠΊΠΎΠ². Если Π²Ρ‹ Π·Π°Ρ‚Π΅ΠΌ ΡƒΠ΄Π°Π»ΠΈΡ‚Π΅ 3 поля ΠΈΠ· этой Π³Ρ€ΡƒΠΏΠΏΡ‹, Ρƒ вас останутся 2 поля. ΠœΠ°Ρ‚Π΅ΠΌΠ°Ρ‚ΠΈΡ‡Π΅ΡΠΊΠΈ:

[латСкс] 5-3 = 2 [/ латСкс]

Π£ΠΌΠ½ΠΎΠΆΠ΅Π½ΠΈΠ΅

Π£ΠΌΠ½ΠΎΠΆΠ΅Π½ΠΈΠ΅ Ρ‚Π°ΠΊΠΆΠ΅ ΠΎΠ±ΡŠΠ΅Π΄ΠΈΠ½ΡΠ΅Ρ‚ нСсколько Π²Π΅Π»ΠΈΡ‡ΠΈΠ½ Π² ΠΎΠ΄Π½Ρƒ Π²Π΅Π»ΠΈΡ‡ΠΈΠ½Ρƒ, которая называСтся ΠΏΡ€ΠΎΠ΄ΡƒΠΊΡ‚ΠΎΠΌ . ЀактичСски, ΡƒΠΌΠ½ΠΎΠΆΠ΅Π½ΠΈΠ΅ ΠΌΠΎΠΆΠ½ΠΎ Ρ€Π°ΡΡΠΌΠ°Ρ‚Ρ€ΠΈΠ²Π°Ρ‚ΡŒ ΠΊΠ°ΠΊ объСдинСниС мноТСства слоТСний.Π’ частности, ΠΏΡ€ΠΎΠΈΠ·Π²Π΅Π΄Π΅Π½ΠΈΠ΅ [latex] x [/ latex] ΠΈ [latex] y [/ latex] являСтся Ρ€Π΅Π·ΡƒΠ»ΡŒΡ‚Π°Ρ‚ΠΎΠΌ слоТСния [latex] x [/ latex] вмСстС [latex] y [/ latex] Ρ€Π°Π·. НапримСр, ΠΎΠ΄ΠΈΠ½ ΠΈΠ· способов подсчСта Ρ‡Π΅Ρ‚Ρ‹Ρ€Π΅Ρ… Π³Ρ€ΡƒΠΏΠΏ ΠΏΠΎ Π΄Π²Π΅ ΠΊΠΎΡ€ΠΎΠ±ΠΊΠΈ — ΡΠ»ΠΎΠΆΠΈΡ‚ΡŒ Π³Ρ€ΡƒΠΏΠΏΡ‹ вмСстС:

[латСкс] 2 + 2 + 2 + 2 = 8 [/ латСкс]

Однако Π΅ΡΡ‚ΡŒ Π΅Ρ‰Π΅ ΠΎΠ΄ΠΈΠ½ способ ΠΏΠΎΡΡ‡ΠΈΡ‚Π°Ρ‚ΡŒ ΠΊΠΎΡ€ΠΎΠ±ΠΊΠΈ — это ΡƒΠΌΠ½ΠΎΠΆΠΈΡ‚ΡŒ количСство:

[латСкс] 2 \ cdot 4 = 8 [/ латСкс]

ΠžΠ±Ρ€Π°Ρ‚ΠΈΡ‚Π΅ Π²Π½ΠΈΠΌΠ°Π½ΠΈΠ΅, Ρ‡Ρ‚ΠΎ ΠΎΠ±Π° ΠΌΠ΅Ρ‚ΠΎΠ΄Π° Π΄Π°ΡŽΡ‚ ΠΎΠ΄ΠΈΠ½ ΠΈ Ρ‚ΠΎΡ‚ ΠΆΠ΅ Ρ€Π΅Π·ΡƒΠ»ΡŒΡ‚Π°Ρ‚ — 8, Π½ΠΎ Π²ΠΎ ΠΌΠ½ΠΎΠ³ΠΈΡ… случаях, особСнно ΠΊΠΎΠ³Π΄Π° Ρƒ вас Π΅ΡΡ‚ΡŒ большиС количСства ΠΈΠ»ΠΈ ΠΌΠ½ΠΎΠ³ΠΎ Π³Ρ€ΡƒΠΏΠΏ, ΡƒΠΌΠ½ΠΎΠΆΠ΅Π½ΠΈΠ΅ ΠΌΠΎΠΆΠ΅Ρ‚ Π±Ρ‹Ρ‚ΡŒ Π½Π°ΠΌΠ½ΠΎΠ³ΠΎ быстрСС.

Дивизия

Π”Π΅Π»Π΅Π½ΠΈΠ΅ — это Π²Π΅Π»ΠΈΡ‡ΠΈΠ½Π°, обратная ΡƒΠΌΠ½ΠΎΠΆΠ΅Π½ΠΈΡŽ. ВмСсто Ρ‚ΠΎΠ³ΠΎ, Ρ‡Ρ‚ΠΎΠ±Ρ‹ ΡƒΠΌΠ½ΠΎΠΆΠ°Ρ‚ΡŒ количСства вмСстС, Ρ‡Ρ‚ΠΎΠ±Ρ‹ ΠΏΠΎΠ»ΡƒΡ‡ΠΈΡ‚ΡŒ большСС Π·Π½Π°Ρ‡Π΅Π½ΠΈΠ΅, Π²Ρ‹ раздСляСтС количСство Π½Π° мСньшСС Π·Π½Π°Ρ‡Π΅Π½ΠΈΠ΅, Π½Π°Π·Ρ‹Π²Π°Π΅ΠΌΠΎΠ΅ частным . ΠžΠΏΡΡ‚ΡŒ ΠΆΠ΅, Π²ΠΎΠ·Π²Ρ€Π°Ρ‰Π°ΡΡΡŒ ΠΊ ΠΏΡ€ΠΈΠΌΠ΅Ρ€Ρƒ с Π±Π»ΠΎΠΊΠΎΠΌ, Ρ€Π°Π·Π΄Π΅Π»Π΅Π½ΠΈΠ΅ Π³Ρ€ΡƒΠΏΠΏΡ‹ ΠΈΠ· 8 Π±Π»ΠΎΠΊΠΎΠ² Π½Π° 4 Ρ€Π°Π²Π½Ρ‹Π΅ Π³Ρ€ΡƒΠΏΠΏΡ‹ ΠΏΡ€ΠΈΠ²ΠΎΠ΄ΠΈΡ‚ ΠΊ ΠΏΠΎΠ»ΡƒΡ‡Π΅Π½ΠΈΡŽ 4 Π³Ρ€ΡƒΠΏΠΏ ΠΏΠΎ 2 Π±Π»ΠΎΠΊΠ°:

[латСкс] 8 \ div 4 = 2 [/ латСкс]

ΠžΡΠ½ΠΎΠ²Π½Ρ‹Π΅ арифмСтичСскиС свойства

ΠšΠΎΠΌΠΌΡƒΡ‚Π°Ρ†ΠΈΠΎΠ½Π½Π°Ρ ΡΠΎΠ±ΡΡ‚Π²Π΅Π½Π½ΠΎΡΡ‚ΡŒ

Бвойство коммутативности описываСт уравнСния, Π² ΠΊΠΎΡ‚ΠΎΡ€Ρ‹Ρ… порядок чисСл Π½Π΅ влияСт Π½Π° Ρ€Π΅Π·ΡƒΠ»ΡŒΡ‚Π°Ρ‚. Π‘Π»ΠΎΠΆΠ΅Π½ΠΈΠ΅ ΠΈ ΡƒΠΌΠ½ΠΎΠΆΠ΅Π½ΠΈΠ΅ — это ΠΊΠΎΠΌΠΌΡƒΡ‚Π°Ρ‚ΠΈΠ²Π½Ρ‹Π΅ ΠΎΠΏΠ΅Ρ€Π°Ρ†ΠΈΠΈ:

  • [латСкс] 2 + 3 = 3 + 2 = 5 [/ латСкс]
  • [латСкс] 5 \ cdot 2 = 2 \ cdot 5 = 10 [/ латСкс]

Однако Π²Ρ‹Ρ‡ΠΈΡ‚Π°Π½ΠΈΠ΅ ΠΈ Π΄Π΅Π»Π΅Π½ΠΈΠ΅ Π½Π΅ ΠΊΠΎΠΌΠΌΡƒΡ‚Π°Ρ‚ΠΈΠ²Π½Ρ‹.

Ассоциативная ΡΠΎΠ±ΡΡ‚Π²Π΅Π½Π½ΠΎΡΡ‚ΡŒ

АссоциативноС свойство описываСт уравнСния, Π² ΠΊΠΎΡ‚ΠΎΡ€Ρ‹Ρ… Π³Ρ€ΡƒΠΏΠΏΠΈΡ€ΠΎΠ²ΠΊΠ° чисСл Π½Π΅ влияСт Π½Π° Ρ€Π΅Π·ΡƒΠ»ΡŒΡ‚Π°Ρ‚. Как ΠΈ Π² случаС с ΠΊΠΎΠΌΠΌΡƒΡ‚Π°Ρ‚ΠΈΠ²Π½ΠΎΡΡ‚ΡŒΡŽ, слоТСниС ΠΈ ΡƒΠΌΠ½ΠΎΠΆΠ΅Π½ΠΈΠ΅ ΡΠ²Π»ΡΡŽΡ‚ΡΡ ассоциативными опСрациями:

  • [латСкс] (2 + 3) + 6 = 2 + (3 + 6) = 11 [/ латСкс]
  • [латСкс] (4 \ cdot 1) \ cdot 2 = 4 \ cdot (1 \ cdot 2) = 8 [/ латСкс]

Π•Ρ‰Π΅ Ρ€Π°Π·, Π²Ρ‹Ρ‡ΠΈΡ‚Π°Π½ΠΈΠ΅ ΠΈ Π΄Π΅Π»Π΅Π½ΠΈΠ΅ Π½Π΅ ассоциативны.

Π Π°ΡΠΏΡ€Π΅Π΄Π΅Π»ΠΈΡ‚Π΅Π»ΡŒΠ½Π°Ρ ΡΠΎΠ±ΡΡ‚Π²Π΅Π½Π½ΠΎΡΡ‚ΡŒ

Бвойство распрСдСлСния ΠΌΠΎΠΆΠ½ΠΎ ΠΈΡΠΏΠΎΠ»ΡŒΠ·ΠΎΠ²Π°Ρ‚ΡŒ, ΠΊΠΎΠ³Π΄Π° сумма Π΄Π²ΡƒΡ… Π²Π΅Π»ΠΈΡ‡ΠΈΠ½ Π·Π°Ρ‚Π΅ΠΌ умноТаСтся Π½Π° Ρ‚Ρ€Π΅Ρ‚ΡŒΠ΅ количСство.

  • [латСкс] (2 + 4) \ cdot 3 = 2 \ cdot 3 + 4 \ cdot 3 = 18 [/ латСкс]

ΠžΡ‚Ρ€ΠΈΡ†Π°Ρ‚Π΅Π»ΡŒΠ½Ρ‹Π΅ числа

АрифмСтичСскиС ΠΎΠΏΠ΅Ρ€Π°Ρ†ΠΈΠΈ ΠΌΠΎΠ³ΡƒΡ‚ Π²Ρ‹ΠΏΠΎΠ»Π½ΡΡ‚ΡŒΡΡ с ΠΎΡ‚Ρ€ΠΈΡ†Π°Ρ‚Π΅Π»ΡŒΠ½Ρ‹ΠΌΠΈ числами Π² соотвСтствии с ΠΎΠΏΡ€Π΅Π΄Π΅Π»Π΅Π½Π½Ρ‹ΠΌΠΈ ΠΏΡ€Π°Π²ΠΈΠ»Π°ΠΌΠΈ.

Π¦Π΅Π»ΠΈ обучСния

ВычислСниС суммы, Ρ€Π°Π·Π½ΠΈΡ†Ρ‹, произвСдСния ΠΈ частного ΠΎΡ‚Ρ€ΠΈΡ†Π°Ρ‚Π΅Π»ΡŒΠ½Ρ‹Ρ… Ρ†Π΅Π»Ρ‹Ρ… чисСл

ΠšΠ»ΡŽΡ‡Π΅Π²Ρ‹Π΅ Π²Ρ‹Π²ΠΎΠ΄Ρ‹

ΠšΠ»ΡŽΡ‡Π΅Π²Ρ‹Π΅ ΠΌΠΎΠΌΠ΅Π½Ρ‚Ρ‹
  • Π‘Π»ΠΎΠΆΠ΅Π½ΠΈΠ΅ Π΄Π²ΡƒΡ… ΠΎΡ‚Ρ€ΠΈΡ†Π°Ρ‚Π΅Π»ΡŒΠ½Ρ‹Ρ… чисСл Π΄Π°Π΅Ρ‚ ΠΎΡ‚Ρ€ΠΈΡ†Π°Ρ‚Π΅Π»ΡŒΠ½ΠΎΠ΅ Π·Π½Π°Ρ‡Π΅Π½ΠΈΠ΅; слоТСниС ΠΏΠΎΠ»ΠΎΠΆΠΈΡ‚Π΅Π»ΡŒΠ½ΠΎΠ³ΠΎ ΠΈ ΠΎΡ‚Ρ€ΠΈΡ†Π°Ρ‚Π΅Π»ΡŒΠ½ΠΎΠ³ΠΎ числа Π΄Π°Π΅Ρ‚ число, ΠΈΠΌΠ΅ΡŽΡ‰Π΅Π΅ Ρ‚ΠΎΡ‚ ΠΆΠ΅ Π·Π½Π°ΠΊ, Ρ‡Ρ‚ΠΎ ΠΈ число большСй Π²Π΅Π»ΠΈΡ‡ΠΈΠ½Ρ‹.
  • Π’Ρ‹Ρ‡ΠΈΡ‚Π°Π½ΠΈΠ΅ ΠΏΠΎΠ»ΠΎΠΆΠΈΡ‚Π΅Π»ΡŒΠ½ΠΎΠ³ΠΎ числа Π΄Π°Π΅Ρ‚ Ρ‚ΠΎΡ‚ ΠΆΠ΅ Ρ€Π΅Π·ΡƒΠ»ΡŒΡ‚Π°Ρ‚, Ρ‡Ρ‚ΠΎ ΠΈ Π΄ΠΎΠ±Π°Π²Π»Π΅Π½ΠΈΠ΅ ΠΎΡ‚Ρ€ΠΈΡ†Π°Ρ‚Π΅Π»ΡŒΠ½ΠΎΠ³ΠΎ числа Ρ€Π°Π²Π½ΠΎΠΉ Π²Π΅Π»ΠΈΡ‡ΠΈΠ½Ρ‹, Π° Π²Ρ‹Ρ‡ΠΈΡ‚Π°Π½ΠΈΠ΅ ΠΎΡ‚Ρ€ΠΈΡ†Π°Ρ‚Π΅Π»ΡŒΠ½ΠΎΠ³ΠΎ числа Π΄Π°Π΅Ρ‚ Ρ‚ΠΎΡ‚ ΠΆΠ΅ Ρ€Π΅Π·ΡƒΠ»ΡŒΡ‚Π°Ρ‚, Ρ‡Ρ‚ΠΎ ΠΈ Π΄ΠΎΠ±Π°Π²Π»Π΅Π½ΠΈΠ΅ ΠΏΠΎΠ»ΠΎΠΆΠΈΡ‚Π΅Π»ΡŒΠ½ΠΎΠ³ΠΎ числа.
  • ΠŸΡ€ΠΎΠΈΠ·Π²Π΅Π΄Π΅Π½ΠΈΠ΅ ΠΎΠ΄Π½ΠΎΠ³ΠΎ ΠΏΠΎΠ»ΠΎΠΆΠΈΡ‚Π΅Π»ΡŒΠ½ΠΎΠ³ΠΎ числа ΠΈ ΠΎΠ΄Π½ΠΎΠ³ΠΎ ΠΎΡ‚Ρ€ΠΈΡ†Π°Ρ‚Π΅Π»ΡŒΠ½ΠΎΠ³ΠΎ числа ΠΎΡ‚Ρ€ΠΈΡ†Π°Ρ‚Π΅Π»ΡŒΠ½ΠΎ, Π° ΠΏΡ€ΠΎΠΈΠ·Π²Π΅Π΄Π΅Π½ΠΈΠ΅ Π΄Π²ΡƒΡ… ΠΎΡ‚Ρ€ΠΈΡ†Π°Ρ‚Π΅Π»ΡŒΠ½Ρ‹Ρ… чисСл ΠΏΠΎΠ»ΠΎΠΆΠΈΡ‚Π΅Π»ΡŒΠ½ΠΎ.
  • ЧастноС ΠΎΠ΄Π½ΠΎΠ³ΠΎ ΠΏΠΎΠ»ΠΎΠΆΠΈΡ‚Π΅Π»ΡŒΠ½ΠΎΠ³ΠΎ числа ΠΈ ΠΎΠ΄Π½ΠΎΠ³ΠΎ ΠΎΡ‚Ρ€ΠΈΡ†Π°Ρ‚Π΅Π»ΡŒΠ½ΠΎΠ³ΠΎ числа ΠΎΡ‚Ρ€ΠΈΡ†Π°Ρ‚Π΅Π»ΡŒΠ½ΠΎ, Π° частноС Π΄Π²ΡƒΡ… ΠΎΡ‚Ρ€ΠΈΡ†Π°Ρ‚Π΅Π»ΡŒΠ½Ρ‹Ρ… чисСл ΠΏΠΎΠ»ΠΎΠΆΠΈΡ‚Π΅Π»ΡŒΠ½ΠΎ.
Π§Π΅Ρ‚Ρ‹Ρ€Π΅ ΠΎΠΏΠ΅Ρ€Π°Ρ†ΠΈΠΈ

Π”ΠΎΠΏΠΎΠ»Π½Π΅Π½ΠΈΠ΅

Π‘Π»ΠΎΠΆΠ΅Π½ΠΈΠ΅ Π΄Π²ΡƒΡ… ΠΎΡ‚Ρ€ΠΈΡ†Π°Ρ‚Π΅Π»ΡŒΠ½Ρ‹Ρ… чисСл ΠΎΡ‡Π΅Π½ΡŒ ΠΏΠΎΡ…ΠΎΠΆΠ΅ Π½Π° слоТСниС Π΄Π²ΡƒΡ… ΠΏΠΎΠ»ΠΎΠΆΠΈΡ‚Π΅Π»ΡŒΠ½Ρ‹Ρ… чисСл. НапримСр:

[латСкс] (- 3) + (βˆ’5) = βˆ’8 [/ латСкс]

ΠžΡΠ½ΠΎΠ²ΠΎΠΏΠΎΠ»Π°Π³Π°ΡŽΡ‰ΠΈΠΉ ΠΏΡ€ΠΈΠ½Ρ†ΠΈΠΏ Π·Π°ΠΊΠ»ΡŽΡ‡Π°Π΅Ρ‚ΡΡ Π² Ρ‚ΠΎΠΌ, Ρ‡Ρ‚ΠΎ Π΄Π²Π° Π΄ΠΎΠ»Π³Π° — ΠΎΡ‚Ρ€ΠΈΡ†Π°Ρ‚Π΅Π»ΡŒΠ½Ρ‹Π΅ числа — ΠΌΠΎΠ³ΡƒΡ‚ Π±Ρ‹Ρ‚ΡŒ ΠΎΠ±ΡŠΠ΅Π΄ΠΈΠ½Π΅Π½Ρ‹ Π² ΠΎΠ΄ΠΈΠ½ Π΄ΠΎΠ»Π³ большСй Π²Π΅Π»ΠΈΡ‡ΠΈΠ½Ρ‹.

ΠŸΡ€ΠΈ слоТСнии ΠΏΠΎΠ»ΠΎΠΆΠΈΡ‚Π΅Π»ΡŒΠ½Ρ‹Ρ… ΠΈ ΠΎΡ‚Ρ€ΠΈΡ†Π°Ρ‚Π΅Π»ΡŒΠ½Ρ‹Ρ… чисСл Π΄Ρ€ΡƒΠ³ΠΎΠΉ способ Π·Π°ΠΏΠΈΡΠ°Ρ‚ΡŒ ΠΎΡ‚Ρ€ΠΈΡ†Π°Ρ‚Π΅Π»ΡŒΠ½Ρ‹Π΅ числа — Π²Ρ‹Ρ‡Π΅ΡΡ‚ΡŒ ΠΏΠΎΠ»ΠΎΠΆΠΈΡ‚Π΅Π»ΡŒΠ½Ρ‹Π΅ Π²Π΅Π»ΠΈΡ‡ΠΈΠ½Ρ‹. НапримСр:

[латСкс] 8 + (βˆ’3) = 8 — 3 = 5 [/ латСкс]

Π—Π΄Π΅ΡΡŒ ΠΊΡ€Π΅Π΄ΠΈΡ‚ 8 сочСтаСтся с Π·Π°Π΄ΠΎΠ»ΠΆΠ΅Π½Π½ΠΎΡΡ‚ΡŒΡŽ 3, Ρ‡Ρ‚ΠΎ Π΄Π°Π΅Ρ‚ ΠΎΠ±Ρ‰ΠΈΠΉ ΠΊΡ€Π΅Π΄ΠΈΡ‚ 5.Однако, Ссли ΠΎΡ‚Ρ€ΠΈΡ†Π°Ρ‚Π΅Π»ΡŒΠ½ΠΎΠ΅ число ΠΈΠΌΠ΅Π΅Ρ‚ Π±ΠΎΠ»ΡŒΡˆΡƒΡŽ Π²Π΅Π»ΠΈΡ‡ΠΈΠ½Ρƒ, Ρ€Π΅Π·ΡƒΠ»ΡŒΡ‚Π°Ρ‚ Π±ΡƒΠ΄Π΅Ρ‚ ΠΎΡ‚Ρ€ΠΈΡ†Π°Ρ‚Π΅Π»ΡŒΠ½Ρ‹ΠΌ:

[латСкс] (- 8) + 3 = 3 — 8 = βˆ’5 [/ латСкс]

Аналогично:

[латСкс] (- 2) + 7 = 7 — 2 = 5 [/ латСкс]

Π—Π΄Π΅ΡΡŒ Π΄ΠΎΠ»Π³ 2 сочСтаСтся с ΠΊΡ€Π΅Π΄ΠΈΡ‚ΠΎΠΌ 7. ΠšΡ€Π΅Π΄ΠΈΡ‚ ΠΈΠΌΠ΅Π΅Ρ‚ Π±ΠΎΠ»ΡŒΡˆΡƒΡŽ Π²Π΅Π»ΠΈΡ‡ΠΈΠ½Ρƒ, Ρ‡Π΅ΠΌ Π΄ΠΎΠ»Π³, поэтому Ρ€Π΅Π·ΡƒΠ»ΡŒΡ‚Π°Ρ‚ ΠΏΠΎΠ»ΠΎΠΆΠΈΡ‚Π΅Π»ΡŒΠ½Ρ‹ΠΉ. Но Ссли ΠΊΡ€Π΅Π΄ΠΈΡ‚ мСньшС Π΄ΠΎΠ»Π³Π°, Ρ€Π΅Π·ΡƒΠ»ΡŒΡ‚Π°Ρ‚ Π±ΡƒΠ΄Π΅Ρ‚ ΠΎΡ‚Ρ€ΠΈΡ†Π°Ρ‚Π΅Π»ΡŒΠ½Ρ‹ΠΌ:

[латСкс] 2 + (βˆ’7) = 2-7 ​​= βˆ’5 [/ латСкс]

Π’Ρ‹Ρ‡ΠΈΡ‚Π°Π½ΠΈΠ΅

Π’Ρ‹Ρ‡ΠΈΡ‚Π°Π½ΠΈΠ΅ ΠΏΠΎΠ»ΠΎΠΆΠΈΡ‚Π΅Π»ΡŒΠ½Ρ‹Ρ… чисСл Π΄Ρ€ΡƒΠ³ ΠΈΠ· Π΄Ρ€ΡƒΠ³Π° ΠΌΠΎΠΆΠ΅Ρ‚ Π΄Π°Ρ‚ΡŒ ΠΎΡ‚Ρ€ΠΈΡ†Π°Ρ‚Π΅Π»ΡŒΠ½Ρ‹ΠΉ ΠΎΡ‚Π²Π΅Ρ‚. НапримСр, вычитая 8 ΠΈΠ· 5:

[латСкс] 5-8 = βˆ’3 [/ латСкс]

Π’Ρ‹Ρ‡ΠΈΡ‚Π°Π½ΠΈΠ΅ ΠΏΠΎΠ»ΠΎΠΆΠΈΡ‚Π΅Π»ΡŒΠ½ΠΎΠ³ΠΎ числа ΠΎΠ±Ρ‹Ρ‡Π½ΠΎ Π°Π½Π°Π»ΠΎΠ³ΠΈΡ‡Π½ΠΎ слоТСнию ΠΎΡ‚Ρ€ΠΈΡ†Π°Ρ‚Π΅Π»ΡŒΠ½ΠΎΠ³ΠΎ числа. Π’ΠΎ Π΅ΡΡ‚ΡŒ:

[латСкс] 5 — 8 = 5 + (βˆ’8) = βˆ’3 [/ латСкс]

ΠΈ

[латСкс] (- 3) — 5 = (βˆ’3) + (βˆ’5) = βˆ’8 [/ латСкс]

Аналогично, ΠΏΡ€ΠΈ Π²Ρ‹Ρ‡ΠΈΡ‚Π°Π½ΠΈΠΈ ΠΎΡ‚Ρ€ΠΈΡ†Π°Ρ‚Π΅Π»ΡŒΠ½ΠΎΠ³ΠΎ числа Π΄Π°Π΅Ρ‚ Ρ‚ΠΎΡ‚ ΠΆΠ΅ Ρ€Π΅Π·ΡƒΠ»ΡŒΡ‚Π°Ρ‚, Ρ‡Ρ‚ΠΎ ΠΈ ΠΏΡ€ΠΈ Π΄ΠΎΠ±Π°Π²Π»Π΅Π½ΠΈΠΈ ΠΏΠΎΠ»ΠΎΠΆΠΈΡ‚Π΅Π»ΡŒΠ½ΠΎΠ³ΠΎ числа ΠΈΠ· этого числа. ИдСя здСсь Π² Ρ‚ΠΎΠΌ, Ρ‡Ρ‚ΠΎ , потСряв Π΄ΠΎΠ»Π³, — это Ρ‚ΠΎ ΠΆΠ΅ самоС, Ρ‡Ρ‚ΠΎ ΠΏΠΎΠ»ΡƒΡ‡ΠΈΡ‚ ΠΊΡ€Π΅Π΄ΠΈΡ‚.Π‘Π»Π΅Π΄ΠΎΠ²Π°Ρ‚Π΅Π»ΡŒΠ½ΠΎ:

[латСкс] 3 — (βˆ’5) = 3 + 5 = 8 [/ латСкс]

ΠΈ

[латСкс] (- 5) — (βˆ’8) = (βˆ’5) + 8 = 3 [/ латСкс]

Π£ΠΌΠ½ΠΎΠΆΠ΅Π½ΠΈΠ΅

ΠŸΡ€ΠΈ ΡƒΠΌΠ½ΠΎΠΆΠ΅Π½ΠΈΠΈ ΠΏΠΎΠ»ΠΎΠΆΠΈΡ‚Π΅Π»ΡŒΠ½Ρ‹Ρ… ΠΈ ΠΎΡ‚Ρ€ΠΈΡ†Π°Ρ‚Π΅Π»ΡŒΠ½Ρ‹Ρ… чисСл Π·Π½Π°ΠΊ произвСдСния опрСдСляСтся ΠΏΠΎ ΡΠ»Π΅Π΄ΡƒΡŽΡ‰ΠΈΠΌ ΠΏΡ€Π°Π²ΠΈΠ»Π°ΠΌ:

  • ΠŸΡ€ΠΎΠΈΠ·Π²Π΅Π΄Π΅Π½ΠΈΠ΅ Π΄Π²ΡƒΡ… ΠΏΠΎΠ»ΠΎΠΆΠΈΡ‚Π΅Π»ΡŒΠ½Ρ‹Ρ… чисСл ΠΏΠΎΠ»ΠΎΠΆΠΈΡ‚Π΅Π»ΡŒΠ½ΠΎ. ΠŸΡ€ΠΎΠΈΠ·Π²Π΅Π΄Π΅Π½ΠΈΠ΅ ΠΎΠ΄Π½ΠΎΠ³ΠΎ ΠΏΠΎΠ»ΠΎΠΆΠΈΡ‚Π΅Π»ΡŒΠ½ΠΎΠ³ΠΎ числа ΠΈ ΠΎΠ΄Π½ΠΎΠ³ΠΎ ΠΎΡ‚Ρ€ΠΈΡ†Π°Ρ‚Π΅Π»ΡŒΠ½ΠΎΠ³ΠΎ числа ΠΎΡ‚Ρ€ΠΈΡ†Π°Ρ‚Π΅Π»ΡŒΠ½ΠΎ.
  • ΠŸΡ€ΠΎΠΈΠ·Π²Π΅Π΄Π΅Π½ΠΈΠ΅ Π΄Π²ΡƒΡ… ΠΎΡ‚Ρ€ΠΈΡ†Π°Ρ‚Π΅Π»ΡŒΠ½Ρ‹Ρ… чисСл ΠΏΠΎΠ»ΠΎΠΆΠΈΡ‚Π΅Π»ΡŒΠ½ΠΎ.

НапримСр:

[латСкс] (- 2) Γ— 3 = βˆ’6 [/ латСкс]

Π­Ρ‚ΠΎ просто ΠΏΠΎΡ‚ΠΎΠΌΡƒ, Ρ‡Ρ‚ΠΎ слоТСниС βˆ’2 Ρ‚Ρ€ΠΈ Ρ€Π°Π·Π° Π΄Π°Π΅Ρ‚ βˆ’6:

[латСкс] (- 2) Γ— 3 = (βˆ’2) + (βˆ’2) + (βˆ’2) = βˆ’6 [/ латСкс]

Однако

[латСкс] (- 2) Γ— (βˆ’3) = 6 [/ латСкс]

Π‘Π½ΠΎΠ²Π° идСя Π·Π°ΠΊΠ»ΡŽΡ‡Π°Π΅Ρ‚ΡΡ Π² Ρ‚ΠΎΠΌ, Ρ‡Ρ‚ΠΎ потСря Π΄ΠΎΠ»Π³Π° — это Ρ‚ΠΎ ΠΆΠ΅ самоС, Ρ‡Ρ‚ΠΎ ΠΈ ΠΏΠΎΠ»ΡƒΡ‡Π΅Π½ΠΈΠ΅ ΠΊΡ€Π΅Π΄ΠΈΡ‚Π°. Π’ этом случаС ΠΏΠΎΡ‚Π΅Ρ€ΡΡ‚ΡŒ Π΄Π²Π° Π΄ΠΎΠ»Π³Π° ΠΏΠΎ Ρ‚Ρ€ΠΈ Π² ΠΊΠ°ΠΆΠ΄ΠΎΠΌ — это Ρ‚ΠΎ ΠΆΠ΅ самоС, Ρ‡Ρ‚ΠΎ ΠΏΠΎΠ»ΡƒΡ‡ΠΈΡ‚ΡŒ ΠΊΡ€Π΅Π΄ΠΈΡ‚ Π² ΡˆΠ΅ΡΡ‚ΡŒ Ρ€Π°Π·:

[латСкс] \ left (βˆ’2 \ text {Π΄ΠΎΠ»Π³ΠΈ} \ right) \ times \ left (βˆ’3 \ text {each} \ right) = +6 \ text {ΠΊΡ€Π΅Π΄ΠΈΡ‚} [/ latex]

Дивизия

Π—Π½Π°ΠΊΠΎΠ²Ρ‹Π΅ ΠΏΡ€Π°Π²ΠΈΠ»Π° дСлСния Ρ‚Π°ΠΊΠΈΠ΅ ΠΆΠ΅, ΠΊΠ°ΠΊ ΠΈ для умноТСния.

  • Π Π°Π·Π΄Π΅Π»Π΅Π½ΠΈΠ΅ Π΄Π²ΡƒΡ… ΠΏΠΎΠ»ΠΎΠΆΠΈΡ‚Π΅Π»ΡŒΠ½Ρ‹Ρ… чисСл Π΄Π°Π΅Ρ‚ ΠΏΠΎΠ»ΠΎΠΆΠΈΡ‚Π΅Π»ΡŒΠ½ΠΎΠ΅ число.
  • Π Π°Π·Π΄Π΅Π»Π΅Π½ΠΈΠ΅ ΠΎΠ΄Π½ΠΎΠ³ΠΎ ΠΏΠΎΠ»ΠΎΠΆΠΈΡ‚Π΅Π»ΡŒΠ½ΠΎΠ³ΠΎ числа ΠΈ ΠΎΠ΄Π½ΠΎΠ³ΠΎ ΠΎΡ‚Ρ€ΠΈΡ†Π°Ρ‚Π΅Π»ΡŒΠ½ΠΎΠ³ΠΎ числа Π΄Π°Π΅Ρ‚ ΠΎΡ‚Ρ€ΠΈΡ†Π°Ρ‚Π΅Π»ΡŒΠ½ΠΎΠ΅ число.
  • Π Π°Π·Π΄Π΅Π»Π΅Π½ΠΈΠ΅ Π΄Π²ΡƒΡ… ΠΎΡ‚Ρ€ΠΈΡ†Π°Ρ‚Π΅Π»ΡŒΠ½Ρ‹Ρ… чисСл Π΄Π°Π΅Ρ‚ ΠΏΠΎΠ»ΠΎΠΆΠΈΡ‚Π΅Π»ΡŒΠ½ΠΎΠ΅ число.

Если Ρƒ Π΄Π΅Π»ΠΈΠΌΠΎΠ³ΠΎ ΠΈ дСлитСля ΠΎΠ΄ΠΈΠ½ ΠΈ Ρ‚ΠΎΡ‚ ΠΆΠ΅ Π·Π½Π°ΠΊ, Ρ‚ΠΎ Π΅ΡΡ‚ΡŒ Ρ€Π΅Π·ΡƒΠ»ΡŒΡ‚Π°Ρ‚ всСгда ΠΏΠΎΠ»ΠΎΠΆΠΈΡ‚Π΅Π»ΡŒΠ½Ρ‹ΠΉ. НапримСр:

[латСкс] 8 Γ· (βˆ’2) = βˆ’4 [/ латСкс]

ΠΈ

[латСкс] (- 8) Γ· 2 = βˆ’4 [/ латСкс]

Π½ΠΎ

[латСкс] (- 8) Γ· (βˆ’2) = 4 [/ латСкс].

Π”ΠΎΠΏΠΎΠ»Π½ΠΈΡ‚Π΅Π»ΡŒΠ½Ρ‹Π΅ сообраТСния

ΠžΡΠ½ΠΎΠ²Π½Ρ‹Π΅ свойства слоТСния (ΠΊΠΎΠΌΠΌΡƒΡ‚Π°Ρ‚ΠΈΠ²Π½ΠΎΡΡ‚ΡŒ, Π°ΡΡΠΎΡ†ΠΈΠ°Ρ‚ΠΈΠ²Π½ΠΎΡΡ‚ΡŒ ΠΈ Π΄ΠΈΡΡ‚Ρ€ΠΈΠ±ΡƒΡ‚ΠΈΠ²Π½ΠΎΡΡ‚ΡŒ) Ρ‚Π°ΠΊΠΆΠ΅ ΠΏΡ€ΠΈΠΌΠ΅Π½ΠΈΠΌΡ‹ ΠΊ ΠΎΡ‚Ρ€ΠΈΡ†Π°Ρ‚Π΅Π»ΡŒΠ½Ρ‹ΠΌ числам. НапримСр, ΡΠ»Π΅Π΄ΡƒΡŽΡ‰Π΅Π΅ ΡƒΡ€Π°Π²Π½Π΅Π½ΠΈΠ΅ дСмонстрируСт свойство распрСдСлСния:

[латСкс] -3 (2 + 5) = (-3) \ cdot 2 + (-3) \ cdot 5 [/ латСкс]

Π€Ρ€Π°ΠΊΡ†ΠΈΠΈ

Π”Ρ€ΠΎΠ±ΡŒ прСдставляСт собой Ρ‡Π°ΡΡ‚ΡŒ Ρ†Π΅Π»ΠΎΠ³ΠΎ ΠΈ состоит ΠΈΠ· Ρ†Π΅Π»ΠΎΠ³ΠΎ числитСля ΠΈ Π½Π΅Π½ΡƒΠ»Π΅Π²ΠΎΠ³ΠΎ Ρ†Π΅Π»ΠΎΠ³ΠΎ знамСнатСля.

Π¦Π΅Π»ΠΈ обучСния

Π’Ρ‹Ρ‡ΠΈΡΠ»ΠΈΡ‚ΡŒ Ρ€Π΅Π·ΡƒΠ»ΡŒΡ‚Π°Ρ‚ ΠΎΠΏΠ΅Ρ€Π°Ρ†ΠΈΠΉ с дробями

ΠšΠ»ΡŽΡ‡Π΅Π²Ρ‹Π΅ Π²Ρ‹Π²ΠΎΠ΄Ρ‹

ΠšΠ»ΡŽΡ‡Π΅Π²Ρ‹Π΅ ΠΌΠΎΠΌΠ΅Π½Ρ‚Ρ‹
  • Для слоТСния ΠΈ вычитания Π΄Ρ€ΠΎΠ±Π΅ΠΉ трСбуСтся Β«ΠΎΠ΄ΠΈΠ½Π°ΠΊΠΎΠ²Ρ‹Π΅ количСства» — ΠΎΠ±Ρ‰ΠΈΠΉ Π·Π½Π°ΠΌΠ΅Π½Π°Ρ‚Π΅Π»ΡŒ.Π§Ρ‚ΠΎΠ±Ρ‹ ΡΠ»ΠΎΠΆΠΈΡ‚ΡŒ ΠΈΠ»ΠΈ Π²Ρ‹Ρ‡Π΅ΡΡ‚ΡŒ Π΄Ρ€ΠΎΠ±ΠΈ, содСрТащиС Ρ€Π°Π·Π»ΠΈΡ‡Π°ΡŽΡ‰ΠΈΠ΅ΡΡ количСства (Π½Π°ΠΏΡ€ΠΈΠΌΠ΅Ρ€, ΠΏΡ€ΠΈΠ±Π°Π²Π»Π΅Π½ΠΈΠ΅ Ρ‡Π΅Ρ‚Π²Π΅Ρ€Ρ‚Π΅ΠΉ ΠΊ трСтям), Π½Π΅ΠΎΠ±Ρ…ΠΎΠ΄ΠΈΠΌΠΎ ΠΏΡ€Π΅ΠΎΠ±Ρ€Π°Π·ΠΎΠ²Π°Ρ‚ΡŒ всС суммы Π² ΠΎΠ΄ΠΈΠ½Π°ΠΊΠΎΠ²Ρ‹Π΅ количСства.
  • Π£ΠΌΠ½ΠΎΠΆΠ΅Π½ΠΈΠ΅ Π΄Ρ€ΠΎΠ±Π΅ΠΉ Ρ‚Ρ€Π΅Π±ΡƒΠ΅Ρ‚ умноТСния числитСлСй Π΄Ρ€ΡƒΠ³ Π½Π° Π΄Ρ€ΡƒΠ³Π°, Π° Π·Π°Ρ‚Π΅ΠΌ Π·Π½Π°ΠΌΠ΅Π½Π°Ρ‚Π΅Π»Π΅ΠΉ Π΄Ρ€ΡƒΠ³ Π½Π° Π΄Ρ€ΡƒΠ³Π°. Быстрый ΠΏΡƒΡ‚ΡŒ — ΠΈΡΠΏΠΎΠ»ΡŒΠ·ΠΎΠ²Π°Ρ‚ΡŒ ΡΡ‚Ρ€Π°Ρ‚Π΅Π³ΠΈΡŽ ΠΎΡ‚ΠΌΠ΅Π½Ρ‹, которая ΡƒΠΌΠ΅Π½ΡŒΡˆΠ°Π΅Ρ‚ числа Π΄ΠΎ минимально Π²ΠΎΠ·ΠΌΠΎΠΆΠ½Ρ‹Ρ… Π·Π½Π°Ρ‡Π΅Π½ΠΈΠΉ ΠΏΠ΅Ρ€Π΅Π΄ ΡƒΠΌΠ½ΠΎΠΆΠ΅Π½ΠΈΠ΅ΠΌ.
  • ΠŸΡ€ΠΈ Π΄Π΅Π»Π΅Π½ΠΈΠΈ Π½Π° Π΄Ρ€ΠΎΠ±ΠΈ ΠΏΠ΅Ρ€Π²ΠΎΠ΅ число умноТаСтся Π½Π° Π²Π΅Π»ΠΈΡ‡ΠΈΠ½Ρƒ, ΠΎΠ±Ρ€Π°Ρ‚Π½ΡƒΡŽ Π²Ρ‚ΠΎΡ€ΠΎΠΌΡƒ числу.
ΠšΠ»ΡŽΡ‡Π΅Π²Ρ‹Π΅ Ρ‚Π΅Ρ€ΠΌΠΈΠ½Ρ‹
  • Ρ‡ΠΈΡΠ»ΠΈΡ‚Π΅Π»ΡŒ : Число, ΠΊΠΎΡ‚ΠΎΡ€ΠΎΠ΅ находится Π½Π°Π΄ Ρ‡Π΅Ρ€Ρ‚ΠΎΠΉ Π΄Ρ€ΠΎΠ±ΠΈ ΠΈ прСдставляСт собой Ρ‡Π°ΡΡ‚ΡŒ Ρ†Π΅Π»ΠΎΠ³ΠΎ числа.
  • обратная : Π΄Ρ€ΠΎΠ±ΡŒ, пСрСвСрнутая Ρ‚Π°ΠΊ, Ρ‡Ρ‚ΠΎ Ρ‡ΠΈΡΠ»ΠΈΡ‚Π΅Π»ΡŒ ΠΈ Π·Π½Π°ΠΌΠ΅Π½Π°Ρ‚Π΅Π»ΡŒ помСнялись мСстами.
  • Π·Π½Π°ΠΌΠ΅Π½Π°Ρ‚Π΅Π»ΡŒ : Число ΠΏΠΎΠ΄ Ρ‡Π΅Ρ€Ρ‚ΠΎΠΉ Π΄Ρ€ΠΎΠ±ΠΈ ΠΈ прСдставляСт собой Ρ†Π΅Π»ΠΎΠ΅ число.
  • Π΄Ρ€ΠΎΠ±ΡŒ : ΠΎΡ‚Π½ΠΎΡˆΠ΅Π½ΠΈΠ΅ Π΄Π²ΡƒΡ… чисСл — числитСля ΠΈ знамСнатСля — ΠΎΠ±Ρ‹Ρ‡Π½ΠΎ записываСмых ΠΎΠ΄Π½ΠΎ Π½Π°Π΄ Π΄Ρ€ΡƒΠ³ΠΈΠΌ ΠΈ Ρ€Π°Π·Π΄Π΅Π»Π΅Π½Π½Ρ‹Ρ… Π³ΠΎΡ€ΠΈΠ·ΠΎΠ½Ρ‚Π°Π»ΡŒΠ½ΠΎΠΉ Ρ‡Π΅Ρ€Ρ‚ΠΎΠΉ.

Π”Ρ€ΠΎΠ±ΡŒ прСдставляСт собой Ρ‡Π°ΡΡ‚ΡŒ Ρ†Π΅Π»ΠΎΠ³ΠΎ. ΠžΠ±Ρ‹Ρ‡Π½Π°Ρ Π΄Ρ€ΠΎΠ±ΡŒ, Π½Π°ΠΏΡ€ΠΈΠΌΠ΅Ρ€ [латСкс] \ frac {1} {2} [/ latex], [latex] \ frac {8} {5} [/ latex] ΠΈΠ»ΠΈ [latex] \ frac {3} {4} [/ latex], состоит ΠΈΠ· Ρ†Π΅Π»ΠΎΠ³ΠΎ числитСля (Π²Π΅Ρ€Ρ…Π½Π΅Π΅ число) ΠΈ Π½Π΅Π½ΡƒΠ»Π΅Π²ΠΎΠ³ΠΎ Ρ†Π΅Π»ΠΎΠ³ΠΎ знамСнатСля (Π½ΠΈΠΆΠ½Π΅Π΅ число). Π§ΠΈΡΠ»ΠΈΡ‚Π΅Π»ΡŒ прСдставляСт ΠΎΠΏΡ€Π΅Π΄Π΅Π»Π΅Π½Π½ΠΎΠ΅ количСство Ρ€Π°Π²Π½Ρ‹Ρ… частСй Ρ†Π΅Π»ΠΎΠ³ΠΎ, Π° Π·Π½Π°ΠΌΠ΅Π½Π°Ρ‚Π΅Π»ΡŒ ΡƒΠΊΠ°Π·Ρ‹Π²Π°Π΅Ρ‚, сколько ΠΈΠ· этих частСй Π½Π΅ΠΎΠ±Ρ…ΠΎΠ΄ΠΈΠΌΠΎ, Ρ‡Ρ‚ΠΎΠ±Ρ‹ ΡΠΎΡΡ‚Π°Π²ΠΈΡ‚ΡŒ ΠΎΠ΄Π½ΠΎ Ρ†Π΅Π»ΠΎΠ΅. ΠŸΡ€ΠΈΠΌΠ΅Ρ€ ΠΌΠΎΠΆΠ½ΠΎ ΡƒΠ²ΠΈΠ΄Π΅Ρ‚ΡŒ Π½Π° ΡΠ»Π΅Π΄ΡƒΡŽΡ‰Π΅ΠΌ рисункС, Π½Π° ΠΊΠΎΡ‚ΠΎΡ€ΠΎΠΌ Ρ‚ΠΎΡ€Ρ‚ Ρ€Π°Π·Π΄Π΅Π»Π΅Π½ Π½Π° Ρ‡Π΅Ρ‚Π²Π΅Ρ€Ρ‚ΠΈΠ½ΠΊΠΈ:

Π§Π΅Ρ‚Π²Π΅Ρ€Ρ‚ΠΈ Ρ‚ΠΎΡ€Ρ‚Π°: Π’ΠΎΡ€Ρ‚ с ΡƒΠ΄Π°Π»Π΅Π½Π½ΠΎΠΉ Ρ‡Π΅Ρ‚Π²Π΅Ρ€Ρ‚ΡŒΡŽ.ΠŸΠΎΠΊΠ°Π·Π°Π½Ρ‹ ΠΎΡΡ‚Π°Π»ΡŒΠ½Ρ‹Π΅ Ρ‚Ρ€ΠΈ Ρ‡Π΅Ρ‚Π²Π΅Ρ€Ρ‚ΠΈ. ΠŸΡƒΠ½ΠΊΡ‚ΠΈΡ€Π½Ρ‹ΠΌΠΈ линиями ΠΎΠ±ΠΎΠ·Π½Π°Ρ‡Π΅Π½Ρ‹ мСста, Π³Π΄Π΅ Ρ‚ΠΎΡ€Ρ‚ ΠΌΠΎΠΆΠ½ΠΎ Ρ€Π°Π·Ρ€Π΅Π·Π°Ρ‚ΡŒ, Ρ‡Ρ‚ΠΎΠ±Ρ‹ Ρ€Π°Π·Π΄Π΅Π»ΠΈΡ‚ΡŒ Π΅Π³ΠΎ Π½Π° Ρ€Π°Π²Π½Ρ‹Π΅ части. КаТдая ΠΎΡΡ‚Π°Π²ΡˆΠ°ΡΡΡ Ρ‡Π΅Ρ‚Π²Π΅Ρ€Ρ‚ΡŒ Ρ‚ΠΎΡ€Ρ‚Π° обозначаСтся Π΄Ρ€ΠΎΠ±ΡŒΡŽ [латСкс] \ frac {1} {4} [/ latex].

Π”ΠΎΠΏΠΎΠ»Π½Π΅Π½ΠΈΠ΅

Π”ΠΎΠ±Π°Π²Π»Π΅Π½ΠΈΠ΅ ΠΎΠ΄ΠΈΠ½Π°ΠΊΠΎΠ²Ρ‹Ρ… количСств

ΠŸΠ΅Ρ€Π²ΠΎΠ΅ ΠΏΡ€Π°Π²ΠΈΠ»ΠΎ слоТСния Π΄Ρ€ΠΎΠ±Π΅ΠΉ — Π½Π°Ρ‡Π°Ρ‚ΡŒ с добавлСния Π΄Ρ€ΠΎΠ±Π΅ΠΉ, ΠΊΠΎΡ‚ΠΎΡ€Ρ‹Π΅ содСрТат ΠΎΠ΄ΠΈΠ½Π°ΠΊΠΎΠ²Ρ‹Π΅ Π·Π½Π°ΠΌΠ΅Π½Π°Ρ‚Π΅Π»ΠΈ, Π½Π°ΠΏΡ€ΠΈΠΌΠ΅Ρ€, ΠΊΡ€Π°Ρ‚Π½Ρ‹Π΅ Ρ‡Π΅Ρ‚Π²Π΅Ρ€Ρ‚ΠΈ ΠΈΠ»ΠΈ Ρ‡Π΅Ρ‚Π²Π΅Ρ€Ρ‚ΠΈ. Π§Π΅Ρ‚Π²Π΅Ρ€Ρ‚ΡŒ прСдставлСна β€‹β€‹Π΄Ρ€ΠΎΠ±ΡŒΡŽ [латСкс] \ frac {1} {4} [/ latex], Π³Π΄Π΅ Ρ‡ΠΈΡΠ»ΠΈΡ‚Π΅Π»ΡŒ 1 прСдставляСт ΠΎΠ΄Π½Ρƒ Ρ‡Π΅Ρ‚Π²Π΅Ρ€Ρ‚ΡŒ, Π° Π·Π½Π°ΠΌΠ΅Π½Π°Ρ‚Π΅Π»ΡŒ 4 прСдставляСт количСство Ρ‡Π΅Ρ‚Π²Π΅Ρ€Ρ‚Π΅ΠΉ, Π½Π΅ΠΎΠ±Ρ…ΠΎΠ΄ΠΈΠΌΠΎΠ΅ для создания Ρ†Π΅Π»ΠΎΠ³ΠΎ , ΠΈΠ»ΠΈ ΠΎΠ΄ΠΈΠ½ Π΄ΠΎΠ»Π»Π°Ρ€.

ΠŸΡ€Π΅Π΄ΡΡ‚Π°Π²ΡŒΡ‚Π΅ сСбС, Ρ‡Ρ‚ΠΎ ΠΎΠ΄ΠΈΠ½ ΠΊΠ°Ρ€ΠΌΠ°Π½ содСрТит Π΄Π²Π΅ Ρ‡Π΅Ρ‚Π²Π΅Ρ€Ρ‚ΠΈΠ½ΠΊΠΈ, Π° Π΄Ρ€ΡƒΠ³ΠΎΠΉ ΠΊΠ°Ρ€ΠΌΠ°Π½ — Ρ‚Ρ€ΠΈ Ρ‡Π΅Ρ‚Π²Π΅Ρ€Ρ‚ΠΈ. ВсСго ΠΏΡΡ‚ΡŒ ΠΊΠ²Π°Ρ€Ρ‚Π°Π»ΠΎΠ². ΠŸΠΎΡΠΊΠΎΠ»ΡŒΠΊΡƒ Ρ‡Π΅Ρ‚Ρ‹Ρ€Π΅ Ρ‡Π΅Ρ‚Π²Π΅Ρ€Ρ‚ΠΈ эквивалСнтны ΠΎΠ΄Π½ΠΎΠΌΡƒ (Π΄ΠΎΠ»Π»Π°Ρ€Ρƒ), это ΠΌΠΎΠΆΠ½ΠΎ ΠΏΡ€Π΅Π΄ΡΡ‚Π°Π²ΠΈΡ‚ΡŒ ΡΠ»Π΅Π΄ΡƒΡŽΡ‰ΠΈΠΌ ΠΎΠ±Ρ€Π°Π·ΠΎΠΌ:

[латСкс] \ displaystyle \ frac {2} {4} + \ frac {3} {4} = \ frac {5} {4} = 1 \ frac {1} {4} [/ latex]

Π”ΠΎΠ±Π°Π²Π»Π΅Π½ΠΈΠ΅ ΠΎΡ‚Π»ΠΈΡ‡Π½Ρ‹Ρ… количСств

Π§Ρ‚ΠΎΠ±Ρ‹ Π΄ΠΎΠ±Π°Π²ΠΈΡ‚ΡŒ Π΄Ρ€ΠΎΠ±ΠΈ, ΠΊΠΎΡ‚ΠΎΡ€Ρ‹Π΅ содСрТат Π·Π½Π°ΠΌΠ΅Π½Π°Ρ‚Π΅Π»ΠΈ Π² ΠΎΡ‚Π»ΠΈΡ‡ΠΈΠ΅ ΠΎΡ‚ (Π½Π°ΠΏΡ€ΠΈΠΌΠ΅Ρ€, Ρ‡Π΅Ρ‚Π²Π΅Ρ€Ρ‚ΠΈ ΠΈ Ρ‚Ρ€Π΅Ρ‚ΠΈ), Π½Π΅ΠΎΠ±Ρ…ΠΎΠ΄ΠΈΠΌΠΎ сначала ΠΏΡ€Π΅ΠΎΠ±Ρ€Π°Π·ΠΎΠ²Π°Ρ‚ΡŒ всС суммы Π² ΠΎΠ΄ΠΈΠ½Π°ΠΊΠΎΠ²Ρ‹Π΅ Π²Π΅Π»ΠΈΡ‡ΠΈΠ½Ρ‹, Ρ‡Ρ‚ΠΎ ΠΎΠ·Π½Π°Ρ‡Π°Π΅Ρ‚, Ρ‡Ρ‚ΠΎ всС Π΄Ρ€ΠΎΠ±ΠΈ Π΄ΠΎΠ»ΠΆΠ½Ρ‹ ΠΈΠΌΠ΅Ρ‚ΡŒ ΠΎΠ±Ρ‰ΠΈΠΉ Π·Π½Π°ΠΌΠ΅Π½Π°Ρ‚Π΅Π»ΡŒ.Один простой способ Π½Π°ΠΉΡ‚ΠΈ Π·Π½Π°ΠΌΠ΅Π½Π°Ρ‚Π΅Π»ΡŒ, ΠΊΠΎΡ‚ΠΎΡ€Ρ‹ΠΉ даст Π²Π°ΠΌ ΠΎΠ΄ΠΈΠ½Π°ΠΊΠΎΠ²Ρ‹Π΅ количСства, — это просто ΠΏΠ΅Ρ€Π΅ΠΌΠ½ΠΎΠΆΠΈΡ‚ΡŒ Π΄Π²Π° знамСнатСля Π΄Ρ€ΠΎΠ±Π΅ΠΉ. (Π’Π°ΠΆΠ½ΠΎ ΠΏΠΎΠΌΠ½ΠΈΡ‚ΡŒ, Ρ‡Ρ‚ΠΎ ΠΊΠ°ΠΆΠ΄Ρ‹ΠΉ Ρ‡ΠΈΡΠ»ΠΈΡ‚Π΅Π»ΡŒ Ρ‚Π°ΠΊΠΆΠ΅ Π΄ΠΎΠ»ΠΆΠ΅Π½ Π±Ρ‹Ρ‚ΡŒ ΡƒΠΌΠ½ΠΎΠΆΠ΅Π½ Π½Π° Ρ‚ΠΎ ΠΆΠ΅ Π·Π½Π°Ρ‡Π΅Π½ΠΈΠ΅, Π½Π° ΠΊΠΎΡ‚ΠΎΡ€ΠΎΠ΅ умноТаСтся Π΅Π³ΠΎ Π·Π½Π°ΠΌΠ΅Π½Π°Ρ‚Π΅Π»ΡŒ, Ρ‡Ρ‚ΠΎΠ±Ρ‹ Π΄Ρ€ΠΎΠ±ΡŒ прСдставляла Ρ‚ΠΎ ΠΆΠ΅ ΠΎΡ‚Π½ΠΎΡˆΠ΅Π½ΠΈΠ΅.)

НапримСр, Ρ‡Ρ‚ΠΎΠ±Ρ‹ ΠΏΡ€ΠΈΠ±Π°Π²ΠΈΡ‚ΡŒ Ρ‡Π΅Ρ‚Π²Π΅Ρ€Ρ‚ΠΈ ΠΊ трСтям, ΠΎΠ±Π° Ρ‚ΠΈΠΏΠ° Π΄Ρ€ΠΎΠ±Π΅ΠΉ ΠΏΡ€Π΅ΠΎΠ±Ρ€Π°Π·ΡƒΡŽΡ‚ΡΡ Π² Π΄Π²Π΅Π½Π°Π΄Ρ†Π°Ρ‚Ρ‹Π΅:

[латСкс] \ displaystyle \ frac {1} {3} + \ frac {1} {4} = \ frac {1 \ cdot 4} {3 \ cdot4} + \ frac {1 \ cdot3} {4 \ cdot3} = \ frac {4} {12} + \ frac {3} {12} = \ frac {7} {12} [/ latex]

Π­Ρ‚ΠΎΡ‚ ΠΌΠ΅Ρ‚ΠΎΠ΄ ΠΌΠΎΠΆΠ½ΠΎ алгСбраичСски Π²Ρ‹Ρ€Π°Π·ΠΈΡ‚ΡŒ ΡΠ»Π΅Π΄ΡƒΡŽΡ‰ΠΈΠΌ ΠΎΠ±Ρ€Π°Π·ΠΎΠΌ:

[латСкс] \ displaystyle \ frac {a} {b} + \ frac {c} {d} = \ frac {ad + cb} {bd} [/ latex]

Π­Ρ‚ΠΎΡ‚ ΠΌΠ΅Ρ‚ΠΎΠ΄ Ρ€Π°Π±ΠΎΡ‚Π°Π΅Ρ‚ всСгда. Однако ΠΈΠ½ΠΎΠ³Π΄Π° Π΅ΡΡ‚ΡŒ Π±ΠΎΠ»Π΅Π΅ быстрый способ — ΠΈΡΠΏΠΎΠ»ΡŒΠ·ΠΎΠ²Π°Ρ‚ΡŒ мСньший Π·Π½Π°ΠΌΠ΅Π½Π°Ρ‚Π΅Π»ΡŒ ΠΈΠ»ΠΈ наимСньший ΠΎΠ±Ρ‰ΠΈΠΉ Π·Π½Π°ΠΌΠ΅Π½Π°Ρ‚Π΅Π»ΡŒ. НапримСр, Ρ‡Ρ‚ΠΎΠ±Ρ‹ Π΄ΠΎΠ±Π°Π²ΠΈΡ‚ΡŒ [latex] \ frac {3} {4} [/ latex] ΠΊ [latex] \ frac {5} {12} [/ latex], Π·Π½Π°ΠΌΠ΅Π½Π°Ρ‚Π΅Π»ΡŒ 48 (ΠΏΡ€ΠΎΠΈΠ·Π²Π΅Π΄Π΅Π½ΠΈΠ΅ 4 ΠΈ 12, Π΄Π²Π° Π·Π½Π°ΠΌΠ΅Π½Π°Ρ‚Π΅Π»ΠΈ), Π½ΠΎ ΠΌΠΎΠΆΠ½ΠΎ ΠΈΡΠΏΠΎΠ»ΡŒΠ·ΠΎΠ²Π°Ρ‚ΡŒ ΠΈ мСньший Π·Π½Π°ΠΌΠ΅Π½Π°Ρ‚Π΅Π»ΡŒ 12 (наимСньшСС ΠΎΠ±Ρ‰Π΅Π΅ ΠΊΡ€Π°Ρ‚Π½ΠΎΠ΅ 4 ΠΈ 12).

Π‘Π»ΠΎΠΆΠ΅Π½ΠΈΠ΅ Π΄Ρ€ΠΎΠ±Π΅ΠΉ ΠΊ Ρ†Π΅Π»Ρ‹ΠΌ числам

Π§Ρ‚ΠΎ Π΄Π΅Π»Π°Ρ‚ΡŒ, Ссли ΠΊ Ρ†Π΅Π»ΠΎΠΌΡƒ числу прибавляСтся дробная Ρ‡Π°ΡΡ‚ΡŒ? ΠŸΡ€ΠΎΡΡ‚ΠΎ Π½Π°Ρ‡Π½ΠΈΡ‚Π΅ с записи Ρ†Π΅Π»ΠΎΠ³ΠΎ числа Π² Π²ΠΈΠ΄Π΅ Π΄Ρ€ΠΎΠ±ΠΈ (Π½Π°ΠΏΠΎΠΌΠ½ΠΈΠΌ, Ρ‡Ρ‚ΠΎ Ρ†Π΅Π»ΠΎΠ΅ число ΠΈΠΌΠ΅Π΅Ρ‚ Π·Π½Π°ΠΌΠ΅Π½Π°Ρ‚Π΅Π»ΡŒ [латСкс] 1 [/ латСкс]), Π° Π·Π°Ρ‚Π΅ΠΌ ΠΏΡ€ΠΎΠ΄ΠΎΠ»ΠΆΠ°ΠΉΡ‚Π΅ описанный Π²Ρ‹ΡˆΠ΅ процСсс слоТСния Π΄Ρ€ΠΎΠ±Π΅ΠΉ.

Π’Ρ‹Ρ‡ΠΈΡ‚Π°Π½ΠΈΠ΅

ΠŸΡ€ΠΎΡ†Π΅ΡΡ вычитания Π΄Ρ€ΠΎΠ±Π΅ΠΉ, ΠΏΠΎ сути, Ρ‚Π°ΠΊΠΎΠΉ ΠΆΠ΅, ΠΊΠ°ΠΊ ΠΈ процСсс ΠΈΡ… слоТСния. НайдитС ΠΎΠ±Ρ‰ΠΈΠΉ Π·Π½Π°ΠΌΠ΅Π½Π°Ρ‚Π΅Π»ΡŒ ΠΈ Π·Π°ΠΌΠ΅Π½ΠΈΡ‚Π΅ ΠΊΠ°ΠΆΠ΄ΡƒΡŽ Π΄Ρ€ΠΎΠ±ΡŒ Π½Π° ΡΠΊΠ²ΠΈΠ²Π°Π»Π΅Π½Ρ‚Π½ΡƒΡŽ Π΄Ρ€ΠΎΠ±ΡŒ, ΠΈΡΠΏΠΎΠ»ΡŒΠ·ΡƒΡ этот ΠΎΠ±Ρ‰ΠΈΠΉ Π·Π½Π°ΠΌΠ΅Π½Π°Ρ‚Π΅Π»ΡŒ. Π—Π°Ρ‚Π΅ΠΌ Π²Ρ‹Ρ‡Ρ‚ΠΈΡ‚Π΅ числитСли. НапримСр:

[латСкс] \ displaystyle \ frac {2} {3} — \ frac {1} {2} = \ frac {2 \ cdot 2} {3 \ cdot2} — \ frac {1 \ cdot3} {2 \ cdot3} = \ frac {4} {6} — \ frac {3} {6} = \ frac {1} {6} [/ latex]

Π§Ρ‚ΠΎΠ±Ρ‹ Π²Ρ‹Ρ‡Π΅ΡΡ‚ΡŒ Π΄Ρ€ΠΎΠ±ΡŒ ΠΈΠ· Ρ†Π΅Π»ΠΎΠ³ΠΎ числа ΠΈΠ»ΠΈ Π²Ρ‹Ρ‡Π΅ΡΡ‚ΡŒ Ρ†Π΅Π»ΠΎΠ΅ число ΠΈΠ· Π΄Ρ€ΠΎΠ±ΠΈ, ΠΏΠ΅Ρ€Π΅ΠΏΠΈΡˆΠΈΡ‚Π΅ Ρ†Π΅Π»ΠΎΠ΅ число ΠΊΠ°ΠΊ Π΄Ρ€ΠΎΠ±ΡŒ, Π° Π·Π°Ρ‚Π΅ΠΌ Π²Ρ‹ΠΏΠΎΠ»Π½ΠΈΡ‚Π΅ описанный Π²Ρ‹ΡˆΠ΅ процСсс вычитания Π΄Ρ€ΠΎΠ±Π΅ΠΉ.

Π£ΠΌΠ½ΠΎΠΆΠ΅Π½ΠΈΠ΅

Π’ ΠΎΡ‚Π»ΠΈΡ‡ΠΈΠ΅ ΠΎΡ‚ слоТСния ΠΈ вычитания, ΠΏΡ€ΠΈ ΡƒΠΌΠ½ΠΎΠΆΠ΅Π½ΠΈΠΈ Π·Π½Π°ΠΌΠ΅Π½Π°Ρ‚Π΅Π»ΠΈ Π½Π΅ ΠΎΠ±ΡΠ·Π°Ρ‚Π΅Π»ΡŒΠ½ΠΎ Π΄ΠΎΠ»ΠΆΠ½Ρ‹ Π±Ρ‹Ρ‚ΡŒ ΠΎΠ΄ΠΈΠ½Π°ΠΊΠΎΠ²Ρ‹ΠΌΠΈ. Π§Ρ‚ΠΎΠ±Ρ‹ ΡƒΠΌΠ½ΠΎΠΆΠΈΡ‚ΡŒ Π΄Ρ€ΠΎΠ±ΠΈ, просто ΡƒΠΌΠ½ΠΎΠΆΡŒΡ‚Π΅ числитСли Π΄Ρ€ΡƒΠ³ Π½Π° Π΄Ρ€ΡƒΠ³Π°, Π° Π·Π½Π°ΠΌΠ΅Π½Π°Ρ‚Π΅Π»ΠΈ Π΄Ρ€ΡƒΠ³ Π½Π° Π΄Ρ€ΡƒΠ³Π°. НапримСр:

[латСкс] \ displaystyle \ frac {2} {3} \ cdot \ frac {3} {4} = \ frac {6} {12} [/ latex]

Если ΠΊΠ°ΠΊΠΎΠΉ-Π»ΠΈΠ±ΠΎ Ρ‡ΠΈΡΠ»ΠΈΡ‚Π΅Π»ΡŒ ΠΈ Π·Π½Π°ΠΌΠ΅Π½Π°Ρ‚Π΅Π»ΡŒ ΠΈΠΌΠ΅ΡŽΡ‚ ΠΎΠ±Ρ‰ΠΈΠΉ ΠΌΠ½ΠΎΠΆΠΈΡ‚Π΅Π»ΡŒ, Π΄Ρ€ΠΎΠ±ΠΈ ΠΌΠΎΠ³ΡƒΡ‚ Π±Ρ‹Ρ‚ΡŒ ΡƒΠΌΠ΅Π½ΡŒΡˆΠ΅Π½Ρ‹ Π΄ΠΎ наимСньшСго значСния Π΄ΠΎ ΠΈΠ»ΠΈ послС умноТСния. НапримСр, получСнная Π΄Ρ€ΠΎΠ±ΡŒ ΠΌΠΎΠΆΠ΅Ρ‚ Π±Ρ‹Ρ‚ΡŒ ΡƒΠΌΠ΅Π½ΡŒΡˆΠ΅Π½Π° Π΄ΠΎ [latex] \ frac {1} {2} [/ latex], ΠΏΠΎΡ‚ΠΎΠΌΡƒ Ρ‡Ρ‚ΠΎ Ρ‡ΠΈΡΠ»ΠΈΡ‚Π΅Π»ΡŒ ΠΈ Π·Π½Π°ΠΌΠ΅Π½Π°Ρ‚Π΅Π»ΡŒ дСлят ΠΌΠ½ΠΎΠΆΠΈΡ‚Π΅Π»ΡŒ 6.Π’ качСствС Π°Π»ΡŒΡ‚Π΅Ρ€Π½Π°Ρ‚ΠΈΠ²Ρ‹ Π΄Ρ€ΠΎΠ±ΠΈ Π² исходном ΡƒΡ€Π°Π²Π½Π΅Π½ΠΈΠΈ ΠΌΠΎΠΆΠ½ΠΎ Π±Ρ‹Π»ΠΎ Π±Ρ‹ ΡƒΠΌΠ΅Π½ΡŒΡˆΠΈΡ‚ΡŒ, ΠΊΠ°ΠΊ ΠΏΠΎΠΊΠ°Π·Π°Π½ΠΎ Π½ΠΈΠΆΠ΅, ΠΏΠΎΡ‚ΠΎΠΌΡƒ Ρ‡Ρ‚ΠΎ 2 ΠΈ 4 ΠΈΠΌΠ΅ΡŽΡ‚ ΠΎΠ±Ρ‰ΠΈΠΉ ΠΌΠ½ΠΎΠΆΠΈΡ‚Π΅Π»ΡŒ 2, Π° 3 ΠΈ 3 ΠΈΠΌΠ΅ΡŽΡ‚ ΠΎΠ±Ρ‰ΠΈΠΉ ΠΌΠ½ΠΎΠΆΠΈΡ‚Π΅Π»ΡŒ 3:

[латСкс] \ displaystyle \ frac {2} {3} \ cdot \ frac {3} {4} = \ frac {1} {1} \ cdot \ frac {1} {2} = \ frac {1} { 2} [/ латСкс]

Π§Ρ‚ΠΎΠ±Ρ‹ ΡƒΠΌΠ½ΠΎΠΆΠΈΡ‚ΡŒ Π΄Ρ€ΠΎΠ±ΡŒ Π½Π° Ρ†Π΅Π»ΠΎΠ΅ число, просто ΡƒΠΌΠ½ΠΎΠΆΡŒΡ‚Π΅ это число Π½Π° Ρ‡ΠΈΡΠ»ΠΈΡ‚Π΅Π»ΡŒ Π΄Ρ€ΠΎΠ±ΠΈ:

[латСкс] \ displaystyle \ frac {3} {4} \ cdot 5 = \ frac {15} {4} [/ latex]

ΠžΠ±Ρ‹Ρ‡Π½Π°Ρ ситуация, ΠΊΠΎΠ³Π΄Π° ΡƒΠΌΠ½ΠΎΠΆΠ΅Π½ΠΈΠ΅ Π΄Ρ€ΠΎΠ±Π΅ΠΉ Π±Ρ‹Π²Π°Π΅Ρ‚ ΠΏΠΎΠ»Π΅Π·Π½Ρ‹ΠΌ, — это Π²ΠΎ врСмя Π³ΠΎΡ‚ΠΎΠ²ΠΊΠΈ. Π§Ρ‚ΠΎ, Ссли ΠΊΡ‚ΠΎ-Ρ‚ΠΎ Π·Π°Ρ…ΠΎΡ‡Π΅Ρ‚ Β«ΠΏΠΎΠ»ΠΎΠ²ΠΈΠ½ΡƒΒ» Ρ€Π΅Ρ†Π΅ΠΏΡ‚Π° ΠΏΠ΅Ρ‡Π΅Π½ΡŒΡ, для ΠΊΠΎΡ‚ΠΎΡ€ΠΎΠ³ΠΎ трСбуСтся [латСкс] \ frac {1} {2} [/ latex] Ρ‡Π°ΡˆΠΊΠΈ шоколадной струТки? Π§Ρ‚ΠΎΠ±Ρ‹ Π½Π°ΠΉΡ‚ΠΈ Π½Π΅ΠΎΠ±Ρ…ΠΎΠ΄ΠΈΠΌΠΎΠ΅ количСство шоколадной ΠΊΡ€ΠΎΡˆΠΊΠΈ, ΡƒΠΌΠ½ΠΎΠΆΡŒΡ‚Π΅ [латСкс] \ frac {1} {2} \ cdot \ frac {1} {2} [/ latex]. Π’ Ρ€Π΅Π·ΡƒΠ»ΡŒΡ‚Π°Ρ‚Π΅ получаСтся [латСкс] \ frac {1} {4} [/ latex], поэтому ΠΏΡ€Π°Π²ΠΈΠ»ΡŒΠ½ΠΎΠ΅ количСство шоколадной струТки составляСт [латСкс] \ frac {1} {4} [/ latex] Ρ‡Π°ΡˆΠΊΠΈ.

Дивизия

ΠŸΡ€ΠΎΡ†Π΅ΡΡ дСлСния числа Π½Π° Π΄Ρ€ΠΎΠ±ΡŒ Π²Π»Π΅Ρ‡Π΅Ρ‚ Π·Π° собой ΡƒΠΌΠ½ΠΎΠΆΠ΅Π½ΠΈΠ΅ числа Π½Π° ΠΎΠ±Ρ€Π°Ρ‚Π½ΡƒΡŽ Π²Π΅Π»ΠΈΡ‡ΠΈΠ½Ρƒ Π΄Ρ€ΠΎΠ±ΠΈ.ΠžΠ±Ρ€Π°Ρ‚Π½Π°Ρ Π²Π΅Π»ΠΈΡ‡ΠΈΠ½Π° — это просто Π΄Ρ€ΠΎΠ±ΡŒ, пСрСвСрнутая Ρ‚Π°ΠΊ, Ρ‡Ρ‚ΠΎ Ρ‡ΠΈΡΠ»ΠΈΡ‚Π΅Π»ΡŒ ΠΈ Π·Π½Π°ΠΌΠ΅Π½Π°Ρ‚Π΅Π»ΡŒ ΠΌΠ΅Π½ΡΡŽΡ‚ΡΡ мСстами. НапримСр:

[латСкс] \ displaystyle \ frac {1} {2} \ div \ frac {3} {4} = \ frac {1} {2} \ cdot \ frac {4} {3} = \ frac {4} { 6} = \ frac {2} {3} [/ latex]

Π§Ρ‚ΠΎΠ±Ρ‹ Ρ€Π°Π·Π΄Π΅Π»ΠΈΡ‚ΡŒ Π΄Ρ€ΠΎΠ±ΡŒ Π½Π° Ρ†Π΅Π»ΠΎΠ΅ число, Π»ΠΈΠ±ΠΎ Ρ€Π°Π·Π΄Π΅Π»ΠΈΡ‚Π΅ Ρ‡ΠΈΡΠ»ΠΈΡ‚Π΅Π»ΡŒ Π΄Ρ€ΠΎΠ±ΠΈ Π½Π° Ρ†Π΅Π»ΠΎΠ΅ число (Ссли ΠΎΠ½ΠΎ дСлится просто):

[латСкс] \ displaystyle \ frac {10} {3} \ div 5 = \ frac {10 \ div 2} {3} = \ frac {2} {3} [/ latex]

ΠΈΠ»ΠΈ ΡƒΠΌΠ½ΠΎΠΆΡŒΡ‚Π΅ Π·Π½Π°ΠΌΠ΅Π½Π°Ρ‚Π΅Π»ΡŒ Π΄Ρ€ΠΎΠ±ΠΈ Π½Π° Ρ†Π΅Π»ΠΎΠ΅ число:

[латСкс] \ displaystyle \ frac {10} {3} \ div 5 = \ displaystyle \ frac {10} {3 \ cdot5} = \ frac {10} {15} = \ frac {2} {3} [/ латСкс]

Π‘Π»ΠΎΠΆΠ½Ρ‹Π΅ Ρ„Ρ€Π°ΠΊΡ†ΠΈΠΈ

КомплСксная Π΄Ρ€ΠΎΠ±ΡŒ — это Π΄Ρ€ΠΎΠ±ΡŒ, Π² ΠΊΠΎΡ‚ΠΎΡ€ΠΎΠΉ Ρ‡ΠΈΡΠ»ΠΈΡ‚Π΅Π»ΡŒ, Π·Π½Π°ΠΌΠ΅Π½Π°Ρ‚Π΅Π»ΡŒ ΠΈΠ»ΠΈ ΠΎΠ±Π° ΡΠ²Π»ΡΡŽΡ‚ΡΡ дробями, ΠΊΠΎΡ‚ΠΎΡ€Ρ‹Π΅ ΠΌΠΎΠ³ΡƒΡ‚ ΡΠΎΠ΄Π΅Ρ€ΠΆΠ°Ρ‚ΡŒ ΠΏΠ΅Ρ€Π΅ΠΌΠ΅Π½Π½Ρ‹Π΅, константы ΠΈΠ»ΠΈ ΠΈ Ρ‚ΠΎ, ΠΈ Π΄Ρ€ΡƒΠ³ΠΎΠ΅.

Π¦Π΅Π»ΠΈ обучСния

Π£ΠΏΡ€ΠΎΡΡ‚ΠΈΡ‚ΡŒ слоТныС Π΄Ρ€ΠΎΠ±ΠΈ

ΠšΠ»ΡŽΡ‡Π΅Π²Ρ‹Π΅ Π²Ρ‹Π²ΠΎΠ΄Ρ‹

ΠšΠ»ΡŽΡ‡Π΅Π²Ρ‹Π΅ ΠΌΠΎΠΌΠ΅Π½Ρ‚Ρ‹
  • Π‘Π»ΠΎΠΆΠ½Ρ‹Π΅ Π΄Ρ€ΠΎΠ±ΠΈ Π²ΠΊΠ»ΡŽΡ‡Π°ΡŽΡ‚ Ρ‚Π°ΠΊΠΈΠ΅ числа, ΠΊΠ°ΠΊ [latex] \ frac {\ left (\ frac {8} {15} \ right)} {\ left (\ frac {2} {3} \ right)} [/ latex] ΠΈ [латСкс] \ frac {3} {1- \ frac {2} {5}} [/ latex], Π³Π΄Π΅ Ρ‡ΠΈΡΠ»ΠΈΡ‚Π΅Π»ΡŒ, Π·Π½Π°ΠΌΠ΅Π½Π°Ρ‚Π΅Π»ΡŒ ΠΈΠ»ΠΈ ΠΎΠ±Π° Π²ΠΊΠ»ΡŽΡ‡Π°ΡŽΡ‚ Π΄Ρ€ΠΎΠ±ΠΈ.
  • ΠŸΡ€Π΅ΠΆΠ΄Π΅ Ρ‡Π΅ΠΌ Ρ€Π΅ΡˆΠ°Ρ‚ΡŒ слоТныС Ρ€Π°Ρ†ΠΈΠΎΠ½Π°Π»ΡŒΠ½Ρ‹Π΅ выраТСния, ΠΏΠΎΠ»Π΅Π·Π½ΠΎ ΠΈΡ… максимально ΡƒΠΏΡ€ΠΎΡΡ‚ΠΈΡ‚ΡŒ.
  • Β«ΠœΠ΅Ρ‚ΠΎΠ΄ комбинирования-дСлСния» для упрощСния слоТных Π΄Ρ€ΠΎΠ±Π΅ΠΉ Π²Π»Π΅Ρ‡Π΅Ρ‚ Π·Π° собой (1) объСдинСниС Ρ‡Π»Π΅Π½ΠΎΠ² Π² числитСлС, (2) объСдинСниС Ρ‡Π»Π΅Π½ΠΎΠ² Π² Π·Π½Π°ΠΌΠ΅Π½Π°Ρ‚Π΅Π»Π΅ ΠΈ, Π½Π°ΠΊΠΎΠ½Π΅Ρ†, (3) Π΄Π΅Π»Π΅Π½ΠΈΠ΅ числитСля Π½Π° Π·Π½Π°ΠΌΠ΅Π½Π°Ρ‚Π΅Π»ΡŒ.
ΠšΠ»ΡŽΡ‡Π΅Π²Ρ‹Π΅ Ρ‚Π΅Ρ€ΠΌΠΈΠ½Ρ‹
  • комплСксная Π΄Ρ€ΠΎΠ±ΡŒ : ΠΎΡ‚Π½ΠΎΡˆΠ΅Π½ΠΈΠ΅, Π² ΠΊΠΎΡ‚ΠΎΡ€ΠΎΠΌ Ρ‡ΠΈΡΠ»ΠΈΡ‚Π΅Π»ΡŒ, Π·Π½Π°ΠΌΠ΅Π½Π°Ρ‚Π΅Π»ΡŒ ΠΈΠ»ΠΈ ΠΎΠ±Π° сами ΡΠ²Π»ΡΡŽΡ‚ΡΡ дробями.

КомплСксная Π΄Ρ€ΠΎΠ±ΡŒ, Ρ‚Π°ΠΊΠΆΠ΅ называСмая комплСксным Ρ€Π°Ρ†ΠΈΠΎΠ½Π°Π»ΡŒΠ½Ρ‹ΠΌ Π²Ρ‹Ρ€Π°ΠΆΠ΅Π½ΠΈΠ΅ΠΌ, — это Π΄Ρ€ΠΎΠ±ΡŒ, Π² ΠΊΠΎΡ‚ΠΎΡ€ΠΎΠΉ Ρ‡ΠΈΡΠ»ΠΈΡ‚Π΅Π»ΡŒ, Π·Π½Π°ΠΌΠ΅Π½Π°Ρ‚Π΅Π»ΡŒ ΠΈΠ»ΠΈ ΠΎΠ±Π° ΡΠ²Π»ΡΡŽΡ‚ΡΡ дробями. НапримСр, [латСкс] \ frac {\ left (\ frac {8} {15} \ right)} {\ left (\ frac {2} {3} \ right)} [/ latex] ΠΈ [latex] \ frac {3} {1- \ frac {2} {5}} [/ latex] — слоТныС Π΄Ρ€ΠΎΠ±ΠΈ. ΠŸΡ€ΠΈ Ρ€Π°Π±ΠΎΡ‚Π΅ с уравнСниями, ΠΊΠΎΡ‚ΠΎΡ€Ρ‹Π΅ Π²ΠΊΠ»ΡŽΡ‡Π°ΡŽΡ‚ слоТныС Π΄Ρ€ΠΎΠ±ΠΈ, ΠΏΠΎΠ»Π΅Π·Π½ΠΎ ΡƒΠΏΡ€ΠΎΡΡ‚ΠΈΡ‚ΡŒ ΡΠ»ΠΎΠΆΠ½ΡƒΡŽ Π΄Ρ€ΠΎΠ±ΡŒ ΠΏΠ΅Ρ€Π΅Π΄ Ρ‚Π΅ΠΌ, ΠΊΠ°ΠΊ Ρ€Π΅ΡˆΠ°Ρ‚ΡŒ ΡƒΡ€Π°Π²Π½Π΅Π½ΠΈΠ΅.

ΠŸΡ€ΠΎΡ†Π΅ΡΡ упрощСния слоТных Π΄Ρ€ΠΎΠ±Π΅ΠΉ, извСстный ΠΊΠ°ΠΊ Β«ΠΌΠ΅Ρ‚ΠΎΠ΄ комбинирования-дСлСния», выглядит ΡΠ»Π΅Π΄ΡƒΡŽΡ‰ΠΈΠΌ ΠΎΠ±Ρ€Π°Π·ΠΎΠΌ:

  1. ΠžΠ±ΡŠΠ΅Π΄ΠΈΠ½ΠΈΡ‚Π΅ Ρ‡Π»Π΅Π½Ρ‹ Π² числитСлС.
  2. ΠžΠ±ΡŠΠ΅Π΄ΠΈΠ½ΠΈΡ‚Π΅ Ρ‡Π»Π΅Π½Ρ‹ Π² Π·Π½Π°ΠΌΠ΅Π½Π°Ρ‚Π΅Π»Π΅.
  3. Π Π°Π·Π΄Π΅Π»ΠΈΡ‚Π΅ Ρ‡ΠΈΡΠ»ΠΈΡ‚Π΅Π»ΡŒ Π½Π° Π·Π½Π°ΠΌΠ΅Π½Π°Ρ‚Π΅Π»ΡŒ.

ΠŸΡ€ΠΈΠΌΠ΅Ρ€ 1

Π”Π°Π²Π°ΠΉΡ‚Π΅ ΠΏΡ€ΠΈΠΌΠ΅Π½ΠΈΠΌ этот ΠΌΠ΅Ρ‚ΠΎΠ΄ ΠΊ ΠΏΠ΅Ρ€Π²ΠΎΠΉ слоТной Π΄Ρ€ΠΎΠ±ΠΈ, прСдставлСнной Π²Ρ‹ΡˆΠ΅:

[латСкс] \ displaystyle {\ frac {\ left (\ frac {8} {15} \ right)} {\ left (\ frac {2} {3} \ right)}} [/ латСкс]

ΠŸΠΎΡΠΊΠΎΠ»ΡŒΠΊΡƒ Π½Π΅Ρ‚ Ρ‚Π΅Ρ€ΠΌΠΈΠ½ΠΎΠ², ΠΊΠΎΡ‚ΠΎΡ€Ρ‹Π΅ ΠΌΠΎΠΆΠ½ΠΎ ΠΎΠ±ΡŠΠ΅Π΄ΠΈΠ½ΠΈΡ‚ΡŒ ΠΈΠ»ΠΈ ΡƒΠΏΡ€ΠΎΡΡ‚ΠΈΡ‚ΡŒ Π½ΠΈ Π² числитСлС, Π½ΠΈ Π² Π·Π½Π°ΠΌΠ΅Π½Π°Ρ‚Π΅Π»Π΅, ΠΌΡ‹ ΠΏΠ΅Ρ€Π΅ΠΉΠ΄Π΅ΠΌ ΠΊ ΡˆΠ°Π³Ρƒ 3, Ρ€Π°Π·Π΄Π΅Π»ΠΈΠ² Ρ‡ΠΈΡΠ»ΠΈΡ‚Π΅Π»ΡŒ Π½Π° Π·Π½Π°ΠΌΠ΅Π½Π°Ρ‚Π΅Π»ΡŒ:

[латСкс] \ displaystyle {\ frac {\ left (\ frac {8} {15} \ right)} {\ left (\ frac {2} {3} \ right)} = \ frac {8} {15} \ div \ frac {2} {3}} [/ латСкс]

Из ΠΏΡ€Π΅Π΄Ρ‹Π΄ΡƒΡ‰ΠΈΡ… Ρ€Π°Π·Π΄Π΅Π»ΠΎΠ² ΠΌΡ‹ Π·Π½Π°Π΅ΠΌ, Ρ‡Ρ‚ΠΎ Π΄Π΅Π»Π΅Π½ΠΈΠ΅ Π½Π° Π΄Ρ€ΠΎΠ±ΡŒ Π°Π½Π°Π»ΠΎΠ³ΠΈΡ‡Π½ΠΎ ΡƒΠΌΠ½ΠΎΠΆΠ΅Π½ΠΈΡŽ Π½Π° ΠΎΠ±Ρ€Π°Ρ‚Π½ΡƒΡŽ Π²Π΅Π»ΠΈΡ‡ΠΈΠ½Ρƒ этой Π΄Ρ€ΠΎΠ±ΠΈ.ΠŸΠΎΡΡ‚ΠΎΠΌΡƒ ΠΌΡ‹ ΠΈΡΠΏΠΎΠ»ΡŒΠ·ΡƒΠ΅ΠΌ ΠΌΠ΅Ρ‚ΠΎΠ΄ сокращСния, Ρ‡Ρ‚ΠΎΠ±Ρ‹ максимально ΡƒΠΏΡ€ΠΎΡΡ‚ΠΈΡ‚ΡŒ числа, Π° Π·Π°Ρ‚Π΅ΠΌ ΡƒΠΌΠ½ΠΎΠΆΠ°Π΅ΠΌ Π΅Π³ΠΎ Π½Π° ΡƒΠΏΡ€ΠΎΡ‰Π΅Π½Π½ΡƒΡŽ ΠΎΠ±Ρ€Π°Ρ‚Π½ΡƒΡŽ Π²Π΅Π»ΠΈΡ‡ΠΈΠ½Ρƒ дСлитСля ΠΈΠ»ΠΈ знамСнатСля Π΄Ρ€ΠΎΠ±ΠΈ:

[латСкс] \ displaystyle {{\ frac 8 {15}} \ cdot {\ frac 32} = {\ frac 4 {5}} \ cdot {\ frac 11} = {\ frac 4 {5}}} [/ латСкс]

Π‘Π»Π΅Π΄ΠΎΠ²Π°Ρ‚Π΅Π»ΡŒΠ½ΠΎ, комплСксная Π΄Ρ€ΠΎΠ±ΡŒ [латСкс] \ frac {\ left (\ frac {8} {15} \ right)} {\ left (\ frac {2} {3} \ right)} [/ latex] упрощаСтся Π΄ΠΎ [ латСкс] \ frac {4} {5} [/ латСкс].

ΠŸΡ€ΠΈΠΌΠ΅Ρ€ 2

Π”Π°Π²Π°ΠΉΡ‚Π΅ ΠΏΠΎΠΏΡ€ΠΎΠ±ΡƒΠ΅ΠΌ Π΄Ρ€ΡƒΠ³ΠΎΠΉ ΠΏΡ€ΠΈΠΌΠ΅Ρ€:

[латСкс] \ displaystyle {\ frac {\ left (\ dfrac {1} {2} + \ dfrac {2} {3} \ right)} {\ left (\ dfrac {2} {3} \ cdot \ dfrac {3} {4} \ right)}} [/ латСкс]

НачнитС с шага 1 описанного Π²Ρ‹ΡˆΠ΅ ΠΌΠ΅Ρ‚ΠΎΠ΄Π° комбинирования-дСлСния: ΠΎΠ±ΡŠΠ΅Π΄ΠΈΠ½ΠΈΡ‚Π΅ Ρ‡Π»Π΅Π½Ρ‹ Π² числитСлС.Π’Ρ‹ ΠΎΠ±Π½Π°Ρ€ΡƒΠΆΠΈΡ‚Π΅, Ρ‡Ρ‚ΠΎ ΠΎΠ±Ρ‰ΠΈΠΉ Π·Π½Π°ΠΌΠ΅Π½Π°Ρ‚Π΅Π»ΡŒ Π΄Π²ΡƒΡ… Π΄Ρ€ΠΎΠ±Π΅ΠΉ Π² числитСлС Ρ€Π°Π²Π΅Π½ 6, Π° Π·Π°Ρ‚Π΅ΠΌ Π²Ρ‹ ΠΌΠΎΠΆΠ΅Ρ‚Π΅ ΡΠ»ΠΎΠΆΠΈΡ‚ΡŒ эти Π΄Π²Π° Ρ‡Π»Π΅Π½Π° вмСстС, Ρ‡Ρ‚ΠΎΠ±Ρ‹ ΠΏΠΎΠ»ΡƒΡ‡ΠΈΡ‚ΡŒ ΠΎΠ΄ΠΈΠ½ Ρ‡Π»Π΅Π½ Π΄Ρ€ΠΎΠ±ΠΈ Π² числитСлС большСй Π΄Ρ€ΠΎΠ±ΠΈ:

[латСкс] \ displaystyle \ frac {\ left (\ dfrac {1} {2} + \ dfrac {2} {3} \ right)} {\ left (\ dfrac {2} {3} \ cdot \ dfrac { 3} {4} \ right)} = \ dfrac {\ left (\ dfrac {3} {6} + \ dfrac {4} {6} \ right)} {\ left (\ dfrac {2} {3} \ cdot \ dfrac {3} {4} \ right)} = \ dfrac {\ left (\ dfrac {7} {6} \ right)} {\ left (\ dfrac {2} {3} \ cdot \ dfrac {3 } {4} \ right)} [/ латСкс]

ΠŸΠ΅Ρ€Π΅ΠΉΠ΄Π΅ΠΌ ΠΊ ΡˆΠ°Π³Ρƒ 2: объСдиним Ρ‡Π»Π΅Π½Ρ‹ Π² Π·Π½Π°ΠΌΠ΅Π½Π°Ρ‚Π΅Π»Π΅.Для этого ΠΌΡ‹ ΡƒΠΌΠ½ΠΎΠΆΠ°Π΅ΠΌ Π΄Ρ€ΠΎΠ±ΠΈ Π² Π·Π½Π°ΠΌΠ΅Π½Π°Ρ‚Π΅Π»Π΅ вмСстС ΠΈ ΡƒΠΏΡ€ΠΎΡ‰Π°Π΅ΠΌ Ρ€Π΅Π·ΡƒΠ»ΡŒΡ‚Π°Ρ‚, сокращая Π΅Π³ΠΎ Π΄ΠΎ Π½Π°ΠΈΠΌΠ΅Π½ΡŒΡˆΠΈΡ… Ρ‡Π»Π΅Π½ΠΎΠ²:

[латСкс] \ dfrac {\ left (\ dfrac {7} {6} \ right)} {\ left (\ dfrac {2} {3} \ cdot \ dfrac {3} {4} \ right)} = \ dfrac {\ left (\ dfrac {7} {6} \ right)} {\ left (\ dfrac {6} {12} \ right)} = \ dfrac {\ left (\ dfrac {7} {6} \ right )} {\ left (\ dfrac {1} {2} \ right)} [/ latex]

ΠŸΠ΅Ρ€Π΅ΠΉΠ΄Π΅ΠΌ ΠΊ ΡˆΠ°Π³Ρƒ 3: Ρ€Π°Π·Π΄Π΅Π»ΠΈΠΌ Ρ‡ΠΈΡΠ»ΠΈΡ‚Π΅Π»ΡŒ Π½Π° Π·Π½Π°ΠΌΠ΅Π½Π°Ρ‚Π΅Π»ΡŒ. Напомним, Ρ‡Ρ‚ΠΎ Π΄Π΅Π»Π΅Π½ΠΈΠ΅ Π½Π° Π΄Ρ€ΠΎΠ±ΡŒ Π°Π½Π°Π»ΠΎΠ³ΠΈΡ‡Π½ΠΎ ΡƒΠΌΠ½ΠΎΠΆΠ΅Π½ΠΈΡŽ Π½Π° ΠΎΠ±Ρ€Π°Ρ‚Π½ΡƒΡŽ Π²Π΅Π»ΠΈΡ‡ΠΈΠ½Ρƒ этой Π΄Ρ€ΠΎΠ±ΠΈ:

[латСкс] \ frac {\ left (\ dfrac {7} {6} \ right)} {\ left (\ dfrac {1} {2} \ right)} = {\ dfrac {7} {6}} \ cdot {\ dfrac {2} {1}} = \ dfrac {14} {6} [/ latex]

НаконСц, упростим ΠΏΠΎΠ»ΡƒΡ‡Π΅Π½Π½ΡƒΡŽ Π΄Ρ€ΠΎΠ±ΡŒ:

[латСкс] \ displaystyle \ frac {14} {6} = 2 \ frac {2} {6} [/ latex]

Π‘Π»Π΅Π΄ΠΎΠ²Π°Ρ‚Π΅Π»ΡŒΠ½ΠΎ, Π² ΠΈΡ‚ΠΎΠ³Π΅:

[латСкс] \ frac {\ left (\ dfrac {1} {2} + \ dfrac {2} {3} \ right)} {\ left (\ dfrac {2} {3} \ cdot \ dfrac {3} {4} \ right)} = 2 \ dfrac {2} {6} [/ latex]

Π’Π²Π΅Π΄Π΅Π½ΠΈΠ΅ Π² экспонСнты

Π­ΠΊΡΠΏΠΎΠ½Π΅Π½Ρ†ΠΈΠ°Π»ΡŒΠ½Π°Ρ Ρ„ΠΎΡ€ΠΌΠ°, записанная [latex] b ^ n [/ latex], прСдставляСт собой ΡƒΠΌΠ½ΠΎΠΆΠ΅Π½ΠΈΠ΅ Π±Π°Π·ΠΎΠ²ΠΎΠ³ΠΎ [latex] b [/ latex] Π½Π° сам [latex] n [/ latex] Ρ€Π°Π·. 5 = 3 \ cdot 3 \ cdot 3 \ cdot 3 \ cdot 3 = 243 [/ латСкс]

ΠŸΠΎΠΊΠ°Π·Π°Ρ‚Π΅Π»ΠΈ 0 ΠΈ 1

Π›ΡŽΠ±ΠΎΠ΅ число, Π²ΠΎΠ·Π²Π΅Π΄Π΅Π½Π½ΠΎΠ΅ Π² ΡΡ‚Π΅ΠΏΠ΅Π½ΡŒ [латСкс] 1 [/ латСкс], являСтся самим числом.0 = 1 [/ латСкс].

ΠŸΠΎΡ€ΡΠ΄ΠΎΠΊ дСйствий

ΠŸΠΎΡ€ΡΠ΄ΠΎΠΊ ΠΎΠΏΠ΅Ρ€Π°Ρ†ΠΈΠΉ — это ΠΏΠΎΠ΄Ρ…ΠΎΠ΄ ΠΊ ΠΎΡ†Π΅Π½ΠΊΠ΅ Π²Ρ‹Ρ€Π°ΠΆΠ΅Π½ΠΈΠΉ, Π²ΠΊΠ»ΡŽΡ‡Π°ΡŽΡ‰ΠΈΡ… нСсколько арифмСтичСских ΠΎΠΏΠ΅Ρ€Π°Ρ†ΠΈΠΉ.

Π¦Π΅Π»ΠΈ обучСния

Π Π°Π·Π»ΠΈΡ‡ΠΈΠ΅ ΠΌΠ΅ΠΆΠ΄Ρƒ ΠΏΡ€Π°Π²ΠΈΠ»ΡŒΠ½Ρ‹ΠΌ ΠΈ Π½Π΅ΠΏΡ€Π°Π²ΠΈΠ»ΡŒΠ½Ρ‹ΠΌ использованиСм порядка ΠΎΠΏΠ΅Ρ€Π°Ρ†ΠΈΠΉ

ΠšΠ»ΡŽΡ‡Π΅Π²Ρ‹Π΅ Π²Ρ‹Π²ΠΎΠ΄Ρ‹

ΠšΠ»ΡŽΡ‡Π΅Π²Ρ‹Π΅ ΠΌΠΎΠΌΠ΅Π½Ρ‚Ρ‹
  • ΠŸΠΎΡ€ΡΠ΄ΠΎΠΊ ΠΎΠΏΠ΅Ρ€Π°Ρ†ΠΈΠΉ ΠΏΡ€Π΅Π΄ΠΎΡ‚Π²Ρ€Π°Ρ‰Π°Π΅Ρ‚ Π΄Π²ΡƒΡΠΌΡ‹ΡΠ»Π΅Π½Π½ΠΎΡΡ‚ΡŒ матСматичСских Π²Ρ‹Ρ€Π°ΠΆΠ΅Π½ΠΈΠΉ.
  • ΠŸΠΎΡ€ΡΠ΄ΠΎΠΊ ΠΎΠΏΠ΅Ρ€Π°Ρ†ΠΈΠΉ ΡΠ»Π΅Π΄ΡƒΡŽΡ‰ΠΈΠΉ: 1) ΡƒΠΏΡ€ΠΎΡΡ‚ΠΈΡ‚ΡŒ Ρ‡Π»Π΅Π½Ρ‹ Π² ΠΊΡ€ΡƒΠ³Π»Ρ‹Ρ… ΠΈΠ»ΠΈ ΠΊΠ²Π°Π΄Ρ€Π°Ρ‚Π½Ρ‹Ρ… скобках, 2) ΡƒΠΏΡ€ΠΎΡΡ‚ΠΈΡ‚ΡŒ ΠΏΠΎΠΊΠ°Π·Π°Ρ‚Π΅Π»ΠΈ ΠΈ ΠΊΠΎΡ€Π½ΠΈ, 3) Π²Ρ‹ΠΏΠΎΠ»Π½ΠΈΡ‚ΡŒ ΡƒΠΌΠ½ΠΎΠΆΠ΅Π½ΠΈΠ΅ ΠΈ Π΄Π΅Π»Π΅Π½ΠΈΠ΅, 4) Π²Ρ‹ΠΏΠΎΠ»Π½ΠΈΡ‚ΡŒ слоТСниС ΠΈ Π²Ρ‹Ρ‡ΠΈΡ‚Π°Π½ΠΈΠ΅.
  • Π£ΠΌΠ½ΠΎΠΆΠ΅Π½ΠΈΠ΅ ΠΈ Π΄Π΅Π»Π΅Π½ΠΈΠ΅ ΠΈΠΌΠ΅ΡŽΡ‚ Ρ€Π°Π²Π½Ρ‹ΠΉ ΠΏΡ€ΠΈΠΎΡ€ΠΈΡ‚Π΅Ρ‚, Ρ€Π°Π²Π½ΠΎ ΠΊΠ°ΠΊ ΠΈ слоТСниС ΠΈ Π²Ρ‹Ρ‡ΠΈΡ‚Π°Π½ΠΈΠ΅. Π­Ρ‚ΠΎ ΠΎΠ·Π½Π°Ρ‡Π°Π΅Ρ‚, Ρ‡Ρ‚ΠΎ ΠΎΠΏΠ΅Ρ€Π°Ρ†ΠΈΠΈ умноТСния ΠΈ дСлСния (Π° Ρ‚Π°ΠΊΠΆΠ΅ ΠΎΠΏΠ΅Ρ€Π°Ρ†ΠΈΠΈ слоТСния ΠΈ вычитания) ΠΌΠΎΠ³ΡƒΡ‚ Π²Ρ‹ΠΏΠΎΠ»Π½ΡΡ‚ΡŒΡΡ Π² Ρ‚ΠΎΠΌ порядкС, Π² ΠΊΠΎΡ‚ΠΎΡ€ΠΎΠΌ ΠΎΠ½ΠΈ ΠΏΠΎΡΠ²Π»ΡΡŽΡ‚ΡΡ Π² Π²Ρ‹Ρ€Π°ΠΆΠ΅Π½ΠΈΠΈ.
  • ΠŸΠΎΠ»Π΅Π·Π½Ρ‹ΠΉ ΠΌΠ½Π΅ΠΌΠΎΠ½ΠΈΠΊ для запоминания порядка дСйствий — PEMDAS, ΠΈΠ½ΠΎΠ³Π΄Π° Ρ€Π°ΡΡˆΠΈΡ€ΡΠ΅ΠΌΡ‹ΠΉ Π΄ΠΎ Β«ΠŸΡ€ΠΎΡˆΡƒ прощСния, моя дорогая тСтя Π‘Π°Π»Π»ΠΈΒ».
ΠšΠ»ΡŽΡ‡Π΅Π²Ρ‹Π΅ Ρ‚Π΅Ρ€ΠΌΠΈΠ½Ρ‹
  • матСматичСская опСрация : дСйствиС ΠΈΠ»ΠΈ ΠΏΡ€ΠΎΡ†Π΅Π΄ΡƒΡ€Π°, которая создаСт Π½ΠΎΠ²ΠΎΠ΅ Π·Π½Π°Ρ‡Π΅Π½ΠΈΠ΅ ΠΈΠ· ΠΎΠ΄Π½ΠΎΠ³ΠΎ ΠΈΠ»ΠΈ Π½Π΅ΡΠΊΠΎΠ»ΡŒΠΊΠΈΡ… Π²Ρ…ΠΎΠ΄Π½Ρ‹Ρ… Π·Π½Π°Ρ‡Π΅Π½ΠΈΠΉ.

ΠŸΠΎΡ€ΡΠ΄ΠΎΠΊ ΠΎΠΏΠ΅Ρ€Π°Ρ†ΠΈΠΉ — это способ вычислСния Π²Ρ‹Ρ€Π°ΠΆΠ΅Π½ΠΈΠΉ, ΠΊΠΎΡ‚ΠΎΡ€Ρ‹Π΅ Π²ΠΊΠ»ΡŽΡ‡Π°ΡŽΡ‚ Π±ΠΎΠ»Π΅Π΅ ΠΎΠ΄Π½ΠΎΠΉ арифмСтичСской ΠΎΠΏΠ΅Ρ€Π°Ρ†ΠΈΠΈ. Π­Ρ‚ΠΈ ΠΏΡ€Π°Π²ΠΈΠ»Π° говорят Π²Π°ΠΌ, ΠΊΠ°ΠΊ Π²Π°ΠΌ слСдуСт ΡƒΠΏΡ€ΠΎΡΡ‚ΠΈΡ‚ΡŒ ΠΈΠ»ΠΈ Ρ€Π΅ΡˆΠΈΡ‚ΡŒ Π²Ρ‹Ρ€Π°ΠΆΠ΅Π½ΠΈΠ΅ ΠΈΠ»ΠΈ ΡƒΡ€Π°Π²Π½Π΅Π½ΠΈΠ΅ Ρ‚Π°ΠΊΠΈΠΌ ΠΎΠ±Ρ€Π°Π·ΠΎΠΌ, Ρ‡Ρ‚ΠΎΠ±Ρ‹ ΠΏΠΎΠ»ΡƒΡ‡ΠΈΡ‚ΡŒ ΠΏΡ€Π°Π²ΠΈΠ»ΡŒΠ½Ρ‹ΠΉ Ρ€Π΅Π·ΡƒΠ»ΡŒΡ‚Π°Ρ‚.

НапримСр, ΠΊΠΎΠ³Π΄Π° Π²Ρ‹ встрСтитС Π²Ρ‹Ρ€Π°ΠΆΠ΅Π½ΠΈΠ΅ [латСкс] 4 + 2 \ cdot 3 [/ latex], ΠΊΠ°ΠΊ Π²Ρ‹ поступитС?

Один Π²Π°Ρ€ΠΈΠ°Π½Ρ‚:

[латСкс] \ begin {align} \ displaystyle 4 + 2 \ cdot3 & = (4 + 2) \ cdot 3 \\ & = 6 \ cdot 3 \\ & = 18 \ end {align} [/ latex]

Π”Ρ€ΡƒΠ³ΠΎΠΉ Π²Π°Ρ€ΠΈΠ°Π½Ρ‚:

[латСкс] \ begin {align} \ displaystyle 4 + 2 \ cdot 3 & = 4+ (2 \ cdot 3) \\ & = 4 + 6 \\ & = 10 \ end {align} [/ latex]

Какой порядок дСйствий ΠΏΡ€Π°Π²ΠΈΠ»ΡŒΠ½Ρ‹ΠΉ?

Π§Ρ‚ΠΎΠ±Ρ‹ ΠΈΠΌΠ΅Ρ‚ΡŒ Π²ΠΎΠ·ΠΌΠΎΠΆΠ½ΠΎΡΡ‚ΡŒ ΠΎΠ±Ρ‰Π°Ρ‚ΡŒΡΡ с ΠΏΠΎΠΌΠΎΡ‰ΡŒΡŽ матСматичСских Π²Ρ‹Ρ€Π°ΠΆΠ΅Π½ΠΈΠΉ, Ρƒ нас Π΄ΠΎΠ»ΠΆΠ΅Π½ Π±Ρ‹Ρ‚ΡŒ согласованный порядок ΠΎΠΏΠ΅Ρ€Π°Ρ†ΠΈΠΉ, Ρ‡Ρ‚ΠΎΠ±Ρ‹ ΠΊΠ°ΠΆΠ΄ΠΎΠ΅ Π²Ρ‹Ρ€Π°ΠΆΠ΅Π½ΠΈΠ΅ Π±Ρ‹Π»ΠΎ ΠΎΠ΄Π½ΠΎΠ·Π½Π°Ρ‡Π½Ρ‹ΠΌ.НапримСр, для ΠΏΡ€ΠΈΠ²Π΅Π΄Π΅Π½Π½ΠΎΠ³ΠΎ Π²Ρ‹ΡˆΠ΅ выраТСния всС ΠΌΠ°Ρ‚Π΅ΠΌΠ°Ρ‚ΠΈΠΊΠΈ согласятся, Ρ‡Ρ‚ΠΎ ΠΏΡ€Π°Π²ΠΈΠ»ΡŒΠ½Ρ‹ΠΉ ΠΎΡ‚Π²Π΅Ρ‚ — 10.

ΠŸΠΎΡ€ΡΠ΄ΠΎΠΊ ΠΎΠΏΠ΅Ρ€Π°Ρ†ΠΈΠΉ, ΠΈΡΠΏΠΎΠ»ΡŒΠ·ΡƒΠ΅ΠΌΡ‹Ρ… Π² ΠΌΠ°Ρ‚Π΅ΠΌΠ°Ρ‚ΠΈΠΊΠ΅, Π½Π°ΡƒΠΊΠ΅, Ρ‚Π΅Ρ…Π½ΠΈΠΊΠ΅ ΠΈ ΠΌΠ½ΠΎΠ³ΠΈΡ… языках программирования, ΡΠ»Π΅Π΄ΡƒΡŽΡ‰ΠΈΠΉ:

  1. УпроститС Ρ‚Π΅Ρ€ΠΌΠΈΠ½Ρ‹ Π² ΠΊΡ€ΡƒΠ³Π»Ρ‹Ρ… ΠΈΠ»ΠΈ ΠΊΠ²Π°Π΄Ρ€Π°Ρ‚Π½Ρ‹Ρ… скобках
  2. Π£ΠΏΡ€ΠΎΡΡ‚ΠΈΡ‚ΡŒ экспонСнты ΠΈ ΠΊΠΎΡ€Π½ΠΈ
  3. Π’Ρ‹ΠΏΠΎΠ»Π½ΠΈΡ‚ΡŒ ΡƒΠΌΠ½ΠΎΠΆΠ΅Π½ΠΈΠ΅ ΠΈ Π΄Π΅Π»Π΅Π½ΠΈΠ΅
  4. Π’Ρ‹ΠΏΠΎΠ»Π½ΠΈΡ‚ΡŒ слоТСниС ΠΈ Π²Ρ‹Ρ‡ΠΈΡ‚Π°Π½ΠΈΠ΅

Π­Ρ‚ΠΈ ΠΏΡ€Π°Π²ΠΈΠ»Π° ΠΎΠ·Π½Π°Ρ‡Π°ΡŽΡ‚, Ρ‡Ρ‚ΠΎ Π² матСматичСском Π²Ρ‹Ρ€Π°ΠΆΠ΅Π½ΠΈΠΈ сначала Π΄ΠΎΠ»ΠΆΠ½Π° Π²Ρ‹ΠΏΠΎΠ»Π½ΡΡ‚ΡŒΡΡ опСрация, Π·Π°Π½ΠΈΠΌΠ°ΡŽΡ‰Π°Ρ Π½Π°ΠΈΠ²Ρ‹ΡΡˆΠ΅Π΅ мСсто Π² спискС.3 \\ & = 6-5 + 8 \\ & = 1 + 8 \\ & = 9 \ end {align} [/ latex]

ΠŸΡ€ΠΈΠΌΠ΅Ρ‡Π°Π½ΠΈΠ΅ ΠΎ Ρ€Π°Π²Π½ΠΎΠΉ приоритСтности

ΠŸΠΎΡΠΊΠΎΠ»ΡŒΠΊΡƒ ΡƒΠΌΠ½ΠΎΠΆΠ΅Π½ΠΈΠ΅ ΠΈ Π΄Π΅Π»Π΅Π½ΠΈΠ΅ ΠΈΠΌΠ΅ΡŽΡ‚ Ρ€Π°Π²Π½Ρ‹ΠΉ ΠΏΡ€ΠΈΠΎΡ€ΠΈΡ‚Π΅Ρ‚, ΠΌΠΎΠΆΠ΅Ρ‚ Π±Ρ‹Ρ‚ΡŒ ΠΏΠΎΠ»Π΅Π·Π½ΠΎ Π΄ΡƒΠΌΠ°Ρ‚ΡŒ ΠΎ Π΄Π΅Π»Π΅Π½ΠΈΠΈ Π½Π° число ΠΊΠ°ΠΊ ΠΎ ΡƒΠΌΠ½ΠΎΠΆΠ΅Π½ΠΈΠΈ Π½Π° ΠΎΠ±Ρ€Π°Ρ‚Π½ΡƒΡŽ Π²Π΅Π»ΠΈΡ‡ΠΈΠ½Ρƒ этого числа. Π’Π°ΠΊΠΈΠΌ ΠΎΠ±Ρ€Π°Π·ΠΎΠΌ, [латСкс] 3 \ div 4 = 3 \ cdot \ frac {1} {4} [/ latex]. Π”Ρ€ΡƒΠ³ΠΈΠΌΠΈ словами, частноС 3 ΠΈ 4 Ρ€Π°Π²Π½ΠΎ ΠΏΡ€ΠΎΠΈΠ·Π²Π΅Π΄Π΅Π½ΠΈΡŽ 3 ΠΈ [латСкс] \ frac {1} {4} [/ latex].

Π’ΠΎΡ‡Π½ΠΎ Ρ‚Π°ΠΊ ΠΆΠ΅, ΠΏΠΎΡΠΊΠΎΠ»ΡŒΠΊΡƒ слоТСниС ΠΈ Π²Ρ‹Ρ‡ΠΈΡ‚Π°Π½ΠΈΠ΅ ΠΈΠΌΠ΅ΡŽΡ‚ Ρ€Π°Π²Π½Ρ‹ΠΉ ΠΏΡ€ΠΈΠΎΡ€ΠΈΡ‚Π΅Ρ‚, ΠΌΡ‹ ΠΌΠΎΠΆΠ΅ΠΌ Π΄ΡƒΠΌΠ°Ρ‚ΡŒ ΠΎ Π²Ρ‹Ρ‡ΠΈΡ‚Π°Π½ΠΈΠΈ числа ΠΊΠ°ΠΊ ΠΎ слоТСнии ΠΎΡ‚Ρ€ΠΈΡ†Π°Ρ‚Π΅Π»ΡŒΠ½ΠΎΠ³ΠΎ числа.Π’Π°ΠΊΠΈΠΌ ΠΎΠ±Ρ€Π°Π·ΠΎΠΌ [латСкс] 3βˆ’4 = 3 + (- 4) [/ латСкс]. Π”Ρ€ΡƒΠ³ΠΈΠΌΠΈ словами, Ρ€Π°Π·Π½ΠΈΡ†Π° 3 ΠΈ 4 Ρ€Π°Π²Π½Π° суммС ΠΏΠΎΠ»ΠΎΠΆΠΈΡ‚Π΅Π»ΡŒΠ½Ρ‹Ρ… Ρ‚Ρ€Π΅Ρ… ΠΈ ΠΎΡ‚Ρ€ΠΈΡ†Π°Ρ‚Π΅Π»ΡŒΠ½Ρ‹Ρ… Ρ‡Π΅Ρ‚Ρ‹Ρ€Π΅Ρ….

ΠŸΡ€ΠΈ Ρ‚Π°ΠΊΠΎΠΌ ΠΏΠΎΠ½ΠΈΠΌΠ°Π½ΠΈΠΈ ΠΏΡ€Π΅Π΄ΡΡ‚Π°Π²ΡŒΡ‚Π΅ [латСкс] 1βˆ’3 + 7 [/ latex] ΠΊΠ°ΠΊ сумму 1, минус 3 ΠΈ 7, Π° Π·Π°Ρ‚Π΅ΠΌ слоТитС эти Ρ‚Π΅Ρ€ΠΌΠΈΠ½Ρ‹ вмСстС. Π’Π΅ΠΏΠ΅Ρ€ΡŒ, ΠΊΠΎΠ³Π΄Π° Π²Ρ‹ ΠΈΠ·ΠΌΠ΅Π½ΠΈΠ»ΠΈ структуру ΠΎΠΏΠ΅Ρ€Π°Ρ†ΠΈΠΉ, любой Π·Π°ΠΊΠ°Π· Π±ΡƒΠ΄Π΅Ρ‚ Ρ€Π°Π±ΠΎΡ‚Π°Ρ‚ΡŒ:

  • [латСкс] (1-3) + 7 = -2 + 7 = 5 [/ латСкс]
  • [латСкс] (7βˆ’3) + 1 = 4 + 1 = 5 [/ латСкс]

Π’Π°ΠΆΠ½ΠΎ ΡΠΎΡ…Ρ€Π°Π½ΡΡ‚ΡŒ ΠΎΡ‚Ρ€ΠΈΡ†Π°Ρ‚Π΅Π»ΡŒΠ½Ρ‹ΠΉ Π·Π½Π°ΠΊ с Π»ΡŽΠ±Ρ‹ΠΌ ΠΎΡ‚Ρ€ΠΈΡ†Π°Ρ‚Π΅Π»ΡŒΠ½Ρ‹ΠΌ числом (здСсь 3).

МнСмоника

Π’ БША Π°Π±Π±Ρ€Π΅Π²ΠΈΠ°Ρ‚ΡƒΡ€Π° PEMDAS являСтся распространСнным мнСмоничСским символом для запоминания порядка ΠΎΠΏΠ΅Ρ€Π°Ρ†ΠΈΠΉ. Π­Ρ‚ΠΎ ΠΎΠ·Π½Π°Ρ‡Π°Π΅Ρ‚ ΠΊΡ€ΡƒΠ³Π»Ρ‹Π΅ скобки, экспонСнты, ΡƒΠΌΠ½ΠΎΠΆΠ΅Π½ΠΈΠ΅, Π΄Π΅Π»Π΅Π½ΠΈΠ΅, слоТСниС ΠΈ Π²Ρ‹Ρ‡ΠΈΡ‚Π°Π½ΠΈΠ΅. PEMDAS часто Ρ€Π°ΡΡˆΠΈΡ€ΡΠ΅Ρ‚ΡΡ Π΄ΠΎ Β«ΠŸΡ€ΠΎΡˆΡƒ прощСния, моя дорогая тСтя Π‘Π°Π»Π»ΠΈΒ».

Π­Ρ‚Π° ΠΌΠ½Π΅ΠΌΠΎΠ½ΠΈΠΊΠ° ΠΌΠΎΠΆΠ΅Ρ‚ Π²Π²ΠΎΠ΄ΠΈΡ‚ΡŒ Π² Π·Π°Π±Π»ΡƒΠΆΠ΄Π΅Π½ΠΈΠ΅, ΠΎΠ΄Π½Π°ΠΊΠΎ, ΠΏΠΎΡΠΊΠΎΠ»ΡŒΠΊΡƒ Β«MDΒ» ΠΏΠΎΠ΄Ρ€Π°Π·ΡƒΠΌΠ΅Π²Π°Π΅Ρ‚, Ρ‡Ρ‚ΠΎ ΡƒΠΌΠ½ΠΎΠΆΠ΅Π½ΠΈΠ΅ Π΄ΠΎΠ»ΠΆΠ½ΠΎ Π²Ρ‹ΠΏΠΎΠ»Π½ΡΡ‚ΡŒΡΡ ΠΏΠ΅Ρ€Π΅Π΄ Π΄Π΅Π»Π΅Π½ΠΈΠ΅ΠΌ, Π° Β«ASΒ» — это слоТСниС ΠΏΠ΅Ρ€Π΅Π΄ Π²Ρ‹Ρ‡ΠΈΡ‚Π°Π½ΠΈΠ΅ΠΌ, Π° Π½Π΅ ΠΏΡ€ΠΈΠ·Π½Π°Π½ΠΈΠ΅ ΠΈΡ… Ρ€Π°Π²Π½ΠΎΠ³ΠΎ ΠΏΡ€ΠΈΠΎΡ€ΠΈΡ‚Π΅Ρ‚Π°.Π§Ρ‚ΠΎΠ±Ρ‹ ΠΏΡ€ΠΎΠΈΠ»Π»ΡŽΡΡ‚Ρ€ΠΈΡ€ΠΎΠ²Π°Ρ‚ΡŒ, ΠΏΠΎΡ‡Π΅ΠΌΡƒ это ΠΏΡ€ΠΎΠ±Π»Π΅ΠΌΠ°, рассмотрим ΡΠ»Π΅Π΄ΡƒΡŽΡ‰Π΅Π΅:

[латСкс] 10-3 + 2 [/ латСкс]

Π­Ρ‚ΠΎ Π²Ρ‹Ρ€Π°ΠΆΠ΅Π½ΠΈΠ΅ ΠΏΡ€Π°Π²ΠΈΠ»ΡŒΠ½ΠΎ упрощаСтся Π΄ΠΎ 9. Однако, Ссли Π²Ρ‹ сначала слоТитС 2 ΠΈ 3, Ρ‡Ρ‚ΠΎΠ±Ρ‹ ΠΏΠΎΠ»ΡƒΡ‡ΠΈΡ‚ΡŒ 5, ΠΈ , Π° Π·Π°Ρ‚Π΅ΠΌ Π²Ρ‹ΠΏΠΎΠ»Π½ΠΈΠ» Π²Ρ‹Ρ‡ΠΈΡ‚Π°Π½ΠΈΠ΅, Π²Ρ‹ ΠΏΠΎΠ»ΡƒΡ‡ΠΈΠ»ΠΈ Π±Ρ‹ 5 Π² качСствС ΠΎΠΊΠΎΠ½Ρ‡Π°Ρ‚Π΅Π»ΡŒΠ½ΠΎΠ³ΠΎ ΠΎΡ‚Π²Π΅Ρ‚Π°, Ρ‡Ρ‚ΠΎ Π½Π΅Π²Π΅Ρ€Π½ΠΎ. Π§Ρ‚ΠΎΠ±Ρ‹ ΠΈΠ·Π±Π΅ΠΆΠ°Ρ‚ΡŒ этой ошибки, Π»ΡƒΡ‡ΡˆΠ΅ всСго Ρ€Π°ΡΡΠΌΠ°Ρ‚Ρ€ΠΈΠ²Π°Ρ‚ΡŒ эту ΠΏΡ€ΠΎΠ±Π»Π΅ΠΌΡƒ ΠΊΠ°ΠΊ сумму ΠΏΠΎΠ»ΠΎΠΆΠΈΡ‚Π΅Π»ΡŒΠ½Ρ‹Ρ… дСсяти, ΠΎΡ‚Ρ€ΠΈΡ†Π°Ρ‚Π΅Π»ΡŒΠ½Ρ‹Ρ… Ρ‚Ρ€Π΅Ρ… ΠΈ ΠΏΠΎΠ»ΠΎΠΆΠΈΡ‚Π΅Π»ΡŒΠ½Ρ‹Ρ… Π΄Π²ΡƒΡ….

[латСкс] 10 + (- 3) +2 [/ латСкс]

Π§Ρ‚ΠΎΠ±Ρ‹ ΠΏΠΎΠ»Π½ΠΎΡΡ‚ΡŒΡŽ ΠΈΠ·Π±Π΅ΠΆΠ°Ρ‚ΡŒ этой ΠΏΡƒΡ‚Π°Π½ΠΈΡ†Ρ‹, Π°Π»ΡŒΡ‚Π΅Ρ€Π½Π°Ρ‚ΠΈΠ²Π½Ρ‹ΠΉ способ записи ΠΌΠ½Π΅ΠΌΠΎΠ½ΠΈΠΊΠΈ:

-P

E

MD

КАК

Или, просто ΠΊΠ°ΠΊ PEMA, Π³Π΄Π΅ ΡƒΡ‡Π°Ρ‚, Ρ‡Ρ‚ΠΎ ΡƒΠΌΠ½ΠΎΠΆΠ΅Π½ΠΈΠ΅ ΠΈ Π΄Π΅Π»Π΅Π½ΠΈΠ΅ ΠΏΠΎ своСй сути ΠΈΠΌΠ΅ΡŽΡ‚ ΠΎΠ΄ΠΈΠ½ ΠΈ Ρ‚ΠΎΡ‚ ΠΆΠ΅ ΠΏΡ€ΠΈΠΎΡ€ΠΈΡ‚Π΅Ρ‚, Π° слоТСниС ΠΈ Π²Ρ‹Ρ‡ΠΈΡ‚Π°Π½ΠΈΠ΅ ΠΏΠΎ своСй сути ΠΈΠΌΠ΅ΡŽΡ‚ ΠΎΠ΄ΠΈΠ½Π°ΠΊΠΎΠ²Ρ‹ΠΉ ΠΏΡ€ΠΈΠΎΡ€ΠΈΡ‚Π΅Ρ‚.Π­Ρ‚Π° ΠΌΠ½Π΅ΠΌΠΎΠ½ΠΈΠΊΠ° проясняСт ΡΠΊΠ²ΠΈΠ²Π°Π»Π΅Π½Ρ‚Π½ΠΎΡΡ‚ΡŒ умноТСния ΠΈ дСлСния, Π° Ρ‚Π°ΠΊΠΆΠ΅ слоТСния ΠΈ вычитания.

Π‘Π»ΠΎΠΆΠ΅Π½ΠΈΠ΅, Π²Ρ‹Ρ‡ΠΈΡ‚Π°Π½ΠΈΠ΅, ΡƒΠΌΠ½ΠΎΠΆΠ΅Π½ΠΈΠ΅ ΠΈ Π΄Π΅Π»Π΅Π½ΠΈΠ΅ Ρ†Π΅Π»Ρ‹Ρ… чисСл

Π¦Π΅Π»ΠΈ обучСния

  • Π˜ΡΠΏΠΎΠ»ΡŒΠ·ΡƒΠΉΡ‚Π΅ слоТСниС, Π²Ρ‹Ρ‡ΠΈΡ‚Π°Π½ΠΈΠ΅, ΡƒΠΌΠ½ΠΎΠΆΠ΅Π½ΠΈΠ΅ ΠΈ Π΄Π΅Π»Π΅Π½ΠΈΠ΅ ΠΏΡ€ΠΈ вычислСнии цСлочислСнных Π²Ρ‹Ρ€Π°ΠΆΠ΅Π½ΠΈΠΉ

Π Π°Π±ΠΎΡ‚Π° с Ρ†Π΅Π»Ρ‹ΠΌΠΈ числами ΠΈ Π²Ρ‹ΠΏΠΎΠ»Π½Π΅Π½ΠΈΠ΅ основных вычислСний — основа всСй ΠΌΠ°Ρ‚Π΅ΠΌΠ°Ρ‚ΠΈΠΊΠΈ. ΠŸΡ€Π΅Π΄ΠΏΠΎΠ»ΠΎΠΆΠΈΠΌ, Π²Ρ‹ ΠΏΠΎΠΌΠ½ΠΈΡ‚Π΅, ΠΊΠ°ΠΊ Π²Ρ‹ΠΏΠΎΠ»Π½ΡΡ‚ΡŒ слоТСниС, Π²Ρ‹Ρ‡ΠΈΡ‚Π°Π½ΠΈΠ΅, ΡƒΠΌΠ½ΠΎΠΆΠ΅Π½ΠΈΠ΅ ΠΈ Π΄Π΅Π»Π΅Π½ΠΈΠ΅ ΠΎΠ΄Π½ΠΎΠΉ Ρ†ΠΈΡ„Ρ€Ρ‹.Для выполнСния этих расчСтов Ρƒ вас часто Π±ΡƒΠ΄Π΅Ρ‚ ΠΏΠΎΠ΄ Ρ€ΡƒΠΊΠΎΠΉ ΠΊΠ°Π»ΡŒΠΊΡƒΠ»ΡΡ‚ΠΎΡ€, Π½ΠΎ быстроС освСТСниС Π·Π½Π°Π½ΠΈΠΉ ΠΏΠΎΠΌΠΎΠΆΠ΅Ρ‚ Π²Π°ΠΌ Π»ΡƒΡ‡ΡˆΠ΅ ΠΏΠΎΠ½ΡΡ‚ΡŒ, ΠΊΠ°ΠΊ Ρ€Π°Π±ΠΎΡ‚Π°Ρ‚ΡŒ с числами, Ρ‡Ρ‚ΠΎΠ±Ρ‹ слоТныС уравнСния Π±Ρ‹Π»ΠΈ ΠΌΠ΅Π½Π΅Π΅ слоТными.

Π”ΠΎΠΏΠΎΠ»Π½Π΅Π½ΠΈΠ΅

ΠΏΡ€ΠΈΠΌΠ΅Ρ€

Π”ΠΎΠ±Π°Π²ΡŒΡ‚Π΅: [латСкс] 28 + 61 [/ латСкс]

РСшСниС
Π§Ρ‚ΠΎΠ±Ρ‹ ΡΠ»ΠΎΠΆΠΈΡ‚ΡŒ числа, состоящиС ΠΈΠ· Π±ΠΎΠ»Π΅Π΅ Ρ‡Π΅ΠΌ ΠΎΠ΄Π½ΠΎΠΉ Ρ†ΠΈΡ„Ρ€Ρ‹, часто ΠΏΡ€ΠΎΡ‰Π΅ Π·Π°ΠΏΠΈΡΠ°Ρ‚ΡŒ числа Π² Π²Π΅Ρ€Ρ‚ΠΈΠΊΠ°Π»ΡŒΠ½ΠΎΠΌ Π½Π°ΠΏΡ€Π°Π²Π»Π΅Π½ΠΈΠΈ Π² столбцы.

ΠΠ°ΠΏΠΈΡˆΠΈΡ‚Π΅ числа Ρ‚Π°ΠΊ, Ρ‡Ρ‚ΠΎΠ±Ρ‹ Ρ†ΠΈΡ„Ρ€Ρ‹ Π΅Π΄ΠΈΠ½ΠΈΡ† ΠΈ дСсятков Ρ€Π°ΡΠΏΠΎΠ»Π°Π³Π°Π»ΠΈΡΡŒ Π²Π΅Ρ€Ρ‚ΠΈΠΊΠ°Π»ΡŒΠ½ΠΎ. [латСкс] \ begin {array} {c} \ hfill 28 \\ \\ \ hfill \ underset {\ text {____}} {+ 61} \ end {array} [/ latex]
Π—Π°Ρ‚Π΅ΠΌ Π΄ΠΎΠ±Π°Π²ΡŒΡ‚Π΅ Ρ†ΠΈΡ„Ρ€Ρ‹ Π² ΠΊΠ°ΠΆΠ΄ΠΎΠ΅ разрядноС Π·Π½Π°Ρ‡Π΅Π½ΠΈΠ΅.

Π‘ΠΊΠ»Π°Π΄Ρ‹Π²Π°Π΅ΠΌ Π΅Π΄ΠΈΠ½ΠΈΡ†Ρ‹: [латСкс] 8 + 1 = 9 [/ латСкс]

Π‘ΠΊΠ»Π°Π΄Ρ‹Π²Π°Π΅ΠΌ дСсятки: [латСкс] 2 + 6 = 8 [/ латСкс]

[латСкс] \ begin {array} {c} \ hfill 28 \\ \\ \ hfill \ underset {\ text {____}} {+ 61} \\ \ hfill 89 \ end {array} [/ latex]

Π’ ΠΏΡ€Π΅Π΄Ρ‹Π΄ΡƒΡ‰Π΅ΠΌ ΠΏΡ€ΠΈΠΌΠ΅Ρ€Π΅ сумма Π΅Π΄ΠΈΠ½ΠΈΡ† ΠΈ сумма дСсятков Π±Ρ‹Π»ΠΈ мСньшС [латСкс] 10 [/ латСкс].Но Ρ‡Ρ‚ΠΎ Π±ΡƒΠ΄Π΅Ρ‚, Ссли сумма [латСкс] 10 [/ латСкс] ΠΈΠ»ΠΈ большС? Π”Π°Π²Π°ΠΉΡ‚Π΅ Π²ΠΎΡΠΏΠΎΠ»ΡŒΠ·ΡƒΠ΅ΠΌΡΡ нашСй модСлью base- [latex] 10 [/ latex], Ρ‡Ρ‚ΠΎΠ±Ρ‹ Π²Ρ‹ΡΡΠ½ΠΈΡ‚ΡŒ это.

На рисункС Π½ΠΈΠΆΠ΅ снова ΠΏΠΎΠΊΠ°Π·Π°Π½ΠΎ Π΄ΠΎΠ±Π°Π²Π»Π΅Π½ΠΈΠ΅ [латСкса] 17 [/ латСкса] ΠΈ [латСкса] 26 [/ латСкса].

Когда ΠΌΡ‹ складываСм Ρ‚Π΅, [latex] 7 + 6 [/ latex], ΠΌΡ‹ ΠΏΠΎΠ»ΡƒΡ‡Π°Π΅ΠΌ [latex] 13 [/ latex]. ΠŸΠΎΡΠΊΠΎΠ»ΡŒΠΊΡƒ Ρƒ нас большС, Ρ‡Π΅ΠΌ [латСкс] 10 [/ латСкс], ΠΌΡ‹ ΠΌΠΎΠΆΠ΅ΠΌ ΠΎΠ±ΠΌΠ΅Π½ΡΡ‚ΡŒ [латСкс] 10 [/ латСкс] Π½Π° Π΄Π΅ΡΡΡ‚ΡŒ [латСкс] 1 [/ латСкс]. Π’Π΅ΠΏΠ΅Ρ€ΡŒ Ρƒ нас Π΅ΡΡ‚ΡŒ [латСксныС] 4 [/ латСксныС] дСсятки ΠΈ [латСксныС] 3 [/ латСксныС] дСсятки.НС ΠΈΡΠΏΠΎΠ»ΡŒΠ·ΡƒΡ модСль, ΠΌΡ‹ ΠΏΠΎΠΊΠ°Π·Ρ‹Π²Π°Π΅ΠΌ это ΠΊΠ°ΠΊ малСнький красный [латСкс] 1 [/ латСкс] Π½Π°Π΄ Ρ†ΠΈΡ„Ρ€Π°ΠΌΠΈ Π² разрядС дСсятков.

Когда сумма Π² столбцС Π·Π½Π°Ρ‡Π΅Π½ΠΈΠΉ разряда большС [latex] 9 [/ latex], ΠΌΡ‹ пСрСносимся Π² ΡΠ»Π΅Π΄ΡƒΡŽΡ‰ΠΈΠΉ столбСц слСва. ΠŸΠ΅Ρ€Π΅Π½ΠΎΡ — это Ρ‚ΠΎ ΠΆΠ΅ самоС, Ρ‡Ρ‚ΠΎ ΠΏΠ΅Ρ€Π΅Π³Ρ€ΡƒΠΏΠΏΠΈΡ€ΠΎΠ²ΠΊΠ° ΠΏΡƒΡ‚Π΅ΠΌ ΠΎΠ±ΠΌΠ΅Π½Π°. НапримСр, [латСкс] 10 [/ латСкс] Π΅Π΄ΠΈΠ½ΠΈΡ† Π·Π° [латСкс] 1 [/ латСкс] дСсятку ΠΈΠ»ΠΈ [латСкс] 10 [/ латСкс] дСсяток Π·Π° [латСкс] 1 [/ латСкс] ΡΠΎΡ‚Π½ΡŽ.

Π‘Π»ΠΎΠΆΠΈΡ‚ΡŒ Ρ†Π΅Π»Ρ‹Π΅ числа

  1. ΠΠ°ΠΏΠΈΡˆΠΈΡ‚Π΅ числа Ρ‚Π°ΠΊ, Ρ‡Ρ‚ΠΎΠ±Ρ‹ ΠΊΠ°ΠΆΠ΄ΠΎΠ΅ разрядноС Π·Π½Π°Ρ‡Π΅Π½ΠΈΠ΅ Ρ€Π°ΡΠΏΠΎΠ»Π°Π³Π°Π»ΠΎΡΡŒ Π²Π΅Ρ€Ρ‚ΠΈΠΊΠ°Π»ΡŒΠ½ΠΎ.
  2. Π”ΠΎΠ±Π°Π²ΡŒΡ‚Π΅ Ρ†ΠΈΡ„Ρ€Ρ‹ Π² ΠΊΠ°ΠΆΠ΄ΠΎΠ΅ разрядноС Π·Π½Π°Ρ‡Π΅Π½ΠΈΠ΅. Π Π°Π±ΠΎΡ‚Π°ΠΉΡ‚Π΅ справа Π½Π°Π»Π΅Π²ΠΎ, начиная с Π΅Π΄ΠΈΠ½ΠΈΡ†. Если сумма Π² разрядС большС [latex] 9 [/ latex], пСрСносится ΠΊ ΡΠ»Π΅Π΄ΡƒΡŽΡ‰Π΅ΠΌΡƒ разряду.
  3. ΠŸΡ€ΠΎΠ΄ΠΎΠ»ΠΆΠ°ΠΉΡ‚Π΅ Π΄ΠΎΠ±Π°Π²Π»ΡΡ‚ΡŒ ΠΊΠ°ΠΆΠ΄ΠΎΠ΅ разрядноС Π·Π½Π°Ρ‡Π΅Π½ΠΈΠ΅ справа Π½Π°Π»Π΅Π²ΠΎ, добавляя ΠΊΠ°ΠΆΠ΄ΠΎΠ΅ разрядноС Π·Π½Π°Ρ‡Π΅Π½ΠΈΠ΅ ΠΈ пСрСнося, Ссли Π½Π΅ΠΎΠ±Ρ…ΠΎΠ΄ΠΈΠΌΠΎ.

ΠΏΡ€ΠΈΠΌΠ΅Ρ€

Π”ΠΎΠ±Π°Π²ΡŒΡ‚Π΅: [латСкс] 43 + 69 [/ латСкс]

ПокаТи ΠΎΡ‚Π²Π΅Ρ‚

РСшСниС

ΠΠ°ΠΏΠΈΡˆΠΈΡ‚Π΅ числа Ρ‚Π°ΠΊ, Ρ‡Ρ‚ΠΎΠ±Ρ‹ Ρ†ΠΈΡ„Ρ€Ρ‹ Π²Ρ‹ΡΡ‚Ρ€Π°ΠΈΠ²Π°Π»ΠΈΡΡŒ Π²Π΅Ρ€Ρ‚ΠΈΠΊΠ°Π»ΡŒΠ½ΠΎ. [латСкс] \ begin {array} {c} \ hfill 43 \\ \\ \ hfill \ underset {\ text {____}} {+ 69} \ end {array} [/ latex]
Π”ΠΎΠ±Π°Π²ΡŒΡ‚Π΅ Ρ†ΠΈΡ„Ρ€Ρ‹ Π² ΠΊΠ°ΠΆΠ΄ΠΎΠΌ мСстС.

Π‘ΠΊΠ»Π°Π΄Ρ‹Π²Π°Π΅ΠΌ Π΅Π΄ΠΈΠ½ΠΈΡ†Ρ‹: [латСкс] 3 + 9 = 12 [/ латСкс]

Π—Π°ΠΏΠΈΡˆΠΈΡ‚Π΅ [латСкс] 2 [/ латСкс] Π² разрядах Π΅Π΄ΠΈΠ½ΠΈΡ† суммы.

Π”ΠΎΠ±Π°Π²ΡŒΡ‚Π΅ [latex] 1 [/ latex] дСсятку ΠΊ разряду дСсятков.

[латСкс] \ begin {массив} {c} \ hfill \ stackrel {1} ​​{4} 3 \\ \ hfill \ underset {\ text {____}} {+ 69} \\ \ hfill 2 \ end {array} [/ латСкс]
Π’Π΅ΠΏΠ΅Ρ€ΡŒ слоТитС дСсятки: [латСкс] 1 + 4 + 6 = 11 [/ латСкс]

Π—Π°ΠΏΠΈΡˆΠΈΡ‚Π΅ 11 Π² суммС.

[латСкс] \ begin {array} {c} \ hfill \ stackrel {1} ​​{4} 3 \\ \ hfill \ underset {\ text {____}} {+ 69} \\ \ hfill 112 \ end {array} [/ латСкс]

Если слагаСмыС содСрТат Ρ€Π°Π·Π½ΠΎΠ΅ количСство Ρ†ΠΈΡ„Ρ€, Π±ΡƒΠ΄ΡŒΡ‚Π΅ остороТны, Ρ‡Ρ‚ΠΎΠ±Ρ‹ Π²Ρ‹Ρ€ΠΎΠ²Π½ΡΡ‚ΡŒ ΡΠΎΠΎΡ‚Π²Π΅Ρ‚ΡΡ‚Π²ΡƒΡŽΡ‰ΠΈΠ΅ разрядныС значСния, начиная с Π΅Π΄ΠΈΠ½ΠΈΡ† ΠΈ двигаясь Π²Π»Π΅Π²ΠΎ.

ΠΏΡ€ΠΈΠΌΠ΅Ρ€

Π”ΠΎΠ±Π°Π²ΠΈΡ‚ΡŒ: [латСкс] 1,683 + 479 [/ латСкс].

ПокаТи ΠΎΡ‚Π²Π΅Ρ‚

РСшСниС

ΠΠ°ΠΏΠΈΡˆΠΈΡ‚Π΅ числа Ρ‚Π°ΠΊ, Ρ‡Ρ‚ΠΎΠ±Ρ‹ Ρ†ΠΈΡ„Ρ€Ρ‹ Π²Ρ‹ΡΡ‚Ρ€Π°ΠΈΠ²Π°Π»ΠΈΡΡŒ Π²Π΅Ρ€Ρ‚ΠΈΠΊΠ°Π»ΡŒΠ½ΠΎ. [латСкс] \ begin {array} {c} \ hfill 1,683 \\ \\ \ hfill \ underset {\ text {______}} {+ 479} \ end {array} [/ latex]
Π‘Π»ΠΎΠΆΠΈΡ‚Π΅ Ρ†ΠΈΡ„Ρ€Ρ‹ Π² ΠΊΠ°ΠΆΠ΄ΠΎΠΌ Π·Π½Π°Ρ‡Π΅Π½ΠΈΠΈ разряда.
Π‘ΠΊΠ»Π°Π΄Ρ‹Π²Π°Π΅ΠΌ Π΅Π΄ΠΈΠ½ΠΈΡ†Ρ‹: [латСкс] 3 + 9 = 12 [/ латСкс].

ΠΠ°ΠΏΠΈΡˆΠΈΡ‚Π΅ [latex] 2 [/ latex] Π² разрядС Π΅Π΄ΠΈΠ½ΠΈΡ† суммы ΠΈ пСрСнСситС [latex] 1 [/ latex] Π΄Π΅ΡΡΡ‚ΡŒ Π² разряды дСсятков.

[латСкс] \ begin {array} {c} \ hfill 1,6 \ stackrel {1} ​​{8} 3 \\ \\ \ hfill \ underset {\ text {______}} {+ 479} \\ \ hfill 2 \ end {array} [/ latex]
Π‘ΠΊΠ»Π°Π΄Ρ‹Π²Π°Π΅ΠΌ дСсятки: [латСкс] 1 + 7 + 8 = 16 [/ латСкс]

ΠΠ°ΠΏΠΈΡˆΠΈΡ‚Π΅ [латСкс] 6 [/ латСкс] Π² разрядС дСсятков ΠΈ отнСситС [латСкс] 1 [/ латСкс] ΡΠΎΡ‚Π½ΡŽ ΠΊ разряду сотСн.

[латСкс] \ begin {array} {c} \ hfill 1, \ stackrel {1} ​​{6} \ stackrel {1} ​​{8} 3 \\ \\ \ hfill \ underset {\ text {______}} {+ 479} \\ \ hfill 62 \ end {array} [/ latex]
Π‘ΠΊΠ»Π°Π΄Ρ‹Π²Π°Π΅ΠΌ сотни: [латСкс] 1 + 6 + 4 = 11 [/ латСкс]

ΠΠ°ΠΏΠΈΡˆΠΈΡ‚Π΅ [латСкс] 1 [/ латСкс] Π² разрядС сотСн ΠΈ пСрСнСситС [латСкс] 1 [/ латСкс] тысячу Π² разряды тысяч.

[латСкс] \ begin {array} {c} \ hfill \ stackrel {1} ​​{1}, \ stackrel {1} ​​{6} \ stackrel {1} ​​{8} 3 \\ \\ \ hfill \ underset {\ тСкст {______}} {+ 479} \\ \ hfill 162 \ end {array} [/ latex]
Π‘Π»ΠΎΠΆΠΈΡ‚Π΅ тысячи [латСкс] 1 + 1 = 2 [/ латСкс].

ΠΠ°ΠΏΠΈΡˆΠΈΡ‚Π΅ [латСкс] 2 [/ латСкс] Π² разрядС тысяч суммы.

[латСкс] \ begin {array} {c} \ hfill \ stackrel {1} ​​{1}, \ stackrel {1} ​​{6} \ stackrel {1} ​​{8} 3 \\ \\ \ hfill \ underset {\ тСкст {______}} {+ 479} \\ \ hfill 2,162 \ end {array} [/ latex]

ΠŸΠΎΡΠΌΠΎΡ‚Ρ€ΠΈΡ‚Π΅ Π²ΠΈΠ΄Π΅ΠΎ Π½ΠΈΠΆΠ΅, Ρ‡Ρ‚ΠΎΠ±Ρ‹ ΡƒΠ²ΠΈΠ΄Π΅Ρ‚ΡŒ Π΅Ρ‰Π΅ ΠΎΠ΄ΠΈΠ½ ΠΏΡ€ΠΈΠΌΠ΅Ρ€ Ρ‚ΠΎΠ³ΠΎ, ΠΊΠ°ΠΊ ΡΠ»ΠΎΠΆΠΈΡ‚ΡŒ Ρ‚Ρ€ΠΈ Ρ†Π΅Π»Ρ‹Ρ… числа, выровняв разряды.

Π’Ρ‹Ρ‡ΠΈΡ‚Π°Π½ΠΈΠ΅

Π‘Π»ΠΎΠΆΠ΅Π½ΠΈΠ΅ ΠΈ Π²Ρ‹Ρ‡ΠΈΡ‚Π°Π½ΠΈΠ΅ — ΠΎΠ±Ρ€Π°Ρ‚Π½Ρ‹Π΅ ΠΎΠΏΠ΅Ρ€Π°Ρ†ΠΈΠΈ. Π‘Π»ΠΎΠΆΠ΅Π½ΠΈΠ΅ отмСняСт Π²Ρ‹Ρ‡ΠΈΡ‚Π°Π½ΠΈΠ΅, Π° Π²Ρ‹Ρ‡ΠΈΡ‚Π°Π½ΠΈΠ΅ отмСняСт слоТСниС.
ΠœΡ‹ Π·Π½Π°Π΅ΠΌ [латСкс] 7 — 3 = 4 [/ latex], ΠΏΠΎΡ‚ΠΎΠΌΡƒ Ρ‡Ρ‚ΠΎ [latex] 4 + 3 = 7 [/ latex]. Π—Π½Π°Π½ΠΈΠ΅ всСх Ρ„Π°ΠΊΡ‚ΠΎΠ² слоТСния чисСл ΠΏΠΎΠΌΠΎΠΆΠ΅Ρ‚ ΠΏΡ€ΠΈ Π²Ρ‹Ρ‡ΠΈΡ‚Π°Π½ΠΈΠΈ. Π—Π°Ρ‚Π΅ΠΌ ΠΌΡ‹ ΠΌΠΎΠΆΠ΅ΠΌ ΠΏΡ€ΠΎΠ²Π΅Ρ€ΠΈΡ‚ΡŒ Π²Ρ‹Ρ‡ΠΈΡ‚Π°Π½ΠΈΠ΅, Π΄ΠΎΠ±Π°Π²ΠΈΠ². Π’ ΠΏΡ€ΠΈΠ²Π΅Π΄Π΅Π½Π½Ρ‹Ρ… Π²Ρ‹ΡˆΠ΅ ΠΏΡ€ΠΈΠΌΠ΅Ρ€Π°Ρ… наши вычитания ΠΌΠΎΠΆΠ½ΠΎ ΠΏΡ€ΠΎΠ²Π΅Ρ€ΠΈΡ‚ΡŒ слоТСниСм.

[латСкс] 7-3 = 4 [/ латСкс] ΠΏΠΎΡ‚ΠΎΠΌΡƒ Ρ‡Ρ‚ΠΎ [латСкс] 4 + 3 = 7 [/ латСкс]
[латСкс] 13-8 = 5 [/ латСкс] ΠΏΠΎΡ‚ΠΎΠΌΡƒ Ρ‡Ρ‚ΠΎ [латСкс] 5 + 8 = 13 [/ латСкс]
[латСкс] 43-26 = 17 [/ латСкс] ΠΏΠΎΡ‚ΠΎΠΌΡƒ Ρ‡Ρ‚ΠΎ [латСкс] 17 + 26 = 43 [/ латСкс]

Для вычитания чисСл, состоящих ΠΈΠ· Π±ΠΎΠ»Π΅Π΅ Ρ‡Π΅ΠΌ ΠΎΠ΄Π½ΠΎΠΉ Ρ†ΠΈΡ„Ρ€Ρ‹, ΠΎΠ±Ρ‹Ρ‡Π½ΠΎ ΠΏΡ€ΠΎΡ‰Π΅ Π·Π°ΠΏΠΈΡΡ‹Π²Π°Ρ‚ΡŒ числа Π²Π΅Ρ€Ρ‚ΠΈΠΊΠ°Π»ΡŒΠ½ΠΎ Π² столбцы, ΠΊΠ°ΠΊ ΠΌΡ‹ это Π΄Π΅Π»Π°Π»ΠΈ для слоТСния.ВыровняйтС Ρ†ΠΈΡ„Ρ€Ρ‹ ΠΏΠΎ разряду, Π° Π·Π°Ρ‚Π΅ΠΌ Π²Ρ‹Ρ‡Ρ‚ΠΈΡ‚Π΅ ΠΊΠ°ΠΆΠ΄Ρ‹ΠΉ столбСц, начиная с Π΅Π΄ΠΈΠ½ΠΈΡ†, Π° Π·Π°Ρ‚Π΅ΠΌ двигаясь Π²Π»Π΅Π²ΠΎ.

Π£ΠΏΡ€Π°ΠΆΠ½Π΅Π½ΠΈΠ΅

Π’Ρ‹Ρ‡Ρ‚ΠΈΡ‚Π΅ ΠΈ ΠΏΡ€ΠΎΠ²Π΅Ρ€ΡŒΡ‚Π΅, Π΄ΠΎΠ±Π°Π²ΠΈΠ²: [латСкс] 89 — 61 [/ латСкс].

ПокаТи ΠΎΡ‚Π²Π΅Ρ‚

РСшСниС

ΠΠ°ΠΏΠΈΡˆΠΈΡ‚Π΅ числа Ρ‚Π°ΠΊ, Ρ‡Ρ‚ΠΎΠ±Ρ‹ Ρ†ΠΈΡ„Ρ€Ρ‹ Π΅Π΄ΠΈΠ½ΠΈΡ† ΠΈ дСсятков Ρ€Π°ΡΠΏΠΎΠ»Π°Π³Π°Π»ΠΈΡΡŒ Π²Π΅Ρ€Ρ‚ΠΈΠΊΠ°Π»ΡŒΠ½ΠΎ. [латСкс] \ begin {array} {c} \ hfill 89 \\ \ hfill \ underset {\ text {____}} {- 61} \ end {array} [/ latex]
Π’Ρ‹Ρ‡Ρ‚ΠΈΡ‚Π΅ Ρ†ΠΈΡ„Ρ€Ρ‹ Π² ΠΊΠ°ΠΆΠ΄ΠΎΠΌ Π·Π½Π°Ρ‡Π΅Π½ΠΈΠΈ разряда.

Π’Ρ‹Ρ‡Ρ‚ΠΈΡ‚Π΅ Π΅Π΄ΠΈΠ½ΠΈΡ†Ρ‹: [латСкс] 9 — 1 = 8 [/ латСкс]

Π’Ρ‹Ρ‡Ρ‚ΠΈΡ‚Π΅ дСсятки: [латСкс] 8–6 = 2 [/ латСкс]

[латСкс] \ begin {array} {c} \ hfill 89 \\ \ hfill \ underset {\ text {____}} {- 61} \\ \ hfill 28 \ end {array} [/ latex]
ΠŸΡ€ΠΎΠ²Π΅Ρ€ΠΈΡ‚ΡŒ с ΠΏΠΎΠΌΠΎΡ‰ΡŒΡŽ дополнСния.

[латСкс] \ begin {массив} {c} \ hfill 28 \\ \ hfill \ underset {\ text {____}} {+ 61} \\ \ hfill 89 \ end {array} \ quad \ checkmark [/ latex]

Наш ΠΎΡ‚Π²Π΅Ρ‚ ΠΏΡ€Π°Π²ΠΈΠ»ΡŒΠ½Ρ‹ΠΉ.

Π’Ρ‹Ρ‡Π΅ΡΡ‚ΡŒ Ρ†Π΅Π»Ρ‹Π΅ числа

  1. ΠΠ°ΠΏΠΈΡˆΠΈΡ‚Π΅ числа Ρ‚Π°ΠΊ, Ρ‡Ρ‚ΠΎΠ±Ρ‹ ΠΊΠ°ΠΆΠ΄ΠΎΠ΅ разрядноС Π·Π½Π°Ρ‡Π΅Π½ΠΈΠ΅ Ρ€Π°ΡΠΏΠΎΠ»Π°Π³Π°Π»ΠΎΡΡŒ Π²Π΅Ρ€Ρ‚ΠΈΠΊΠ°Π»ΡŒΠ½ΠΎ.
  2. Π’Ρ‹Ρ‡Ρ‚ΠΈΡ‚Π΅ Ρ†ΠΈΡ„Ρ€Ρ‹ Π² ΠΊΠ°ΠΆΠ΄ΠΎΠΌ Π·Π½Π°Ρ‡Π΅Π½ΠΈΠΈ разряда. Π Π°Π±ΠΎΡ‚Π°ΠΉΡ‚Π΅ справа Π½Π°Π»Π΅Π²ΠΎ, начиная с Π΅Π΄ΠΈΠ½ΠΈΡ†. Если Ρ†ΠΈΡ„Ρ€Π° Π²Π²Π΅Ρ€Ρ…Ρƒ мСньшС Ρ†ΠΈΡ„Ρ€Ρ‹ Π²Π½ΠΈΠ·Ρƒ, ΠΏΡ€ΠΈ нСобходимости заимствуйтС.
  3. ΠŸΡ€ΠΎΠ΄ΠΎΠ»ΠΆΠ°ΠΉΡ‚Π΅ Π²Ρ‹Ρ‡ΠΈΡ‚Π°Ρ‚ΡŒ ΠΊΠ°ΠΆΠ΄ΠΎΠ΅ разрядноС Π·Π½Π°Ρ‡Π΅Π½ΠΈΠ΅ справа Π½Π°Π»Π΅Π²ΠΎ, ΠΏΡ€ΠΈ нСобходимости заимствуя.
  4. ΠŸΡ€ΠΎΠ²Π΅Ρ€ΠΈΡ‚ΡŒ Π΄ΠΎΠ±Π°Π²Π»Π΅Π½ΠΈΠ΅ΠΌ.

ΡƒΠΏΡ€Π°ΠΆΠ½Π΅Π½ΠΈΠ΅

Π’Ρ‹Ρ‡Ρ‚ΠΈΡ‚Π΅: [латСкс] 43 — 26 [/ латСкс].

Π’ ΠΏΡ€ΠΈΠ²Π΅Π΄Π΅Π½Π½ΠΎΠΌ Π²Ρ‹ΡˆΠ΅ ΠΏΡ€ΠΈΠΌΠ΅Ρ€Π΅, Ссли ΠΌΡ‹ ΠΌΠΎΠ΄Π΅Π»ΠΈΡ€ΡƒΠ΅ΠΌ Π²Ρ‹Ρ‡ΠΈΡ‚Π°Π½ΠΈΠ΅ [latex] 26 [/ latex] ΠΈΠ· [latex] 43 [/ latex], ΠΌΡ‹ Π±Ρ‹ Π·Π°ΠΌΠ΅Π½ΠΈΠ»ΠΈ [latex] 1 [/ latex] Π΄Π΅ΡΡΡ‚ΡŒ Π½Π° [latex] 10 [/ latex]. .Когда ΠΌΡ‹ Π΄Π΅Π»Π°Π΅ΠΌ это Π±Π΅Π· ΠΌΠΎΠ΄Π΅Π»Π΅ΠΉ, ΠΌΡ‹ Π³ΠΎΠ²ΠΎΡ€ΠΈΠΌ, Ρ‡Ρ‚ΠΎ заимствуСм [latex] 1 [/ latex] ΠΈΠ· разряда дСсятков ΠΈ добавляСм [latex] 10 [/ latex] Π² разряды Π΅Π΄ΠΈΠ½ΠΈΡ†.

Π£ΠΏΡ€Π°ΠΆΠ½Π΅Π½ΠΈΠ΅

Π’Ρ‹Ρ‡Ρ‚ΠΈΡ‚Π΅ ΠΈ ΠΏΡ€ΠΎΠ²Π΅Ρ€ΡŒΡ‚Π΅, Π΄ΠΎΠ±Π°Π²ΠΈΠ²: [латСкс] 207 — 64 [/ латСкс].

Π£ΠΏΡ€Π°ΠΆΠ½Π΅Π½ΠΈΠ΅

Π’Ρ‹Ρ‡Ρ‚ΠΈΡ‚Π΅ ΠΈ ΠΏΡ€ΠΎΠ²Π΅Ρ€ΡŒΡ‚Π΅, Π΄ΠΎΠ±Π°Π²ΠΈΠ²: [латСкс] 2,162 — 479 [/ латСкс].

ПокаТи ΠΎΡ‚Π²Π΅Ρ‚

РСшСниС

Наш ΠΎΡ‚Π²Π΅Ρ‚ ΠΏΡ€Π°Π²ΠΈΠ»ΡŒΠ½Ρ‹ΠΉ.

ΠŸΠΎΡΠΌΠΎΡ‚Ρ€ΠΈΡ‚Π΅ Π²ΠΈΠ΄Π΅ΠΎ Π½ΠΈΠΆΠ΅, Ρ‡Ρ‚ΠΎΠ±Ρ‹ ΡƒΠ²ΠΈΠ΄Π΅Ρ‚ΡŒ Π΅Ρ‰Π΅ ΠΎΠ΄ΠΈΠ½ ΠΏΡ€ΠΈΠΌΠ΅Ρ€ вычитания Ρ†Π΅Π»Ρ‹Ρ… чисСл ΠΏΡƒΡ‚Π΅ΠΌ выравнивания Π·Π½Π°Ρ‡Π΅Π½ΠΈΠΉ разряда.

Π£ΠΌΠ½ΠΎΠΆΠ΅Π½ΠΈΠ΅

Π§Ρ‚ΠΎΠ±Ρ‹ ΡƒΠΌΠ½ΠΎΠΆΠ°Ρ‚ΡŒ Π±Π΅Π· использования ΠΌΠΎΠ΄Π΅Π»Π΅ΠΉ, Π²Π°ΠΌ Π½Π΅ΠΎΠ±Ρ…ΠΎΠ΄ΠΈΠΌΠΎ Π·Π½Π°Ρ‚ΡŒ всС Ρ„Π°ΠΊΡ‚Ρ‹ ΠΎΠ΄Π½ΠΎΠ·Π½Π°Ρ‡Π½ΠΎΠ³ΠΎ умноТСния. Π£Π±Π΅Π΄ΠΈΡ‚Π΅ΡΡŒ, Ρ‡Ρ‚ΠΎ Π²Ρ‹ Ρ…ΠΎΡ€ΠΎΡˆΠΎ ΠΈΡ… Π·Π½Π°Π΅Ρ‚Π΅, ΠΏΡ€Π΅ΠΆΠ΄Π΅ Ρ‡Π΅ΠΌ ΠΏΠ΅Ρ€Π΅Ρ…ΠΎΠ΄ΠΈΡ‚ΡŒ ΠΊ этому Ρ€Π°Π·Π΄Π΅Π»Ρƒ. Π’ Ρ‚Π°Π±Π»ΠΈΡ†Π΅ Π½ΠΈΠΆΠ΅ ΠΏΠΎΠΊΠ°Π·Π°Π½Ρ‹ Ρ„Π°ΠΊΡ‚Ρ‹ умноТСния.

КаТдоС ΠΏΠΎΠ»Π΅ ΠΏΠΎΠΊΠ°Π·Ρ‹Π²Π°Π΅Ρ‚ ΠΏΡ€ΠΎΠΈΠ·Π²Π΅Π΄Π΅Π½ΠΈΠ΅ числа Π² Π»Π΅Π²ΠΎΠΌ столбцС ΠΈ числа Π² Π²Π΅Ρ€Ρ…Π½Π΅ΠΌ ряду. Если Π²Ρ‹ Π½Π΅ ΡƒΠ²Π΅Ρ€Π΅Π½Ρ‹ Π² ΠΏΡ€ΠΎΠ΄ΡƒΠΊΡ‚Π΅, смодСлируйтС Π΅Π³ΠΎ. Π’Π°ΠΆΠ½ΠΎ, Ρ‡Ρ‚ΠΎΠ±Ρ‹ Π²Ρ‹ Π·Π°ΠΏΠΎΠΌΠ½ΠΈΠ»ΠΈ Π»ΡŽΠ±Ρ‹Π΅ числовыС Ρ„Π°ΠΊΡ‚Ρ‹, ΠΊΠΎΡ‚ΠΎΡ€Ρ‹Ρ… Π²Ρ‹ Π΅Ρ‰Π΅ Π½Π΅ Π·Π½Π°Π΅Ρ‚Π΅, Ρ‡Ρ‚ΠΎΠ±Ρ‹ Π²Ρ‹ Π±Ρ‹Π»ΠΈ Π³ΠΎΡ‚ΠΎΠ²Ρ‹ ΡƒΠΌΠ½ΠΎΠΆΠ°Ρ‚ΡŒ большиС числа.

[латСкс] x [/ латСкс] [латСкс] 0 [/ латСкс] [латСкс] 1 [/ латСкс] [латСкс] 2 [/ латСкс] [латСкс] 3 [/ латСкс] [латСкс] 4 [/ латСкс] [латСкс] 5 [/ латСкс] [латСкс] 6 [/ латСкс] [латСкс] 7 [/ латСкс] [латСкс] 8 [/ латСкс] [латСкс] 9 [/ латСкс]
[латСкс] 0 [/ латСкс] [латСкс] 0 [/ латСкс] [латСкс] 0 [/ латСкс] [латСкс] 0 [/ латСкс] [латСкс] 0 [/ латСкс] [латСкс] 0 [/ латСкс] [латСкс] 0 [/ латСкс] [латСкс] 0 [/ латСкс] [латСкс] 0 [/ латСкс] [латСкс] 0 [/ латСкс] [латСкс] 0 [/ латСкс]
[латСкс] 1 [/ латСкс] [латСкс] 0 [/ латСкс] [латСкс] 1 [/ латСкс] [латСкс] 2 [/ латСкс] [латСкс] 3 [/ латСкс] [латСкс] 4 [/ латСкс] [латСкс] 5 [/ латСкс] [латСкс] 6 [/ латСкс] [латСкс] 7 [/ латСкс] [латСкс] 8 [/ латСкс] [латСкс] 9 [/ латСкс]
[латСкс] 2 [/ латСкс] [латСкс] 0 [/ латСкс] [латСкс] 2 [/ латСкс] [латСкс] 4 [/ латСкс] [латСкс] 6 [/ латСкс] [латСкс] 8 [/ латСкс] [латСкс] 10 [/ латСкс] [латСкс] 12 [/ латСкс] [латСкс] 14 [/ латСкс] [латСкс] 16 [/ латСкс] [латСкс] 18 [/ латСкс]
[латСкс] 3 [/ латСкс] [латСкс] 0 [/ латСкс] [латСкс] 3 [/ латСкс] [латСкс] 6 [/ латСкс] [латСкс] 9 [/ латСкс] [латСкс] 12 [/ латСкс] [латСкс] 15 [/ латСкс] [латСкс] 18 [/ латСкс] [латСкс] 21 [/ латСкс] [латСкс] 24 [/ латСкс] [латСкс] 27 [/ латСкс]
[латСкс] 4 [/ латСкс] [латСкс] 0 [/ латСкс] [латСкс] 4 [/ латСкс] [латСкс] 8 [/ латСкс] [латСкс] 12 [/ латСкс] [латСкс] 16 [/ латСкс] [латСкс] 20 [/ латСкс] [латСкс] 24 [/ латСкс] [латСкс] 28 [/ латСкс] [латСкс] 32 [/ латСкс] [латСкс] 36 [/ латСкс]
[латСкс] 5 [/ латСкс] [латСкс] 0 [/ латСкс] [латСкс] 5 [/ латСкс] [латСкс] 10 [/ латСкс] [латСкс] 15 [/ латСкс] [латСкс] 20 [/ латСкс] [латСкс] 25 [/ латСкс] [латСкс] 30 [/ латСкс] [латСкс] 35 [/ латСкс] [латСкс] 40 [/ латСкс] [латСкс] 45 [/ латСкс]
[латСкс] 6 [/ латСкс] [латСкс] 0 [/ латСкс] [латСкс] 6 [/ латСкс] [латСкс] 12 [/ латСкс] [латСкс] 18 [/ латСкс] [латСкс] 24 [/ латСкс] [латСкс] 30 [/ латСкс] [латСкс] 36 [/ латСкс] [латСкс] 42 [/ латСкс] [латСкс] 48 [/ латСкс] [латСкс] 54 [/ латСкс]
[латСкс] 7 [/ латСкс] [латСкс] 0 [/ латСкс] [латСкс] 7 [/ латСкс] [латСкс] 14 [/ латСкс] [латСкс] 21 [/ латСкс] [латСкс] 28 [/ латСкс] [латСкс] 35 [/ латСкс] [латСкс] 42 [/ латСкс] [латСкс] 49 [/ латСкс] [латСкс] 56 [/ латСкс] [латСкс] 63 [/ латСкс]
[латСкс] 8 [/ латСкс] [латСкс] 0 [/ латСкс] [латСкс] 8 [/ латСкс] [латСкс] 16 [/ латСкс] [латСкс] 24 [/ латСкс] [латСкс] 32 [/ латСкс] [латСкс] 40 [/ латСкс] [латСкс] 48 [/ латСкс] [латСкс] 56 [/ латСкс] [латСкс] 64 [/ латСкс] [латСкс] 72 [/ латСкс]
[латСкс] 9 [/ латСкс] [латСкс] 0 [/ латСкс] [латСкс] 9 [/ латСкс] [латСкс] 18 [/ латСкс] [латСкс] 27 [/ латСкс] [латСкс] 36 [/ латСкс] [латСкс] 45 [/ латСкс] [латСкс] 54 [/ латСкс] [латСкс] 63 [/ латСкс] [латСкс] 72 [/ латСкс] [латСкс] 81 [/ латСкс]

ΠœΡ‹ Π·Π½Π°Π΅ΠΌ, Ρ‡Ρ‚ΠΎ ΠΈΠ·ΠΌΠ΅Π½Π΅Π½ΠΈΠ΅ порядка слоТСния Π½Π΅ мСняСт суммы.ΠœΡ‹ Π²ΠΈΠ΄Π΅Π»ΠΈ, Ρ‡Ρ‚ΠΎ [латСкс] 8 + 9 = 17 [/ латСкс] Ρ‚ΠΎ ΠΆΠ΅ самоС, Ρ‡Ρ‚ΠΎ [латСкс] 9 + 8 = 17 [/ латСкс].

Π’Π΅Ρ€Π½ΠΎ Π»ΠΈ это ΠΈ для умноТСния? Π”Π°Π²Π°ΠΉΡ‚Π΅ посмотрим Π½Π° нСсколько ΠΏΠ°Ρ€ Ρ„Π°ΠΊΡ‚ΠΎΡ€ΠΎΠ².

[латСкс] 4 \ cdot 7 = 28 \ quad 7 \ cdot 4 = 28 [/ латСкс]
[латСкс] 9 \ cdot 7 = 63 \ quad 7 \ cdot 9 = 63 [/ латСкс]
[латСкс] 8 \ cdot 9 = 72 \ quad 9 \ cdot 8 = 72 [/ латСкс]

ΠŸΡ€ΠΈ ΠΎΠ±Ρ€Π°Ρ‚Π½ΠΎΠΌ порядкС Ρ„Π°ΠΊΡ‚ΠΎΡ€ΠΎΠ² ΠΏΡ€ΠΎΠ΄ΡƒΠΊΡ‚ Π½Π΅ измСняСтся. Π­Ρ‚ΠΎ называСтся ΠΊΠΎΠΌΠΌΡƒΡ‚Π°Ρ‚ΠΈΠ²Π½Ρ‹ΠΌ свойством умноТСния.

ΠšΠΎΠΌΠΌΡƒΡ‚Π°Ρ‚ΠΈΠ²Π½ΠΎΠ΅ свойство умноТСния

ИзмСнСниС порядка Ρ„Π°ΠΊΡ‚ΠΎΡ€ΠΎΠ² Π½Π΅ мСняСт ΠΈΡ… произвСдСния.

[латСкс] a \ cdot b = b \ cdot a [/ латСкс]

ΠΏΡ€ΠΈΠΌΠ΅Ρ€

Π£ΠΌΠ½ΠΎΠΆΠΈΡ‚ΡŒ:

[латСкс] 8 \ cdot 7 [/ латСкс]
[латСкс] 7 \ cdot 8 [/ латСкс]

ПокаТи ΠΎΡ‚Π²Π΅Ρ‚

РСшСниС:

1. [латСкс] 8 \ cdot 7 [/ латСкс]
Π£ΠΌΠ½ΠΎΠΆΠΈΡ‚ΡŒ. [латСкс] 56 [/ латСкс]
2. [латСкс] 7 \ cdot 8 [/ латСкс]
Π£ΠΌΠ½ΠΎΠΆΠΈΡ‚ΡŒ. [латСкс] 56 [/ латСкс]

ИзмСнСниС порядка Ρ„Π°ΠΊΡ‚ΠΎΡ€ΠΎΠ² Π½Π΅ ΠΏΡ€ΠΈΠ²ΠΎΠ΄ΠΈΡ‚ ΠΊ измСнСнию ΠΏΡ€ΠΎΠ΄ΡƒΠΊΡ‚Π°.

Π§Ρ‚ΠΎΠ±Ρ‹ ΡƒΠΌΠ½ΠΎΠΆΠ°Ρ‚ΡŒ числа, состоящиС Π±ΠΎΠ»Π΅Π΅ Ρ‡Π΅ΠΌ ΠΈΠ· ΠΎΠ΄Π½ΠΎΠΉ Ρ†ΠΈΡ„Ρ€Ρ‹, ΠΎΠ±Ρ‹Ρ‡Π½ΠΎ ΠΏΡ€ΠΎΡ‰Π΅ Π·Π°ΠΏΠΈΡΡ‹Π²Π°Ρ‚ΡŒ числа Π²Π΅Ρ€Ρ‚ΠΈΠΊΠ°Π»ΡŒΠ½ΠΎ Π² столбцы, ΠΊΠ°ΠΊ ΠΌΡ‹ это Π΄Π΅Π»Π°Π»ΠΈ для слоТСния ΠΈ вычитания.

[латСкс] \ begin {array} {c} \ hfill 27 \\ \ hfill \ underset {\ text {___}} {\ times 3} \ end {array} [/ latex]

НачнСм с умноТСния [латСкс] 3 [/ латСкс] Π½Π° [латСкс] 7 [/ латСкс].

[латСкс] 3 \ Ρ€Π°Π· 7 = 21 [/ латСкс]

ΠœΡ‹ пишСм [латСкс] 1 [/ латСкс] Π½Π° Π΅Π΄ΠΈΠ½ΠΈΡ†Π°Ρ… Ρ‚ΠΎΠ²Π°Ρ€Π°. ΠŸΠ΅Ρ€Π΅Π½ΠΎΡΠΈΠΌ [latex] 2 [/ latex] дСсятки, написав [latex] 2 [/ latex] Π½Π°Π΄ разрядами дСсятков.


Π—Π°Ρ‚Π΅ΠΌ ΠΌΡ‹ ΡƒΠΌΠ½ΠΎΠΆΠ°Π΅ΠΌ [latex] 3 [/ latex] Π½Π° [latex] 2 [/ latex] ΠΈ добавляСм [latex] 2 [/ latex] Π½Π°Π΄ разрядами дСсятков ΠΊ ΠΏΡ€ΠΎΠ΄ΡƒΠΊΡ‚Ρƒ. Π˜Ρ‚Π°ΠΊ, [латСкс] 3 \ times 2 = 6 [/ латСкс], ΠΈ [латСкс] 6 + 2 = 8 [/ латСкс]. ΠΠ°ΠΏΠΈΡˆΠΈΡ‚Π΅ [латСкс] 8 [/ латСкс] Π² разрядС дСсятков ΠΏΡ€ΠΎΠ΄ΡƒΠΊΡ‚Π°.


ИздСлиС [латСкс] 81 [/ латСкс].

Когда ΠΌΡ‹ ΡƒΠΌΠ½ΠΎΠΆΠ°Π΅ΠΌ Π΄Π²Π° числа Π½Π° Ρ€Π°Π·Π½ΠΎΠ΅ количСство Ρ†ΠΈΡ„Ρ€, ΠΎΠ±Ρ‹Ρ‡Π½ΠΎ ΠΏΡ€ΠΎΡ‰Π΅ Π½Π°ΠΏΠΈΡΠ°Ρ‚ΡŒ мСньшСС число Π²Π½ΠΈΠ·Ρƒ. МоТно Π±Ρ‹Π»ΠΎ Π±Ρ‹ Π½Π°ΠΏΠΈΡΠ°Ρ‚ΡŒ ΠΈ ΠΏΠΎ-Π΄Ρ€ΡƒΠ³ΠΎΠΌΡƒ, Π½ΠΎ с этим ΠΏΡ€ΠΎΡ‰Π΅ Ρ€Π°Π±ΠΎΡ‚Π°Ρ‚ΡŒ.

ΠΏΡ€ΠΈΠΌΠ΅Ρ€

Π£ΠΌΠ½ΠΎΠΆΠ΅Π½ΠΈΠ΅: [латСкс] 15 \ cdot 4 [/ латСкс]

ПокаТи ΠΎΡ‚Π²Π΅Ρ‚

РСшСниС

ΠΠ°ΠΏΠΈΡˆΠΈΡ‚Π΅ числа Ρ‚Π°ΠΊ, Ρ‡Ρ‚ΠΎΠ±Ρ‹ Ρ†ΠΈΡ„Ρ€Ρ‹ [латСкс] 5 [/ латСкс] ΠΈ [латСкс] 4 [/ латСкс] Π²Ρ‹ΡΡ‚Ρ€ΠΎΠΈΠ»ΠΈΡΡŒ Π²Π΅Ρ€Ρ‚ΠΈΠΊΠ°Π»ΡŒΠ½ΠΎ. [латСкс] \ begin {array} {c} \ hfill 15 \\ \ hfill \ underset {\ text {_____}} {\ times 4} \ end {array} [/ latex]
Π£ΠΌΠ½ΠΎΠΆΡŒΡ‚Π΅ [латСкс] 4 [/ латСкс] Π½Π° Ρ†ΠΈΡ„Ρ€Ρƒ Π² разрядС Π΅Π΄ΠΈΠ½ΠΈΡ† [латСкс] 15 [/ латСкс]. [латСкс] 4 \ cdot 5 = 20 [/ латСкс].
ΠΠ°ΠΏΠΈΡˆΠΈΡ‚Π΅ [латСкс] 0 [/ латСкс] Π² Π΅Π΄ΠΈΠ½ΠΈΡ†Π°Ρ… издСлия ΠΈ нСситС [латСкс] 2 [/ латСкс] дСсятки. [латСкс] \ begin {массив} {c} \ hfill \ stackrel {2} {1} 5 \\ \ hfill \ underset {\ text {_____}} {\ times 4} \\ \ hfill 0 \ end {массив } [/ latex]
Π£ΠΌΠ½ΠΎΠΆΡŒΡ‚Π΅ [латСкс] 4 [/ латСкс] Π½Π° Ρ†ΠΈΡ„Ρ€Ρƒ Π² разрядС дСсятков [латСкс] 15 [/ латСкс]. [латСкс] 4 \ cdot 1 = 4 [/ латСкс].

Π”ΠΎΠ±Π°Π²ΡŒΡ‚Π΅ [латСкс] 2 [/ латСкс] дСсятки, ΠΊΠΎΡ‚ΠΎΡ€Ρ‹Π΅ ΠΌΡ‹ нСсли. [латСкс] 4 + 2 = 6 [/ латСкс].

ΠΠ°ΠΏΠΈΡˆΠΈΡ‚Π΅ [латСкс] 6 [/ латСкс] Π² разрядС дСсятков ΠΏΡ€ΠΎΠ΄ΡƒΠΊΡ‚Π°. [латСкс] \ begin {array} {c} \ hfill \ stackrel {2} {1} 5 \\ \ hfill \ underset {\ text {_____}} {\ times 4} \\ \ hfill 60 \ end {массив } [/ latex]

ΠΏΡ€ΠΈΠΌΠ΅Ρ€

Π£ΠΌΠ½ΠΎΠΆΠ΅Π½ΠΈΠ΅: [латСкс] 286 \ cdot 5 [/ латСкс]

ПокаТи ΠΎΡ‚Π²Π΅Ρ‚

РСшСниС

ΠΠ°ΠΏΠΈΡˆΠΈΡ‚Π΅ числа Ρ‚Π°ΠΊ, Ρ‡Ρ‚ΠΎΠ±Ρ‹ Ρ†ΠΈΡ„Ρ€Ρ‹ [латСкс] 5 [/ латСкс] ΠΈ [латСкс] 6 [/ латСкс] Π²Ρ‹ΡΡ‚Ρ€ΠΎΠΈΠ»ΠΈΡΡŒ Π²Π΅Ρ€Ρ‚ΠΈΠΊΠ°Π»ΡŒΠ½ΠΎ. [латСкс] \ begin {array} {c} \ hfill 286 \\ \ hfill \ underset {\ text {_____}} {\ times 5} \ end {array} [/ latex]
Π£ΠΌΠ½ΠΎΠΆΡŒΡ‚Π΅ [латСкс] 5 [/ латСкс] Π½Π° Ρ†ΠΈΡ„Ρ€Ρƒ Π² разрядС Π΅Π΄ΠΈΠ½ΠΈΡ† [латСкс] 286 [/ латСкс].

[латСкс] 5 \ cdot 6 = 30 [/ латСкс]

ΠΠ°ΠΏΠΈΡˆΠΈΡ‚Π΅ [латСкс] 0 [/ латСкс] Π½Π° мСстС Π΅Π΄ΠΈΠ½ΠΈΡ†Ρ‹ ΠΏΡ€ΠΎΠ΄ΡƒΠΊΡ‚Π° ΠΈ пСрСнСситС [латСкс] 3 [/ латСкс] Π² разряд дСсятков. Π£ΠΌΠ½ΠΎΠΆΡŒΡ‚Π΅ [латСкс] 5 [/ латСкс] Π½Π° Ρ†ΠΈΡ„Ρ€Ρƒ Π² дСсятки Ρ€Π°Π· [латСкс] 286 [/ латСкс].

[латСкс] 5 \ cdot 8 = 40 [/ латСкс]

[латСкс] \ begin {массив} {} \\ \ hfill 2 \ stackrel {3} {8} 6 \\ \ hfill \ underset {\ text {_____}} {\ times 5} \\ \ hfill 0 \ end {array} [/ latex]
Π”ΠΎΠ±Π°Π²ΡŒΡ‚Π΅ [латСкс] 3 [/ латСкс] дСсятки, ΠΊΠΎΡ‚ΠΎΡ€Ρ‹Π΅ ΠΌΡ‹ принСсли, Ρ‡Ρ‚ΠΎΠ±Ρ‹ ΠΏΠΎΠ»ΡƒΡ‡ΠΈΡ‚ΡŒ [латСкс] 40 + 3 = 43 [/ латСкс].

ΠΠ°ΠΏΠΈΡˆΠΈΡ‚Π΅ [латСкс] 3 [/ латСкс] Π² разрядС дСсятков ΠΏΡ€ΠΎΠ΄ΡƒΠΊΡ‚Π° ΠΈ отнСситС [латСкс] 4 [/ латСкс] ΠΊ разряду сотСн.

[латСкс] \ begin {array} {c} \ hfill \ stackrel {4} {2} \ stackrel {3} {8} 6 \\ \ hfill \ underset {\ text {_____}} {\ times 5} \ \ \ hfill 30 \ end {array} [/ latex]
Π£ΠΌΠ½ΠΎΠΆΡŒΡ‚Π΅ [латСкс] 5 [/ латСкс] Π½Π° Ρ†ΠΈΡ„Ρ€Ρƒ Π² разрядС сотСн [латСкс] 286 [/ латСкс]. [латСкс] 5 \ cdot 2 = 10 [/ латСкс].

Π”ΠΎΠ±Π°Π²ΡŒΡ‚Π΅ [латСкс] 4 [/ латСкс] сотни, ΠΊΠΎΡ‚ΠΎΡ€Ρ‹Π΅ ΠΌΡ‹ принСсли, Ρ‡Ρ‚ΠΎΠ±Ρ‹ ΠΏΠΎΠ»ΡƒΡ‡ΠΈΡ‚ΡŒ [латСкс] 10 + 4 = 14 [/ латСкс].

ΠΠ°ΠΏΠΈΡˆΠΈΡ‚Π΅ [латСкс] 4 [/ латСкс] Π² разрядС сотСн ΠΏΡ€ΠΎΠ΄ΡƒΠΊΡ‚Π° ΠΈ [латСкс] 1 [/ латСкс] Π² разрядС тысяч.

[латСкс] \ begin {array} {c} \ hfill \ stackrel {4} {2} \ stackrel {3} {8} 6 \\ \ hfill \ underset {\ text {_____}} {\ times 5} \ \ \ hfill 1,430 \ end {array} [/ latex]

Когда ΠΌΡ‹ ΡƒΠΌΠ½ΠΎΠΆΠ°Π΅ΠΌ Π½Π° число, состоящСС ΠΈΠ· Π΄Π²ΡƒΡ… ΠΈΠ»ΠΈ Π±ΠΎΠ»Π΅Π΅ Ρ†ΠΈΡ„Ρ€, ΠΌΡ‹ ΡƒΠΌΠ½ΠΎΠΆΠ°Π΅ΠΌ Π½Π° ΠΊΠ°ΠΆΠ΄ΡƒΡŽ ΠΈΠ· Ρ†ΠΈΡ„Ρ€ ΠΎΡ‚Π΄Π΅Π»ΡŒΠ½ΠΎ, работая справа Π½Π°Π»Π΅Π²ΠΎ. КаТдоС ΠΎΡ‚Π΄Π΅Π»ΡŒΠ½ΠΎΠ΅ ΠΏΡ€ΠΎΠΈΠ·Π²Π΅Π΄Π΅Π½ΠΈΠ΅ Ρ†ΠΈΡ„Ρ€ называСтся частичным ΠΏΡ€ΠΎΠΈΠ·Π²Π΅Π΄Π΅Π½ΠΈΠ΅ΠΌ.Когда ΠΌΡ‹ пишСм частичныС ΠΏΡ€ΠΎΠ΄ΡƒΠΊΡ‚Ρ‹, ΠΌΡ‹ Π΄ΠΎΠ»ΠΆΠ½Ρ‹ ΡƒΠ±Π΅Π΄ΠΈΡ‚ΡŒΡΡ, Ρ‡Ρ‚ΠΎ Π²Ρ‹Ρ€ΠΎΠ²Π½Π΅Π½Ρ‹ значСния ΠΏΠΎΠ·ΠΈΡ†ΠΈΠΉ.

Π£ΠΌΠ½ΠΎΠΆΠ΅Π½ΠΈΠ΅ Ρ†Π΅Π»Ρ‹Ρ… чисСл

  1. ΠΠ°ΠΏΠΈΡˆΠΈΡ‚Π΅ числа Ρ‚Π°ΠΊ, Ρ‡Ρ‚ΠΎΠ±Ρ‹ ΠΊΠ°ΠΆΠ΄ΠΎΠ΅ разрядноС Π·Π½Π°Ρ‡Π΅Π½ΠΈΠ΅ Ρ€Π°ΡΠΏΠΎΠ»Π°Π³Π°Π»ΠΎΡΡŒ Π²Π΅Ρ€Ρ‚ΠΈΠΊΠ°Π»ΡŒΠ½ΠΎ.
  2. Π£ΠΌΠ½ΠΎΠΆΡŒΡ‚Π΅ Ρ†ΠΈΡ„Ρ€Ρ‹ Π² ΠΊΠ°ΠΆΠ΄ΠΎΠΌ Π·Π½Π°Ρ‡Π΅Π½ΠΈΠΈ разряда.
    • Π Π°Π±ΠΎΡ‚Π°ΠΉΡ‚Π΅ справа Π½Π°Π»Π΅Π²ΠΎ, начиная с Π΅Π΄ΠΈΠ½ΠΈΡ† Π² Π½ΠΈΠΆΠ½Π΅ΠΌ ряду.
      • Π£ΠΌΠ½ΠΎΠΆΡŒΡ‚Π΅ Π½ΠΈΠΆΠ½Π΅Π΅ число Π½Π° Ρ†ΠΈΡ„Ρ€Ρƒ Π΅Π΄ΠΈΠ½ΠΈΡ† Π² Π²Π΅Ρ€Ρ…Π½Π΅ΠΌ числС, Π·Π°Ρ‚Π΅ΠΌ Π½Π° Ρ†ΠΈΡ„Ρ€Ρƒ дСсятков ΠΈ Ρ‚Π°ΠΊ Π΄Π°Π»Π΅Π΅.
      • Если ΠΏΡ€ΠΎΠ΄ΡƒΠΊΡ‚ Π² разрядС большС, Ρ‡Π΅ΠΌ [латСкс] 9 [/ латСкс], ΠΏΠ΅Ρ€Π΅Ρ…ΠΎΠ΄ΠΈΡ‚ΡŒ ΠΊ ΡΠ»Π΅Π΄ΡƒΡŽΡ‰Π΅ΠΌΡƒ разряду.
      • ΠΠ°ΠΏΠΈΡˆΠΈΡ‚Π΅ частичныС ΠΏΡ€ΠΎΠ΄ΡƒΠΊΡ‚Ρ‹, выровняв Ρ†ΠΈΡ„Ρ€Ρ‹ Π² ΠΏΠΎΠ·ΠΈΡ†ΠΈΠΎΠ½Π½Ρ‹Ρ… значСниях с числами, ΡƒΠΊΠ°Π·Π°Π½Π½Ρ‹ΠΌΠΈ Π²Ρ‹ΡˆΠ΅.
    • ΠŸΠΎΠ²Ρ‚ΠΎΡ€ΠΈΡ‚Π΅ эти дСйствия для разряда дСсятков Π² Π½ΠΈΠΆΠ½Π΅ΠΌ числС, разряда сотСн ΠΈ Ρ‚Π°ΠΊ Π΄Π°Π»Π΅Π΅.
    • ВставляйтС ноль Π² качСствС заполнитСля с ΠΊΠ°ΠΆΠ΄Ρ‹ΠΌ Π΄ΠΎΠΏΠΎΠ»Π½ΠΈΡ‚Π΅Π»ΡŒΠ½Ρ‹ΠΌ частичным ΠΏΡ€ΠΎΠ΄ΡƒΠΊΡ‚ΠΎΠΌ.
  3. Π”ΠΎΠ±Π°Π²ΡŒΡ‚Π΅ частичныС ΠΏΡ€ΠΎΠ΄ΡƒΠΊΡ‚Ρ‹.

ΠΏΡ€ΠΈΠΌΠ΅Ρ€

Π£ΠΌΠ½ΠΎΠΆΠ΅Π½ΠΈΠ΅: [латСкс] 62 \ Π²Π»Π΅Π²ΠΎ (87 \ Π²ΠΏΡ€Π°Π²ΠΎ) [/ латСкс]

ПокаТи ΠΎΡ‚Π²Π΅Ρ‚

РСшСниС

ΠΠ°ΠΏΠΈΡˆΠΈΡ‚Π΅ числа Ρ‚Π°ΠΊ, Ρ‡Ρ‚ΠΎΠ±Ρ‹ ΠΊΠ°ΠΆΠ΄ΠΎΠ΅ мСсто Π±Ρ‹Π»ΠΎ располоТСно Π²Π΅Ρ€Ρ‚ΠΈΠΊΠ°Π»ΡŒΠ½ΠΎ.
НачнитС с умноТСния 7 Π½Π° 62. Π£ΠΌΠ½ΠΎΠΆΡŒΡ‚Π΅ 7 Π½Π° Ρ†ΠΈΡ„Ρ€Ρƒ 62 Π² разрядС Π΅Π΄ΠΈΠ½ΠΈΡ†.

[латСкс] 7 \ cdot 2 = 14 [/ латСкс].

ΠΠ°ΠΏΠΈΡˆΠΈΡ‚Π΅ 4 Π² разрядС Π΅Π΄ΠΈΠ½ΠΈΡ† ΠΏΡ€ΠΎΠ΄ΡƒΠΊΡ‚Π° ΠΈ пСрСнСситС 1 Π² разряды дСсятков.

Π£ΠΌΠ½ΠΎΠΆΡŒΡ‚Π΅ 7 Π½Π° Ρ†ΠΈΡ„Ρ€Ρƒ Π² разрядС дСсятков 62. [латСкс] 7 \ cdot 6 = 42 [/ латСкс]. Π”ΠΎΠ±Π°Π²ΡŒΡ‚Π΅ 1 дСсятку, ΠΊΠΎΡ‚ΠΎΡ€ΡƒΡŽ ΠΌΡ‹ нСсли.

[латСкс] 42 + 1 = 43 [/ латСкс] латСкс].

Π—Π°ΠΏΠΈΡˆΠΈΡ‚Π΅ 3 Π² разрядС дСсятков произвСдСния ΠΈ 4 Π² разрядах сотСн.

ΠŸΠ΅Ρ€Π²Ρ‹ΠΉ частичный ΠΏΡ€ΠΎΠ΄ΡƒΠΊΡ‚ — [латСкс] 434 [/ латСкс].
Π’Π΅ΠΏΠ΅Ρ€ΡŒ Π½Π°ΠΏΠΈΡˆΠΈΡ‚Π΅ [latex] 0 [/ latex] ΠΏΠΎΠ΄ [latex] 4 [/ latex] Π² мСстС Π΅Π΄ΠΈΠ½ΠΈΡ† ΡΠ»Π΅Π΄ΡƒΡŽΡ‰Π΅Π³ΠΎ частичного ΠΏΡ€ΠΎΠ΄ΡƒΠΊΡ‚Π° Π² качСствС заполнитСля, Ρ‚Π°ΠΊ ΠΊΠ°ΠΊ Ρ‚Π΅ΠΏΠ΅Ρ€ΡŒ ΠΌΡ‹ ΡƒΠΌΠ½ΠΎΠΆΠ°Π΅ΠΌ Ρ†ΠΈΡ„Ρ€Ρƒ Π½Π° разряды дСсятков [latex ] 87 [/ латСкс] [латСкс] 62 [/ латСкс].

Π£ΠΌΠ½ΠΎΠΆΠΈΡ‚ΡŒ [латСкс] 8 [/ латСкс] Π½Π° Ρ†ΠΈΡ„Ρ€Ρƒ Π² разрядС Π΅Π΄ΠΈΠ½ΠΈΡ† [латСкс] 62 [/ латСкс]

[латСкс] 8 \ cdot 2 = 16 [/ латСкс]. ΠΠ°ΠΏΠΈΡˆΠΈΡ‚Π΅ [латСкс] 6 [/ латСкс] Π½Π° ΡΠ»Π΅Π΄ΡƒΡŽΡ‰Π΅ΠΌ мСстС ΠΏΡ€ΠΎΠ΄ΡƒΠΊΡ‚Π°, Ρ‚ΠΎ Π΅ΡΡ‚ΡŒ Π½Π° разрядС дСсятков.ΠžΡ‚Π½Π΅ΡΠΈΡ‚Π΅ [латСкс] 1 [/ латСкс] ΠΊ разряду дСсятков.

Π£ΠΌΠ½ΠΎΠΆΡŒΡ‚Π΅ [latex] 8 [/ latex] Π½Π° [latex] 6 [/ latex], Ρ†ΠΈΡ„Ρ€Ρƒ Π² дСсятках [latex] 62 [/ latex], Π·Π°Ρ‚Π΅ΠΌ Π΄ΠΎΠ±Π°Π²ΡŒΡ‚Π΅ [latex] 1 [/ latex] Π΄Π΅ΡΡΡ‚ΡŒ. ΠΏΠΎΠ²Π΅Π·Π»ΠΈ ΠΏΠΎΠ»ΡƒΡ‡ΠΈΡ‚ΡŒ [латСкс] 49 [/ латСкс].

ΠΠ°ΠΏΠΈΡˆΠΈΡ‚Π΅ [латСкс] 9 [/ латСкс] Π² разрядС сотСн ΠΏΡ€ΠΎΠ΄ΡƒΠΊΡ‚Π° ΠΈ [латСкс] 4 [/ латСкс] Π² разрядС тысяч.

Π’Ρ‚ΠΎΡ€ΠΎΠΉ частичный ΠΏΡ€ΠΎΠ΄ΡƒΠΊΡ‚ [латСкс] 4960 [/ латСкс]. Π”ΠΎΠ±Π°Π²ΡŒΡ‚Π΅ частичныС ΠΏΡ€ΠΎΠ΄ΡƒΠΊΡ‚Ρ‹.

ΠŸΡ€ΠΎΠ΄ΡƒΠΊΡ‚ [латСкс] 5,394 [/ латСкс].

Если имССтся Ρ‚Ρ€ΠΈ ΠΈΠ»ΠΈ Π±ΠΎΠ»Π΅Π΅ Ρ„Π°ΠΊΡ‚ΠΎΡ€ΠΎΠ², ΠΌΡ‹ ΡƒΠΌΠ½ΠΎΠΆΠ°Π΅ΠΌ ΠΏΠ΅Ρ€Π²Ρ‹Π΅ Π΄Π²Π°, Π° Π·Π°Ρ‚Π΅ΠΌ ΡƒΠΌΠ½ΠΎΠΆΠ°Π΅ΠΌ ΠΈΡ… ΠΏΡ€ΠΎΠΈΠ·Π²Π΅Π΄Π΅Π½ΠΈΠ΅ Π½Π° ΡΠ»Π΅Π΄ΡƒΡŽΡ‰ΠΈΠΉ коэффициСнт. НапримСр:

для умноТСния [латСкс] 8 \ cdot 3 \ cdot 2 [/ латСкс]
ΠΏΠ΅Ρ€Π²ΠΎΠ΅ ΡƒΠΌΠ½ΠΎΠΆΠ΅Π½ΠΈΠ΅ [латСкс] 8 \ cdot 3 [/ латСкс] [латСкс] 24 \ cdot 2 [/ латСкс]
, Π·Π°Ρ‚Π΅ΠΌ ΡƒΠΌΠ½ΠΎΠΆΡŒΡ‚Π΅ [латСкс] 24 \ cdot 2 [/ latex] [латСкс] 48 [/ латСкс]

Π’ Π²ΠΈΠ΄Π΅ΠΎ Π½ΠΈΠΆΠ΅ ΠΌΡ‹ суммируСм ΠΊΠΎΠ½Ρ†Π΅ΠΏΡ†ΠΈΠΈ, прСдставлСнныС Π½Π° этой страницС, Π²ΠΊΠ»ΡŽΡ‡Π°Ρ свойство умноТСния нуля, свойство идСнтичности умноТСния ΠΈ свойство коммутативности умноТСния.ΠΌ

Дивизия

ΠœΡ‹ сказали, Ρ‡Ρ‚ΠΎ слоТСниС ΠΈ Π²Ρ‹Ρ‡ΠΈΡ‚Π°Π½ΠΈΠ΅ — ΠΎΠ±Ρ€Π°Ρ‚Π½Ρ‹Π΅ ΠΎΠΏΠ΅Ρ€Π°Ρ†ΠΈΠΈ, ΠΏΠΎΡ‚ΠΎΠΌΡƒ Ρ‡Ρ‚ΠΎ ΠΎΠ΄Π½ΠΎ отмСняСт Π΄Ρ€ΡƒΠ³ΠΎΠ΅. Π’ΠΎΡ‡Π½ΠΎ Ρ‚Π°ΠΊ ΠΆΠ΅ Π΄Π΅Π»Π΅Π½ΠΈΠ΅ — это опСрация, обратная ΡƒΠΌΠ½ΠΎΠΆΠ΅Π½ΠΈΡŽ. ΠœΡ‹ Π·Π½Π°Π΅ΠΌ [латСкс] 12 \ div 4 = 3 [/ latex], ΠΏΠΎΡ‚ΠΎΠΌΡƒ Ρ‡Ρ‚ΠΎ [латСкс] 3 \ cdot 4 = 12 [/ latex]. ΠŸΡ€ΠΈ Π΄Π΅Π»Π΅Π½ΠΈΠΈ ΠΎΡ‡Π΅Π½ΡŒ Π²Π°ΠΆΠ½ΠΎ Π·Π½Π°Ρ‚ΡŒ всС Ρ„Π°ΠΊΡ‚Ρ‹ ΠΎ числах умноТСния.

ΠœΡ‹ провСряСм наш ΠΎΡ‚Π²Π΅Ρ‚ Π½Π° Π΄Π΅Π»Π΅Π½ΠΈΠ΅, умноТая частноС Π½Π° Π΄Π΅Π»ΠΈΡ‚Π΅Π»ΡŒ, Ρ‡Ρ‚ΠΎΠ±Ρ‹ ΠΎΠΏΡ€Π΅Π΄Π΅Π»ΠΈΡ‚ΡŒ, Ρ€Π°Π²Π½ΠΎ Π»ΠΈ ΠΎΠ½ΠΎ Π΄ΠΈΠ²ΠΈΠ΄Π΅Π½Π΄Ρƒ.ΠœΡ‹ Π·Π½Π°Π΅ΠΌ, Ρ‡Ρ‚ΠΎ [latex] 24 \ div 8 = 3 [/ latex] ΠΏΡ€Π°Π²ΠΈΠ»ΡŒΠ½ΠΎ, ΠΏΠΎΡ‚ΠΎΠΌΡƒ Ρ‡Ρ‚ΠΎ [latex] 3 \ cdot 8 = 24 [/ latex].

ΠΏΡ€ΠΈΠΌΠ΅Ρ€

Π Π°Π·Π΄Π΅Π»ΠΈΡ‚ΡŒ. Π—Π°Ρ‚Π΅ΠΌ ΠΏΡ€ΠΎΠ²Π΅Ρ€ΡŒΡ‚Π΅ ΡƒΠΌΠ½ΠΎΠΆΠ΅Π½ΠΈΠ΅ΠΌ.

  1. [латСкс] 42 \ div 6 [/ латСкс]
  2. [латСкс] \ frac {72} {9} [/ латСкс]
  3. [латСкс] 7 \ overline {) 63} [/ латСкс]

РСшСниС:

1.
[латСкс] 42 \ div 6 [/ латСкс]
Π Π°Π·Π΄Π΅Π»ΠΈΡ‚Π΅ [латСкс] 42 [/ латСкс] Π½Π° [латСкс] 6 [/ латСкс]. [латСкс] 7 [/ латСкс]
ΠŸΡ€ΠΎΠ²Π΅Ρ€ΠΈΡ‚ΡŒ ΡƒΠΌΠ½ΠΎΠΆΠ΅Π½ΠΈΠ΅ΠΌ.

[латСкс] 7 \ cdot 6 [/ латСкс]

[латСкс] 42 \ ΠΊΠ²Π°Π΄Ρ€Π°Ρ‚ΠΈΠΊ \ Π³Π°Π»ΠΎΡ‡ΠΊΠ° [/ латСкс]
2.
[латСкс] \ frac {72} {9} [/ латСкс]
Π Π°Π·Π΄Π΅Π»ΠΈΡ‚Π΅ [латСкс] 72 [/ латСкс] Π½Π° [латСкс] 9 [/ латСкс]. [латСкс] 8 [/ латСкс]
ΠŸΡ€ΠΎΠ²Π΅Ρ€ΠΈΡ‚ΡŒ ΡƒΠΌΠ½ΠΎΠΆΠ΅Π½ΠΈΠ΅ΠΌ.

[латСкс] 8 \ cdot 9 [/ латСкс]

[латСкс] 72 \ ΠΊΠ²Π°Π΄Ρ€Π°Ρ‚ΠΈΠΊ \ Π³Π°Π»ΠΎΡ‡ΠΊΠ° [/ латСкс]
3.
[латСкс] 7 \ overline {) 63} [/ латСкс]
Π Π°Π·Π΄Π΅Π»ΠΈΡ‚Π΅ [латСкс] 63 [/ латСкс] Π½Π° [латСкс] 7 [/ латСкс]. [латСкс] 9 [/ латСкс]
ΠŸΡ€ΠΎΠ²Π΅Ρ€ΠΈΡ‚ΡŒ ΡƒΠΌΠ½ΠΎΠΆΠ΅Π½ΠΈΠ΅ΠΌ.

[латСкс] 9 \ cdot 7 [/ латСкс]

[латСкс] 63 \ ΠΊΠ²Π°Π΄Ρ€Π°Ρ‚ΠΈΠΊ \ Π³Π°Π»ΠΎΡ‡ΠΊΠ° [/ латСкс]

Каково частноС ΠΏΡ€ΠΈ Π΄Π΅Π»Π΅Π½ΠΈΠΈ числа Π½Π° само сСбя?

[латСкс] \ frac {15} {15} = 1 \ text {ΠΏΠΎΡ‚ΠΎΠΌΡƒ Ρ‡Ρ‚ΠΎ} 1 \ cdot 15 = 15 [/ латСкс]

Π Π°Π·Π΄Π΅Π»Π΅Π½ΠΈΠ΅ любого числа [латСкс] \ text {(ΠΊΡ€ΠΎΠΌΠ΅ 0)} [/ latex] Π½Π° само ΠΏΠΎ сСбС Π΄Π°Π΅Ρ‚ частноС [латСкс] 1 [/ latex].ΠšΡ€ΠΎΠΌΠ΅ Ρ‚ΠΎΠ³ΠΎ, любоС число, Π΄Π΅Π»Π΅Π½Π½ΠΎΠ΅ Π½Π° [latex] 1 [/ latex], Π΄Π°Π΅Ρ‚ частноС ΠΎΡ‚ числа. Π­Ρ‚ΠΈ Π΄Π²Π΅ ΠΈΠ΄Π΅ΠΈ ΠΈΠ·Π»ΠΎΠΆΠ΅Π½Ρ‹ Π² Ρ€Π°Π·Π΄Π΅Π»Π΅ «Бвойства Π΅Π΄ΠΈΠ½ΠΈΡ†Ρ‹Β».

ΠŸΠΎΠ΄Ρ€Π°Π·Π΄Π΅Π»Π΅Π½ΠΈΠ΅ нСдвиТимости ΠΎΠ΄Π½ΠΎΠ³ΠΎ

Π›ΡŽΠ±ΠΎΠ΅ число (ΠΊΡ€ΠΎΠΌΠ΅ 0), Ρ€Π°Π·Π΄Π΅Π»Π΅Π½Π½ΠΎΠ΅ само ΠΏΠΎ сСбС, Ρ€Π°Π²Π½ΠΎ Π΅Π΄ΠΈΠ½ΠΈΡ†Π΅. [латСкс] a \ div a = 1 [/ латСкс]
Π›ΡŽΠ±ΠΎΠ΅ число, Ρ€Π°Π·Π΄Π΅Π»Π΅Π½Π½ΠΎΠ΅ Π½Π° Π΅Π΄ΠΈΠ½ΠΈΡ†Ρƒ, являСтся ΠΎΠ΄Π½ΠΈΠΌ ΠΈ Ρ‚Π΅ΠΌ ΠΆΠ΅ числом. [латСкс] a \ div 1 = a [/ латСкс]

ΠΏΡ€ΠΈΠΌΠ΅Ρ€

Π Π°Π·Π΄Π΅Π»ΠΈΡ‚ΡŒ. Π—Π°Ρ‚Π΅ΠΌ ΠΏΡ€ΠΎΠ²Π΅Ρ€ΡŒΡ‚Π΅ ΡƒΠΌΠ½ΠΎΠΆΠ΅Π½ΠΈΠ΅ΠΌ:

  1. [латСкс] 11 \ div 11 [/ латСкс]
  2. [латСкс] \ frac {19} {1} [/ латСкс]
ΠŸΠΎΠΊΠ°Π·Π°Ρ‚ΡŒ ΠΎΡ‚Π²Π΅Ρ‚

РСшСниС:

1.
[латСкс] 11 \ div 11 [/ латСкс]
Число, Ρ€Π°Π·Π΄Π΅Π»Π΅Π½Π½ΠΎΠ΅ само ΠΏΠΎ сСбС, — [латСкс] 1 [/ латСкс]. [латСкс] 1 [/ латСкс]
ΠŸΡ€ΠΎΠ²Π΅Ρ€ΠΈΡ‚ΡŒ ΡƒΠΌΠ½ΠΎΠΆΠ΅Π½ΠΈΠ΅ΠΌ.

[латСкс] 1 \ cdot 11 [/ латСкс]

[латСкс] 11 \ ΠΊΠ²Π°Π΄Ρ€Π°Ρ‚ΠΈΠΊ \ Π³Π°Π»ΠΎΡ‡ΠΊΠ° [/ латСкс]

2.
[латСкс] \ frac {19} {1} [/ латСкс]
Число, Π΄Π΅Π»Π΅Π½Π½ΠΎΠ΅ Π½Π° [латСкс] 1 [/ латСкс], Ρ€Π°Π²Π½ΠΎ самому сСбС. [латСкс] 19 [/ латСкс]
ΠŸΡ€ΠΎΠ²Π΅Ρ€ΠΈΡ‚ΡŒ ΡƒΠΌΠ½ΠΎΠΆΠ΅Π½ΠΈΠ΅ΠΌ.

[латСкс] 19 \ cdot 1 [/ латСкс]

[латСкс] 19 \ ΠΊΠ²Π°Π΄Ρ€Π°Ρ‚ΠΈΠΊ \ Π³Π°Π»ΠΎΡ‡ΠΊΠ° [/ латСкс]

ΠŸΡ€Π΅Π΄ΠΏΠΎΠ»ΠΎΠΆΠΈΠΌ, Ρƒ нас Π΅ΡΡ‚ΡŒ [latex] \ text {\ $ 0} [/ latex], ΠΈ ΠΌΡ‹ Ρ…ΠΎΡ‚ΠΈΠΌ Ρ€Π°Π·Π΄Π΅Π»ΠΈΡ‚ΡŒ Π΅Π³ΠΎ ΠΌΠ΅ΠΆΠ΄Ρƒ [latex] 3 [/ latex] людьми. Бколько ΠΏΠΎΠ»ΡƒΡ‡ΠΈΡ‚ ΠΊΠ°ΠΆΠ΄Ρ‹ΠΉ Ρ‡Π΅Π»ΠΎΠ²Π΅ΠΊ? ΠšΠ°ΠΆΠ΄Ρ‹ΠΉ ΠΏΠΎΠ»ΡƒΡ‡ΠΈΡ‚ [латСкс] \ тСкст {\ $ 0} [/ латСкс]. Ноль Ρ€Π°Π·Π΄Π΅Π»Π΅Π½Π½Ρ‹ΠΉ Π½Π° любоС число — [латСкс] 0 [/ латСкс].

Π’Π΅ΠΏΠ΅Ρ€ΡŒ ΠΏΡ€Π΅Π΄ΠΏΠΎΠ»ΠΎΠΆΠΈΠΌ, Ρ‡Ρ‚ΠΎ ΠΌΡ‹ Ρ…ΠΎΡ‚ΠΈΠΌ Ρ€Π°Π·Π΄Π΅Π»ΠΈΡ‚ΡŒ [latex] \ text {\ $ 10} [/ latex] Π½Π° [latex] 0 [/ latex].Π­Ρ‚ΠΎ ΠΎΠ·Π½Π°Ρ‡Π°Π΅Ρ‚, Ρ‡Ρ‚ΠΎ Π½Π°ΠΌ Π½ΡƒΠΆΠ½ΠΎ Π½Π°ΠΉΡ‚ΠΈ число, ΠΊΠΎΡ‚ΠΎΡ€ΠΎΠ΅ ΠΌΡ‹ ΡƒΠΌΠ½ΠΎΠΆΠΈΠΌ Π½Π° [latex] 0 [/ latex], Ρ‡Ρ‚ΠΎΠ±Ρ‹ ΠΏΠΎΠ»ΡƒΡ‡ΠΈΡ‚ΡŒ [latex] 10 [/ latex]. Π­Ρ‚ΠΎΠ³ΠΎ Π½Π΅ ΠΌΠΎΠΆΠ΅Ρ‚ ΠΏΡ€ΠΎΠΈΠ·ΠΎΠΉΡ‚ΠΈ, ΠΏΠΎΡ‚ΠΎΠΌΡƒ Ρ‡Ρ‚ΠΎ [латСкс] 0 [/ латСкс] ΡƒΠΌΠ½ΠΎΠΆΠ΅Π½Π½ΠΎΠ΅ Π½Π° любоС число Ρ€Π°Π²Π½ΠΎ [латСкс] 0 [/ латСкс]. Π”Π΅Π»Π΅Π½ΠΈΠ΅ Π½Π° ноль называСтся undefined .

Π­Ρ‚ΠΈ Π΄Π²Π΅ ΠΈΠ΄Π΅ΠΈ ΡΠΎΡΡ‚Π°Π²Π»ΡΡŽΡ‚ свойство дСлСния нуля.

ΠŸΠΎΠ΄Ρ€Π°Π·Π΄Π΅Π»Π΅Π½ΠΈΠ΅ нСдвиТимости Zero

ΠΡƒΠ»ΡŒ, Π΄Π΅Π»Π΅Π½Π½Ρ‹ΠΉ Π½Π° любоС число, Π±ΡƒΠ΄Π΅Ρ‚ [латСкс] 0 [/ латСкс]. [латСкс] 0 \ div a = 0 [/ латСкс]
Π”Π΅Π»Π΅Π½ΠΈΠ΅ числа Π½Π° ноль Π½Π΅ ΠΎΠΏΡ€Π΅Π΄Π΅Π»Π΅Π½ΠΎ. [латСкс] a \ div 0 [/ латСкс] undefined

Π”Ρ€ΡƒΠ³ΠΎΠΉ способ ΠΎΠ±ΡŠΡΡΠ½ΠΈΡ‚ΡŒ, ΠΏΠΎΡ‡Π΅ΠΌΡƒ Π΄Π΅Π»Π΅Π½ΠΈΠ΅ Π½Π° ноль Π½Π΅ ΠΎΠΏΡ€Π΅Π΄Π΅Π»Π΅Π½ΠΎ, — это ΠΏΠΎΠΌΠ½ΠΈΡ‚ΡŒ, Ρ‡Ρ‚ΠΎ Π΄Π΅Π»Π΅Π½ΠΈΠ΅ Π½Π° самом Π΄Π΅Π»Π΅ являСтся ΠΏΠΎΠ²Ρ‚ΠΎΡ€Π½Ρ‹ΠΌ Π²Ρ‹Ρ‡ΠΈΡ‚Π°Π½ΠΈΠ΅ΠΌ. Бколько Ρ€Π°Π· ΠΌΡ‹ ΠΌΠΎΠΆΠ΅ΠΌ ΡƒΠ±Ρ€Π°Ρ‚ΡŒ [latex] 0 [/ latex] ΠΈΠ· [latex] 10? [/ Latex] ΠŸΠΎΡΠΊΠΎΠ»ΡŒΠΊΡƒ Π²Ρ‹Ρ‡ΠΈΡ‚Π°Π½ΠΈΠ΅ [latex] 0 [/ latex] Π½ΠΈΠΊΠΎΠ³Π΄Π° Π½Π΅ ΠΈΠ·ΠΌΠ΅Π½ΠΈΡ‚ ΠΎΠ±Ρ‰ΡƒΡŽ сумму, ΠΌΡ‹ Π½ΠΈΠΊΠΎΠ³Π΄Π° Π½Π΅ ΠΏΠΎΠ»ΡƒΡ‡ΠΈΠΌ ΠΎΡ‚Π²Π΅Ρ‚Π°. Π’Π°ΠΊΠΈΠΌ ΠΎΠ±Ρ€Π°Π·ΠΎΠΌ, ΠΌΡ‹ Π½Π΅ ΠΌΠΎΠΆΠ΅ΠΌ Ρ€Π°Π·Π΄Π΅Π»ΠΈΡ‚ΡŒ число Π½Π° [латСкс] 0 [/ латСкс].

ΠΏΡ€ΠΈΠΌΠ΅Ρ€

Π Π°Π·Π΄Π΅Π»ΠΈΡ‚ΡŒ. ΠŸΡ€ΠΎΠ²Π΅Ρ€ΠΈΡ‚ΡŒ ΡƒΠΌΠ½ΠΎΠΆΠ΅Π½ΠΈΠ΅ΠΌ:

  1. [латСкс] 0 \ div 3 [/ латСкс]
  2. [латСкс] \ frac {10} {0} [/ латСкс]
ΠŸΠΎΠΊΠ°Π·Π°Ρ‚ΡŒ ΠΎΡ‚Π²Π΅Ρ‚

РСшСниС

1.
[латСкс] 0 \ div 3 [/ латСкс]
Ноль, Π΄Π΅Π»Π΅Π½Π½Ρ‹ΠΉ Π½Π° любоС число, равняСтся Π½ΡƒΠ»ΡŽ. [латСкс] 0 [/ латСкс]
ΠŸΡ€ΠΎΠ²Π΅Ρ€ΠΈΡ‚ΡŒ ΡƒΠΌΠ½ΠΎΠΆΠ΅Π½ΠΈΠ΅ΠΌ.

[латСкс] 0 \ cdot 3 [/ латСкс]

[латСкс] 0 \ Ρ‡Π΅Ρ‚Ρ‹Ρ€Π΅Ρ…ΡƒΠ³ΠΎΠ»ΡŒΠ½ΠΈΠΊ \ Π³Π°Π»ΠΎΡ‡ΠΊΠ° [/ латСкс]

2.
[латСкс] 10/0 [/ латСкс]
Π”Π΅Π»Π΅Π½ΠΈΠ΅ Π½Π° ноль Π½Π΅ ΠΎΠΏΡ€Π΅Π΄Π΅Π»Π΅Π½ΠΎ. undefined

Когда Π΄Π΅Π»ΠΈΡ‚Π΅Π»ΡŒ ΠΈΠ»ΠΈ Π΄Π΅Π»ΠΈΠΌΠΎΠ΅ состоит Π±ΠΎΠ»Π΅Π΅ Ρ‡Π΅ΠΌ ΠΈΠ· ΠΎΠ΄Π½ΠΎΠΉ Ρ†ΠΈΡ„Ρ€Ρ‹, ΠΎΠ±Ρ‹Ρ‡Π½ΠΎ ΠΏΡ€ΠΎΡ‰Π΅ ΠΈΡΠΏΠΎΠ»ΡŒΠ·ΠΎΠ²Π°Ρ‚ΡŒ Π½ΠΎΡ‚Π°Ρ†ΠΈΡŽ [latex] 4 \ overline {) 12} [/ latex]. Π­Ρ‚ΠΎΡ‚ процСсс называСтся Π΄Π»ΠΈΠ½Π½Ρ‹ΠΌ Π΄Π΅Π»Π΅Π½ΠΈΠ΅ΠΌ. Π”Π°Π²Π°ΠΉΡ‚Π΅ ΠΏΠΎΡ€Π°Π±ΠΎΡ‚Π°Π΅ΠΌ Π½Π°Π΄ процСссом, Ρ€Π°Π·Π΄Π΅Π»ΠΈΠ² [латСкс] 78 [/ латСкс] Π½Π° [латСкс] 3 [/ латСкс].

Π Π°Π·Π΄Π΅Π»ΠΈΡ‚Π΅ ΠΏΠ΅Ρ€Π²ΡƒΡŽ Ρ†ΠΈΡ„Ρ€Ρƒ Π΄Π΅Π»ΠΈΠΌΠΎΠ³ΠΎ, [латСкс] 7 [/ латСкс], Π½Π° Π΄Π΅Π»ΠΈΡ‚Π΅Π»ΡŒ, [латСкс] 3 [/ латСкс].
Π”Π΅Π»ΠΈΡ‚Π΅Π»ΡŒ [латСкс] 3 [/ латСкс] ΠΌΠΎΠΆΠ΅Ρ‚ Π²Ρ…ΠΎΠ΄ΠΈΡ‚ΡŒ Π² [латСкс] 7 [/ латСкс] Π΄Π²Π° Ρ€Π°Π·Π°, Ρ‚Π°ΠΊ ΠΊΠ°ΠΊ [латСкс] 2 \ times 3 = 6 [/ латСкс].ΠΠ°ΠΏΠΈΡˆΠΈΡ‚Π΅ [латСкс] 2 [/ латСкс] Π½Π°Π΄ [латСксом] 7 [/ латСкс] Π² частном.
Π£ΠΌΠ½ΠΎΠΆΡŒΡ‚Π΅ [латСкс] 2 [/ латСкс] Π² частном Π½Π° [латСкс] 2 [/ латСкс] ΠΈ Π·Π°ΠΏΠΈΡˆΠΈΡ‚Π΅ ΠΏΡ€ΠΎΠΈΠ·Π²Π΅Π΄Π΅Π½ΠΈΠ΅ [латСкс] 6 [/ латСкс] ΠΏΠΎΠ΄ [латСкс] 7 [/ латСкс].
Π’Ρ‹Ρ‡Ρ‚ΠΈΡ‚Π΅ это ΠΏΡ€ΠΎΠΈΠ·Π²Π΅Π΄Π΅Π½ΠΈΠ΅ ΠΈΠ· ΠΏΠ΅Ρ€Π²ΠΎΠΉ Ρ†ΠΈΡ„Ρ€Ρ‹ Π΄ΠΈΠ²ΠΈΠ΄Π΅Π½Π΄Π°. Π’Ρ‹Ρ‡Ρ‚ΠΈΡ‚Π΅ [латСкс] 7 — 6 [/ латСкс]. ΠΠ°ΠΏΠΈΡˆΠΈΡ‚Π΅ Ρ€Π°Π·Π½ΠΈΡ†Ρƒ, 1, ΠΏΠΎΠ΄ ΠΏΠ΅Ρ€Π²ΠΎΠΉ Ρ†ΠΈΡ„Ρ€ΠΎΠΉ Π΄Π΅Π»ΠΈΠΌΠΎΠ³ΠΎ.
ΠžΠΏΡƒΡΡ‚ΠΈΡ‚Π΅ ΡΠ»Π΅Π΄ΡƒΡŽΡ‰ΡƒΡŽ Ρ†ΠΈΡ„Ρ€Ρƒ Π΄Π΅Π»ΠΈΠΌΠΎΠ³ΠΎ.Π‘Π±ΠΈΠ²Π°Π΅ΠΌ [латСкс] 8 [/ латСкс].
Π Π°Π·Π΄Π΅Π»ΠΈΡ‚Π΅ [латСкс] 18 [/ латСкс] Π½Π° Π΄Π΅Π»ΠΈΡ‚Π΅Π»ΡŒ, [латСкс] 3 [/ латСкс]. Π”Π΅Π»ΠΈΡ‚Π΅Π»ΡŒ [латСкс] 3 [/ латСкс] ΠΏΠ΅Ρ€Π΅Ρ…ΠΎΠ΄ΠΈΡ‚ Π² [латСкс] 18 [/ латСкс] ΡˆΠ΅ΡΡ‚ΡŒ Ρ€Π°Π·.
ΠΠ°ΠΏΠΈΡˆΠΈΡ‚Π΅ [латСкс] 6 [/ латСкс] Π² частном Π½Π°Π΄ [латСксом] 8 [/ латСксом].
Π£ΠΌΠ½ΠΎΠΆΡŒΡ‚Π΅ [латСкс] 6 [/ латСкс] Π² частном Π½Π° Π΄Π΅Π»ΠΈΡ‚Π΅Π»ΡŒ ΠΈ Π·Π°ΠΏΠΈΡˆΠΈΡ‚Π΅ ΠΏΡ€ΠΎΠΈΠ·Π²Π΅Π΄Π΅Π½ΠΈΠ΅ [латСкс] 18 [/ латСкс] ΠΏΠΎΠ΄ Π΄Π΅Π»ΠΈΠΌΡ‹ΠΌ. Π’Ρ‹Ρ‡Ρ‚ΠΈΡ‚Π΅ [латСкс] 18 [/ латСкс] ΠΈΠ· [латСкс] 18 [/ латСкс].

ΠœΡ‹ Π±ΡƒΠ΄Π΅ΠΌ ΠΏΠΎΠ²Ρ‚ΠΎΡ€ΡΡ‚ΡŒ процСсс Π΄ΠΎ Ρ‚Π΅Ρ… ΠΏΠΎΡ€, ΠΏΠΎΠΊΠ° Π² Π΄ΠΈΠ²ΠΈΠ΄Π΅Π½Π΄Π΅ Π½Π΅ останСтся Ρ†ΠΈΡ„Ρ€, ΠΊΠΎΡ‚ΠΎΡ€Ρ‹Π΅ Π½ΡƒΠΆΠ½ΠΎ ΡƒΠΌΠ΅Π½ΡŒΡˆΠΈΡ‚ΡŒ. Π’ этой Π·Π°Π΄Π°Ρ‡Π΅ большС Π½Π΅Ρ‚ Ρ†ΠΈΡ„Ρ€, ΠΊΠΎΡ‚ΠΎΡ€Ρ‹Π΅ Π½ΡƒΠΆΠ½ΠΎ ΡΠ±ΠΈΠ²Π°Ρ‚ΡŒ, поэтому Π΄Π΅Π»Π΅Π½ΠΈΠ΅ Π·Π°ΠΊΠΎΠ½Ρ‡Π΅Π½ΠΎ.

[латСкс] \ text {So} 78 \ div 3 = 26 [/ латСкс].

ΠŸΡ€ΠΎΠ²Π΅Ρ€ΡŒΡ‚Π΅, ΡƒΠΌΠ½ΠΎΠΆΠΈΠ² частноС Π½Π° Π΄Π΅Π»ΠΈΡ‚Π΅Π»ΡŒ, Ρ‡Ρ‚ΠΎΠ±Ρ‹ ΠΏΠΎΠ»ΡƒΡ‡ΠΈΡ‚ΡŒ Π΄ΠΈΠ²ΠΈΠ΄Π΅Π½Π΄. Π£ΠΌΠ½ΠΎΠΆΡŒΡ‚Π΅ [латСкс] 26 \ Π½Π° 3 [/ латСкс], Ρ‡Ρ‚ΠΎΠ±Ρ‹ ΡƒΠ±Π΅Π΄ΠΈΡ‚ΡŒΡΡ, Ρ‡Ρ‚ΠΎ ΠΏΡ€ΠΎΠ΄ΡƒΠΊΡ‚ Ρ€Π°Π²Π΅Π½ Π΄ΠΈΠ²ΠΈΠ΄Π΅Π½Π΄Ρƒ, [латСкс] 78 [/ латСкс].

[латСкс] \ begin {array} {c} \ hfill \ stackrel {1} ​​{2} 6 \\ \ hfill \ underset {\ text {___}} {\ times 3} \\ \ hfill 78 \ end {массив } [/ латСкс]

Π”Π°, поэтому наш ΠΎΡ‚Π²Π΅Ρ‚ ΠΏΡ€Π°Π²ΠΈΠ»ΡŒΠ½Ρ‹ΠΉ.[латСкс] \ Π³Π°Π»ΠΎΡ‡ΠΊΠ° [/ латСкс]

Π”Π΅Π»Π΅Π½ΠΈΠ΅ Ρ†Π΅Π»Ρ‹Ρ… чисСл

  1. Π Π°Π·Π΄Π΅Π»ΠΈΡ‚Π΅ ΠΏΠ΅Ρ€Π²ΡƒΡŽ Ρ†ΠΈΡ„Ρ€Ρƒ Π΄Π΅Π»ΠΈΠΌΠΎΠ³ΠΎ Π½Π° Π΄Π΅Π»ΠΈΡ‚Π΅Π»ΡŒ. Если Π΄Π΅Π»ΠΈΡ‚Π΅Π»ΡŒ большС, Ρ‡Π΅ΠΌ пСрвая Ρ†ΠΈΡ„Ρ€Π° Π΄Π΅Π»ΠΈΠΌΠΎΠ³ΠΎ, Ρ€Π°Π·Π΄Π΅Π»ΠΈΡ‚Π΅ ΠΏΠ΅Ρ€Π²Ρ‹Π΅ Π΄Π²Π΅ Ρ†ΠΈΡ„Ρ€Ρ‹ Π΄Π΅Π»ΠΈΠΌΠΎΠ³ΠΎ Π½Π° Π΄Π΅Π»ΠΈΡ‚Π΅Π»ΡŒ ΠΈ Ρ‚. Π”.
  2. ΠΠ°ΠΏΠΈΡˆΠΈΡ‚Π΅ частноС Π½Π°Π΄ Π΄ΠΈΠ²ΠΈΠ΄Π΅Π½Π΄ΠΎΠΌ.
  3. Π£ΠΌΠ½ΠΎΠΆΡŒΡ‚Π΅ частноС Π½Π° Π΄Π΅Π»ΠΈΡ‚Π΅Π»ΡŒ ΠΈ Π·Π°ΠΏΠΈΡˆΠΈΡ‚Π΅ ΠΏΡ€ΠΎΠΈΠ·Π²Π΅Π΄Π΅Π½ΠΈΠ΅ ΠΏΠΎΠ΄ Π΄Π΅Π»ΠΈΠΌΡ‹ΠΌ.
  4. Π’Ρ‹Ρ‡Ρ‚ΠΈΡ‚Π΅ этот ΠΏΡ€ΠΎΠ΄ΡƒΠΊΡ‚ ΠΈΠ· Π΄ΠΈΠ²ΠΈΠ΄Π΅Π½Π΄Π°.
  5. Π’Π²Π΅Π΄ΠΈΡ‚Π΅ ΡΠ»Π΅Π΄ΡƒΡŽΡ‰ΡƒΡŽ Ρ†ΠΈΡ„Ρ€Ρƒ Π΄Π΅Π»ΠΈΠΌΠΎΠ³ΠΎ.
  6. ΠŸΠΎΠ²Ρ‚ΠΎΡ€ΡΠΉΡ‚Π΅ дСйствия с шага 1 Π΄ΠΎ Ρ‚Π΅Ρ… ΠΏΠΎΡ€, ΠΏΠΎΠΊΠ° Π² Π΄Π΅Π»ΠΈΠΌΠΎΠΌ Π½Π΅ останСтся Ρ†ΠΈΡ„Ρ€, ΠΊΠΎΡ‚ΠΎΡ€Ρ‹Π΅ Π½ΡƒΠΆΠ½ΠΎ ΡƒΠΌΠ΅Π½ΡŒΡˆΠΈΡ‚ΡŒ.
  7. ΠŸΡ€ΠΎΠ²Π΅Ρ€ΡŒΡ‚Π΅, ΡƒΠΌΠ½ΠΎΠΆΠΈΠ² частноС Π½Π° Π΄Π΅Π»ΠΈΡ‚Π΅Π»ΡŒ.

На Π²ΠΈΠ΄Π΅ΠΎ Π½ΠΈΠΆΠ΅ ΠΌΡ‹ ΠΏΠΎΠΊΠ°Π·Ρ‹Π²Π°Π΅ΠΌ Π΅Ρ‰Π΅ ΠΎΠ΄ΠΈΠ½ ΠΏΡ€ΠΈΠΌΠ΅Ρ€ использования дСлСния Π² столбик.

ΠΏΡ€ΠΈΠΌΠ΅Ρ€

Π Π°Π·Π΄Π΅Π»ΠΈΡ‚ΡŒ [латСкс] 2,596 \ div 4 [/ латСкс]. ΠŸΡ€ΠΎΠ²Π΅Ρ€ΠΈΡ‚ΡŒ ΡƒΠΌΠ½ΠΎΠΆΠ΅Π½ΠΈΠ΅ΠΌ:

ПокаТи ΠΎΡ‚Π²Π΅Ρ‚

РСшСниС

Π­Ρ‚ΠΎ равняСтся Π΄ΠΈΠ²ΠΈΠ΄Π΅Π½Π΄Ρƒ, поэтому наш ΠΎΡ‚Π²Π΅Ρ‚ ΠΏΡ€Π°Π²ΠΈΠ»ΡŒΠ½Ρ‹ΠΉ.

ΠΏΡ€ΠΈΠΌΠ΅Ρ€

Π Π°Π·Π΄Π΅Π»ΠΈΡ‚Π΅ [латСкс] 4,506 \ div 6 [/ латСкс].ΠŸΡ€ΠΎΠ²Π΅Ρ€ΠΈΡ‚ΡŒ ΡƒΠΌΠ½ΠΎΠΆΠ΅Π½ΠΈΠ΅ΠΌ:

ПокаТи ΠΎΡ‚Π²Π΅Ρ‚

РСшСниС

Π­Ρ‚ΠΎ равняСтся Π΄ΠΈΠ²ΠΈΠ΄Π΅Π½Π΄Ρƒ, поэтому наш ΠΎΡ‚Π²Π΅Ρ‚ ΠΏΡ€Π°Π²ΠΈΠ»ΡŒΠ½Ρ‹ΠΉ.

ΠŸΠΎΡΠΌΠΎΡ‚Ρ€ΠΈΡ‚Π΅ это Π²ΠΈΠ΄Π΅ΠΎ, Ρ‡Ρ‚ΠΎΠ±Ρ‹ ΡƒΠ²ΠΈΠ΄Π΅Ρ‚ΡŒ Π΅Ρ‰Π΅ ΠΎΠ΄ΠΈΠ½ ΠΏΡ€ΠΈΠΌΠ΅Ρ€ Ρ‚ΠΎΠ³ΠΎ, ΠΊΠ°ΠΊ ΠΈΡΠΏΠΎΠ»ΡŒΠ·ΠΎΠ²Π°Ρ‚ΡŒ Π΄Π»ΠΈΠ½Π½ΠΎΠ΅ Π΄Π΅Π»Π΅Π½ΠΈΠ΅ для дСлСния Ρ‡Π΅Ρ‚Ρ‹Ρ€Π΅Ρ…Π·Π½Π°Ρ‡Π½ΠΎΠ³ΠΎ Ρ†Π΅Π»ΠΎΠ³ΠΎ числа Π½Π° Π΄Π²ΡƒΠ·Π½Π°Ρ‡Π½ΠΎΠ΅ Ρ†Π΅Π»ΠΎΠ΅ число.

Пока всС ΠΏΡ€ΠΎΠ±Π»Π΅ΠΌΡ‹ с Ρ€Π°Π·Π΄Π΅Π»Π΅Π½ΠΈΠ΅ΠΌ Ρ€Π΅ΡˆΠ°ΡŽΡ‚ΡΡ Ρ€Π°Π²Π½ΠΎΠΌΠ΅Ρ€Π½ΠΎ. НапримСр, Ссли Π±Ρ‹ Ρƒ нас Π±Ρ‹Π»ΠΎ [латСксноС] 24 [/ латСксноС] ΠΏΠ΅Ρ‡Π΅Π½ΡŒΠ΅ ΠΈ ΠΌΡ‹ Ρ…ΠΎΡ‚Π΅Π»ΠΈ Π±Ρ‹ ΡΠ΄Π΅Π»Π°Ρ‚ΡŒ ΠΏΠ°ΠΊΠ΅Ρ‚Ρ‹ ΠΈΠ· [латСксного] 8 [/ латСксного] ΠΏΠ΅Ρ‡Π΅Π½ΡŒΡ, Ρƒ нас Π±Ρ‹Π»ΠΈ Π±Ρ‹ [латСксныС] 3 [/ латСксныС] ΠΏΠ°ΠΊΠ΅Ρ‚Ρ‹.Но Ρ‡Ρ‚ΠΎ, Ссли Π±Ρ‹ Π±Ρ‹Π»ΠΎ ΠΏΠ΅Ρ‡Π΅Π½ΡŒΠ΅ [latex] 28 [/ latex], ΠΈ ΠΌΡ‹ Ρ…ΠΎΡ‚Π΅Π»ΠΈ Π±Ρ‹ ΡΠ΄Π΅Π»Π°Ρ‚ΡŒ ΠΏΠ°ΠΊΠ΅Ρ‚Ρ‹ ΠΈΠ· [latex] 8? [/ Latex] НачнСм с Ρ„Π°ΠΉΠ»ΠΎΠ² cookie [latex] 28 [/ latex].


ΠŸΠΎΠΏΡ€ΠΎΠ±ΡƒΠΉΡ‚Π΅ Ρ€Π°Π·ΠΌΠ΅ΡΡ‚ΠΈΡ‚ΡŒ Ρ„Π°ΠΉΠ»Ρ‹ cookie Π³Ρ€ΡƒΠΏΠΏΠ°ΠΌΠΈ ΠΏΠΎ восСмь ΡˆΡ‚ΡƒΠΊ.


ΠžΡΡ‚Π°Π»ΠΈΡΡŒ [latex] 3 [/ latex] Π³Ρ€ΡƒΠΏΠΏΡ‹ ΠΈΠ· восьми Ρ„Π°ΠΉΠ»ΠΎΠ² cookie ΠΈ [latex] 4 [/ latex] Ρ„Π°ΠΉΠ»ΠΎΠ² cookie. ΠœΡ‹ Π²Ρ‹Π·Ρ‹Π²Π°Π΅ΠΌ ΠΎΡΡ‚Π°Π²ΡˆΠΈΠ΅ΡΡ Ρ„Π°ΠΉΠ»Ρ‹ cookie [latex] 4 [/ latex] ΠΈ ΠΏΠΎΠΊΠ°Π·Ρ‹Π²Π°Π΅ΠΌ ΠΈΡ…, записывая R4 рядом с [latex] 3 [/ latex]. (R ΠΎΠ·Π½Π°Ρ‡Π°Π΅Ρ‚ остаток.) ​​

Π§Ρ‚ΠΎΠ±Ρ‹ ΠΏΡ€ΠΎΠ²Π΅Ρ€ΠΈΡ‚ΡŒ это Π΄Π΅Π»Π΅Π½ΠΈΠ΅, ΠΌΡ‹ ΡƒΠΌΠ½ΠΎΠΆΠ°Π΅ΠΌ [latex] 3 [/ latex] Π½Π° [latex] 8 [/ latex], Ρ‡Ρ‚ΠΎΠ±Ρ‹ ΠΏΠΎΠ»ΡƒΡ‡ΠΈΡ‚ΡŒ [latex] 24 [/ latex], Π° Π·Π°Ρ‚Π΅ΠΌ складываСм остаток [latex] 4 [/ latex].

[латСкс] \ begin {array} {c} \ hfill 3 \\ \ hfill \ underset {\ text {___}} {\ times 8} \\ \ hfill 24 \\ \ hfill \ underset {\ text {___} } {+ 4} \\ \ hfill 28 \ end {array} [/ latex]

ΠΏΡ€ΠΈΠΌΠ΅Ρ€

Π Π°Π·Π΄Π΅Π»ΠΈΡ‚ΡŒ [латСкс] 1,439 \ div 4 [/ латСкс]. ΠŸΡ€ΠΎΠ²Π΅Ρ€ΠΈΡ‚ΡŒ ΡƒΠΌΠ½ΠΎΠΆΠ΅Π½ΠΈΠ΅ΠΌ.

ПокаТи ΠΎΡ‚Π²Π΅Ρ‚

РСшСниС

Π˜Ρ‚Π°ΠΊ [латСкс] 1,439 \ div 4 [/ latex] — это [латСкс] 359 [/ latex] с остатком [латСкс] 3 [/ latex]. Наш ΠΎΡ‚Π²Π΅Ρ‚ ΠΏΡ€Π°Π²ΠΈΠ»ΡŒΠ½Ρ‹ΠΉ.

ΠΏΡ€ΠΈΠΌΠ΅Ρ€

Π Π°Π·Π΄Π΅Π»ΠΈΡ‚Π΅, Π° Π·Π°Ρ‚Π΅ΠΌ ΠΏΡ€ΠΎΠ²Π΅Ρ€ΡŒΡ‚Π΅ ΡƒΠΌΠ½ΠΎΠΆΠ΅Π½ΠΈΠ΅ΠΌ: [латСкс] 1,461 \ div 13 [/ латСкс].

ПокаТи ΠΎΡ‚Π²Π΅Ρ‚

РСшСниС

Наш ΠΎΡ‚Π²Π΅Ρ‚ ΠΏΡ€Π°Π²ΠΈΠ»ΡŒΠ½Ρ‹ΠΉ.

ΠŸΠΎΡΠΌΠΎΡ‚Ρ€ΠΈΡ‚Π΅ Π²ΠΈΠ΄Π΅ΠΎ Π½ΠΈΠΆΠ΅, Ρ‡Ρ‚ΠΎΠ±Ρ‹ ΡƒΠ²ΠΈΠ΄Π΅Ρ‚ΡŒ Π΅Ρ‰Π΅ ΠΎΠ΄ΠΈΠ½ ΠΏΡ€ΠΈΠΌΠ΅Ρ€ Ρ‚ΠΎΠ³ΠΎ, ΠΊΠ°ΠΊ ΠΈΡΠΏΠΎΠ»ΡŒΠ·ΠΎΠ²Π°Ρ‚ΡŒ Π΄Π»ΠΈΠ½Π½ΠΎΠ΅ Π΄Π΅Π»Π΅Π½ΠΈΠ΅ для дСлСния Ρ†Π΅Π»Ρ‹Ρ… чисСл, ΠΊΠΎΠ³Π΄Π° Π΅ΡΡ‚ΡŒ остаток.

ВнСситС свой Π²ΠΊΠ»Π°Π΄!

Π£ вас Π±Ρ‹Π»Π° идСя ΡƒΠ»ΡƒΡ‡ΡˆΠΈΡ‚ΡŒ этот ΠΊΠΎΠ½Ρ‚Π΅Π½Ρ‚? Нам ΠΎΡ‡Π΅Π½ΡŒ понравится ваш Π²ΠΊΠ»Π°Π΄.

Π£Π»ΡƒΡ‡ΡˆΠΈΡ‚ΡŒ эту ΡΡ‚Ρ€Π°Π½ΠΈΡ†ΡƒΠŸΠΎΠ΄Ρ€ΠΎΠ±Π½Π΅Π΅

ΠœΠ°Ρ‚Π΅ΠΌΠ°Ρ‚ΠΈΠΊΠ° для 5-Ρ… классов — Π‘Π»ΠΎΠΊ 5: Π£ΠΌΠ½ΠΎΠΆΠ΅Π½ΠΈΠ΅ ΠΈ Π΄Π΅Π»Π΅Π½ΠΈΠ΅ Π΄Ρ€ΠΎΠ±Π΅ΠΉ

Π‘Π²ΠΎΠ΄ΠΊΠ° ΠΏΠΎ установкС

Π’ 5 классС 5 класса учащиСся ΠΏΡ€ΠΎΠ΄ΠΎΠ»ΠΆΠ°ΡŽΡ‚ ΠΈΠ·ΡƒΡ‡Π΅Π½ΠΈΠ΅ ΠΎΠΏΠ΅Ρ€Π°Ρ†ΠΈΠΉ с дробями, углубляя своС ΠΏΠΎΠ½ΠΈΠΌΠ°Π½ΠΈΠ΅ умноТСния Π΄Ρ€ΠΎΠ±Π΅ΠΉ с 4 класса ΠΈ знакомя ΠΈΡ… с Π΄Π΅Π»Π΅Π½ΠΈΠ΅ΠΌ Π½Π° Π΄Ρ€ΠΎΠ±ΠΈ.

УчащиСся ΠΎΡ‡Π΅Π½ΡŒ Ρ€Π°Π½ΠΎ Π½Π°Ρ‡Π°Π»ΠΈ ΠΈΠ·ΡƒΡ‡Π°Ρ‚ΡŒ Π΄Ρ€ΠΎΠ±ΠΈ, ΠΊΠ°ΠΊ описано Π² Ρ€Π°Π·Π΄Π΅Π»Π΅ Β«ΠšΡ€Π°Ρ‚ΠΊΠΎΠ΅ содСрТаниС Ρ€Π°Π·Π΄Π΅Π»Π° 4Β». Однако ΡƒΡ‡Π΅Π½ΠΈΠΊΠΈ Π½Π°Ρ‡Π°Π»ΠΈ Π·Π½Π°ΠΊΠΎΠΌΠΈΡ‚ΡŒΡΡ с ΡƒΠΌΠ½ΠΎΠΆΠ΅Π½ΠΈΠ΅ΠΌ Π΄Ρ€ΠΎΠ±Π΅ΠΉ Ρ‚ΠΎΠ»ΡŒΠΊΠΎ Π² 4 классС, ΠΊΠΎΠ³Π΄Π° ΠΎΠ½ΠΈ Π½Π°ΡƒΡ‡ΠΈΠ»ΠΈΡΡŒ ΡƒΠΌΠ½ΠΎΠΆΠ°Ρ‚ΡŒ Π΄Ρ€ΠΎΠ±ΡŒ Π½Π° Ρ†Π΅Π»ΠΎΠ΅ число, интСрпрСтируя это ΠΊΠ°ΠΊ ΠΏΠΎΠ²Ρ‚ΠΎΡ€Π½ΠΎΠ΅ слоТСниС. НапримСр, $$ {4 \ times {2 \ over3}} $$ рассматриваСтся ΠΊΠ°ΠΊ 4 ΠΊΠΎΠΏΠΈΠΈ 2 Ρ‚Ρ€Π΅Ρ‚Π΅ΠΉ. Π­Ρ‚ΠΎ ΠΏΠΎΠ½ΠΈΠΌΠ°Π½ΠΈΠ΅ основано Π½Π° ΠΏΠΎΠ½ΠΈΠΌΠ°Π½ΠΈΠΈ умноТСния ΠΊΠ°ΠΊ Ρ€Π°Π²Π½Ρ‹Ρ… Π³Ρ€ΡƒΠΏΠΏ (3.OA.1). Однако Π² 4-ΠΌ классС ΡƒΡ‡Π΅Π½ΠΈΠΊΠΈ Ρ‚Π°ΠΊΠΆΠ΅ Ρ€Π°Π·Π²ΠΈΠ»ΠΈ ΠΏΠΎΠ½ΠΈΠΌΠ°Π½ΠΈΠ΅ ΠΌΡƒΠ»ΡŒΡ‚ΠΈΠΏΠ»ΠΈΠΊΠ°Ρ‚ΠΈΠ²Π½ΠΎΠ³ΠΎ сравнСния (4.OA.1), Ρ‡Ρ‚ΠΎ Π±ΡƒΠ΄Π΅Ρ‚ ΠΈΠΌΠ΅Ρ‚ΡŒ особоС Π·Π½Π°Ρ‡Π΅Π½ΠΈΠ΅ для Π½ΠΎΠ²Ρ‹Ρ… способов ΠΈΠ½Ρ‚Π΅Ρ€ΠΏΡ€Π΅Ρ‚Π°Ρ†ΠΈΠΈ учащимися умноТСния Π΄Ρ€ΠΎΠ±Π΅ΠΉ Π² этом ΠΌΠΎΠ΄ΡƒΠ»Π΅.

Π Π°Π·Π΄Π΅Π» начинаСтся с Ρ‚ΠΎΠ³ΠΎ, Ρ‡Ρ‚ΠΎ учащиСся Ρ€Π°Π·Π²ΠΈΠ²Π°ΡŽΡ‚ Π½ΠΎΠ²ΠΎΠ΅ ΠΏΠΎΠ½ΠΈΠΌΠ°Π½ΠΈΠ΅ Π΄Ρ€ΠΎΠ±Π΅ΠΉ ΠΊΠ°ΠΊ дСлСния. РаньшС ΠΎΠ½ΠΈ Π΄ΡƒΠΌΠ°Π»ΠΈ ΠΎ дробях ΠΊΠ°ΠΊ ΠΎ Ρ€Π°Π·Π΄Π΅Π»Π°Ρ… Ρ†Π΅Π»ΠΎΠ³ΠΎ Ρ€Π°Π²Π½ΠΎΠ³ΠΎ Ρ€Π°Π·ΠΌΠ΅Ρ€Π°, Π½ΠΎ здСсь ΠΎΠ½ΠΈ Ρ€Π°Π·Π²ΠΈΠ²Π°ΡŽΡ‚ ΠΏΠΎΠ½ΠΈΠΌΠ°Π½ΠΈΠ΅ Π΄Ρ€ΠΎΠ±ΠΈ ΠΊΠ°ΠΊ самой ΠΎΠΏΠ΅Ρ€Π°Ρ†ΠΈΠΈ ΠΈ ΠΏΡ€Π΅Π΄ΡΡ‚Π°Π²Π»ΡΡŽΡ‚ ΠΏΡ€ΠΎΠ±Π»Π΅ΠΌΡ‹ дСлСния ΠΊΠ°ΠΊ Π΄Ρ€ΠΎΠ±ΠΈ (5.NF.3). Π’Π΅ΠΏΠ΅Ρ€ΡŒ студСнты видят, Ρ‡Ρ‚ΠΎ остатки ΠΌΠΎΠΆΠ½ΠΎ ΠΈΠ½Ρ‚Π΅Ρ€ΠΏΡ€Π΅Ρ‚ΠΈΡ€ΠΎΠ²Π°Ρ‚ΡŒ Π΅Ρ‰Π΅ ΠΏΠΎ-Π΄Ρ€ΡƒΠ³ΠΎΠΌΡƒ, Π° ΠΈΠΌΠ΅Π½Π½ΠΎ Π΄Π΅Π»ΠΈΡ‚ΡŒ Π½Π° Π΄Π΅Π»ΠΈΡ‚Π΅Π»ΡŒ, Ρ‡Ρ‚ΠΎΠ±Ρ‹ ΠΏΠΎΠ»ΡƒΡ‡ΠΈΡ‚ΡŒ частноС со ΡΠΌΠ΅ΡˆΠ°Π½Π½Ρ‹ΠΌΠΈ числами.Π—Π°Ρ‚Π΅ΠΌ учащиСся Ρ€Π°Π·Π²ΠΈΠ²Π°ΡŽΡ‚ Π½ΠΎΠ²ΠΎΠ΅ ΠΏΠΎΠ½ΠΈΠΌΠ°Π½ΠΈΠ΅ умноТСния Π΄Ρ€ΠΎΠ±Π΅ΠΉ ΠΊΠ°ΠΊ Π΄Ρ€ΠΎΠ±Π½Ρ‹Ρ… частСй Π½Π°Π±ΠΎΡ€Π° ΠΎΠΏΡ€Π΅Π΄Π΅Π»Π΅Π½Π½ΠΎΠ³ΠΎ Ρ€Π°Π·ΠΌΠ΅Ρ€Π° (5.NF.4), Ρ‡Ρ‚ΠΎ прСдставляСт собой Π½ΠΎΠ²ΡƒΡŽ ΠΈΠ½Ρ‚Π΅Ρ€ΠΏΡ€Π΅Ρ‚Π°Ρ†ΠΈΡŽ ΠΌΡƒΠ»ΡŒΡ‚ΠΈΠΏΠ»ΠΈΠΊΠ°Ρ‚ΠΈΠ²Π½ΠΎΠ³ΠΎ сравнСния. Π‘Ρ‚ΡƒΠ΄Π΅Π½Ρ‚Ρ‹ ΠΈΡΠΏΠΎΠ»ΡŒΠ·ΡƒΡŽΡ‚ это ΠΏΠΎΠ½ΠΈΠΌΠ°Π½ΠΈΠ΅ для Ρ€Π°Π·Ρ€Π°Π±ΠΎΡ‚ΠΊΠΈ ΠΎΠ±Ρ‰ΠΈΡ… ΠΌΠ΅Ρ‚ΠΎΠ΄ΠΎΠ² умноТСния Π΄Ρ€ΠΎΠ±Π΅ΠΉ Π½Π° Ρ†Π΅Π»Ρ‹Π΅ числа ΠΈ Π΄Ρ€ΠΎΠ±ΠΈ, Π²ΠΊΠ»ΡŽΡ‡Π°Ρ ΡΠΌΠ΅ΡˆΠ°Π½Π½Ρ‹Π΅ числа. На протяТСнии этой Ρ€Π°Π±ΠΎΡ‚Ρ‹ студСнты Ρ€Π°Π·Π²ΠΈΠ²Π°ΡŽΡ‚ ΠΏΠΎΠ½ΠΈΠΌΠ°Π½ΠΈΠ΅ умноТСния ΠΊΠ°ΠΊ ΡˆΠΊΠ°Π»Ρ‹ (5.NF.5), Β«Π²Π°ΠΆΠ½ΠΎΠΉ возмоТности для студСнтов Ρ€Π°ΡΡΡƒΠΆΠ΄Π°Ρ‚ΡŒ абстрактно» (MP.2), ΠΊΠ°ΠΊ отмСчаСтся Π² Progressions (Progressions for the Common Core State Standards in Mathematics, Число ΠΈ ΠΎΠΏΠ΅Ρ€Π°Ρ†ΠΈΠΈ — Π”Ρ€ΠΎΠ±ΠΈ, 3-5, стр.14). Π—Π°Ρ‚Π΅ΠΌ учащиСся ΠΈΠ·ΡƒΡ‡Π°ΡŽΡ‚ Π΄Π΅Π»Π΅Π½ΠΈΠ΅ Π΅Π΄ΠΈΠ½ΠΈΡ‡Π½ΠΎΠΉ Π΄Ρ€ΠΎΠ±ΠΈ Π½Π° Ρ†Π΅Π»ΠΎΠ΅ число ΠΈ Ρ†Π΅Π»ΠΎΠ³ΠΎ числа Π½Π° Π΅Π΄ΠΈΠ½ΠΈΡ‡Π½ΡƒΡŽ Π΄Ρ€ΠΎΠ±ΡŒ (5.NF.7), подготавливая учащихся ΠΊ дСлСнию дробями Π²ΠΎ всСх случаях Π² 6 классС (6.NS.1). Π—Π°Ρ‚Π΅ΠΌ студСнты Ρ‚Π°ΠΊΠΆΠ΅ Ρ€Π΅ΡˆΠ°ΡŽΡ‚ мноТСство Π·Π°Π΄Π°Ρ‡ со словами, видя, Ρ‡Ρ‚ΠΎ стратСгии, ΠΊΠΎΡ‚ΠΎΡ€Ρ‹Π΅ ΠΎΠ½ΠΈ использовали для Ρ€Π΅ΡˆΠ΅Π½ΠΈΡ Π·Π°Π΄Π°Ρ‡ со словами с Ρ†Π΅Π»Ρ‹ΠΌΠΈ числами, всС Π΅Ρ‰Π΅ ΠΏΡ€ΠΈΠΌΠ΅Π½ΠΈΠΌΡ‹, Π½ΠΎ особоС Π²Π½ΠΈΠΌΠ°Π½ΠΈΠ΅ слСдуСт ΡƒΠ΄Π΅Π»ΡΡ‚ΡŒ обсуТдаСмому Ρ†Π΅Π»ΠΎΠΌΡƒ (5.NF.6, MP.4), Π° Ρ‚Π°ΠΊΠΆΠ΅ ΠΏΠΈΡΠ°Ρ‚ΡŒ ΠΈ Ρ€Π΅ΡˆΠ°Ρ‚ΡŒ выраТСния, содСрТащиС Π΄Ρ€ΠΎΠ±ΠΈ, ΠΊΠ°ΠΊ способ ΠΏΠΎΠ΄Π΄Π΅Ρ€ΠΆΠΊΠΈ основной Ρ€Π°Π±ΠΎΡ‚Ρ‹ (5.OA.1, 5.OA.2). НаконСц, учащиСся ΡΠΎΠ·Π΄Π°ΡŽΡ‚ Π»ΠΈΠ½Π΅ΠΉΠ½Ρ‹Π΅ Π³Ρ€Π°Ρ„ΠΈΠΊΠΈ для отобраТСния Π½Π°Π±ΠΎΡ€Π° Π΄Π°Π½Π½Ρ‹Ρ… ΠΈΠ·ΠΌΠ΅Ρ€Π΅Π½ΠΈΠΉ Π² долях Π΅Π΄ΠΈΠ½ΠΈΡ†Ρ‹ ΠΈ Ρ€Π΅ΡˆΠ΅Π½ΠΈΡ Π·Π°Π΄Π°Ρ‡, связанных с ΠΈΠ½Ρ„ΠΎΡ€ΠΌΠ°Ρ†ΠΈΠ΅ΠΉ, прСдставлСнной Π½Π° Π»ΠΈΠ½Π΅ΠΉΠ½Ρ‹Ρ… Π³Ρ€Π°Ρ„ΠΈΠΊΠ°Ρ… (5.MD.2), ΠΏΠΎΠ΄Π΄Π΅Ρ€ΠΆΠΈΠ²Π°ΡŽΡ‰Π΅ΠΌ кластСрном стандартС, ΠΊΠΎΡ‚ΠΎΡ€Ρ‹ΠΉ ΠΏΠΎΠ΄Π΄Π΅Ρ€ΠΆΠΈΠ²Π°Π΅Ρ‚ ΠΎΡΠ½ΠΎΠ²Π½ΡƒΡŽ Ρ€Π°Π±ΠΎΡ‚Ρƒ этого ΠΈ ΠΏΡ€Π΅Π΄Ρ‹Π΄ΡƒΡ‰Π΅Π³ΠΎ модуля. использования всСх Ρ‡Π΅Ρ‚Ρ‹Ρ€Π΅Ρ… ΠΎΠΏΠ΅Ρ€Π°Ρ†ΠΈΠΉ с дробями (5.NF).

Π’ Π‘Π»ΠΎΠΊΠ΅ 6 студСнты научатся ΡƒΠΌΠ½ΠΎΠΆΠ°Ρ‚ΡŒ ΠΈ Π΄Π΅Π»ΠΈΡ‚ΡŒ дСсятичныС Π΄Ρ€ΠΎΠ±ΠΈ, полагаясь Π½Π° своС ΠΏΠΎΠ½ΠΈΠΌΠ°Π½ΠΈΠ΅ этих ΠΎΠΏΠ΅Ρ€Π°Ρ†ΠΈΠΉ с дробями, Ρ€Π°Π·Ρ€Π°Π±ΠΎΡ‚Π°Π½Π½ΠΎΠ΅ для этого Π² этом Π±Π»ΠΎΠΊΠ΅. Π’ 6 классС ΡƒΡ‡Π΅Π½ΠΈΠΊΠΈ ΡΡ‚Π°Π»ΠΊΠΈΠ²Π°ΡŽΡ‚ΡΡ с ΠΎΡΡ‚Π°Π²ΡˆΠΈΠΌΠΈΡΡ случаями Π΄Ρ€ΠΎΠ±Π½ΠΎΠ³ΠΎ дСлСния (6.NS.1). Β«Π Π°Π±ΠΎΡ‚Π° с дробями ΠΈ ΡƒΠΌΠ½ΠΎΠΆΠ΅Π½ΠΈΠ΅ΠΌ — это ΡΡ‚Ρ€ΠΎΠΈΡ‚Π΅Π»ΡŒΠ½Ρ‹ΠΉ Π±Π»ΠΎΠΊ для Ρ€Π°Π±ΠΎΡ‚Ρ‹ с ΠΎΡ‚Π½ΠΎΡˆΠ΅Π½ΠΈΡΠΌΠΈ. Π’ 6 ΠΈ 7 классах учащиСся ΠΈΡΠΏΠΎΠ»ΡŒΠ·ΡƒΡŽΡ‚ своС ΠΏΠΎΠ½ΠΈΠΌΠ°Π½ΠΈΠ΅ Ρ†Π΅Π»Ρ‹Ρ… ΠΈ частСй, Ρ‡Ρ‚ΠΎΠ±Ρ‹ Ρ€Π°ΡΡΡƒΠΆΠ΄Π°Ρ‚ΡŒ ΠΎ ΡΠΎΠΎΡ‚Π½ΠΎΡˆΠ΅Π½ΠΈΡΡ… Π΄Π²ΡƒΡ… Π²Π΅Π»ΠΈΡ‡ΠΈΠ½, составляя ΠΈ анализируя Ρ‚Π°Π±Π»ΠΈΡ†Ρ‹ эквивалСнтных ΡΠΎΠΎΡ‚Π½ΠΎΡˆΠ΅Π½ΠΈΠΉ, ΠΈ отобраТая ΠΏΠ°Ρ€Ρ‹ ΠΈΠ· этих Ρ‚Π°Π±Π»ΠΈΡ† Π² ΠΊΠΎΠΎΡ€Π΄ΠΈΠ½Π°Ρ‚Π½ΠΎΠΉ плоскости. Π­Ρ‚ΠΈ Ρ‚Π°Π±Π»ΠΈΡ†Ρ‹ ΠΈ Π³Ρ€Π°Ρ„ΠΈΠΊΠΈ ΠΏΡ€Π΅Π΄ΡΡ‚Π°Π²Π»ΡΡŽΡ‚ собой ΠΏΡ€ΠΎΠΏΠΎΡ€Ρ†ΠΈΠΎΠ½Π°Π»ΡŒΠ½Ρ‹Π΅ ΠΎΡ‚Π½ΠΎΡˆΠ΅Π½ΠΈΡ, ΠΊΠΎΡ‚ΠΎΡ€Ρ‹Π΅ учащиСся Ρ€Π°ΡΡΠΌΠ°Ρ‚Ρ€ΠΈΠ²Π°ΡŽΡ‚ ΠΊΠ°ΠΊ Ρ„ΡƒΠ½ΠΊΡ†ΠΈΠΈ Π² 8-ΠΌ классС Β»(NF Progression, стр. 20). Π‘Ρ‚ΡƒΠ΄Π΅Π½Ρ‚Ρ‹ Π² дальнСйшСм Π±ΡƒΠ΄ΡƒΡ‚ ΠΏΠΎΠ»Π°Π³Π°Ρ‚ΡŒΡΡ Π½Π° эту ΠΎΠΏΠ΅Ρ€Π°Ρ‚ΠΈΠ²Π½ΡƒΡŽ Π±Π΅Π³Π»ΠΎΡΡ‚ΡŒ Π½Π° протяТСнии всСй ΠΎΡΡ‚Π°Π²ΡˆΠ΅ΠΉΡΡ части своСй матСматичСской ΠΊΠ°Ρ€ΡŒΠ΅Ρ€Ρ‹, ΠΎΡ‚ Π΄Ρ€ΠΎΠ±Π½Ρ‹Ρ… коэффициСнтов Π² функциях Π΄ΠΎ связи ΠΌΠ΅ΠΆΠ΄Ρƒ ΠΈΡ€Ρ€Π°Ρ†ΠΈΠΎΠ½Π°Π»ΡŒΠ½Ρ‹ΠΌΠΈ числами ΠΈ Π½Π΅ΠΏΠΎΠ²Ρ‚ΠΎΡ€ΡΡŽΡ‰ΠΈΠΌΠΈΡΡ дСсятичными Π·Π½Π°ΠΊΠ°ΠΌΠΈ.

Π’Π΅ΠΌΠΏ: 27 ΡƒΡ‡Π΅Π±Π½Ρ‹Ρ… Π΄Π½Π΅ΠΉ (24 ΡƒΡ€ΠΎΠΊΠ°, 2 Π³ΠΈΠ±ΠΊΠΈΡ… дня, 1 ΠΎΡ†Π΅Π½ΠΎΡ‡Π½Ρ‹ΠΉ дСнь)

Π§Ρ‚ΠΎΠ±Ρ‹ ΡƒΠ·Π½Π°Ρ‚ΡŒ, ΠΊΠ°ΠΊ ΠΈΠ·ΠΌΠ΅Π½ΠΈΡ‚ΡŒ Ρ‚Π΅ΠΌΠΏ Π½Π° 2020-2021 ΡƒΡ‡Π΅Π±Π½Ρ‹ΠΉ Π³ΠΎΠ΄ Π² связи с Π·Π°ΠΊΡ€Ρ‹Ρ‚ΠΈΠ΅ΠΌ школ, см. Наши Π Π΅ΠΊΠΎΠΌΠ΅Π½Π΄ΡƒΠ΅ΠΌΡ‹Π΅ ΠΊΠΎΡ€Ρ€Π΅ΠΊΡ‚ΠΈΡ€ΠΎΠ²ΠΊΠΈ объСма ΠΈ очСрСдности 5-Π³ΠΎ класса.

РСшСниС Π·Π°Π΄Π°Ρ‡ ΡƒΠΌΠ½ΠΎΠΆΠ΅Π½ΠΈΠ΅ΠΌ ΠΈ Π΄Π΅Π»Π΅Π½ΠΈΠ΅ΠΌ Π΄Ρ€ΠΎΠ±Π΅ΠΉ ΠΈ ΡΠΌΠ΅ΡˆΠ°Π½Π½Ρ‹Ρ… чисСл

Π—Π°Π΄Π°Ρ‡ΠΈ Π½Π° Π΄Ρ€ΠΎΠ±Π½Ρ‹Π΅ слова с ΠΏΠΎΠΌΠΎΡ‰ΡŒΡŽ ΠΈΠ½Ρ‚Π΅Ρ€Π°ΠΊΡ‚ΠΈΠ²Π½Ρ‹Ρ… ΡƒΠΏΡ€Π°ΠΆΠ½Π΅Π½ΠΈΠΉ

ΠŸΡ€ΠΈΠΌΠ΅Ρ€ 1. Если для изготовлСния ΠΏΠ»Π°Ρ‚ΡŒΡ трСбуСтся 5/6 ярдов Ρ‚ΠΊΠ°Π½ΠΈ, Ρ‚ΠΎ сколько ярдов потрСбуСтся для изготовлСния 8 ΠΏΠ»Π°Ρ‚ΡŒΠ΅Π²?

Анализ: Π§Ρ‚ΠΎΠ±Ρ‹ Ρ€Π΅ΡˆΠΈΡ‚ΡŒ эту ΠΏΡ€ΠΎΠ±Π»Π΅ΠΌΡƒ, ΠΌΡ‹ ΠΏΡ€Π΅ΠΎΠ±Ρ€Π°Π·ΡƒΠ΅ΠΌ Ρ†Π΅Π»ΠΎΠ΅ число Π² Π½Π΅ΠΏΡ€Π°Π²ΠΈΠ»ΡŒΠ½ΡƒΡŽ Π΄Ρ€ΠΎΠ±ΡŒ.Π—Π°Ρ‚Π΅ΠΌ ΠΌΡ‹ ΡƒΠΌΠ½ΠΎΠΆΠΈΠΌ Π΄Π²Π΅ Π΄Ρ€ΠΎΠ±ΠΈ.

РСшСниС:

ΠžΡ‚Π²Π΅Ρ‚: Для изготовлСния 8 ΠΏΠ»Π°Ρ‚ΡŒΠ΅Π² потрСбуСтся 6 ΠΈ 2/3 ярда Ρ‚ΠΊΠ°Π½ΠΈ.


ΠŸΡ€ΠΈΠΌΠ΅Ρ€ 2: Π£ Π Π΅Π½Π΅ Π±Ρ‹Π»Π° ΠΊΠΎΡ€ΠΎΠ±ΠΊΠ° кСксов, ΠΏΠΎΠ»ΠΎΠ²ΠΈΠ½Ρƒ ΠΊΠΎΡ‚ΠΎΡ€ΠΎΠΉ ΠΎΠ½Π° ΠΎΡ‚Π΄Π°Π»Π° своСму Π΄Ρ€ΡƒΠ³Ρƒ Π₯ΡƒΠ°Π½Ρƒ. Π₯ΡƒΠ°Π½ ΠΎΡ‚Π΄Π°Π» 3/4 своСй Π΄ΠΎΠ»ΠΈ своСй ΠΏΠΎΠ΄Ρ€ΡƒΠ³Π΅ Π•Π»Π΅Π½Π΅. Какая дробная Ρ‡Π°ΡΡ‚ΡŒ ΠΎΡ€ΠΈΠ³ΠΈΠ½Π°Π»ΡŒΠ½ΠΎΠΉ ΠΊΠΎΡ€ΠΎΠ±ΠΊΠΈ кСксов Π΄ΠΎΡΡ‚Π°Π»Π°ΡΡŒ Π•Π»Π΅Π½Π΅?

Анализ: Π§Ρ‚ΠΎΠ±Ρ‹ Ρ€Π΅ΡˆΠΈΡ‚ΡŒ эту ΠΏΡ€ΠΎΠ±Π»Π΅ΠΌΡƒ, ΠΌΡ‹ ΡƒΠΌΠ½ΠΎΠΆΠΈΠΌ эти Π΄Π²Π΅ Π΄Ρ€ΠΎΠ±ΠΈ.

РСшСниС:

ΠžΡ‚Π²Π΅Ρ‚: Π•Π»Π΅Π½Π΅ Π΄ΠΎΡΡ‚Π°Π»ΠΎΡΡŒ 3/8 ΠΎΡ€ΠΈΠ³ΠΈΠ½Π°Π»ΡŒΠ½ΠΎΠΉ ΠΊΠΎΡ€ΠΎΠ±ΠΊΠΈ кСксов.


ΠŸΡ€ΠΈΠΌΠ΅Ρ€ 3: Класс ΠΌΠ°Ρ‚Π΅ΠΌΠ°Ρ‚ΠΈΠΊΠΈ Нины ΠΈΠΌΠ΅Π΅Ρ‚ Π΄Π»ΠΈΠ½Ρƒ 6 ΠΈ 4/5 ΠΌΠ΅Ρ‚Ρ€Π° ΠΈ ΡˆΠΈΡ€ΠΈΠ½Ρƒ 1 ΠΈ 3/8 ΠΌΠ΅Ρ‚Ρ€Π°. Какая ΠΏΠ»ΠΎΡ‰Π°Π΄ΡŒ классной ΠΊΠΎΠΌΠ½Π°Ρ‚Ρ‹?

Анализ: Π§Ρ‚ΠΎΠ±Ρ‹ Ρ€Π΅ΡˆΠΈΡ‚ΡŒ эту ΠΏΡ€ΠΎΠ±Π»Π΅ΠΌΡƒ, ΠΌΡ‹ ΡƒΠΌΠ½ΠΎΠΆΠΈΠΌ эти ΡΠΌΠ΅ΡˆΠ°Π½Π½Ρ‹Π΅ числа. Но сначала ΠΌΡ‹ Π΄ΠΎΠ»ΠΆΠ½Ρ‹ ΠΏΡ€Π΅ΠΎΠ±Ρ€Π°Π·ΠΎΠ²Π°Ρ‚ΡŒ ΠΊΠ°ΠΆΠ΄ΠΎΠ΅ смСшанноС число Π² Π½Π΅ΠΏΡ€Π°Π²ΠΈΠ»ΡŒΠ½ΡƒΡŽ Π΄Ρ€ΠΎΠ±ΡŒ.

РСшСниС:

ΠžΡ‚Π²Π΅Ρ‚: ΠŸΠ»ΠΎΡ‰Π°Π΄ΡŒ Π°ΡƒΠ΄ΠΈΡ‚ΠΎΡ€ΠΈΠΈ 9 ΠΈ 7/20 ΠΊΠ²Π°Π΄Ρ€Π°Ρ‚Π½Ρ‹Ρ… ΠΌΠ΅Ρ‚Ρ€ΠΎΠ².


ΠŸΡ€ΠΈΠΌΠ΅Ρ€ 4: ΠŸΠ»ΠΈΡ‚ΠΊΠ° шоколада ΠΈΠΌΠ΅Π΅Ρ‚ Π΄Π»ΠΈΠ½Ρƒ 3/4 дюйма. Если ΠΎΠ½ Ρ€Π°Π·Π΄Π΅Π»Π΅Π½ Π½Π° части Π΄Π»ΠΈΠ½ΠΎΠΉ 3/8 дюйма, Ρ‚ΠΎ сколько это частСй?

Анализ: Π§Ρ‚ΠΎΠ±Ρ‹ Ρ€Π΅ΡˆΠΈΡ‚ΡŒ эту Π·Π°Π΄Π°Ρ‡Ρƒ, ΠΌΡ‹ Ρ€Π°Π·Π΄Π΅Π»ΠΈΠΌ ΠΏΠ΅Ρ€Π²ΡƒΡŽ Π΄Ρ€ΠΎΠ±ΡŒ Π½Π° Π²Ρ‚ΠΎΡ€ΡƒΡŽ.

РСшСниС:

ΠžΡ‚Π²Π΅Ρ‚: 2 ΡˆΡ‚.


ΠŸΡ€ΠΈΠΌΠ΅Ρ€ 5. Π£ элСктрика Π΅ΡΡ‚ΡŒ кусок ΠΏΡ€ΠΎΠ²ΠΎΠ΄Π° Π΄Π»ΠΈΠ½ΠΎΠΉ 4 ΠΈ 3/8 сантимСтра. Она Π΄Π΅Π»ΠΈΡ‚ ΠΏΡ€ΠΎΠ²ΠΎΠ»ΠΎΠΊΡƒ Π½Π° кусочки Π΄Π»ΠΈΠ½ΠΎΠΉ 1 ΠΈ 2/3 сантимСтра. Бколько Ρƒ Π½Π΅Π΅ ΡˆΡ‚ΡƒΠΊ?

Анализ: Π§Ρ‚ΠΎΠ±Ρ‹ Ρ€Π΅ΡˆΠΈΡ‚ΡŒ эту ΠΏΡ€ΠΎΠ±Π»Π΅ΠΌΡƒ, ΠΌΡ‹ Ρ€Π°Π·Π΄Π΅Π»ΠΈΠΌ ΠΏΠ΅Ρ€Π²ΠΎΠ΅ смСшанноС число Π½Π° Π²Ρ‚ΠΎΡ€ΠΎΠ΅.

РСшСниС:

ΠžΡ‚Π²Π΅Ρ‚: Π£ элСктрика 2 ΠΈ 5/8 куска ΠΏΡ€ΠΎΠ²ΠΎΠ΄Π°.


ΠŸΡ€ΠΈΠΌΠ΅Ρ€ 6: На складС 1 ΠΈ 3/10 ΠΌΠ΅Ρ‚Ρ€ΠΎΠ² Π»Π΅Π½Ρ‚Ρ‹. Если ΠΎΠ½ΠΈ раздСлят Π»Π΅Π½Ρ‚Ρƒ Π½Π° куски Π΄Π»ΠΈΠ½ΠΎΠΉ 5/8 ΠΌΠ΅Ρ‚Ρ€ΠΎΠ², Ρ‚ΠΎ сколько кусков Ρƒ Π½ΠΈΡ… получится?

Анализ: Π§Ρ‚ΠΎΠ±Ρ‹ Ρ€Π΅ΡˆΠΈΡ‚ΡŒ эту ΠΏΡ€ΠΎΠ±Π»Π΅ΠΌΡƒ, ΠΌΡ‹ Ρ€Π°Π·Π΄Π΅Π»ΠΈΠΌ ΠΏΠ΅Ρ€Π²ΠΎΠ΅ смСшанноС число Π½Π° Π²Ρ‚ΠΎΡ€ΠΎΠ΅. Π‘Π½Π°Ρ‡Π°Π»Π° ΠΌΡ‹ ΠΏΡ€Π΅ΠΎΠ±Ρ€Π°Π·ΡƒΠ΅ΠΌ ΠΊΠ°ΠΆΠ΄ΠΎΠ΅ смСшанноС число Π² Π½Π΅ΠΏΡ€Π°Π²ΠΈΠ»ΡŒΠ½ΡƒΡŽ Π΄Ρ€ΠΎΠ±ΡŒ.

РСшСниС:

ΠžΡ‚Π²Π΅Ρ‚: На складС Π±ΡƒΠ΄Π΅Ρ‚ 2 ΠΈ 2/25 кусков Π»Π΅Π½Ρ‚Ρ‹.


РСзюмС: Π’ этом ΡƒΡ€ΠΎΠΊΠ΅ ΠΌΡ‹ ΡƒΠ·Π½Π°Π»ΠΈ, ΠΊΠ°ΠΊ Ρ€Π΅ΡˆΠ°Ρ‚ΡŒ Π·Π°Π΄Π°Ρ‡ΠΈ со словами, связанныС с ΡƒΠΌΠ½ΠΎΠΆΠ΅Π½ΠΈΠ΅ΠΌ ΠΈ Π΄Π΅Π»Π΅Π½ΠΈΠ΅ΠΌ Π΄Ρ€ΠΎΠ±Π΅ΠΉ ΠΈ ΡΠΌΠ΅ΡˆΠ°Π½Π½Ρ‹Ρ… чисСл.


УпраТнСния

Указания: Π²Ρ‹Ρ‡Ρ‚ΠΈΡ‚Π΅ ΡΠΌΠ΅ΡˆΠ°Π½Π½Ρ‹Π΅ числа Π² ΠΊΠ°ΠΆΠ΄ΠΎΠΌ ΡƒΠΏΡ€Π°ΠΆΠ½Π΅Π½ΠΈΠΈ Π½ΠΈΠΆΠ΅. ΠžΠ±ΡΠ·Π°Ρ‚Π΅Π»ΡŒΠ½ΠΎ упроститС ваш Ρ€Π΅Π·ΡƒΠ»ΡŒΡ‚Π°Ρ‚, Ссли Π½Π΅ΠΎΠ±Ρ…ΠΎΠ΄ΠΈΠΌΠΎ. Π©Π΅Π»ΠΊΠ½ΠΈΡ‚Π΅ ΠΎΠ΄ΠΈΠ½ Ρ€Π°Π· Π² ОКНО ΠžΠ’Π’Π•Π’Π ΠΈ Π²Π²Π΅Π΄ΠΈΡ‚Π΅ свой ΠΎΡ‚Π²Π΅Ρ‚; Π·Π°Ρ‚Π΅ΠΌ Π½Π°ΠΆΠΌΠΈΡ‚Π΅ ENTER. ПослС Ρ‚ΠΎΠ³ΠΎ, ΠΊΠ°ΠΊ Π²Ρ‹ Π½Π°ΠΆΠΌΠ΅Ρ‚Π΅ ENTER, Π² Π‘Π›ΠžΠšΠ• Π Π•Π—Π£Π›Π¬Π’ΠΠ’ΠžΠ’ появится сообщСниС, ΡƒΠΊΠ°Π·Ρ‹Π²Π°ΡŽΡ‰Π΅Π΅, ΠΏΡ€Π°Π²ΠΈΠ»ΡŒΠ½Ρ‹ΠΉ ΠΈΠ»ΠΈ Π½Π΅ΠΏΡ€Π°Π²ΠΈΠ»ΡŒΠ½Ρ‹ΠΉ ваш ΠΎΡ‚Π²Π΅Ρ‚. Π§Ρ‚ΠΎΠ±Ρ‹ Π½Π°Ρ‡Π°Ρ‚ΡŒ Π·Π°Π½ΠΎΠ²ΠΎ, Π½Π°ΠΆΠΌΠΈΡ‚Π΅ ОЧИБВИВЬ.

ΠŸΡ€ΠΈΠΌΠ΅Ρ‡Π°Π½ΠΈΠ΅. Π§Ρ‚ΠΎΠ±Ρ‹ Π½Π°ΠΏΠΈΡΠ°Ρ‚ΡŒ смСшанноС число Ρ‡Π΅Ρ‚Ρ‹Ρ€Π΅ ΠΈ Π΄Π²Π΅ Ρ‚Ρ€Π΅Ρ‚ΠΈ, Π²Π²Π΅Π΄ΠΈΡ‚Π΅ Π² Ρ„ΠΎΡ€ΠΌΡƒ 4, ΠΏΡ€ΠΎΠ±Π΅Π» ΠΈ Π·Π°Ρ‚Π΅ΠΌ 2/3.

1. Одна партия ΠΏΠ΅Ρ‡Π΅Π½ΡŒΡ содСрТит 1 ΠΈ 3/4 стакана растоплСнного шоколада. Бколько Ρ‡Π°ΡˆΠ΅ΠΊ растоплСнного шоколада Π½ΡƒΠΆΠ½ΠΎ для изготовлСния 8 ΠΏΠ°Ρ€Ρ‚ΠΈΠΉ ΠΏΠ΅Ρ‡Π΅Π½ΡŒΡ?
2. Π’ΠΎΠ΄Π΄ Π²Ρ‹ΠΏΠΈΠ» 5/8 Π±Π°Π½ΠΊΠΈ сока объСмом 24 ΡƒΠ½Ρ†ΠΈΠΈ. Π›Π°ΠΉΠ»Π° Π²Ρ‹ΠΏΠΈΠ»Π° Π½Π° Ρ‚Ρ€Π΅Ρ‚ΡŒ мСньшС сока, Ρ‡Π΅ΠΌ Π’ΠΎΠ΄Π΄. Бколько ΡƒΠ½Ρ†ΠΈΠΉ Π²Ρ‹ΠΏΠΈΠ»Π° Π›ΠΈΠ»Π°?
3. ΠŸΡ€ΡΠΌΠΎΡƒΠ³ΠΎΠ»ΡŒΠ½Ρ‹ΠΉ ΠΊΠΎΠ²Ρ€ΠΈΠΊ ΠΈΠΌΠ΅Π΅Ρ‚ Π΄Π»ΠΈΠ½Ρƒ 3 ΠΈ 2/3 Ρ„ΡƒΡ‚Π° ΠΈ ΡˆΠΈΡ€ΠΈΠ½Ρƒ 2 ΠΈ 3/4 Ρ„ΡƒΡ‚Π°. Какова ΠΏΠ»ΠΎΡ‰Π°Π΄ΡŒ ΠΊΠΎΠ²Ρ€ΠΈΠΊΠ°?
4. Π£ Π”ΠΆΠ°Π½Π΅Ρ‚ 5 ΠΈ 3/4 сантимСтра Π»Π°ΠΊΡ€ΠΈΡ‡Π½ΠΈΠΊΠ°. Она Π΄Π΅Π»ΠΈΡ‚ Π»Π°ΠΊΡ€ΠΈΡ†Ρƒ Π½Π° кусочки Π΄Π»ΠΈΠ½ΠΎΠΉ 1 ΠΈ 7/8 сантимСтра. Бколько кусочков солодки Ρƒ Π½Π΅Π΅ Π±ΡƒΠ΄Π΅Ρ‚?
5. ΠšΡƒΡΠΎΠΊ Π΄Π΅Ρ€Π΅Π²Π° Π΄Π»ΠΈΠ½ΠΎΠΉ 15 Ρ„ΡƒΡ‚ΠΎΠ².Бколько 3/4 Ρ„ΡƒΡ‚Π° ΠΌΠΎΠΆΠ½ΠΎ Π²Ρ‹Ρ€Π΅Π·Π°Ρ‚ΡŒ ΠΈΠ· Π½Π΅Π³ΠΎ?

Π’Π°ΠΆΠ½Ρ‹Π΅ матСматичСскиС Π½Π°Π²Ρ‹ΠΊΠΈ для пятиклассников

Π₯ΠΎΡ‚ΠΈΡ‚Π΅ ΠΏΠΎΠΌΠΎΡ‡ΡŒ своСму пятикласснику ΠΎΡΠ²ΠΎΠΈΡ‚ΡŒ ΠΌΠ°Ρ‚Π΅ΠΌΠ°Ρ‚ΠΈΠΊΡƒ? Π’ΠΎΡ‚ Π½Π΅ΠΊΠΎΡ‚ΠΎΡ€Ρ‹Π΅ ΠΈΠ· Π½Π°Π²Ρ‹ΠΊΠΎΠ², ΠΊΠΎΡ‚ΠΎΡ€Ρ‹Π΅ ваш пятиклассник Π±ΡƒΠ΄Π΅Ρ‚ ΠΈΠ·ΡƒΡ‡Π°Ρ‚ΡŒ Π² классС.

Π‘Π»ΠΎΠΆΠ΅Π½ΠΈΠ΅, Π²Ρ‹Ρ‡ΠΈΡ‚Π°Π½ΠΈΠ΅, ΡƒΠΌΠ½ΠΎΠΆΠ΅Π½ΠΈΠ΅ ΠΈ Π΄Π΅Π»Π΅Π½ΠΈΠ΅

ΠœΠ½ΠΎΠ³ΠΎΠ·Π½Π°Ρ‡Π½Ρ‹Π΅ Ρ†Π΅Π»Ρ‹Π΅ числа

Быстро ΠΈ Ρ‚ΠΎΡ‡Π½ΠΎ ΡƒΠΌΠ½ΠΎΠΆΠ°ΠΉΡ‚Π΅ ΠΌΠ½ΠΎΠ³ΠΎΠ·Π½Π°Ρ‡Π½Ρ‹Π΅ Ρ†Π΅Π»Ρ‹Π΅ числа. Π Π°Π·Π΄Π΅Π»ΠΈΡ‚Π΅ Ρ†Π΅Π»Ρ‹Π΅ числа (Π΄ΠΎ Ρ‡Π΅Ρ‚Ρ‹Ρ€Π΅Ρ… Ρ†ΠΈΡ„Ρ€) Π½Π° Π΄Π²ΡƒΠ·Π½Π°Ρ‡Π½Ρ‹Π΅ числа.

ΠŸΡ€ΠΈΠΌΠ΅Ρ€:

Π Π΅ΡˆΠΈΡ‚ΡŒ 4,824 Γ· 12 =?

ΠžΠ±ΡŠΡΡΠ½ΠΈΡ‚Π΅ ΠΈΠ»ΠΈ ΠΏΡ€ΠΎΠΈΠ»Π»ΡŽΡΡ‚Ρ€ΠΈΡ€ΡƒΠΉΡ‚Π΅, ΠΊΠ°ΠΊ Π²Ρ‹ Ρ€Π΅ΡˆΠΈΠ»ΠΈ эту ΠΏΡ€ΠΎΠ±Π»Π΅ΠΌΡƒ.

Π‘ΠΎΠ²Π΅Ρ‚: Π²Ρ‹Π΄Π΅Π»ΠΈΡ‚Π΅ Ρ€Π΅Π°Π»ΡŒΠ½Ρ‹Π΅ ΠΏΡ€ΠΈΠΌΠ΅Ρ€Ρ‹ использования ΠΌΠ°Ρ‚Π΅ΠΌΠ°Ρ‚ΠΈΠΊΠΈ.

По ΠΌΠ΅Ρ€Π΅ Ρ‚ΠΎΠ³ΠΎ, ΠΊΠ°ΠΊ ΠΌΠ°Ρ‚Π΅ΠΌΠ°Ρ‚ΠΈΠΊΠ°, ΠΊΠΎΡ‚ΠΎΡ€ΡƒΡŽ ΠΎΠ½ΠΈ ΠΈΠ·ΡƒΡ‡Π°ΡŽΡ‚, становится Π±ΠΎΠ»Π΅Π΅ слоТной ΠΈ ΠΌΠ΅Π½Π΅Π΅ ΠΎΡ‡Π΅Π²ΠΈΠ΄Π½ΠΎ связанной с ΠΈΡ… повсСднСвным ΠΎΠΏΡ‹Ρ‚ΠΎΠΌ, Ρƒ Π½Π΅ΠΊΠΎΡ‚ΠΎΡ€Ρ‹Ρ… Π΄Π΅Ρ‚Π΅ΠΉ Π½Π°Ρ‡ΠΈΠ½Π°Π΅Ρ‚ Ρ€Π°Π·Π²ΠΈΠ²Π°Ρ‚ΡŒΡΡ матСматичСская Ρ‚Ρ€Π΅Π²ΠΎΠ³Π°. Π’Π°ΠΆΠ½ΠΎ, Ρ‡Ρ‚ΠΎΠ±Ρ‹ ваш Ρ€Π΅Π±Π΅Π½ΠΎΠΊ занимался ΠΌΠ°Ρ‚Π΅ΠΌΠ°Ρ‚ΠΈΠΊΠΎΠΉ ΠΈ ΠΏΠΎΠΌΠΎΠ³Π°Π» Π΅ΠΌΡƒ ΠΏΠΎΠ½ΡΡ‚ΡŒ, ΠΊΠ°ΠΊ Π² Ρ€Π΅Π°Π»ΡŒΠ½ΠΎΠΉ ΠΆΠΈΠ·Π½ΠΈ ΠΏΡ€ΠΈΠΌΠ΅Π½ΡΡŽΡ‚ΡΡ ΠΊΠΎΠ½Ρ†Π΅ΠΏΡ†ΠΈΠΈ, ΠΊΠΎΡ‚ΠΎΡ€Ρ‹Π΅ Ρ€Π΅Π±Π΅Π½ΠΎΠΊ ΠΈΠ·ΡƒΡ‡Π°Π΅Ρ‚ Π² школС.БоставлСниС Π±ΡŽΠ΄ΠΆΠ΅Ρ‚Π° Π½Π° ΡˆΠΊΠΎΠ»ΡŒΠ½Ρ‹Π΅ принадлСТности ΠΈΠ»ΠΈ ΠΈΡ… СТСмСсячноС пособиС — ΠΎΠ΄ΠΈΠ½ ΠΈΠ· способов ΠΏΡ€Π°ΠΊΡ‚ΠΈΠΊΠΎΠ²Π°Ρ‚ΡŒ слоТСниС ΠΈ Π²Ρ‹Ρ‡ΠΈΡ‚Π°Π½ΠΈΠ΅. ΠŸΠΎΠΏΡ€ΠΎΡΠΈΡ‚Π΅ ΠΈΡ… ΠΏΠΎΠΌΠΎΡ‡ΡŒ Π²Π°ΠΌ с ΠΏΡ€ΠΈΠ³ΠΎΡ‚ΠΎΠ²Π»Π΅Π½ΠΈΠ΅ΠΌ ΠΈΠ»ΠΈ Π²Ρ‹ΠΏΠ΅Ρ‡ΠΊΠΎΠΉ, Ρ‡Ρ‚ΠΎΠ±Ρ‹ ΠΏΠΎΠΊΠ°Π·Π°Ρ‚ΡŒ, ΠΊΠ°ΠΊ Ρ€Π°Π±ΠΎΡ‚Π°ΡŽΡ‚ Π΄Ρ€ΠΎΠ±ΠΈ. ΠŸΠΎΠΌΠΎΠ³Π°Ρ‚ΡŒ Ρ€Π°ΡΡΡ‡ΠΈΡ‚Ρ‹Π²Π°Ρ‚ΡŒ Ρ†Π΅Π½Ρ‹ ΠΏΡ€ΠΈ ΠΏΠΎΠΊΡƒΠΏΠΊΠ΅ ΠΏΡ€ΠΎΠ΄ΡƒΠΊΡ‚ΠΎΠ² — Ρ‚ΠΎΠΆΠ΅ Ρ…ΠΎΡ€ΠΎΡˆΠ°Ρ ΠΏΡ€Π°ΠΊΡ‚ΠΈΠΊΠ°.

БвязанныС

ПониманиС разряда

Π Π°ΡΡˆΠΈΡ€ΡŒΡ‚Π΅ ΠΏΠΎΠ½ΠΈΠΌΠ°Π½ΠΈΠ΅ разряда: Π² ΠΌΠ½ΠΎΠ³ΠΎΠ·Π½Π°Ρ‡Π½ΠΎΠΌ числС Ρ†ΠΈΡ„Ρ€Π° Π² ΠΎΠ΄Π½ΠΎΠΌ мСстС прСдставляСт 1⁄10 Ρ‚ΠΎΠ³ΠΎ, Ρ‡Ρ‚ΠΎ ΠΎΠ½Π° прСдставляСт Π² мСстС слСва ΠΎΡ‚ Π½Π΅Π³ΠΎ, ΠΈ Π² 10 Ρ€Π°Π· большС ΠΊΠ°ΠΊ ΠΎΠ½ ΠΈΠ·ΠΎΠ±Ρ€Π°ΠΆΠ΅Π½ справа ΠΎΡ‚ Π½Π΅Π³ΠΎ.

Π‘Ρ€Π°Π²Π½Π΅Π½ΠΈΠ΅ дСсятичных Π·Π½Π°ΠΊΠΎΠ²

Π§Ρ‚Π΅Π½ΠΈΠ΅, запись ΠΈ сравнСниС дСсятичных Π·Π½Π°ΠΊΠΎΠ² с разрядами тысячных, ΠΈΡΠΏΠΎΠ»ΡŒΠ·ΡƒΡ символы> (большС Ρ‡Π΅ΠΌ) ΠΈ <(мСньшС Ρ‡Π΅ΠΌ). НапримСр:

  • ΠŸΡ€ΠΎΡ‡Ρ‚ΠΈΡ‚Π΅ это дСсятичноС число: 23,002.
  • Π—Π°ΠΏΠΈΡˆΠΈΡ‚Π΅ Π΄Π²Π΅ ΠΈ ΡˆΠ΅ΡΡ‚ΡŒΠ΄Π΅ΡΡΡ‚ Π΄Π²Π΅ тысячныС Π² Π²ΠΈΠ΄Π΅ дСсятичного числа.
  • Какой Π·Π½Π°ΠΊ ΠΏΠΎΠ΄Ρ‚Π²Π΅Ρ€ΠΆΠ΄Π°Π΅Ρ‚ это ΡƒΡ‚Π²Π΅Ρ€ΠΆΠ΄Π΅Π½ΠΈΠ΅: 5,389 _? _ 5,420
  • Π˜ΡΡΠ»Π΅Π΄ΠΎΠ²Π°Ρ‚Π΅Π»ΡŒ измСряСт количСство Π±Π°ΠΊΡ‚Π΅Ρ€ΠΈΠΉ, Π²Ρ‹Ρ€ΠΎΡΡˆΠΈΡ… Π½Π° ΠΎΠ±Ρ€Π°Π·Ρ†Π°Ρ… Π½Π΅ΠΎΡ…Π»Π°ΠΆΠ΄Π΅Π½Π½Ρ‹Ρ… ΠΏΡ€ΠΎΠ΄ΡƒΠΊΡ‚ΠΎΠ². Π’Π°Ρˆ Ρ€Π΅Π±Π΅Π½ΠΎΠΊ насчитываСт 73.343 ΠΌΠΈΠ»Π»ΠΈΠΎΠ½Π° Π±Π°ΠΊΡ‚Π΅Ρ€ΠΈΠΉ Π² ΠΎΠ±Ρ€Π°Π·Ρ†Π΅ A, 73,431 ΠΌΠΈΠ»Π»ΠΈΠΎΠ½Π° Π±Π°ΠΊΡ‚Π΅Ρ€ΠΈΠΉ Π² ΠΎΠ±Ρ€Π°Π·Ρ†Π΅ B ΠΈ 74,399 ΠΌΠΈΠ»Π»ΠΈΠΎΠ½Π° Π±Π°ΠΊΡ‚Π΅Ρ€ΠΈΠΉ Π² ΠΎΠ±Ρ€Π°Π·Ρ†Π΅ C. РасполоТитС ΠΎΠ±Ρ€Π°Π·Ρ†Ρ‹ Π² порядкС ΠΎΡ‚ наибольшСго количСства Π±Π°ΠΊΡ‚Π΅Ρ€ΠΈΠΉ ΠΊ Π½Π°ΠΈΠΌΠ΅Π½ΡŒΡˆΠ΅ΠΌΡƒ. ΠžΠ±ΡŠΡΡΠ½ΠΈΡ‚Π΅ ΠΈΠ»ΠΈ ΠΏΡ€ΠΎΠΈΠ»Π»ΡŽΡΡ‚Ρ€ΠΈΡ€ΡƒΠΉΡ‚Π΅, ΠΊΠ°ΠΊ Π²Ρ‹ ΠΏΡ€ΠΈΠ²ΠΎΠ΄ΠΈΡ‚Π΅ эти ΠΎΠ±Ρ€Π°Π·Ρ†Ρ‹ Π² порядок.

БвязанныС

ДСсятичныС Π΄ΠΎΠ»ΠΈ Π΄ΠΎ сотых

Π‘Π»ΠΎΠΆΠ΅Π½ΠΈΠ΅, Π²Ρ‹Ρ‡ΠΈΡ‚Π°Π½ΠΈΠ΅, ΡƒΠΌΠ½ΠΎΠΆΠ΅Π½ΠΈΠ΅ ΠΈ Π΄Π΅Π»Π΅Π½ΠΈΠ΅ дСсятичных Π΄ΠΎΠ»Π΅ΠΉ Π΄ΠΎ сотых.

Π‘ΠΎΠ²Π΅Ρ‚: ΠΏΠΎΡ‚Ρ€Π΅Π½ΠΈΡ€ΡƒΠΉΡ‚Π΅ΡΡŒ Π² вычислСниях с использованиСм дСсятичных Π·Π½Π°ΠΊΠΎΠ².

БвяТитС Ρ€Π°Π±ΠΎΡ‚Ρƒ с дСсятичными Π·Π½Π°ΠΊΠ°ΠΌΠΈ, ΠΊΠΎΡ‚ΠΎΡ€ΡƒΡŽ ваш Ρ€Π΅Π±Π΅Π½ΠΎΠΊ Π΄Π΅Π»Π°Π΅Ρ‚ Π² классС, с Ρ€Π΅Π°Π»ΡŒΠ½Ρ‹ΠΌ ΠΌΠΈΡ€ΠΎΠΌ, поощряя ΠΈΡ… Π΄Π΅Π»Π°Ρ‚ΡŒ ΠΏΠΎΠΊΡƒΠΏΠΊΠΈ ΠΏΠΎ Π²Ρ‹Π³ΠΎΠ΄Π½Ρ‹ΠΌ Ρ†Π΅Π½Π°ΠΌ.ΠŸΠΎΠΏΡ€ΠΎΡΠΈΡ‚Π΅ ΠΈΡ… Ρ€Π°Π·Π΄Π΅Π»ΠΈΡ‚ΡŒ ΡΡ‚ΠΎΠΈΠΌΠΎΡΡ‚ΡŒ Ρ‚ΠΎΠ²Π°Ρ€ΠΎΠ², ΡƒΠΏΠ°ΠΊΠΎΠ²Π°Π½Π½Ρ‹Ρ… ΠΎΠΏΡ‚ΠΎΠΌ, Π½Π° количСство ΠΎΡ‚Π΄Π΅Π»ΡŒΠ½Ρ‹Ρ… Ρ‚ΠΎΠ²Π°Ρ€ΠΎΠ², Ρ‡Ρ‚ΠΎΠ±Ρ‹ ΠΎΠΏΡ€Π΅Π΄Π΅Π»ΠΈΡ‚ΡŒ ΡΡ‚ΠΎΠΈΠΌΠΎΡΡ‚ΡŒ ΠΊΠ°ΠΆΠ΄ΠΎΠ³ΠΎ Ρ‚ΠΎΠ²Π°Ρ€Π°. Π˜Ρ‚Π°ΠΊ, сколько Π²Ρ‹ ΠΏΠ»Π°Ρ‚ΠΈΡ‚Π΅ Π·Π° Ρ€ΡƒΠ»ΠΎΠ½ Π±ΡƒΠΌΠ°ΠΆΠ½ΠΎΠ³ΠΎ ΠΏΠΎΠ»ΠΎΡ‚Π΅Π½Ρ†Π° ΠΈΠ»ΠΈ Π·Π° Π±Π°Π½ΠΊΡƒ Π³Π°Π·ΠΈΡ€ΠΎΠ²ΠΊΠΈ ΠΏΡ€ΠΈ ΠΏΠΎΠΊΡƒΠΏΠΊΠ΅ ΠΎΠΏΡ‚ΠΎΠΌ? Или попроситС Ρ€Π΅Π±Π΅Π½ΠΊΠ° ΠΏΠΎΠ΄ΡΡ‡ΠΈΡ‚Π°Ρ‚ΡŒ, сколько Π²Ρ‹ сэкономитС Π½Π° ΠΊΠ°ΠΆΠ΄ΠΎΠΌ Ρ‚ΠΎΠ²Π°Ρ€Π΅, Ссли Ρ†Π΅Π½Ρ‹ со скидкой ΠΏΡ€Π΅Π΄ΠΏΠΎΠ»Π°Π³Π°ΡŽΡ‚ ΠΎΠΏΡ‚ΠΎΠ²Ρ‹Π΅ скидки.

ΠŸΠΎΠΊΠ°Π·Π°Ρ‚Π΅Π»ΠΈ стСпСни

Π Π°Π·Π±Π΅Ρ€ΠΈΡ‚Π΅ΡΡŒ, Ρ‡Ρ‚ΠΎ Ρ‚Π°ΠΊΠΎΠ΅ ΠΏΠΎΠΊΠ°Π·Π°Ρ‚Π΅Π»ΡŒ стСпСни. НапримСр, Β«2Β» Π² 10Β² ΡƒΠΊΠ°Π·Ρ‹Π²Π°Π΅Ρ‚, сколько Ρ€Π°Π· Π½ΡƒΠΆΠ½ΠΎ ΡƒΠΌΠ½ΠΎΠΆΠΈΡ‚ΡŒ число само Π½Π° сСбя. 10Β² ΠΌΠΎΠΆΠ½ΠΎ Ρ‡ΠΈΡ‚Π°Ρ‚ΡŒ ΠΊΠ°ΠΊ Β«10 Π² стСпСни 2Β», Β«10 Π² стСпСни 2Β» ΠΈΠ»ΠΈ Β«10 Π² ΠΊΠ²Π°Π΄Ρ€Π°Ρ‚Π΅Β» ΠΈ ΠΎΠ·Π½Π°Ρ‡Π°Π΅Ρ‚ 10 x 10 ΠΈΠ»ΠΈ 100.10Β³ (ΠΈΠ»ΠΈ Β«10 Π² Ρ‚Ρ€Π΅Ρ‚ΡŒΠ΅ΠΉ стСпСни» ΠΈΠ»ΠΈ Β«10 Π² ΠΊΡƒΠ±Π΅Β») ΠΎΠ·Π½Π°Ρ‡Π°Π΅Ρ‚ 10 x 10 x 10, ΠΈΠ»ΠΈ 1000.

Π”Ρ€ΠΎΠ±ΠΈ

РСшСниС Π·Π°Π΄Π°Ρ‡ со словами

РСшСниС Π·Π°Π΄Π°Ρ‡ со словами, Π²ΠΊΠ»ΡŽΡ‡Π°ΡŽΡ‰ΠΈΡ… слоТСниС ΠΈ Π²Ρ‹Ρ‡ΠΈΡ‚Π°Π½ΠΈΠ΅ Π΄Ρ€ΠΎΠ±Π΅ΠΉ.

ΠŸΡ€ΠΈΠΌΠ΅Ρ€:

Π£Ρ‡Π΅Π½ΠΈΠΊΠΈ пятого класса ΡΠΎΠ±ΠΈΡ€Π°ΡŽΡ‚ ΠΏΠ°Π·Π» ΠΈΠ· 600 Π΄Π΅Ρ‚Π°Π»Π΅ΠΉ. Они Π½Π°Ρ‡Π°Π»ΠΈ Π²Ρ‡Π΅Ρ€Π° ΠΈ собрали 100 частСй — всСго ΠΎΠ΄Π½Ρƒ ΡˆΠ΅ΡΡ‚ΡƒΡŽ (1⁄6) Π³ΠΎΠ»ΠΎΠ²ΠΎΠ»ΠΎΠΌΠΊΠΈ. БСгодня ΠΈΡ… собрано 400 ΡˆΡ‚ΡƒΠΊ. Какая Ρ‡Π°ΡΡ‚ΡŒ Π³ΠΎΠ»ΠΎΠ²ΠΎΠ»ΠΎΠΌΠΊΠΈ Π·Π°Π²Π΅Ρ€ΡˆΠ΅Π½Π°? НарисуйтС ΠΊΠ°Ρ€Ρ‚ΠΈΠ½ΠΊΡƒ И Π·Π°ΠΏΠΈΡˆΠΈΡ‚Π΅ ΠΌΠ°Ρ‚Π΅ΠΌΠ°Ρ‚ΠΈΠΊΡƒ, Ρ‡Ρ‚ΠΎΠ±Ρ‹ ΠΏΠΎΠΊΠ°Π·Π°Ρ‚ΡŒ, ΠΊΠ°ΠΊ Π²Ρ‹ Ρ€Π΅ΡˆΠΈΠ»ΠΈ Π·Π°Π΄Π°Ρ‡Ρƒ.

Π‘ΠΎΠ²Π΅Ρ‚: Π²Ρ‹Π΄Π΅Π»ΠΈΡ‚Π΅ Ρ€Π΅Π°Π»ΡŒΠ½Ρ‹Π΅ ΠΏΡ€ΠΈΠΌΠ΅Ρ€Ρ‹ использования ΠΌΠ°Ρ‚Π΅ΠΌΠ°Ρ‚ΠΈΠΊΠΈ.

По ΠΌΠ΅Ρ€Π΅ Ρ‚ΠΎΠ³ΠΎ, ΠΊΠ°ΠΊ ΠΌΠ°Ρ‚Π΅ΠΌΠ°Ρ‚ΠΈΠΊΠ°, ΠΊΠΎΡ‚ΠΎΡ€ΡƒΡŽ ΠΎΠ½ΠΈ ΠΈΠ·ΡƒΡ‡Π°ΡŽΡ‚, становится Π±ΠΎΠ»Π΅Π΅ слоТной ΠΈ ΠΌΠ΅Π½Π΅Π΅ ΠΎΡ‡Π΅Π²ΠΈΠ΄Π½ΠΎ связанной с ΠΈΡ… повсСднСвным ΠΎΠΏΡ‹Ρ‚ΠΎΠΌ, Ρƒ Π½Π΅ΠΊΠΎΡ‚ΠΎΡ€Ρ‹Ρ… Π΄Π΅Ρ‚Π΅ΠΉ Π½Π°Ρ‡ΠΈΠ½Π°Π΅Ρ‚ Ρ€Π°Π·Π²ΠΈΠ²Π°Ρ‚ΡŒΡΡ матСматичСская Ρ‚Ρ€Π΅Π²ΠΎΠ³Π°. Π’Π°ΠΆΠ½ΠΎ, Ρ‡Ρ‚ΠΎΠ±Ρ‹ ваш Ρ€Π΅Π±Π΅Π½ΠΎΠΊ занимался ΠΌΠ°Ρ‚Π΅ΠΌΠ°Ρ‚ΠΈΠΊΠΎΠΉ ΠΈ ΠΏΠΎΠΌΠΎΠ³Π°Π» Π΅ΠΌΡƒ ΠΏΠΎΠ½ΡΡ‚ΡŒ, ΠΊΠ°ΠΊ Π² Ρ€Π΅Π°Π»ΡŒΠ½ΠΎΠΉ ΠΆΠΈΠ·Π½ΠΈ ΠΏΡ€ΠΈΠΌΠ΅Π½ΡΡŽΡ‚ΡΡ ΠΊΠΎΠ½Ρ†Π΅ΠΏΡ†ΠΈΠΈ, ΠΊΠΎΡ‚ΠΎΡ€Ρ‹Π΅ ΠΎΠ½ ΠΈΠ·ΡƒΡ‡Π°Π΅Ρ‚ Π² школС. БоставлСниС Π±ΡŽΠ΄ΠΆΠ΅Ρ‚Π° Π½Π° ΡˆΠΊΠΎΠ»ΡŒΠ½Ρ‹Π΅ принадлСТности ΠΈΠ»ΠΈ СТСмСсячноС пособиС — ΠΎΠ΄ΠΈΠ½ ΠΈΠ· способов для Π½Π΅Π΅ ΠΏΡ€Π°ΠΊΡ‚ΠΈΠΊΠΎΠ²Π°Ρ‚ΡŒ слоТСниС ΠΈ Π²Ρ‹Ρ‡ΠΈΡ‚Π°Π½ΠΈΠ΅.Если Π²Ρ‹ попроситС Π΅Π΅ ΠΏΠΎΠΌΠΎΡ‡ΡŒ Π²Π°ΠΌ с ΠΏΡ€ΠΈΠ³ΠΎΡ‚ΠΎΠ²Π»Π΅Π½ΠΈΠ΅ΠΌ ΠΈΠ»ΠΈ Π²Ρ‹ΠΏΠ΅Ρ‡ΠΊΠΎΠΉ, это ΠΏΠΎΠΊΠ°ΠΆΠ΅Ρ‚ Π΅ΠΉ, ΠΊΠ°ΠΊ Ρ€Π°Π±ΠΎΡ‚Π°ΡŽΡ‚ Π΄Ρ€ΠΎΠ±ΠΈ. ΠŸΠΎΠΌΠΎΠ³Π°Ρ‚ΡŒ Ρ€Π°ΡΡΡ‡ΠΈΡ‚Ρ‹Π²Π°Ρ‚ΡŒ Ρ†Π΅Π½Ρ‹ ΠΏΡ€ΠΈ ΠΏΠΎΠΊΡƒΠΏΠΊΠ΅ ΠΏΡ€ΠΎΠ΄ΡƒΠΊΡ‚ΠΎΠ² — Ρ‚ΠΎΠΆΠ΅ Ρ…ΠΎΡ€ΠΎΡˆΠ°Ρ ΠΏΡ€Π°ΠΊΡ‚ΠΈΠΊΠ°.

НахоТдСниС ΠΎΠ±Ρ‰Π΅Π³ΠΎ знамСнатСля

Π Π΅ΡˆΠΈΡ‚Π΅ Π·Π°Π΄Π°Ρ‡ΠΈ со словами, Π²ΠΊΠ»ΡŽΡ‡Π°ΡŽΡ‰ΠΈΠ΅ слоТСниС ΠΈ Π²Ρ‹Ρ‡ΠΈΡ‚Π°Π½ΠΈΠ΅ Π΄Ρ€ΠΎΠ±Π΅ΠΉ с Ρ€Π°Π·Π½Ρ‹ΠΌΠΈ знамСнатСлями (Π½ΠΈΠΆΠ½ΠΈΠ΅ числа), прСобразовывая ΠΈΡ… Π² Π΄Ρ€ΠΎΠ±ΠΈ с ΠΎΠ΄ΠΈΠ½Π°ΠΊΠΎΠ²Ρ‹ΠΌ Π·Π½Π°ΠΌΠ΅Π½Π°Ρ‚Π΅Π»Π΅ΠΌ, Π½Π°Π·Ρ‹Π²Π°Π΅ΠΌΡ‹Π΅ ΠΎΠ±Ρ‰ΠΈΠΌ Π·Π½Π°ΠΌΠ΅Π½Π°Ρ‚Π΅Π»Π΅ΠΌ.

ΠŸΡ€ΠΈΠΌΠ΅Ρ€:

Бамая высокая Π΄Π΅Π²ΠΎΡ‡ΠΊΠ° Π² пятом классС ΠΈΠΌΠ΅Π΅Ρ‚ рост 51 7⁄8 дюйма.Π‘Π°ΠΌΡ‹ΠΉ высокий ΠΌΠ°Π»ΡŒΡ‡ΠΈΠΊ Π² пятом классС ΠΈΠΌΠ΅Π΅Ρ‚ рост 49 1⁄2 дюйма. Какая Ρ€Π°Π·Π½ΠΈΡ†Π° Π² ΠΈΡ… ростС?

ПослС Π²Π΅Ρ‡Π΅Ρ€ΠΈΠ½ΠΊΠΈ ΠΎΡΡ‚Π°Π»ΠΈΡΡŒ Π΄Π²Π΅ Ρ‡Π°ΡˆΠΊΠΈ Π»ΠΈΠΌΠΎΠ½Π°Π΄Π°. Π’ ΠΎΠ΄Π½ΠΎΠΉ мискС 1⁄3 Π³Π°Π»Π»ΠΎΠ½Π°. Π’ Π΄Ρ€ΡƒΠ³ΠΎΠΌ — 1⁄2 Π³Π°Π»Π»ΠΎΠ½Π° Π»ΠΈΠΌΠΎΠ½Π°Π΄Π°. Π”Ρ€ΡƒΠ³ Π³ΠΎΠ²ΠΎΡ€ΠΈΡ‚, Ρ‡Ρ‚ΠΎ Π½Π΅ стоит ΠΏΡ‹Ρ‚Π°Ρ‚ΡŒΡΡ ΠΎΠ±ΡŠΠ΅Π΄ΠΈΠ½ΠΈΡ‚ΡŒ ΠΈΡ… Π² 1-Π³Π°Π»Π»ΠΎΠ½Π½Ρ‹ΠΉ ΠΊΠΎΠ½Ρ‚Π΅ΠΉΠ½Π΅Ρ€, ΠΏΠΎΡ‚ΠΎΠΌΡƒ Ρ‡Ρ‚ΠΎ Π»ΠΈΠΌΠΎΠ½Π°Π΄ Π²Ρ‹Ρ‚Π΅Ρ‡Π΅Ρ‚ Π½Π°Π²Π΅Ρ€Ρ…. Π’Ρ‹ согласны? ΠŸΠΎΡ‡Π΅ΠΌΡƒ ΠΈΠ»ΠΈ ΠΏΠΎΡ‡Π΅ΠΌΡƒ Π½Π΅Ρ‚?

Π£ΠΌΠ½ΠΎΠΆΠ΅Π½ΠΈΠ΅ Π΄Ρ€ΠΎΠ±Π΅ΠΉ

Π Π΅ΡˆΠ°ΠΉΡ‚Π΅ Π·Π°Π΄Π°Ρ‡ΠΈ со словами, Π²ΠΊΠ»ΡŽΡ‡Π°ΡŽΡ‰ΠΈΠ΅ ΡƒΠΌΠ½ΠΎΠΆΠ΅Π½ΠΈΠ΅ Π΄Ρ€ΠΎΠ±Π΅ΠΉ Π½Π° Π΄Ρ€ΡƒΠ³ΠΈΠ΅ Π΄Ρ€ΠΎΠ±ΠΈ ΠΈ ΡƒΠΌΠ½ΠΎΠΆΠ΅Π½ΠΈΠ΅ Π΄Ρ€ΠΎΠ±Π΅ΠΉ Π½Π° ΡΠΌΠ΅ΡˆΠ°Π½Π½Ρ‹Π΅ числа (Ρ†Π΅Π»ΠΎΠ΅ число ΠΈ Π΄Ρ€ΠΎΠ±ΡŒ, Π½Π°ΠΏΡ€ΠΈΠΌΠ΅Ρ€ 11⁄4 ΠΈΠ»ΠΈ 21⁄2).

ΠŸΡ€ΠΈΠΌΠ΅Ρ€:

  • Π’ оркСстрС срСднСй ΡˆΠΊΠΎΠ»Ρ‹ 1⁄3 учащихся-ΠΌΡƒΠ·Ρ‹ΠΊΠ°Π½Ρ‚ΠΎΠ² ΠΈΠ³Ρ€Π°ΡŽΡ‚ Π½Π° струнных инструмСнтах. Из ΡƒΡ‡Π΅Π½ΠΈΠΊΠΎΠ², ΠΈΠ³Ρ€Π°ΡŽΡ‰ΠΈΡ… Π½Π° струнных инструмСнтах, 3⁄4 ΠΈΠ³Ρ€Π°ΡŽΡ‚ Π½Π° скрипкС. Какая Ρ‡Π°ΡΡ‚ΡŒ оркСстра ΠΈΠ³Ρ€Π°Π΅Ρ‚ Π½Π° скрипкС?
  • Π£Ρ‚Ρ€ΠΎΠΌ Π²ΠΎ врСмя экскурсии Π² яблонСвый сад пятиклассники собрали 4⁄5 Π±ΡƒΡˆΠ΅Π»Ρ яблок. ПослС ΠΎΠ±Π΅Π΄Π° Π² полдСнь ΠΎΠ½ΠΈ собрали Π² 2,5 Ρ€Π°Π·Π° большС яблок. УмСстятся Π»ΠΈ всС яблоки, собранныС ΠΈΠΌΠΈ Π΄Π½Π΅ΠΌ, Π² ящик Π½Π° 2 Π±ΡƒΡˆΠ΅Π»Ρ? ΠžΡ‚ΠΊΡƒΠ΄Π° Π²Ρ‹ Π·Π½Π°Π΅Ρ‚Π΅?

Π‘ΠΎΠ²Π΅Ρ‚: ΠΏΠΎΡ‚Ρ€Π΅Π½ΠΈΡ€ΡƒΠΉΡ‚Π΅ΡΡŒ ΠΈΡΠΏΠΎΠ»ΡŒΠ·ΠΎΠ²Π°Ρ‚ΡŒ Π΄Ρ€ΠΎΠ±ΠΈ.

ΠŸΠΎΠΌΠΎΠ³ΠΈΡ‚Π΅ своСму Ρ€Π΅Π±Π΅Π½ΠΊΡƒ ΠΏΠΎΠ·Π½Π°ΠΊΠΎΠΌΠΈΡ‚ΡŒΡΡ с дробями, попросив Π΅Π³ΠΎ ΠΌΠ°ΡΡˆΡ‚Π°Π±ΠΈΡ€ΠΎΠ²Π°Ρ‚ΡŒ Ρ€Π΅Ρ†Π΅ΠΏΡ‚Ρ‹ для вашСй сСмьи. ΠŸΡƒΡΡ‚ΡŒ ΠΎΠ½ΠΈ Π½Π°Ρ‡Π½ΡƒΡ‚ с Ρ‚ΠΎΠ³ΠΎ, Ρ‡Ρ‚ΠΎ сократят ΠΈΠ»ΠΈ удвоят Ρ€Π΅Ρ†Π΅ΠΏΡ‚. Когда ΠΎΠ½ΠΈ ΠΏΠΎΡ‡ΡƒΠ²ΡΡ‚Π²ΡƒΡŽΡ‚ сСбя ΠΊΠΎΠΌΡ„ΠΎΡ€Ρ‚Π½ΠΎ, попроситС ΠΈΡ… ΠΏΡ€Π΅ΠΎΠ±Ρ€Π°Π·ΠΎΠ²Π°Ρ‚ΡŒ Π΅Π³ΠΎ Π½Π° 1 1/2, Ρ‡Ρ‚ΠΎ ΠΏΠΎΠ·Π²ΠΎΠ»ΠΈΡ‚ Ρ€Π΅Ρ†Π΅ΠΏΡ‚Ρƒ, ΠΊΠΎΡ‚ΠΎΡ€Ρ‹ΠΉ Π΄ΠΎΠ»ΠΆΠ΅Π½ Π½Π°ΠΊΠΎΡ€ΠΌΠΈΡ‚ΡŒ сСмью ΠΈΠ· Ρ‡Π΅Ρ‚Ρ‹Ρ€Π΅Ρ… Ρ‡Π΅Π»ΠΎΠ²Π΅ΠΊ, Ρ€Π°Π±ΠΎΡ‚Π°Ρ‚ΡŒ Π½Π° сСмью ΠΈΠ· ΡˆΠ΅ΡΡ‚ΠΈ Ρ‡Π΅Π»ΠΎΠ²Π΅ΠΊ.

Π”Ρ€ΠΎΠ±ΠΈ Π΅Π΄ΠΈΠ½ΠΈΡ†Ρ‹ дСлСния

Π Π°Π·Π΄Π΅Π»ΠΈΡ‚Π΅ Π΄Ρ€ΠΎΠ±ΠΈ Π΅Π΄ΠΈΠ½ΠΈΡ†Ρ‹ (Π΄Ρ€ΠΎΠ±ΠΈ с 1 Π² числитСлС ΠΈΠ»ΠΈ Π²Π΅Ρ€Ρ…Π½ΠΈΠΌ числом) Π½Π° Ρ†Π΅Π»Ρ‹Π΅ числа. Π Π°Π·Π΄Π΅Π»ΠΈΡ‚Π΅ Ρ†Π΅Π»Ρ‹Π΅ числа Π½Π° Π΅Π΄ΠΈΠ½ΠΈΡ‡Π½Ρ‹Π΅ Π΄Ρ€ΠΎΠ±ΠΈ.

ΠŸΡ€ΠΈΠΌΠ΅Ρ€:

Если Ρ‚Ρ€ΠΈ Ρ‡Π΅Π»ΠΎΠ²Π΅ΠΊΠ° раздСлят Β½ Ρ„ΡƒΠ½Ρ‚Π° шоколада ΠΏΠΎΡ€ΠΎΠ²Π½Ρƒ, сколько шоколада ΠΏΠΎΠ»ΡƒΡ‡ΠΈΡ‚ ΠΊΠ°ΠΆΠ΄Ρ‹ΠΉ? ΠžΠ±ΡŠΡΡΠ½ΠΈΡ‚Π΅ ΠΈΠ»ΠΈ ΠΏΡ€ΠΎΠΈΠ»Π»ΡŽΡΡ‚Ρ€ΠΈΡ€ΡƒΠΉΡ‚Π΅, ΠΊΠ°ΠΊ Π²Ρ‹ Ρ€Π΅ΡˆΠΈΠ»ΠΈ эту ΠΏΡ€ΠΎΠ±Π»Π΅ΠΌΡƒ.

Π£ΠΌΠ½ΠΎΠΆΠ΅Π½ΠΈΠ΅ Π½Π° Π΄Ρ€ΠΎΠ±ΠΈ

ΠŸΠΎΠΌΠ½ΠΈΡ‚Π΅, Ρ‡Ρ‚ΠΎ ΡƒΠΌΠ½ΠΎΠΆΠ΅Π½ΠΈΠ΅ числа Π½Π° Π΄Ρ€ΠΎΠ±ΡŒ мСньшС 1 даст ΠΎΡ‚Π²Π΅Ρ‚ мСньшС числа — Π½Π°ΠΏΡ€ΠΈΠΌΠ΅Ρ€: 12 x ΒΎ = 9. Π£ΠΌΠ½ΠΎΠΆΠ΅Π½ΠΈΠ΅ числа Π½Π° Π΄Ρ€ΠΎΠ±ΡŒ большС 1 даст Ρ€Π΅Π·ΡƒΠ»ΡŒΡ‚Π°Ρ‚ Π² ΠΎΡ‚Π²Π΅Ρ‚Π΅ большС числа — Π½Π°ΠΏΡ€ΠΈΠΌΠ΅Ρ€: 12 x 2 Β½ = 30.

Π˜Π·ΠΌΠ΅Ρ€Π΅Π½ΠΈΡ ΠΈ Π΄Π°Π½Π½Ρ‹Π΅

ΠŸΡ€Π΅ΠΎΠ±Ρ€Π°Π·ΠΎΠ²Π°Π½ΠΈΠ΅ Π΅Π΄ΠΈΠ½ΠΈΡ† ΠΈ Π΄Ρ€ΠΎΠ±Π΅ΠΉ

ΠŸΡ€Π΅ΠΎΠ±Ρ€Π°Π·ΠΎΠ²Π°Π½ΠΈΠ΅ Π΅Π΄ΠΈΠ½ΠΈΡ† ΠΈ Π΄ΠΎΠ»Π΅ΠΉ Π΅Π΄ΠΈΠ½ΠΈΡ† Π² ΠΎΠ΄Π½ΠΎΠΉ систСмС измСрСния.

ΠŸΡ€ΠΈΠΌΠ΅Ρ€:

Бколько ΠΌΠΈΠ½ΡƒΡ‚ составляСт 1⁄5 часа? ΠžΠ±ΡŠΡΡΠ½ΠΈΡ‚Π΅ ΠΈΠ»ΠΈ ΠΏΡ€ΠΎΠΈΠ»Π»ΡŽΡΡ‚Ρ€ΠΈΡ€ΡƒΠΉΡ‚Π΅, ΠΊΠ°ΠΊ Π²Ρ‹ Ρ€Π΅ΡˆΠΈΠ»ΠΈ эту ΠΏΡ€ΠΎΠ±Π»Π΅ΠΌΡƒ.

ΠŸΡ€ΠΎΠ±Π»Π΅ΠΌΡ‹ многоступСнчатого прСобразования Π΅Π΄ΠΈΠ½ΠΈΡ† измСрСния

Π Π΅ΡˆΠ°ΠΉΡ‚Π΅ многоступСнчатыС ΠΏΡ€ΠΎΠ±Π»Π΅ΠΌΡ‹ со словами, ΠΈΡΠΏΠΎΠ»ΡŒΠ·ΡƒΡ ΠΏΡ€Π΅ΠΎΠ±Ρ€Π°Π·ΠΎΠ²Π°Π½ΠΈΠ΅ стандартных Π΅Π΄ΠΈΠ½ΠΈΡ† измСрСния Ρ€Π°Π·Π½ΠΎΠ³ΠΎ Ρ€Π°Π·ΠΌΠ΅Ρ€Π°.

ΠŸΡ€ΠΈΠΌΠ΅Ρ€:

Π£ мСня 75 см Π»Π΅Π½Ρ‚Ρ‹.Для выполнСния ΠΏΡ€ΠΎΠ΅ΠΊΡ‚Π° ΠΌΠ½Π΅ Π½ΡƒΠΆΠ½ΠΎ Π² сСмь Ρ€Π°Π· большС Π»Π΅Π½Ρ‚Ρ‹. Бколько Π΅Ρ‰Π΅ ΠΌΠ΅Ρ‚Ρ€ΠΎΠ² Π»Π΅Π½Ρ‚Ρ‹ ΠΌΠ½Π΅ Π½ΡƒΠΆΠ½ΠΎ?

ΠžΠ±ΡŠΡΡΠ½ΠΈΡ‚Π΅ ΠΈΠ»ΠΈ ΠΏΡ€ΠΎΠΈΠ»Π»ΡŽΡΡ‚Ρ€ΠΈΡ€ΡƒΠΉΡ‚Π΅, ΠΊΠ°ΠΊ Π²Ρ‹ Ρ€Π΅ΡˆΠΈΠ»ΠΈ эту ΠΏΡ€ΠΎΠ±Π»Π΅ΠΌΡƒ.

ИспользованиС Π»ΠΈΠ½Π΅ΠΉΠ½ΠΎΠ³ΠΎ Π³Ρ€Π°Ρ„ΠΈΠΊΠ°

Π Π΅ΡˆΠ°ΠΉΡ‚Π΅ ΠΏΡ€ΠΎΠ±Π»Π΅ΠΌΡ‹, ΠΈΡΠΏΠΎΠ»ΡŒΠ·ΡƒΡ ΠΈΠ½Ρ„ΠΎΡ€ΠΌΠ°Ρ†ΠΈΡŽ (Π² Π΅Π΄ΠΈΠ½ΠΈΡ†Π°Ρ… Π΄Ρ€ΠΎΠ±ΠΈ), ΠΏΡ€Π΅Π΄ΡΡ‚Π°Π²Π»Π΅Π½Π½ΡƒΡŽ Π½Π° Π»ΠΈΠ½Π΅ΠΉΠ½ΠΎΠΌ Π³Ρ€Π°Ρ„ΠΈΠΊΠ΅.

ГСомСтрия

ОбъСм

Под объСмом понимаСтся ΠΈΠ·ΠΌΠ΅Ρ€Π΅Π½ΠΈΠ΅ пространства Π²Π½ΡƒΡ‚Ρ€ΠΈ Ρ‚Ρ€Π΅Ρ…ΠΌΠ΅Ρ€Π½ΠΎΠΉ ΠΈΠ»ΠΈ Ρ‚Π²Π΅Ρ€Π΄ΠΎΠΉ Ρ„ΠΈΠ³ΡƒΡ€Ρ‹. Π˜ΡΠΏΠΎΠ»ΡŒΠ·ΡƒΠΉΡ‚Π΅ Ρ„ΠΎΡ€ΠΌΡƒΠ»Ρ‹ Π΄Π»ΠΈΠ½Π° x ΡˆΠΈΡ€ΠΈΠ½Π° x высота ΠΈΠ»ΠΈ основаниС x высота , Ρ‡Ρ‚ΠΎΠ±Ρ‹ ΠΈΠ·ΠΌΠ΅Ρ€ΠΈΡ‚ΡŒ объСм Ρ‚Ρ€Π΅Ρ…ΠΌΠ΅Ρ€Π½ΠΎΠ³ΠΎ ΠΈΠ»ΠΈ Ρ‚Π²Π΅Ρ€Π΄ΠΎΠ³ΠΎ ΠΎΠ±ΡŠΠ΅ΠΊΡ‚Π° с ΠΏΡ€ΡΠΌΠΎΡƒΠ³ΠΎΠ»ΡŒΠ½Ρ‹ΠΌΠΈ сторонами, Π½Π°ΠΏΡ€ΠΈΠΌΠ΅Ρ€ ΠΊΡƒΠ±Π°.Π˜Π·ΠΌΠ΅Ρ€ΡΠΉΡ‚Π΅ объСм для Ρ€Π΅ΡˆΠ΅Π½ΠΈΡ Ρ€Π΅Π°Π»ΡŒΠ½Ρ‹Ρ… ΠΏΡ€ΠΎΠ±Π»Π΅ΠΌ.

ΠŸΡ€ΠΈΠΌΠ΅Ρ€:

ΠŸΡ€ΡΠΌΠΎΡƒΠ³ΠΎΠ»ΡŒΠ½Ρ‹ΠΉ ΠΊΠΎΠ½Ρ‚Π΅ΠΉΠ½Π΅Ρ€ для ΠΌΠΎΡ€ΠΎΠΆΠ΅Π½ΠΎΠ³ΠΎ ΠΈΠΌΠ΅Π΅Ρ‚ Π΄Π»ΠΈΠ½Ρƒ 8 дюймов ΠΈ высоту 4 дюйма. Каков объСм ΠΊΠΎΠ½Ρ‚Π΅ΠΉΠ½Π΅Ρ€Π°, Π²Ρ‹Ρ€Π°ΠΆΠ΅Π½Π½Ρ‹ΠΉ Π² кубичСских Π΄ΡŽΠΉΠΌΠ°Ρ…?

Π‘ΠΎΠ²Π΅Ρ‚Ρ‹, ΠΊΠΎΡ‚ΠΎΡ€Ρ‹Π΅ ΠΏΠΎΠΌΠΎΠ³ΡƒΡ‚ Π²Π°ΡˆΠ΅ΠΌΡƒ пятикласснику Π² ΡƒΡ€ΠΎΠΊΠ΅ ΠΌΠ°Ρ‚Π΅ΠΌΠ°Ρ‚ΠΈΠΊΠΈ, ΠΌΠΎΠΆΠ½ΠΎ Π½Π°ΠΉΡ‚ΠΈ Π½Π° нашСй страницС с совСтами ΠΏΠΎ ΠΌΠ°Ρ‚Π΅ΠΌΠ°Ρ‚ΠΈΠΊΠ΅ для пятого класса.

РСсурсы Parent Toolkit Π±Ρ‹Π»ΠΈ Ρ€Π°Π·Ρ€Π°Π±ΠΎΡ‚Π°Π½Ρ‹ NBC News Learn с ΠΏΠΎΠΌΠΎΡ‰ΡŒΡŽ ΠΏΡ€ΠΎΡ„ΠΈΠ»ΡŒΠ½Ρ‹Ρ… экспСртов ΠΈ ΡΠΎΠΎΡ‚Π²Π΅Ρ‚ΡΡ‚Π²ΡƒΡŽΡ‚ ΠžΠ±Ρ‰ΠΈΠΌ основным государствСнным стандартам.

Π‘Π½Π°Ρ‡Π°Π»Π° ΡƒΠΌΠ½ΠΎΠΆΠΈΡ‚ΡŒ ΠΈΠ»ΠΈ ΡΠ»ΠΎΠΆΠΈΡ‚ΡŒ? ΠŸΠΎΡ€ΡΠ΄ΠΎΠΊ обучСния ΠΏΡ€Π°Π²ΠΈΠ»Π°ΠΌ дСйствий

Когда ΡƒΡ‡Π΅Π½ΠΈΠΊΠΈ 3-Ρ… классов ΠΈ Π²Ρ‹ΡˆΠ΅ учатся ΡΠΊΠ»Π°Π΄Ρ‹Π²Π°Ρ‚ΡŒ, Π²Ρ‹Ρ‡ΠΈΡ‚Π°Ρ‚ΡŒ, ΡƒΠΌΠ½ΠΎΠΆΠ°Ρ‚ΡŒ, Π΄Π΅Π»ΠΈΡ‚ΡŒ ΠΈ Ρ€Π°Π±ΠΎΡ‚Π°Ρ‚ΡŒ с основными числовыми выраТСниями, ΠΎΠ½ΠΈ Π½Π°Ρ‡ΠΈΠ½Π°ΡŽΡ‚ с выполнСния ΠΎΠΏΠ΅Ρ€Π°Ρ†ΠΈΠΉ Π½Π°Π΄ двумя числами. Но Ρ‡Ρ‚ΠΎ происходит, ΠΊΠΎΠ³Π΄Π° Π²Ρ‹Ρ€Π°ΠΆΠ΅Π½ΠΈΠ΅ Ρ‚Ρ€Π΅Π±ΡƒΠ΅Ρ‚ Π½Π΅ΡΠΊΠΎΠ»ΡŒΠΊΠΈΡ… ΠΎΠΏΠ΅Ρ€Π°Ρ†ΠΈΠΉ? НапримСр, Π²Ρ‹ сначала складываСтС ΠΈΠ»ΠΈ ΡƒΠΌΠ½ΠΎΠΆΠ°Π΅Ρ‚Π΅? А ΠΊΠ°ΠΊ насчСт умноТСния ΠΈΠ»ΠΈ дСлСния? Π’ этой ΡΡ‚Π°Ρ‚ΡŒΠ΅ ΠΎΠ±ΡŠΡΡΠ½ΡΠ΅Ρ‚ΡΡ, Π² ΠΊΠ°ΠΊΠΎΠΌ порядкС Π²Ρ‹ΠΏΠΎΠ»Π½ΡΡŽΡ‚ΡΡ ΠΎΠΏΠ΅Ρ€Π°Ρ†ΠΈΠΈ, ΠΈ приводятся ΠΏΡ€ΠΈΠΌΠ΅Ρ€Ρ‹, ΠΊΠΎΡ‚ΠΎΡ€Ρ‹Π΅ Π²Ρ‹ Ρ‚Π°ΠΊΠΆΠ΅ ΠΌΠΎΠΆΠ΅Ρ‚Π΅ ΠΈΡΠΏΠΎΠ»ΡŒΠ·ΠΎΠ²Π°Ρ‚ΡŒ со студСнтами.Он Ρ‚Π°ΠΊΠΆΠ΅ содСрТит Π΄Π²Π° ΡƒΡ€ΠΎΠΊΠ°, ΠΊΠΎΡ‚ΠΎΡ€Ρ‹Π΅ ΠΏΠΎΠΌΠΎΠ³ΡƒΡ‚ Π²Π°ΠΌ ΠΏΡ€Π΅Π΄ΡΡ‚Π°Π²ΠΈΡ‚ΡŒ ΠΈ Ρ€Π°Π·Π²ΠΈΡ‚ΡŒ ΠΊΠΎΠ½Ρ†Π΅ΠΏΡ†ΠΈΡŽ.

ΠšΠ»ΡŽΡ‡Π΅Π²ΠΎΠΉ стандарт:

  • ВыполняйтС арифмСтичСскиС ΠΎΠΏΠ΅Ρ€Π°Ρ†ΠΈΠΈ, Π²ΠΊΠ»ΡŽΡ‡Π°ΡŽΡ‰ΠΈΠ΅ слоТСниС, Π²Ρ‹Ρ‡ΠΈΡ‚Π°Π½ΠΈΠ΅, ΡƒΠΌΠ½ΠΎΠΆΠ΅Π½ΠΈΠ΅ ΠΈ Π΄Π΅Π»Π΅Π½ΠΈΠ΅ Π² ΠΎΠ±Ρ‹Ρ‡Π½ΠΎΠΌ порядкС, нСзависимо ΠΎΡ‚ Ρ‚ΠΎΠ³ΠΎ, ΠΏΡ€ΠΈΡΡƒΡ‚ΡΡ‚Π²ΡƒΡŽΡ‚ Π»ΠΈ скобки ΠΈΠ»ΠΈ Π½Π΅Ρ‚. (ΠžΡ†Π΅Π½ΠΊΠ° 3)

ΠŸΠΎΡ€ΡΠ΄ΠΎΠΊ ΠΎΠΏΠ΅Ρ€Π°Ρ†ΠΈΠΉ — ΠΏΡ€ΠΈΠΌΠ΅Ρ€ ΠΌΠ°Ρ‚Π΅ΠΌΠ°Ρ‚ΠΈΠΊΠΈ, которая ΠΎΡ‡Π΅Π½ΡŒ ΠΏΡ€ΠΎΡ†Π΅Π΄ΡƒΡ€Π½Π°. Π›Π΅Π³ΠΊΠΎ ΠΎΡˆΠΈΠ±ΠΈΡ‚ΡŒΡΡ, ΠΏΠΎΡ‚ΠΎΠΌΡƒ Ρ‡Ρ‚ΠΎ это Π½Π΅ ΡΡ‚ΠΎΠ»ΡŒΠΊΠΎ концСпция, ΠΊΠΎΡ‚ΠΎΡ€ΡƒΡŽ Π²Ρ‹ усвоили, Π° скорСС список ΠΏΡ€Π°Π²ΠΈΠ», ΠΊΠΎΡ‚ΠΎΡ€Ρ‹Π΅ Π²Π°ΠΌ Π½ΡƒΠΆΠ½ΠΎ Π·Π°ΠΏΠΎΠΌΠ½ΠΈΡ‚ΡŒ.Но Π½Π΅ ΠΎΠ±ΠΌΠ°Π½Ρ‹Π²Π°ΠΉΡ‚Π΅ΡΡŒ, думая, Ρ‡Ρ‚ΠΎ ΠΏΡ€ΠΎΡ†Π΅Π΄ΡƒΡ€Π½Ρ‹Π΅ Π½Π°Π²Ρ‹ΠΊΠΈ Π½Π΅ ΠΌΠΎΠ³ΡƒΡ‚ Π±Ρ‹Ρ‚ΡŒ Π³Π»ΡƒΠ±ΠΎΠΊΠΈΠΌΠΈ! Он ΠΌΠΎΠΆΠ΅Ρ‚ ΠΏΡ€Π΅Π΄ΡΡ‚Π°Π²Π»ΡΡ‚ΡŒ слоТныС ΠΏΡ€ΠΎΠ±Π»Π΅ΠΌΡ‹, подходящиС для ΡΡ‚Π°Ρ€ΡˆΠΈΡ… школьников ΠΈ ΡΠΎΠ·Ρ€Π΅Π²ΡˆΠΈΠ΅ для обсуТдСния Π² классС:

  • ΠœΠ΅Π½ΡΠ΅Ρ‚ΡΡ Π»ΠΈ ΠΏΡ€Π°Π²ΠΈΠ»ΠΎ слСва Π½Π°ΠΏΡ€Π°Π²ΠΎ, ΠΊΠΎΠ³Π΄Π° ΡƒΠΌΠ½ΠΎΠΆΠ΅Π½ΠΈΠ΅ подразумСваСтся, Π° Π½Π΅ прописано? (НапримСр, \ (3g \) ΠΈΠ»ΠΈ \ (8 (12) \) вмСсто \ (3 \ times g \) ΠΈΠ»ΠΈ \ (8 \ cdot 12 \).)
  • Π“Π΄Π΅ Ρ„Π°ΠΊΡ‚ΠΎΡ€ΠΈΠ°Π» ΠΏΠΎΠΏΠ°Π΄Π°Π΅Ρ‚ Π² порядок ΠΎΠΏΠ΅Ρ€Π°Ρ†ΠΈΠΈ?
  • Π§Ρ‚ΠΎ происходит, ΠΊΠΎΠ³Π΄Π° ΠΏΠΎΠΊΠ°Π·Π°Ρ‚Π΅Π»ΡŒ стСпСни возводится Π² Π΄Ρ€ΡƒΠ³ΠΎΠΉ ΠΏΠΎΠΊΠ°Π·Π°Ρ‚Π΅Π»ΡŒ, Π½ΠΎ скобок Π½Π΅Ρ‚? (ΠžΠ±Ρ€Π°Ρ‚ΠΈΡ‚Π΅ Π²Π½ΠΈΠΌΠ°Π½ΠΈΠ΅, Ρ‡Ρ‚ΠΎ этот ΡƒΡ€ΠΎΠΊ Π½Π΅ Π²ΠΊΠ»ΡŽΡ‡Π°Π΅Ρ‚ экспонСнты, хотя, Ссли учащиСся Π³ΠΎΡ‚ΠΎΠ²Ρ‹, Π²Ρ‹ ΠΌΠΎΠΆΠ΅Ρ‚Π΅ Ρ€Π°ΡΡˆΠΈΡ€ΠΈΡ‚ΡŒ свой ΡƒΡ€ΠΎΠΊ, Π²ΠΊΠ»ΡŽΡ‡ΠΈΠ² ΠΈΡ….)

Π§Ρ‚ΠΎ ΠΏΠ΅Ρ€Π²ΠΈΡ‡Π½ΠΎ Π² порядкС Ρ€Π°Π±ΠΎΡ‚Ρ‹?

Π‘ΠΎ Π²Ρ€Π΅ΠΌΠ΅Π½Π΅ΠΌ ΠΌΠ°Ρ‚Π΅ΠΌΠ°Ρ‚ΠΈΠΊΠΈ согласовали Π½Π°Π±ΠΎΡ€ ΠΏΡ€Π°Π²ΠΈΠ», Π½Π°Π·Ρ‹Π²Π°Π΅ΠΌΡ‹ΠΉ порядком ΠΎΠΏΠ΅Ρ€Π°Ρ†ΠΈΠΉ , Ρ‡Ρ‚ΠΎΠ±Ρ‹ ΠΎΠΏΡ€Π΅Π΄Π΅Π»ΠΈΡ‚ΡŒ, ΠΊΠ°ΠΊΡƒΡŽ ΠΎΠΏΠ΅Ρ€Π°Ρ†ΠΈΡŽ Π²Ρ‹ΠΏΠΎΠ»Π½ΠΈΡ‚ΡŒ Π² ΠΏΠ΅Ρ€Π²ΡƒΡŽ ΠΎΡ‡Π΅Ρ€Π΅Π΄ΡŒ. Когда Π²Ρ‹Ρ€Π°ΠΆΠ΅Π½ΠΈΠ΅ Π²ΠΊΠ»ΡŽΡ‡Π°Π΅Ρ‚ Ρ‚ΠΎΠ»ΡŒΠΊΠΎ Ρ‡Π΅Ρ‚Ρ‹Ρ€Π΅ основных ΠΎΠΏΠ΅Ρ€Π°Ρ†ΠΈΠΈ, Π²ΠΎΡ‚ ΠΏΡ€Π°Π²ΠΈΠ»Π°:

  1. Π£ΠΌΠ½ΠΎΠΆΠ°ΠΉΡ‚Π΅ ΠΈ Π΄Π΅Π»ΠΈΡ‚Π΅ слСва Π½Π°ΠΏΡ€Π°Π²ΠΎ.
  2. Π‘Π»ΠΎΠΆΠΈΡ‚ΡŒ ΠΈ Π²Ρ‹Ρ‡Π΅ΡΡ‚ΡŒ слСва Π½Π°ΠΏΡ€Π°Π²ΠΎ.

ΠŸΡ€ΠΈ ΡƒΠΏΡ€ΠΎΡ‰Π΅Π½ΠΈΠΈ выраТСния, Ρ‚Π°ΠΊΠΎΠ³ΠΎ ΠΊΠ°ΠΊ \ (12 \ div 4 + 5 \ times 3-6 \), сначала вычислитС \ (12 \ div 4 \), ΠΏΠΎΡΠΊΠΎΠ»ΡŒΠΊΡƒ порядок ΠΎΠΏΠ΅Ρ€Π°Ρ†ΠΈΠΉ Ρ‚Ρ€Π΅Π±ΡƒΠ΅Ρ‚ сначала ΠΎΡ†Π΅Π½ΠΊΠΈ любого умноТСния ΠΈ дСлСния (Π² зависимости ΠΎΡ‚ Ρ‚ΠΎΠ³ΠΎ, Ρ‡Ρ‚ΠΎ ΠΏΡ€ΠΎΠΈΠ·ΠΎΠΉΠ΄Π΅Ρ‚ ΠΏΠ΅Ρ€Π²Ρ‹ΠΉ) слСва Π½Π°ΠΏΡ€Π°Π²ΠΎ ΠΏΠ΅Ρ€Π΅Π΄ вычислСниСм слоТСния ΠΈΠ»ΠΈ вычитания.Π’ Π΄Π°Π½Π½ΠΎΠΌ случаС это ΠΎΠ·Π½Π°Ρ‡Π°Π΅Ρ‚ сначала вычислСниС \ (12 \ div 4 \), Π° Π·Π°Ρ‚Π΅ΠΌ \ (5 \ times 3 \). ПослС Ρ‚ΠΎΠ³ΠΎ, ΠΊΠ°ΠΊ всС ΡƒΠΌΠ½ΠΎΠΆΠ΅Π½ΠΈΠ΅ ΠΈ Π΄Π΅Π»Π΅Π½ΠΈΠ΅ Π±ΡƒΠ΄ΡƒΡ‚ Π·Π°Π²Π΅Ρ€ΡˆΠ΅Π½Ρ‹, ΠΏΡ€ΠΎΠ΄ΠΎΠ»ΠΆΠ°ΠΉΡ‚Π΅, добавляя ΠΈΠ»ΠΈ вычитая (Π² зависимости ΠΎΡ‚ Ρ‚ΠΎΠ³ΠΎ, Ρ‡Ρ‚ΠΎ наступит Ρ€Π°Π½ΡŒΡˆΠ΅) слСва Π½Π°ΠΏΡ€Π°Π²ΠΎ. Π¨Π°Π³ΠΈ ΠΏΠΎΠΊΠ°Π·Π°Π½Ρ‹ Π½ΠΈΠΆΠ΅.

\ (12 \ div 4 + 5 \ times 3-6 \)
\ (3 + 5 \ times 3-6 \) ΠŸΠΎΡ‚ΠΎΠΌΡƒ Ρ‡Ρ‚ΠΎ \ (12 \ div 4 = 3 \)
\ (3 + 15 — 6 \) ΠŸΠΎΡ‚ΠΎΠΌΡƒ Ρ‡Ρ‚ΠΎ \ (5 \ times 3 = 15 \)
\ (18-6 \) ΠŸΠΎΡ‚ΠΎΠΌΡƒ Ρ‡Ρ‚ΠΎ \ (3 + 15 = 18 \)
\ (12 \) ΠŸΠΎΡ‚ΠΎΠΌΡƒ Ρ‡Ρ‚ΠΎ \ (18-6 = 12 \)

Рассмотрим Π² качСствС ΠΏΡ€ΠΈΠΌΠ΅Ρ€Π° Π΄Ρ€ΡƒΠ³ΠΎΠ΅ Π²Ρ‹Ρ€Π°ΠΆΠ΅Π½ΠΈΠ΅:

\ (6 + 4 \ times 7-3 \)
\ (6 + 28-3 \) ΠŸΠΎΡ‚ΠΎΠΌΡƒ Ρ‡Ρ‚ΠΎ \ (4 \ times 7 = 28 \), Ρ‡Ρ‚ΠΎ выполняСтся ΠΏΠ΅Ρ€Π²Ρ‹ΠΌ, ΠΏΠΎΡ‚ΠΎΠΌΡƒ Ρ‡Ρ‚ΠΎ ΡƒΠΌΠ½ΠΎΠΆΠ΅Π½ΠΈΠ΅ ΠΈ Π΄Π΅Π»Π΅Π½ΠΈΠ΅ ΠΎΡ†Π΅Π½ΠΈΠ²Π°ΡŽΡ‚ΡΡ Π² ΠΏΠ΅Ρ€Π²ΡƒΡŽ ΠΎΡ‡Π΅Ρ€Π΅Π΄ΡŒ.
\ (34-3 \) ΠŸΠΎΡ‚ΠΎΠΌΡƒ Ρ‡Ρ‚ΠΎ \ (6 + 28 = 34 \)
\ (31 \) ΠŸΠΎΡ‚ΠΎΠΌΡƒ Ρ‡Ρ‚ΠΎ \ (34-3 = 1 \)

Иногда ΠΌΡ‹ ΠΌΠΎΠΆΠ΅ΠΌ Π·Π°Ρ…ΠΎΡ‚Π΅Ρ‚ΡŒ ΡƒΠ±Π΅Π΄ΠΈΡ‚ΡŒΡΡ, Ρ‡Ρ‚ΠΎ сначала выполняСтся слоТСниС ΠΈΠ»ΠΈ Π²Ρ‹Ρ‡ΠΈΡ‚Π°Π½ΠΈΠ΅. Π“Ρ€ΡƒΠΏΠΏΠΈΡ€ΠΎΠ²ΠΊΠ° символов , Ρ‚Π°ΠΊΠΈΡ… ΠΊΠ°ΠΊ скобки , \ (() \), скобки , \ ([] \) ΠΈΠ»ΠΈ Ρ„ΠΈΠ³ΡƒΡ€Π½Ρ‹Π΅ скобки , \ (\ {\} \), ΠΏΠΎΠ·Π²ΠΎΠ»ΡΡŽΡ‚ Π½Π°ΠΌ ΠΎΠΏΡ€Π΅Π΄Π΅Π»ΡΡ‚ΡŒ порядок, Π² ΠΊΠΎΡ‚ΠΎΡ€ΠΎΠΌ Π²Ρ‹ΠΏΠΎΠ»Π½ΡΡŽΡ‚ΡΡ ΠΎΠΏΡ€Π΅Π΄Π΅Π»Π΅Π½Π½Ρ‹Π΅ ΠΎΠΏΠ΅Ρ€Π°Ρ†ΠΈΠΈ. Π²Ρ‹ΠΏΠΎΠ»Π½Π΅Π½ΠΎ.

ΠŸΠΎΡ€ΡΠ΄ΠΎΠΊ ΠΎΠΏΠ΅Ρ€Π°Ρ†ΠΈΠΉ Ρ‚Ρ€Π΅Π±ΡƒΠ΅Ρ‚, Ρ‡Ρ‚ΠΎΠ±Ρ‹ ΠΎΠΏΠ΅Ρ€Π°Ρ†ΠΈΠΈ Π²Π½ΡƒΡ‚Ρ€ΠΈ символов Π³Ρ€ΡƒΠΏΠΏΠΈΡ€ΠΎΠ²ΠΊΠΈ Π²Ρ‹ΠΏΠΎΠ»Π½ΡΠ»ΠΈΡΡŒ ΠΏΠ΅Ρ€Π΅Π΄ опСрациями Π²Π½Π΅ ΠΈΡ….НапримСр, ΠΏΡ€Π΅Π΄ΠΏΠΎΠ»ΠΎΠΆΠΈΠΌ, Ρ‡Ρ‚ΠΎ Π²Ρ‹Ρ€Π°ΠΆΠ΅Π½ΠΈΠ΅ 6 + 4 Π·Π°ΠΊΠ»ΡŽΡ‡Π΅Π½ΠΎ Π² ΠΊΡ€ΡƒΠ³Π»Ρ‹Π΅ скобки:

\ ((6 + 4) \ times 7-3 \)
\ (10 ​​\ times 7-3 \) ΠŸΠΎΡ‚ΠΎΠΌΡƒ Ρ‡Ρ‚ΠΎ \ (6 + 4 = 10 \), Ρ‡Ρ‚ΠΎ ΠΈ дСлаСтся Π²ΠΎ-ΠΏΠ΅Ρ€Π²Ρ‹Ρ…, ΠΏΠΎΡ‚ΠΎΠΌΡƒ Ρ‡Ρ‚ΠΎ ΠΎΠ½ Π·Π°ΠΊΠ»ΡŽΡ‡Π΅Π½ Π² ΠΊΡ€ΡƒΠ³Π»Ρ‹Π΅ скобки.
\ (70 — 3 \) ΠŸΠΎΡ‚ΠΎΠΌΡƒ Ρ‡Ρ‚ΠΎ \ (10 ​​\ times 7 = 70 \), ΠΈ скобок большС Π½Π΅Ρ‚.
\ (67 \) ΠŸΠΎΡ‚ΠΎΠΌΡƒ Ρ‡Ρ‚ΠΎ \ (70-3 = 67 \)

ΠžΠ±Ρ€Π°Ρ‚ΠΈΡ‚Π΅ Π²Π½ΠΈΠΌΠ°Π½ΠΈΠ΅, Ρ‡Ρ‚ΠΎ Π²Ρ‹Ρ€Π°ΠΆΠ΅Π½ΠΈΠ΅ ΠΈΠΌΠ΅Π΅Ρ‚ ΡΠΎΠ²Π΅Ρ€ΡˆΠ΅Π½Π½ΠΎ Π΄Ρ€ΡƒΠ³ΠΎΠ΅ Π·Π½Π°Ρ‡Π΅Π½ΠΈΠ΅! Π§Ρ‚ΠΎ, Ссли вмСсто этого ΠΌΡ‹ Π·Π°ΠΊΠ»ΡŽΡ‡ΠΈΠΌ \ (7 — 3 \) Π² ΠΊΡ€ΡƒΠ³Π»Ρ‹Π΅ скобки?

\ (6 + 4 \ times (7-3) \)
\ (6 + 4 \ times 4 \) На этот Ρ€Π°Π· \ (7-3 \) находится Π² скобках, Ρ‚Π°ΠΊ Ρ‡Ρ‚ΠΎ ΠΌΡ‹ Π΄Π΅Π»Π°Π΅ΠΌ это Π² ΠΏΠ΅Ρ€Π²ΡƒΡŽ ΠΎΡ‡Π΅Ρ€Π΅Π΄ΡŒ.

Π”ΠΎΠ±Π°Π²ΠΈΡ‚ΡŒ ΠΊΠΎΠΌΠΌΠ΅Π½Ρ‚Π°Ρ€ΠΈΠΉ

Π’Π°Ρˆ адрСс email Π½Π΅ Π±ΡƒΠ΄Π΅Ρ‚ ΠΎΠΏΡƒΠ±Π»ΠΈΠΊΠΎΠ²Π°Π½. ΠžΠ±ΡΠ·Π°Ρ‚Π΅Π»ΡŒΠ½Ρ‹Π΅ поля ΠΏΠΎΠΌΠ΅Ρ‡Π΅Π½Ρ‹ *

Β© 2015 - 2019 ΠœΡƒΠ½ΠΈΡ†ΠΈΠΏΠ°Π»ΡŒΠ½ΠΎΠ΅ ΠΊΠ°Π·Ρ‘Π½Π½ΠΎΠ΅ ΠΎΠ±Ρ‰Π΅ΠΎΠ±Ρ€Π°Π·ΠΎΠ²Π°Ρ‚Π΅Π»ΡŒΠ½ΠΎΠ΅ ΡƒΡ‡Ρ€Π΅ΠΆΠ΄Π΅Π½ΠΈΠ΅ «Валовская срСдняя школа»

ΠšΠ°Ρ€Ρ‚Π° сайта